You are on page 1of 612

FA 2022

Advanced Taxation
(ATX - UK) Course
notes
In the exam ............................................................................................... 4
Syllabus A1: Income Tax ...........................................................................6
Syllabus A1a. TX - UK Recap The scope of income tax .....................................................6
Syllabus A1a. TX - UK Recap Income from employment ..................................................11
Syllabus: A1a. TX - UK Recap Income from self-employment ............................................49
Syllabus A1a. TX - UK Recap Property and investment income .......................................110
Syllabus A1a. TX - UK Recap The Comprehensive computation of taxable income ..........135
Syllabus A1a. TX - UK Recap National insurance contributions ........................................143
Syllabus A1a. TX - UK Recap Exemptions and reliefs .....................................................152
Syllabus A1b. The Scope of Income Tax .......................................................................165
Syllabus A1c. Income from employment .......................................................................181
Syllabus A1d. Income from self employment ..................................................................195
Syllabus A1e. Property and investment income ..............................................................202
Syllabus A1f. Income tax computation and income tax liability ..........................................212
Syllabus A1g. Exemptions and Reliefs for I.T. .................................................................215
Syllabus A2: Chargeable Gains For Individuals ......................................220
Syllabus A2a. TX - UK Recap The scope of the taxation of capital gains ...........................220
Syllabus A2a. TX - UK Recap The basic principles of computing gains and losses ............223
Syllabus A2a. TX - UK Recap Gains and losses on the disposal of property .....................236
Syllabus A2a. TX - UK Recap: Gains and losses on the disposal of shares and securities ..246
Syllabus A2a. TX - UK Recap Entrepreneurs’ relief/Business Asset Disposal Relief ............251
Syllabus A2b. Chargeable gains ...................................................................................270
Syllabus A2c. Trusts ...................................................................................................287
Syllabus A2d. Principles of computing gains and losses .................................................289
Syllabus A2e. Disposal of movable and immovable property ............................................295
Syllabus A2f. Disposals of shares and securities ............................................................307
Syllabus A2g. Exemptions and Reliefs for C.G.T. ............................................................316
Syllabus A3: Inheritance Tax ..................................................................326
Syllabus A3a. TX - UK Recap: Basic principles of computing transfers of value .................326
Syllabus A3a. TX - UK Recap: IHT arising on lifetime transfers and on death .....................338
Syllabus A3a. TX - UK Recap: Exemptions to defer / minimise IHT ...................................350
Syllabus A3b. The scope of IHT ...................................................................................354
Syllabus A3c. Computing transfers of value ................................................................... 361
Syllabus A3d. IHT liabilities on lifetime transfers during life and death .................................372

1
Syllabus A3e. Trusts ...................................................................................................390
Syllabus A3f. IHT planning ........................................................................................... 393
Syllabus A3g. IHT administration ...................................................................................394
Syllabus A4: Corporation Tax ................................................................402
Syllabus A4a. TX - UK Recap: The scope of corporation tax ...........................................402
Syllabus A4a. TX - UK Recap: Taxable total profits .........................................................406
Syllabus A4a. TX - UK Recap: Chargeable gains for companies ......................................456
Syllabus A4a. TX - UK Recap: The comprehensive computation of corporation tax liability ..474
Syllabus A4a. TX - UK Recap: Group corporate structure for C.T. ...................................479
Syllabus A4b. The scope of CT ...................................................................................489
Syllabus A4c. Taxable total profits .................................................................................499
Syllabus A4d. The corporation tax liability .......................................................................510
Syllabus A4e. Group Structure for C.T. ......................................................................... 517
Syllabus A5: Stamp Taxes .....................................................................527
Syllabus A5a. The scope of stamp taxes ...................................................................... 527
Syllabus A5b. Liabilities arising on transfers ....................................................................528
Syllabus A5c/d. Exemptions and Reliefs .......................................................................532
Syllabus A6: Value Added Tax ............................................................... 534
Syllabus A6a. TX - UK Recap: The VAT registration requirements .....................................534
Syllabus A6a. TX - UK Recap: Computation of VAT liabilities ............................................546
Syllabus A6a. TX - UK Recap: The effect of special schemes .........................................569
Syllabus A6b. TX - UK Recap: The overall function and purpose of taxation. .....................582
Syllabus A6b. TX - UK Recap: Principal sources of revenue law and practice ...................585
Syllabus A6b. TX - UK Recap: The systems for self-assessment and the making of returns .....
591
Syllabus A6b. TX - UK Recap: The Time Limits ..............................................................593
Syllabus A6b. TX - UK Recap: Compliance checks, appeals and disputes .......................598
Syllabus A6b. TX - UK Recap: Penalties for non-compliance ...........................................600
Syllabus A6bi. Offshore Matters ...................................................................................602
Syllabus B: Financial Decisions made by a business ..............................603
Syllabus B2. Alternative ways of achieving outcomes ......................................................603
Syllabus B3. Different types of finance and investment ....................................................604
Syllabus C. Ethics .................................................................................606
Syllabus C5/6. Ethics ................................................................................................. 606

2
Syllabus D. Communication in an appropriate manner ...........................609
Syllabus D. Communication .........................................................................................609

3
In the exam

The style and format of the Advanced Taxation (ATX - UK) exam

The ATX - UK exam is offered in paper format only.

Candidates are given 3 hours and 15 minutes.

ATX - UK consists of two sections, A and B. All questions are compulsory.

Questions 1 and 2 in Section A are worth 35 and 25 marks respectively. 

Question 1 will include 4 professional marks. 

Questions 3 and 4 in Section B are each worth 20 marks. 

The whole of the syllabus is examinable throughout both sections of the exam.

The Section A questions require candidates to analyse the information provided and to
use any guidance given to help address the requirements. 

Both questions are likely to deal with a number of different taxes and Question 1 will require
a report, letter, memo or meeting notes as part of the answer. 

There will be 5 marks for Ethics across Section A.

Section A questions are relatively large and so careful time management is important, and
candidates are advised to use the number of marks allocated to each requirement to
determine how much time to spend on each part.

The Section B questions contain an introductory paragraph, which outlines the technical
areas within the question, together with concise structured information and sub-headings to
make them easier to assimilate and navigate. 

Throughout the exam, candidates are expected to be able to identify issues, as well as
demonstrate detailed knowledge of the tax system.

In line with this emphasis on practicality, questions may require candidates to address ‘the
UK tax consequences’ of a given situation without indicating which particular taxes to
consider. 

It is up to candidates to identify the relevant taxes, and the issues in respect of those taxes,
before beginning their answers.

Calculations are normally only required in support of explanations and advice, and not in
isolation. 

4
Again, it is often up to candidates to decide what calculations to produce in order to do this
in the most efficient manner. 

Advice on how to approach a given problem may be provided in the question.

Numerical calculation versus narrative balance:

ATX - UK is weighted 65% towards narrative explanations. 

The ability of candidates to be able to explain their treatments and opinions is vital. 

It is important to note that this does not mean that candidates need to have perfect
grammar or spelling; it means that they need to make themselves understood.

ATX - UK has 4 marks within Question 1 for professional skills, known as professional
marks. 

In order to score well, candidates first have to satisfy the requirement in relation to the style
and format of the document requested. 

Further marks are then available for providing clear explanations and coherent calculations.

ATX - UK is directly underpinned by TX - UK

Knowledge and understanding of the technical content of TX - UK is, therefore, vital if


candidates are to be successful at ATX - UK. 

It is quite possible that the technical content of a ATX - UK question could be drawn almost
wholly from the TX - UK syllabus. 

However, such a question will require the analysis of information provided, and the
application of technical knowledge to the situation in order to solve the problem.

The ATX - UK syllabus extends the coverage of income tax, corporation tax, capital gains
tax and inheritance tax and introduces stamp taxes (stamp duty land tax and stamp duty). 

While no part of the syllabus is more important than any other, it should be recognised from
the above that knowledge of the technical areas that are exclusive to ATX - UK will not, on
its own, be sufficient to pass the exam.

Candidates are required to explain, calculate and apply their knowledge of the system of
taxation in the UK.

5
Syllabus A1: Income Tax

Syllabus A1a. TX - UK Recap The scope of income tax

The contents of the Paper TX - UK study guide for income tax and national insurance, under
headings: 
- The scope of income tax

How is the residence of an individual determined?

3 Stages to determine UK residency

If a person is UK resident in a particular tax year they must pay income tax on their
UK and overseas income.

If a person is not UK resident, then they do not pay UK income tax on their overseas
income, they only pay on their UK income.

The determination of whether a person is UK resident is based on 3 major stages


in the following order:

1. Is the person automatically resident overseas? (2 tests)

2. Is the person automatically UK resident? (3 tests)

3. None above satisfied, use table.

6
STAGE 1: Is the person automatically resident overseas?

If one of the two tests below are satisfied, then the person is automatically resident
overseas and will only pay UK income tax on his UK income.

Test 1 – Short stay in UK Test 2 – Employed abroad

In UK for less than 16 days in the tax Employed overseas and visits UK for
year. less than 91 days during the tax year.

OR

In UK for less than 46 days in tax year


and never previously UK resident.

Illustration:

Shayna is in the UK for 40 days during the tax year. She was not previously UK resident.

Will she be considered to be UK resident for this tax year?

Solution:

No - Shayna satisfies Test 1 – Short stay in the UK. This is because she is in the UK for
less than 46 days and has not been UK resident previously.

7
STAGE 2: Is the person automatically UK resident?

If any of the tests below are satisfied, then the individual is automatically UK resident
and will pay UK income tax on his worldwide earnings. If none of the above tests are
met, then there are detailed rules to follow to determine residency.

Test 2 –  Nowhere else to Test 3 -  Employed in


Test 1 -  Long stay 
go the UK
Spends at least 183
Employed full time in
days in the UK in the Only home is in the UK
the UK. 
tax year

Illustration:

Hemant is in the UK for 60 days during the tax year, his only house is in the UK.

Will he be considered to be UK resident for this tax year?

Solution

Yes - Hemant will be considered to be UK resident for this tax year because he satisfies
Test 2 – nowhere else to go.

8
The foundation of the detailed rules is based on:

Was the individual previously UK resident?

How many days did they spend in the UK in this tax year?

Do they have the amount of UK ties necessary to be considered UK resident in this tax
year?

Not previously UK
Days in the UK Previously UK resident
resident
Less than 16 Automatically not UK Automatically not UK
days resident resident
Automatically not UK
16-45 Resident if 4 UK ties
resident

46-90 Resident if 3 UK ties Resident if 4 UK ties

91-120 Resident if 2 UK ties Resident if 3 UK ties

121-182 Resident if 1 UK ties Resident if 2 UK ties

183 days Automatically UK resident Automatically UK resident

Ties

1. Close family in the UK (Wife, child).

2. House in the UK which is used during the tax year. (at least 1 night during the tax year)

3. In the UK for more time than any other country.

4. In the UK for more than 90 days in either of the previous 2 tax years.

5. Doing substantive work in the UK. (40 days or more)

9
Illustration:

Candice has always been UK resident.

During this tax year she purchased a villa in India where she lived for most of this tax
year.

She has a house in the UK where her husband and children stay. During this tax year
she spent 100 days in the UK.

Will Candice be considered to be UK resident in this tax year?

Solution:

Yes - Candice will be considered to be UK resident.

This is because:

1. She spent 100 days in the UK. (Therefore: Resident if 2 UK ties)

2. She was previously UK resident.

3. She has 2 ties in the UK, close family and a house.

10
Syllabus A1a. TX - UK Recap Income from employment

The contents of the Paper TX - UK study guide for income tax and national insurance, under
headings: 
- Income from employment

Employed or self-employed?

Why do we want to know whether an individual is employed or self


employed?

We want to know whether an individual is employed or self employed because there is a


difference in the tax allowable deductions for the employed and the self employed.

Additionally, there is a difference in the National Insurance Contributions payable by the


employed and the self employed.

How to determine whether an engagement is treated as employment or self


employment?

The main test of an employment as opposed to self-employment is the existence of a


contract of service (employee) compared with a contract for services (self employed).

11
If there is no contract of service, the following suggest employment:

Factors Explanation

If the individual holds an integral position (e.g.) chairman of the


Integral
organisation, they must be employed. You cannot hold key
Position
positions and be self employed.
An employee will be paid his regular wage regardless of the
Risk organisation making a profit or not. He does not bear the
financial risk.

The employer controls the manner and method of work, the


Control
employee must obey.

Legal The employee is entitled to benefits normally provided to


rights employees, for example: holiday pay and sick pay. 

Equipment The employee does not provide his own equipment.

The employee is obliged to work personally and exclusively for


Exclusivity
the employer, and cannot hire their own helpers.

Illustration:

Liam is a driver of luxury cars and started working for Super Cars Ltd on 6 April.

He works a set number of hours each week and is paid an hourly rate for the work that
he does.

When Liam works more than the set number of hours, he is paid overtime.

Liam is under an obligation to accept the work offered by Super Cars Ltd., and the work
is carried out under the control of the customer services manager.

All the vehicles used by Liam are provided by Super Cars Ltd.

What are the factors that indicate that he should be treated as an employee?

12
Solution:

Risk – none. He works for a set number of hours and is paid at an hourly rate. If he
works more than the set number of hours, he is paid overtime.

Control – the customer services manager controls the manner in which the work is
done.

Equipment – all of the cars driven are provided by Super Cars Ltd.

Exclusivity – Liam must do the work provided to him personally.

Conclusion:

Liam will pay income tax under the employment income rules.

Liam will also pay Employee Class 1 NIC.

Super Cars Ltd will pay Employer's Class 1 NIC and Class 1 A NIC on behalf of Liam.

13
Which employment income to tax in a tax year?

Emoluments

Emoluments are amounts that an employee will pay income tax and national insurance
contributions on. 

They include:

• salary

• bonus

• benefits

Emoluments are taxable on the earlier of 2 dates:

1 When a person becomes entitled to payment of the earnings

2 When payment is made

Example

Peter became entitled to be paid a bonus on 31 January 2023 (Tax year 22/23).

He was actually paid the bonus on 30 April 2023 (Tax year 23/24).

• He should pay income tax on it in the tax year 22/23.

Illustration:

Frank is employed in Cow plc and his annual salary is £36,000.

On 06/01/23 his annual salary was increased to £42,000.

Note His salary is due for a tax year which runs from 6th April - 5th April and salaries
accrue evenly over a tax year.

His salary has changed in between the tax year.

Therefore, the salary per month must be calculated and then totalled to give a figure for the
entire year.

14
Additionally, Frank receives the following bonuses based on the company’s results:

For the year ending 31/12/2021 £3000. He became entitled to the bonus on 31/12/21 and
was paid the bonus on 31/5/22.

For the year ending 31/12/2022 £10,000. He became entitled to the bonus on 31/12/22 and
was paid the bonus on 31/5/23.

• Required:

Calculate the taxable income in 2022/23.

Solution:

His salary accrues evenly over the year from 06/04/2022


- 05/04/2023, therefore:
For the 9 months (06/04/22-05/01/23), he will be entitled £36,000 * 9/12 =  £27,000
to:
For the remaining 3 months (06/01/23-05/04/23), he will £42,000 * 3/12 =  £ 10,500
be entitled to:
Salary in Total 27 + 10.5 = £ 37,500

His bonus of £3,000 will be taxed in the tax year 21/22 as he became entitled to it on 31
December 2021.

His bonus of £10,000 will be taxed in the tax year 22/23 as he became entitled to it on 31
December 2022.

Conclusion:

Salary= £37,500

Bonus = £10,000

Total income = £47,500

Less personal allowance = (£12,570)

Taxable income = £ 34,930

15
What is taxable employment income?

Employment income to be assessed for tax include:

1. Salary

2. Wages

3. Bonus

4. Commission

5. Benefits in kind - there are 2 types of benefits - taxable benefits and exempt benefits.

Only taxable benefits will have income tax paid on them, exempt benefits will not.

16
Expenses that you are allowed to deduct from
employment income

Total employment income calculation:

Salary x

Commission x
Benefits in kind x

= Gross emoluments  x

Less allowable deductions (x)


= Total employment income x

Allowable deductions:

1. Contributions to an occupational pension scheme.

Note that Payments to a personal pension scheme are NOT allowable deductions.

More in Topic Pensions.

2. Travel, subsistence and entertaining incurred wholly, exclusively and necessarily in the
performance of duties of employment

3. Subscription to a professional body (e.g.) ACCA

Note that payments for gym memberships are NOT allowable deductions.

4. Deficit on a mileage allowance - Topic The authorised mileage allowances.

5. Donations to charity

Note that Donations to political parties are not allowable deductions.

6. Capital allowances are available for plant and machinery provided by an employee for
us in his duties Topic Capital allowances

17
Illustration:

Paresh had a salary of £30,000.

He paid £500 for his ACCA subscription.

He also paid £1,000 for travel to Scotland for business purposes entirely.

Finally, he paid £1,000 into a charity under a payroll deduction scheme.

What will his employment income be?

Solution:

Salary £30,000

Less:

ACCA subscription (£500)

Business travel expense (£1,000)

Gift aid donation (£1,000)

Employment income £27,500

18
The authorised mileage allowances

This arises when an employee uses their own car on employer’s business.

• The employee is entitled to receive this mileage allowance from their employer to
compensate for additional costs of running the vehicle due to business miles.

• The amount that can be received tax free is set by HMRC.

Anything in excess that is received from the employer will be taxable, and anything
below the allowance that is received from the employer will be deductible.

Motor car Up to 10,000 miles Above 10,000 miles

Authorised mileage allowance 45p 25p

Illustration:

Kerry has a salary of £20,000 per annum.

She uses her own car for employer’s business, the mileage allowance received from her
employer is 50p per mile.

She drove 12,000 business miles in the tax year.

• What is her taxable income?

Solution:

Mileage allowance received = (12 000*0.5) = £6,000

Authorised mileage allowance = (10 000*.45) + (2 000*0.25) = £5,000

Therefore, £1,000 excess will be taxable

19
Conclusion:

Salary £20,000

Unauthorised mileage allowance £1,000

Total employment income £21,000

Less personal allowance (£12,570)

Taxable income £8,430

Illustration:

Instead, Kerry receives 35p per business mile.

What is her taxable income?

Solution:

Mileage allowance received = (12 000*0.35) = £4,200

Authorised mileage allowance = (10 000*.45) + (2 000*0.25) = £5,000

Therefore, the deficit of £800 will reduce her employment income.

Salary £20,000

Mileage allowance deficit (£800)

Total employment income £19,200

Less personal allowance (£12,570)

Taxable income £6,630

Note Travelling between home and work does not count as business miles. This is
considered to be ordinary commuting.

Note Travelling to a temporary workplace will count as business miles. A temporary


workplace is one that you attend irregularly or for less than 24 months.

Note Travelling between work and client's offices will count as business miles.

20
The system for paying income tax for employed
individuals

How does PAYE work?

Most tax in respect of employment income is deducted under the PAYE system.

The objective of the PAYE system is to collect the correct amount of tax over the year.

An employee’s PAYE code is assigned to ensure that their allowances etc. are given
evenly over the year.

The PAYE system applies to most cash payments, other than reimbursed expenses, and
to certain non-cash payments.

• In addition, PAYE applies to round sum expense allowances and payments instead
of benefits.

It is the employer’s duty to deduct income tax and national insurance contributions from
the pay of his employees.

If he fails to do this, he must pay over the tax which he should have deducted and the
employer may be subject to penalties.

It is now possible for an employer to choose to include most taxable benefits within
their normal payroll, with the employee’s income tax liability being collected under
PAYE. This is referred to as the payrolling of benefits. Living accommodation benefits
and beneficial loans must still be reported on the P11D.

Any “Payrolled” benefits do not now have to be reported on the P11D. The P11D is a
form submitted by the employer that lists the benefits provided to an employee in a tax
year.

PAYE codes

An employee’s PAYE code indicates the amount of tax free pay he is entitled to.

The PAYE code will include the employee’s personal allowance and any allowable
deductions and be restricted be various taxable amounts.

• The codes are determined by HMRC.

The employer must act according to the code unless further notified by HMRC, even if
the employee appeals against the code.

21
PAYE Forms

Employers must complete forms P60, P11D and P45.

• A P45 is needed when an employee leaves

When an employee leaves, a form P45 (particulars of Employee Leaving) must be


prepared.

This form shows the employee's code and details of his income and tax paid to date
and is a two part form handed to the employee.

One of the parts is the employee's personal copy.

If the employee takes up a new employment, he must hand the other part of the form
P45 to the new employer.

• Form P11D records details of benefits

Following the end of each tax year, the employer must submit the P11D to HMRC by 6
July.

A copy of the form P11D must also be provided to the employee by 6 July.

The details shown on the P11D include the full cash equivalent of all benefits, so that
the employee may enter the details on his self-assessment tax return.

Specific reference numbers for the entries on the P11D are given to assist with the
preparation of the employee's self assessment tax return.

• Forms P60 is a year-end return

At the end of each tax year, the employer must provide each employee with a form
P60

This shows total taxable earnings for the year, tax deducted, code number, NI number
and the employer's name and address.

The P60 must be provided by 31 May following the year of assessment.

22
Use benefit

If an employer lends an asset (e.g. a computer) to an employee, and the employee uses
this asset privately, then the employee must pay income tax on this benefit.

Examples of assets lent:

1. Computers

2. TV sets

3. Boats

4. Furniture

5. Motorcycles

How much income tax needs to be paid on this benefit?

1. We need to find the value of the benefit in money terms.

2. On this money value, we will apply the income tax rate.

Monetary value of benefit

1. We must find the market value of the asset when it was first given to the employee.

2. Multiply this by 20%.

3. Multiply this value by the number of weeks/months the employee had access to the
asset.

For example, multiplying by 9/12 means that the employee had access to the asset
for 9 out of 12 months in the tax year.

4. Deduct any rent that the employee pays to the employer to use the asset.


23
Proforma:

Assessable benefit: 20% * market value * x/12 X

Less: Rent paid to employer to use asset  (X)

Use benefit  X

Illustration:

Manish’s employer purchased a dishwasher for Manish’s use on 1 April costing £900.
Manish paid his employer £150 to use the dishwasher for the tax year.

What use benefit will be assessable on Manish?

Assessable benefit: 20%*£900*12/12 =  180

Less: Rent paid to employer to use asset  (150)

Use benefit  30

24
Gift benefit

This benefit is linked with the use benefit explained above.

After an employer has given the employee an asset to use privately, the employer may
then decide to give this asset to the employee as a gift.

For example, an employer gave his employee a computer to use for private purposes
for 2 years and the employee was taxed on a benefit for the use of the asset in each of
those 2 years.

After 2 years, the employer then decided to give this computer to the employee as a gift
and the employee was then taxed on an additional benefit for the gift.

The employee will need to pay income tax on the monetary value of gift benefit.

How to calculate the monetary value of this gift?

The monetary value will be the higher of 2 figures:

Figure 1:

• Find the cost to the employer (the original market value of the asset).

• Deduct any use benefits that the employee has already paid income tax on.

For example, if the computer cost the employer £750 2 years ago when he purchased it,
and the use benefit that the employee paid income tax on for each year was £150, then
the gift benefit will be:

Original market value £750

Year 1 (£150)

Year 2 (£150)

Gift benefit £450 - Income tax will be paid on this figure.

Figure 2:

• Find the market value of the asset at the date of the gift to the employee.

For example, after 2 years, if the computer had a market value of £500, the benefit
would be:

Market value at date of gift £500

Therefore, the Gift benefit is £500 as this is the higher of the two figures.


25
Note that we must take the higher figure out of Figure 1 and Figure 2:

• Figure 1: £450

• Figure 2: £500

Therefore, the gift benefit in this case would be £500.

Illustration:

Manish’s employer purchased a dishwasher for Manish’s use on 06/04/2021, costing


£400.

On 06/04/2022 Manish was given the dishwasher by his employer. It’s market value
then being £200.

• What gift benefit will be assessable on Manish?

Solution:

Use benefit assessed in 21/22:

Use benefit assessed  20% * £400 * 12/12 = 80

Use benefit  80

Gift benefit assessed in 22/23:

Figure 1: £

Market value when first provided 400

Less: Use benefit already assessed (80)

Gift benefit 320

Figure 2: £

Market value on date of gift 200

Gift benefit 200

26
Higher of:

Figure 1 :  £320

Figure 2 :  £200

The gift benefit that will be assessed on Manish is £320

27
Living accommodation benefit

If an employer provides an employee with a home to live in, without the home being
necessary for the employee to do his or her job, the employee will have to pay income
tax on a living accommodation benefit.


If the home is necessary for the employee to do his or her job, then this benefit will not
arise.

When is a home necessary for an employee to do their job?

Examples of when a home is necessary for an employee to do their job properly


include:

1. A nanny needs to live in the same home that their child lives in to do their job.

Otherwise, they will not be able to do their job. In this case, providing
accommodation will be considered to be work related and a benefit will not arise.

2. Someone in an army needs to live at the army base, otherwise they will not be able
to do their job.

3. A hotel-worker being provided living accommodation at the hotel will help them
perform their duties better.

4. If there is a special threat to the employee’s security and he lives in the


accommodation as part of special security arrangements, this will ensure that he is
safe to do his job. For example, the Prime Minister or President.

How to calculate the living accommodation benefit?

• If the employer owns the home, then the home’s annual value will be used.

• If the employer is renting the home, then the amount of the benefit is the higher of
the rent paid by the employer and the annual value.

• The amount paid by the employee to the employer will be deducted to give the living
accommodation benefit.

28
Illustration - If the employer is renting the home

Ashok, a sales manager, lives in a flat that his employer has given him to live in.

The employer pays rent of £5,000 per annum for the flat, and Ashok pays the employer
£1,000 per year to use the flat. The annual value of the flat is £4,900.

• What is the living accommodation benefit that Ashok will have to pay income tax
on?

Solution:

Rent paid by employer (as this


5,000
is higher than annual value)
Rent paid by employee to
Less: (1,000)
employer 

Living accommodation benefit 4,000

Illustration - If the employer owns the home

Vandana, a marketing manager, lives in a flat owned by her employer.

The flat has an annual value of £4,000 and she pays the employer £500 per annum to
use the flat.

• What is the living accommodation benefit that she will have to pay income tax on?

Solution:

Annual value 4,000


Rent paid by employee to
Less: (500)
employer 
Living accommodation benefit 3,500

29
Additional benefit

There is an additional benefit that can arise if the employer owns the home and it cost
the employer more than £75,000 when they purchased it.

How to calculate the money value of the additional benefit?

Did the employer buy the home more than 6 years before he gave it to the employee to
use?

1. No

(Cost  - £75,000) * Official rate of interest (2% for 22/23) = Additional benefit

Note

The Cost will include the actual cost of the home plus any amount spent on extending/
enhancing the home before the start of the current tax year.

2. Yes

(Market value - £75,000) * Official rate of interest = Additional benefit

Note:

The market value of the home when it was first given to the employee to be used (the
purchase price is not used here).

Illustration - LESS than 6 years

Vandana, a marketing manager, lives in a flat owned by her employer.

She has occupied the flat for the last 5 years.

The employer bought this flat 5 years ago and paid £85,000.

4 years ago, he spent £10,000 to add a garage onto the flat.

What additional benefit that will she have to pay income tax on?

Solution:

(Cost  - £75,000) * Official rate of interest (2%) = Additional benefit

(£85,000 + £10,000) = £95,000 (cost plus enhancement expenditure)

(£95,000 - £75,000) *2% = £400

£400 is the additional benefit that Vandana will have to pay income tax on.

30
Illustration - MORE than 6 years

Vandana, a marketing manager, lives in a flat owned by her employer.

She has occupied this flat for the last 7 years, and moved in when the flat had a market
value of £100,000.

The employer bought this flat 15 years ago and paid £85,000.

What additional benefit that will she have to pay income tax on?

Solution:

(Market value - £75,000) * Official rate of interest = Additional benefit

(£100,000 - £75,000) *2% = £500

£500 is the additional benefit that Vandana will have to pay income tax on.

Conclusion:

The normal benefit and the additional benefit are added together to give the total living
accommodation benefit that the employee will pay income tax on.

• For example, Vandana will either pay income tax on:

£3,500 + £400 = £3,900 (Flat was not purchased more than 6 years ago)

or

£3,500 + £500 = £4,000 (Flat was purchased more than 6 years ago)

31
Motor cars - ACCA ARTICLE

If an employer gives an employee a motor car to use for business and private purposes,
then a benefit will arise which income tax must be paid on by the employee.

• If there is no private use of the car there is no taxable benefit

• The benefit is a percentage of the car’s list price.

The list price of the car will be given to you in the exam.

The list price includes the list price of any accessories fitted to the motor car.

If an employee pays a capital amount towards receiving the car from the employer, then
this list price will be reduced by this capital contribution.

The list price can be reduced by a maximum of £5,000 - even if the employee has
contributed more than this, it will only be reduced by £5,000.

For example the list price of a car is £15,000 and an employee has made a capital
contribution towards it of £7,000 - the list price of the car will only reduce to £10,000
(£15,000 - £5,000).

Then, this list price will be multiplied by a percentage to give the amount of benefit to be
taxed on.

• How to determine the percentage?

Electric cars

2% applies to electric powered motor cars with zero CO2 emissions.

For hybrid electric motor cars with CO2 emissions between 1-50 grams per
kilometre, the electric range of the motor car is relevant

Electric range

Electric range Percentage


130 miles or more 2%
70 - 129 miles 5%
40-69 miles 8%
30- 39 miles 12%
Less than 30 miles 14%

32
As emissions ride beyond 50 grams, the following percentages apply:

CO2 emissions Percentage Petrol


51 to 54 g/km 15%
55g 16%
Maximum percentage 37%

The base percentage of 16% rises in 1% for each 5 grams per kilometre above the
base level of 55 grams per kilometre, up to a maximum of 37%.

The percentage rates are increased by 4% for diesel cars unless the diesel car in
question is registered after 1 September 2017 and it meets the RDE2 standards.
You will be told in the exam if the car meets the RDE2 standards.

Reductions

So far, we have calculated the benefit as:

(List price-capital contribution) * % = Car benefit

This benefit can be reduced by 2 things:

1. If the motor car is unavailable for periods of at least 30 days of the tax year for
example if the car was not available to the individual for one month in the tax
year, then the benefit will be multiplied by 11/12 - because the car was only
available for 11 months in the tax year, and

2. Where the employee makes a contribution to the employer for the use of the
motor car, this is also known as making a contribution towards the running costs
of the car.

Note contributing towards the running costs of a car are different to the capital
contribution to reduce the list price of a car.

For example An employer gave an employee a motor car to use for private and
business purposes that had a list price of £17,000 - the employee made a capital
contribution of £8,000 towards the list price. The employee also contributed
£1,200 per annum towards the running costs of the car.

33
The taxable benefit would be:

List price less capital contribution: £17,000 - £5,000 (max) = £12,000

£12,000 * % = Benefit - £1,200 (running cost contribution). = Taxable benefit.

Pool cars

The use of a pool car does not result in a company car benefit.

A pool car is one provided for the use of any employee to use for business
purposes and is kept at the business place of work.

Illustration:

Arora plc provided the following employees with company motor cars:

• 1) Lina was provided with a new diesel powered company car on 6 August.

(the car does not meet the RDE2 standards)

The motor car has a list price of £13,500 and an official CO2 emission rate of
97 grams per kilometre.

Lina had an accident in October and was unable to use the car for 2 months,
however the car was always available to her to use.

• 2) Naina was provided with a hybrid electric company car throughout the tax
year. The car had a list price of £32,200 and an official CO2 emission rate of 24
grams per kilometre and an electric range of 90 miles.

• 3) Falak was provided with a new petrol powered company car throughout the
year.

The motor car has a list price of £22,600 and an official CO2 emission rate of
239 grams per kilometre.

Falak paid Arora plc £1,200 for the use of the motor car.

34
• 4) Jayna was provided with a new petrol powered company car throughout the
year.

The motor car had a list price of £16,000 and an official CO2 emission rate of
52 grams per kilometre.

• 5) Saaya was provided with a new diesel car throughout the year.

The motor car had a list price of £11,000 and an official CO2 emission rate of
55 grams per kilometre. The car meets the RDE2 standards.

Required:

Calculate the taxable benefit for Lina, Naina, Falak, Jayna and Saaya.

Solution:

1) Lina

was provided with a diesel powered company car. The CO2 emissions are 97g/km.

The CO2 emissions are rounded down to 95g/km so that it is divisible by five.

The base level percentage of 16% is increased in 1% steps for each complete 5
grams per kilometre above the base level, so the relevant percentage is 28% (16%
+ 4%(diesel car) + (95-55/5) 8%)

The motor car was only available for 8 months during the year, so the benefit is
£2,520 (13,500 × 28% × 8/12).

Note that even if Lina was unable to use the car herself for 2 months, the fact that
the car was available for her to use at all times means that the benefit will not be
reduced because of these 2 months.

2) Naina

With CO2 emissions between 1-50 grams, the electric range of the motor car is
relevant. This is between 70 - 129 miles, so the relevant percentage is 5%.

The motor car was available throughout the tax year so the benefit is £32,200 x 5%
= £1,610

35
3) Falak

The CO2 emissions are 239g/km.

The CO2 emissions are above the base level figure of 55 grams per kilometre.

The relevant percentage is 52% (16% + 36% (235 – 55 = 180/5)), but this is
restricted to the maximum of 37%.

The motor car was available throughout the year so the benefit is £7,162 (22,600 ×
37% = 8,362 - 1,200).

The contribution by Falak towards the use of the motor car reduces the benefit.

4) Jayna

The CO2 emissions are between 51 grams per kilometre and 54 grams per
kilometre so the relevant percentage is 15% as it is a petrol car.

The benefit is £16,000 × 15% = £2,400

5) Saaya

The CO2 emissions is 55g per kilometre and the relevant percentage is 16% (16% +
0% (diesel car)). (diesel car meeting the RDE2 standards does not have the 4%
supplement).

The benefit is £11,000 × 16% = £1,760

Fuel provided for private use

1. The car benefit also covers the running costs of the car BUT does not take
account of fuel provided for private use.

2. The amount of fuel benefit is computed on a base figure of £25,300 multiplied by


the percentage used for calculating the car benefit.

The fuel benefit is reduced proportionately where private use fuel is withdrawn
(and not reintroduced during the year) or the car is only given part way through
the tax year.

3. No reduction is made if the employee contributes towards the cost of petrol for
private use.

If he pays for all fuel used for private motoring the charge is cancelled.

36
Illustration:

Calculate the fuel benefit for Lina, Naina, Falak, Jayna and Saaya assuming also
that Falak pays Arora plc £600 during the year towards the cost of private fuel,
although the actual cost of this fuel was £1,000.

Solution:

Lina

£25,300 × 28% × 8/12 = £4,723



The fuel was not available for first 4 months

Naina

£25,300 × 5% = £1,265

Falak

£25,300 × 37% = £9,361

There is no reduction for the contribution made by Falak since the cost of private
fuel was not fully reimbursed.

Jayna

£25,300 × 15% = £3,795

Saaya

£25,300 × 16% = £4,048

37
Vans and heavier commercial vehicles

1. Where an employee uses an employer’s van for journeys between home and work
and other private use is insignificant there is no benefit.

2. Where private use is not insignificant the tax charge is £3,600 p.a.

3. An additional charge is made for fuel provided for unrestricted private use equal to
£688 p.a.

4. Both benefits are time apportioned if the van is unavailable to the employee for 30
days or more during any part of the tax year.

5. Vans producing zero CO2 emissions (zero emission vans) have a zero benefit charge.
There is also no fuel benefit for zero emission van.

Van benefit

If an employer gives an employee a van to use for private journeys, if the amount of
usage is not significant, then no benefit will arise that an employee needs to pay income
tax on.

However, if the private usage of the van is significant, then a benefit will arise that an
employee will pay income tax on

How to calculate the money value of the benefit?

The money value is a flat tax charge of £3,600 per annum.

Therefore, if the van was only provided for 9 months in the tax year, then the tax charge
would be £3,600 * 9/12 = £2,700

How to calculate the monetary value of the private fuel benefit?

If the employer also provides the employee with fuel for their private journeys, another
benefit will arise that the employee must pay income tax on.

The money value is a flat tax charge of £688 per annum.

Therefore, if the fuel for private journeys were only provided for 9 months in the tax year,
then the benefit for the private fuel provided would be £688 * 9/12 = £516

38
Illustration:

An employee is given use of his employer’s van on 01/06/2022 and uses it for a
significant number of private journeys.

The employer also pays for the fuel for the employee’s private journeys.

What taxable benefit would arise in 2022/23 because of the employer paying for private
fuel?

Solution:

£688 * 10/12 = £573

39
Beneficial Loan Benefit

This benefit arises when an employer gives an employee a loan at an interest rate that is
cheaper than the official interest rate (2% for 22/23). 


For example, a beneficial loan benefit would arise if an employer gave an employee a
£15,000 loan at 1% per annum.

This is because 1% is cheaper than the 2% official interest rate.

Carefully note – if the loan is £10,000 or less, no beneficial loan benefit will arise at all.

If the loan is above £10,000 – then a benefit on the full amount of the loan will arise.

Proforma

Money value of the loan benefit X

Less: Interest actually paid by employee (X)

Beneficial loan benefit X

How to calculate the monetary value of the loan benefit?

There are 2 ways to calculate the monetary value of the loan benefit.

The average method applies automatically but the taxpayer or HMRC can elect for the
strict method if it is more beneficial for them

eg the taxpayer will elect when the strict method produces a smaller benefit figure and
HMRC can elect if the strict method gives a MUCH higher benefit figure.

1. Average method

(Loan outstanding at start of year + Loan outstanding at end of year)/2 * Official interest
rate = £x

2. Strict method

You must find the interest payable for the actual loan outstanding at all times. (Much
easier to understand with an example)

40
Illustration - Average method

Vijay was given a loan of £35,000 by his employer on 06/04/2022. Interest is payable on
this loan at 1% per annum.

On 01/06/2022, Vijay repaid £5,000 of the loan and on 01/12/2022 Vijay repaid another
£15,000 of the loan.

The remaining £15,000 was still outstanding at 05/04/2023.

• What is the taxable benefit of the beneficial loan using the average method?

Solution:

Average method £

Money value of the loan


(£35,000 + £15,000)/2 * 2%  500
benefit

Less: Interest actually paid by employee (w1) (258)

Beneficial loan benefit 242

£242 – is the beneficial loan benefit that Vijay would have to pay income tax on IF the
average method was used.

(W1) - Interest paid by Vijay on loan

From £35,000 *
(2 months) Vijay paid 1% interest on
06/04/2022 – 1% * 2/12 =
£35,000 loan outstanding =
01/06/2022  £58.30

From (6 months) Vijay paid 1% interest on £30,000 *


01/06/2022 – £30,000 loan outstanding (£5,000 was 1% * 6/12 =
01/12/2022 repaid)= £150

From (4 months) Vijay paid 1% interest on £15,000 *


01/12/2022 – £15,000 loan outstanding (another £15,000 1% * 4/12 =
05/04/2023 was repaid)= £50

Total interest paid


£ 258.30
by Vijay

41
Illustration - Strict method

Vijay was given a loan of £35,000 by his employer on 06/04/2022. Interest is payable on
this loan at 1% per annum.

On 01/06/2022, Vijay repaid £5,000 of the loan and on 01/12/2022, Vijay repaid another
£15,000 of the loan.

The remaining £15,000 was still outstanding at 05/04/2023.

• What is the taxable benefit of the beneficial loan using the Strict method?

Solution:

Strict method £

Monetary value of the loan benefit Vijay should have paid (w1) 517
Vijay actually paid (figure
Less: (258)
from previous example)
Beneficial loan benefit 259

Vijay should have paid: £117 + £300 + £100 = £517

£35,000 *
From (2 months) Vijay should pay 2% interest on 2.25% *
06/04/2022 –
£35,000 loan outstanding =  2/12 =
01/06/2022 
£117

£30,000 *
From (6 months) Vijay should pay 2% interest on
2.25% *
01/06/2022 – £30,000 loan outstanding (£5,000 was repaid)
6/12 =
01/12/2022 =
£300

£15,000 *
From (4 months) Vijay should pay 2% interest on
2.25% *
01/12/2022 – £15,000 loan outstanding (another £15,000
4/12 =
05/04/2023 was repaid) =
£100

Conclusion

Vijay’s loan benefit with the average method is :  £242

Vijay’s loan benefit with the strict method is :  £259

42
Which one will he pay income tax on?

Vijay will pay tax on the loan benefit arising from the average method - £242 - as this is
the automatic method and the strict method is not so different that HMRC would elect
for it.

Summary

1. A beneficial loan is one made to an employee below the official rate of interest
(2%)

2. The benefit is the interest on the loan at the official rate, less any interest actually
paid by the employee.

3. There is no benefit if the loans do not exceed £10,000 in total at any time in the
tax year

4. The benefit is calculated using the average method or the strict method.

Average method

This uses the loan outstanding at the beginning and the end of the tax year.

If the loan is taken out or paid back during the tax year, that date is used instead of the
beginning or end the tax year. 


Strict method

This calculates benefit day by day on the balance actually outstanding.

Either the taxpayer or HMRC can elect to use the strict method. HMRC will elect if the
benefit is significantly higher under this method.

43
Other benefits

Generally, the basis for calculating the taxable value of any other benefit is the cost to
the employer.

There are various benefits which are exempt or partially exempt.

Although correctly identifying the tax treatment of such a benefit may result in only a
half mark or one mark, it is important that you correctly identify such benefits so that
time is not wasted with unnecessary calculations.

Finally, there is no exhaustive list of benefits, but you must keep in mind that if an
employee is using something for personal purposes and the employer is paying for it,
then the benefit that is likely to arise is the cost to the employer.

For example if an employer provides education for an employee's child that cost the
employer £600, then the benefit that will arise on the employee is the cost to the
employer of £600.

Payments for home working are exempt up to £6 per week.

An employee can claim a deduction for the additional cost of working from home, such
as expenditure on lighting and heating.

Employers can pay up to £6 per week (without the need for supporting evidence of the
costs incurred by the employee).

Payments above the £6 require evidence of the employee’s actual costs.

Relocation cost - only £8,000 of relocation cost is exempt.

Illustration:

Vary plc provides its employees with various benefits.

The benefits were provided:

1) Denzil was provided with two mobile telephones.

44
The telephones had each cost £250 when purchased by Vary plc in January.

The company paid for all of Denzil’s business and private telephone calls.

2) Emily had her health club membership fee of £710 paid for by Vary plc

3) Frederick spent 5 nights overseas on business for Vary plc.

The company paid him a daily allowance of £10 to cover the cost of personal expenses
such as telephone calls to his family.

4) Grace was paid £11,000 towards the cost of her removal expenses when she
permanently moved to take up her new employment with Vary plc, as she did not live
within a reasonable commuting distance.

The £11,000 covered both her removal expenses and the legal costs of acquiring a new
main residence.

5) Hillary’s three year old daughter was provided with a place at Vary plc’s workplace
nursery.

The total cost to the company of providing this nursery place was £10,800 (240 days at
£45 per day)

6) June had the use of Vary plc’s company gym which is only open to employees of the
company.

The cost to Vary plc of providing this benefit to June was £340.

7) Kristin was provided with free meals in Vary plc’s staff canteen.

The total cost of these meals to the company was £1,460.

The canteen is available to all of the company’s employees.

8) Larry regularly works from home two days per week, and was paid an allowance of
£288 (48 weeks at £6 per week) to cover the extra light and heat costs that were incurred
due to this home working.

9) Marge was given a watch valued at £750 as an award for her 20 years of employment
at Vary plc.

10) Nile had £440 of her medical costs paid for by Vary plc.

She had been away from work for two months due to an injury, and the recommended
medical treatment was to assist her return to work.

What taxable benefits will arise on the employees?


45
Solution:

1) Denzil

Providing one mobile phone to an employee does not result in a taxable benefit.

If additional mobile phones are provided to employees then 20% of the market value of
the phone will be the tax benefit assessed on the employee.

This is very similar to the use benefit

• The provision of one mobile telephone does not give rise to a taxable benefit.

• The taxable benefit for the use of the second telephone is £50 (250 x 20%).

2) Emily

• The benefit of the health club membership is the cost to Vary plc of £710.

3) Frederick

• Payments for private incidental expenses are exempt up to £10 per night when
spent outside the UK, so the allowance does not result in a taxable benefit.

• Note that the equivalent UK allowance is only £5 per night.

4) Grace

• Only £8,000 of the relocation costs is exempt, and so the taxable benefit is £3,000
(11,000 – 8,000).

5) Hillary

• The provision of a place in a workplace nursery does not give rise to a taxable
benefit.

6) June

• The use of a company gym does not give rise to a taxable benefit as the benefit is
available to all employees.

7) Kristin

• The provision of meals in a staff canteen does not give rise to a taxable benefit as
the benefit is available to all employees.

8) Larry

46
• Payments for home working are exempt up to £6 per week, so the allowance does
not result in a taxable benefit.

9) Marge

• A non-cash long-service award is not a taxable benefit if it is for a period of service


of at least 20 years, and the cost of the award does not exceed £50 per year of
service.

10) Nile

• The payment of medical costs of up to £500 does not result in a taxable benefit.

The exemption applies where medical treatment is provided to an employee to assist


them to return to work after a period of absence due to ill-health or injury.

47
RTI reporting

Employers must send income tax and NIC information to HMRC electronically every
time employees are paid, and make their PAYE payments electronically on the 22nd of
the month under the Real Time Information reporting system.

• There are penalties if submissions made during the tax year are made late, though
there is no penalty for the first month in a tax year that submissions are paid late.

• Thereafter a monthly late filing penalty of between £100 and £400 is charged
depending on the number of employees.

An additional penalty of 5% of the tax and NIC due may be charged where the
submission is more than 3 months late.

48
Syllabus: A1a. TX - UK Recap Income from self-
employment
The contents of the Paper TX - UK) study guide for income tax and national insurance, under
headings: 
- Income from self-employment

The basis of assessment for self-employment income

The basis of assessment for a sole trader is the taxable trade profits (self-employed
income) for a 12 month period of account ending in a tax year.

The tax year runs from 6 April to 5 April.



For example: The tax year 2022/23 runs from 6 April 2022 to 5 April 2023.

How are trading profits assessed to tax?

An individual can choose a date to end their accounts.

Whichever tax year that ending date falls into, the profits of that entire year will be assessed
in that tax year.

For example If an individual has a year ending of 30 November, then for the year ending 30
November 2022 - this ends in the tax year 22/23 (06/04/22 - 05/04/23), therefore the profits
for that year ending 30 November 2022 will be assessed to tax in the tax year 22/23.

This is very simple, it is known as the current year basis of assessment

49
Badges of trade

These are used to help differentiate whether a person is trading or whether they
are selling their capital assets.

• The need to differentiate between these 2 types of transactions arises because


an individual who is trading will be assessed to income tax and national
insurance contributions based on self employment.

• However, an individual who is making sales of their capital assets will be


assessed to capital gains tax.

Badges

1. Subject matter

– anything can be trading stock but some items are more likely to be so than others.

For example, the purchase and resale of a substantial number of toilet rolls is
considered trading.

2. Length of period of ownership

– normally, trading stock is held for a short period of time.

For example an item that is held for less than 12 months will be considered trading
stock, but an item that is held for more than 12 months is likely to be considered a
capital asset.

3. Frequency of similar operations

– the more often a deal takes place, the greater the assumption that it is a disposal of
trading stock.

For example if cars are bought and sold throughout the year, this will be considered to
be the trading of cars; however if there is a one time sale of one car, then that is likely to
be considered to be the sale of a capital asset.

4. Subsequent work

– change of character of an asset to make it more saleable is likely to be indicative of


trading.

For example, buying bulk marble for flooring and breaking it down into smaller saleable
units to use for individual floors, will be considered trading. Also, advertising and
making the item more marketable may be indicative of trading.

50
5. Circumstances

– sudden emergency, for example the urgent need of cash can negate the presumption
of trading.

For example selling a vintage car from a collection of vintage cars because of the
urgent need of cash will be considered to be the sale of capital assets.

This is because the motive is not to trade cars, it is to obtain cash quickly.

If the sale is to pay off a business loan then this would be considered to be trading and
subject to income tax on the profit.

6. Motive

– intention of making a profit is necessary for trading.

For example, selling cars at a loss just to get immediate cash will not be considered
trading, but waiting to sell cars at a price that will earn a profit will indicate trading.

51
Illustration:

On 1 May, Tony was made redundant from his job as a marketing executive. Tony
decided to purchase a house for £180,000.

• Tony lived in the house as his main residence and whilst living there he
refurbished it at a cost of £27,700. The renovations were completed on 10
August.

After completing the refurbishment, he was offered a new job and he immediately put
the house up for sale; and it was sold for £275,000 on 31 August.

• Tony had no other income or capital gains.

• Will this be considered to be trading income or a capital gain?

Solution:

Trading income
Badge Reason or Capital
Gain?

A house can be considered to be both Trading +


Subject matter
a trading and capital asset. Capital

Length of period
Less than one year Trading
of ownership

Frequency of
None before Capital
similar operations

Subsequent
Yes the house was refurbished Trading
work 
Circumstance New job resulted in sale Capital

He earned a profit, however this was


Motive not his motive, he sold due to the new Capital
job.

As there are 4 badges pointing towards this being a capital gain and only 3 pointing
towards this being trading income, the profit made on sale will be considered to be a
capital gain.

52
Adjusting the accounting profit

Allowable or Disallowable expenses?

Net Profit (PBIT) is adjusted to arrive at Trading profit

The main adjustments are to disallow for tax certain non-allowable expenses and to
exclude any non-trading income.

You must keep in mind that allowable expenses are usually expenses incurred directly
because of the business.

Personal expenses are not allowable.

The general rule is that expenditure not wholly and exclusively for the purpose of the
trade is not allowable

Net profit X

ADD BACK: Disallowable expenses X

LESS: Income assessable elsewhere (X)

LESS: Non-taxable income (X)

LESS: Capital allowances (X)

Tax adjusted trading profit X

Note:

When preparing this calculation, be careful to start with the NET profit per accounts.

53
Disallowable expenses

The following expenses are disallowed by a business.

Therefore, if they have been deducted to arrive at net profit, they must be added back.

For example, if the net profit is £10,000 and a disallowed expenses (£1,000) has been
deducted, then tax adjusted profit will be £10,000 + £1,000 = £11,000

• Capital expenditure including depreciation is not allowable

Note: 

Repair to an asset is revenue expenditure and is allowable

Improvement to an asset is capital expenditure and is not allowable

For example, purchasing a building to carry out trade in is a capital expense and
disallowed, but repairing this building is revenue expense and allowed.

• Reliefs

- such as qualifying loan interest payments are not allowable as they are dealt with
as a deduction from total income.

For example, paying interest on a loan used to purchase inventory is known as a


qualifying loan interest, this is already deducted under a different heading in the income
tax computation, therefore it cannot be deducted again.

• Entertaining and gifts

Entertaining is disallowed, unless entertaining employees

For example, spending money to entertain suppliers of stock is disallowed.

54
• Subscriptions and donations

National charity donations are not allowable

Charitable donations (made under Gift Aid) these are not allowable as tax relief is given
by extending the tax bands when calculating income tax.

Political donations - these are not allowable

• Fines and penalties

Disallowed unless the fine is paid on behalf of an employee and incurred whilst on
business

For example, if the employee receives a parking ticket whilst delivering goods to a
customer, this is generally allowable by HMRC

A fine for poor health and safety would not be allowable

• The owner’s salary, or drawings or interest on capital invested in the business are
disallowed

For example, if the owner of the business pays himself 10% interest on the capital that
he has invested of £200,000 - this £20,000 interest is disallowed and cannot be
deducted.

• Interest paid on overdue tax is not deductible and interest received on overpaid
tax is not taxable

For example, if interest is payable of £100 due to the late payment of tax  - this is not
allowed and cannot be deducted to arrive at taxable income.

• Irrecoverable Debts (Trade debt write offs & allowances)

These are allowable; the tax treatment follows the accounting treatment

However non trade write offs are not allowable and so the expense is added back.

For example if an item of trading inventory was sold on credit, but the actual cash
never came from the customer, then this is an irrecoverable debt which is an allowable
expense.

For example a loan was made to an employee and then the employee left without
paying it back and it was written off, then this is an irrecoverable debt, but it is
disallowed because it is not a trading item.

55
Allowable expenses

If they have not been deducted to arrive at tax adjusted profit, they must be.

For example if net profit is £10,000 and there is an allowed expense of £1,000 that has
not been deducted, then the tax adjusted profit will be £10,000 - £1,000 = £9,000

However, If they are correctly deducted, you will indicate these expenses with 0 in the
exam, because these items do not require adjustment.

For example if net profit is £10,000 and there is an allowed expense of £1,000 that has
been deducted, then the tax adjusted profit will be £10,000 and you will indicate the
allowed expense with 0.

1. Entertaining and gifts

Gifts to employees are allowable

Gifts to customers are only allowable if

• they cost less than £50 per person per year, and

• the gift is not food, drink, tobacco or vouchers exchangeable for goods and services

• the gift carries a conspicuous advertisement for the business.

For example as Christmas presents, a sole trader can give his customers pens with the
company logo printed on them as gifts, as long as they cost less than £50 per
customer.

2. Subscriptions and donations

Trade or professional association subscriptions are allowable

Charitable donation (Not made under Gift Aid)

• if it is wholly and exclusively for trading purposes (e.g promoting business’ name), and
it is to a local charity then it is allowable

For example if a donation was made to a local charity and in return, at the charity
fundraiser, the business was shown as a sponsor/organiser, then this would be an
allowable expense.

3. Legal and professional charges

Allowable if connected with the trade and are not related to capital items specifically
allowed by statute:

• costs of obtaining loan finance

• costs of renewing a short lease (50 years or less)

56
For example if a loan was taken out to purchase inventory for trading, the legal costs
associated with obtaining this loan and the interest cost of the loan are considered to be
allowable expenses.

4. Interest payable

Interest paid on borrowings for trading purposes is allowable on an accruals basis


therefore no adjustment is needed.

For example if a loan was taken out to purchase inventory for trading, the interest cost
of the loan are considered to be allowable expenses.

1. Premium paid for the grant of a lease

The premium itself is disallowed as is any amortisation of the premium.

The allowable amount per year is:

(51 – n)/50 ×    Premium

An alternative calculation that you may have seen before is:

Premium - (2% x (n-1) x Premium)/n

n = number of years of the lease

This is shown in Topic Premiums granted for short leases

5. The private expenditure of the business owner

If the owner uses a car in the business and 20% of his mileages private, then only 80%
of motor expenses are allowable.

However if the owner provides an employee with a car, and 20% of the mileage is for
private use by the employee, then the full amount of motor expenses is allowable. (The
employee is taxed on the private use under Employment Income).

Private expenses example



If the owner uses his own 4 storey premises as his place to conduct trading and incurs
expenses of £1,000, but he also lives on one of the floors, then 1/4 of the expenses
have been used for private purposes and £250 will be disallowed and £750 will be
allowed.

6. Any salary paid to the family of the owner of the business must not be excessive.
Only salary at the commercial rate for the work done is allowable.

57
For example John ran his business as sole trader and employed his wife Mary to work
for him as a sales executive. Other sales executives are paid £750 per week, but his
wife was paid £1,000. Only £750 will be an allowable expense, because the remaining
£250 will be considered to be excessive.

7. If an owner removes goods from the business for his own use he must add back the
item as a sale at market value, unless the owner accounts for the cost of the goods
in the business accounts then they need only add back the lost profit on the item.

For example if the owner of a business takes goods from his store that cost him £750
and would normally be sold for £1,000. He must either record the sale of £1,000 and
deduct cost of £750 - therefore increasing the profit of his store by £250.

Or, he can simply add £250 to the net profit in his accounts. This would only work if the
owner has not included the purchase figure in the accounts.

Read the question carefully, for example, if the owner had included the item in
purchases, then the sale of £1,000 would need to be added to profits in order for there
to be an overall profit of £250 left to tax.

8. Pre-trading expenditure

– allowable if it is expenditure incurred in the seven years before a business commences


to trade then it is treated as an expense incurred on the day the business starts trading
and follows the above rules.

This is shown in Topic Relief for pre-trading expenditure.

58
Illustration:

Sunny is self-employed running a retail shop. 

Sunny’s statement of profit or loss for the year is as follows:

£ £
Gross Profit 140,880

Expenses:

Depreciation 4,760
Light and heat (Note 1) 1,525

Motor expenses (Note 2) 4,720

Professional fees (Note 3) 2,300


Rent and rates (Note 1) 3,900

Repairs and renewals (Note 4) 5,660

Sundry expenses (Note 5) 2,990


Wages and salaries (Note 6) 84,825

110,680

Net profit 30,200

Notes

Note 1: Private accommodation

Sunny and his wife live in a flat that is situated above the clothing shop.

Of the expenditure included in the statement of profit or loss for light, heat, rent and
rates, 40% relates to the flat.

Note 2: Motor expenses

During the year, Sunny drove a total of 12,000 miles, of which 9,000 were for private
journeys.

59
Note 3: Professional fees

Professional fees are as follows:

Accountancy - including £250 in respect of a capital gains tax


700
computation.

Legal fees in connection with the purchase of the clothing shop 1,200

Debt collection 400

Total 2300

Note 4: Repairs and renewals

The figure of £5,660 for repairs and renewals includes £2,200 for decorating the clothing
shop, and £1,050 for decorating the private flat.

The building was in a usable state when it was purchased.

Note 5: Sundry expenses

The figure of £2,990 for sundry expenses, includes £640 for gifts to customers of food
hampers costing £40 each, £320 for gifts to customers of pens carrying an
advertisement for the clothing shop costing £1.60 each, £100 for a donation to a
national charity, and £40 for a donation to a local charity’s fete.

The fete’s programme carried a free advertisement for the clothing shop.

Note 6: Wages and salaries

The figure of £84,825 for wages and salaries includes the annual salary of £15,500 paid
to Sunny’s wife.

She works in the clothing shop as a sales assistant.

The other sales assistants doing the same job are paid an annual salary of £11,000.

Note 7: Goods for own use

During the year, Sunny took clothes out of the shop for his personal use without paying
or accounting for them.

The cost of these clothes was £460, and they had a selling price of £650.

60
Note 8: Plant and machinery

The capital allowances available for the year are £13,060.

(In the actual examination you may be required to prepare a capital allowances
computation and work out this figure. - see Topic Capital allowances)

Calculate Sunny’s tax adjusted trading profit for the year.

Solution:

Tax adjusted trading profit

£ £

Net profit as per accounts 30,200 

Add: Items debited in P&L – not allowed for tax purposes 

4,760
Depreciation
 

Light and heat (40% × £1,525)  610

Motor expenses (9,000/12,000 × £4,720) 3,540

Personal tax work  250

Legal fees re purchase of new shop (capital) 1,200

Rent and rates (40% × £3,900)  1,560

Decorating private flat  1,050

Gift of food hampers  640

Donation to national charity 100

Excessive remuneration to Sunny’s wife 

(£15,500 – £11,000)  4,500

Own consumption (goods were included in purchases) 650

18,860

Adjusted trading profIt  49,060

(13,060)
Less: Capital allowances (given - note 8) 
 

Tax adjusted trading profIt  36,000

61
Note: Personal taxation expense has been added to the net profit because personal
expenses are not allowable.

Note: The legal fees for the purchase amount for the shop has been added back
because this is not a trading revenue expense, this is a capital expense.

Note: The donation to the local charity is allowable but the donation to the national
charity is not allowable.

62
Small businesses are allowed to use the cash basis

if the business’ turnover does not exceed £150,000

This basis may result in a lower profit to be taxable, and therefore will reduce the
income tax payable.

• The business may continue to use the cash basis until the turnover exceeds
£300,000.

Calculation of profit

1. Total cash receipts of the business plus the sale of capital items are included.

For example If the business sells trading stock worth £25,000 and sells a capital asset
worth £50,000, both of these will be included to give a sales figure of £75,000.

2. Total cash expenses of the business including purchase of capital items used for
business are deducted.

For example If the business purchases trading stock worth £25,000 and purchases a
capital asset worth £50,000, both of these will be included to give a purchase figure of
£75,000.

3. There is an exception of the purchase of motor cars, these will not be included in the
calculation of profit under the cash basis. (Vans purchased will be allowable).

For example, a motor car was purchased for £40,000. Even though this is a capital
item, this will not be included in the purchases of the businesses under the cash basis.

Illustration:

Sales for the period were £61,000 of which £4,000 was still owed by business
customers at the end of the period.

Inventory on May 31, amounted to £1,800.

Purchases and expenses of the period (all allowable) amounted to £29,000 of which
£2,000 was still owed to suppliers at the end of the period.

• What is profit according to the normal basis and cash basis?

63
Solution:

Normal Basis £

Revenues 61,000

Cost of sales (29,000 – 1,800) (27,200)

33,800

Cash Basis £

Receipts (61,000 – 4,000) 57,000

Payments (29,000 – 2,000) 27,000

30,000

Understanding operation

The above calculation of profit only includes cash item, therefore things such as:
receivables, payables, opening and closing inventory will be ignored.

The business will only pay income tax on its cash profits.

Simple proforma to use:

Cash sales received in the tax year x

Cash sales of plant and machinery in the tax year (not car) x

Less:

Cash purchase of inventories in the tax year (x)

Cash allowable expenses in the tax year   (x)

Cash purchases of plant and machinery in the tax year (x)

Motor expenses (Authorised mileage allowance) (x)

Tax adjusted trading profit

64
Motor expenses

The purchase capital expense of motor cars and the running expenses will not be
allowed.

Instead, under this scheme, to replace these expenses, an authorised mileage


allowance will be given.

This is:

1. For the first 10,000 business miles – 45p/mile

2. For any business miles after 10,000 – 25p/mile

For example, the motor car that was purchased for £40,000, drove 15,000 business
miles.

The allowable deduction will be (10,000 miles * 0.45) + (5,000 miles *0.25) = £5,750

Illustration:

Barry commenced a new self employment business on 06/04/2022.

The following information relates to the year ended 05/04/2023:

1. Sales receipts of £81,000 with a further £750 owing on 05/04/2023.

2. Purchase of inventories of £20,000.

3. Closing inventories of £680 at 05/04/2023.

4. On 10/06/2022 Barry purchased a car with C02 emissions of 185g at a cost of


£20,000. Barry drove 12,000 business miles and 4,000 private miles. The total motor
expenses amounted in £3,600.

5. Ventilation system purchase cost £2,500.

6. Other expenses (allowable) cost £17,600 with a further £400 owed as a payable at
05/04/2023.

Calculate the tax adjusted trading profit according to the accruals basis and the
cash basis.

65
Solution:

A) Accruals basis

Sales  £81,750 W1

COS  (£19,320) W2

Gross profit £62,430

Capital allowances (£3,400) W5

Motor expenses (£2,700) W3

Other allowable expenses (£18,000) W4

Tax adjusted trading profit £38,330

W1

Cash and credit sales calculation:

• £81,000 + £750 = £81,750

W2

Cost of sales calculation:

• Opening inventory + purchases – closing inventory.

0 + £20,000 - £680 = £19,320

W3

Motor running expense calculation:

• The motor running expense must be adjusted for business use only, as private
expenses are not allowable.

£3,600 * 12,000/16,000 = £2,700

W4

Other allowable expenses

• £17,600 + £400 = £18,000

W5

Capital allowance calculation:

• AIA £2,500

• WDA £20,000 * 6% = £1,200 * 12,000/16,000 = £900 (car: CO2 >50g/km: special


rate pool)

66
B) The Cash Basis

Cash sales £81,000

Cash purchases (£20,000)

Gross profit £61,000

Cash expenses (£17,600)

Cash capital expenditure (£2,500)

AMAP (£5,000) W1

Tax adjusted trading profit £35,900

W1

AMAP

• 10,000 miles * 0.45 = £4,500

• 2,000 miles * 0.25 = £500

Barry should choose to elect into the cash basis scheme as this results in a
lower taxable profit for him

67
When does trading commence?

Trading commences on the first day on which a trader makes a sale.

However, the trader would have incurred expenditure before this date, for example,
advertising expenditure and/or rent paid in advance.

• This expenditure incurred before trading has commenced is known as “pre-trading


expenditure”.

Pre-trading expenditure will get tax relief by being treated as though it was incurred on
the first day that a sale is made, if the following conditions are satisfied.

Conditions for pre-trading expenditure to be allowable

• 1) It is incurred within 7 years of the commencement of the trade.

• 2) It is an allowable expense.

• For example, if goods were purchased for sale for the business 4 years before the
business had its first sale; this purchase price will be deducted from the first profits
also.

Illustration:

Manny made his first sale in his packaging business on 04/05/2022.

Before this he incurred the material expenses of £3,000 on 31/12/2021.

• Will this expenditure be deducted from the sales revenue to arrive at tax adjusted
trading profit?

Solution:

Yes, this expenditure will be deducted from his sales revenue to arrive at the tax
adjusted trading profit.

It will be treated as though the expenditure was incurred on 04/05/2022.

This is because money spent on materials used in the business are an allowable
expense and it was incurred within 5 months of the trade starting.

68
Assessable profits on commencement

Step 1: The first tax year (TY1)

The first tax year is the year during which the trade commences.

For example:

if a trade commences on 1 June 2022 the first tax year is 2022/23 (06/04/2022 –
05/04/2023)

The basis period for the first tax year runs from the date the trade starts to the next 5 April.

So a trader commencing a business on 1 June 2022 will be taxed on profits arising from 1
June 2022 to 5 April 2023 in 2022/23.

Step 2: The second tax year (TY2)

Does the accounting date fall in Tax Y2?

• YES

How long is this accounting period?

o < 12 months long                                                   

Calculate profits for the first 12 months of trading

o ≥ 12 months long    

Calculate profits for the 12 months to the accounting date ending in Y2

• NO

o Assess the actual profits in tax year 2

 i.e 6 April to 5 April

For example if trade commenced on 01/01/2022 and accounts were prepared to


31/07/2023, the first tax year is 21/22 and the second tax year is 22/23 - but there is no
accounting date ending in the second tax year.

Therefore, profits from 06/04/2022-05/04/2023 will be taxed in the second tax year (22/23).

So, let's go back to our original example:

Assuming that the accounting date is 31 May 2023

69
The second tax year (TY2) - 2023/24

• Does the accounting date (31 May 2023) fall in Tax Y2 (2023/24)?

YES

• How long is this accounting period?

1 June 2022 - 31 May 2023

≥ 12 months long    

Therefore, calculate profits for the 12 months to the accounting date ending in TY2:

1 June 2022 - 31 May 2023

Step 3: The third tax year (TY3)



• Use a current 12 months accounting period

So, let's go back to our example (assuming they continue to make accounts up to 31
May each year):

• The third tax year (TY3) - 2024/25:

1 June 2023 - 31 May 2024

Step 4: Comment on Overlap profits

If the same profits have been taxed two times, at the end of the question, mention how
much profit has been taxed 2 times.

For example if a business commenced trading on 01/01/2022, prepared accounts to


31/12/2022 and made a trading profit of £12,000 during the 12 months.

The first tax year would be 21/22 (01/01/2022-05/04/2022) and 3 months of profit would be
of £3,000 (3/12*£12,000) would be taxed.

The second tax year would be 22/23 and as the accounting period is 12 months long, the
full £12,000 would be taxed.

Therefore, the £3,000 of 21/22 has been taxed 2 times, this is the overlap profit.

You must mention this in your answer.

70
Illustration 1: ≥ 12 months long

Peter starts to trade on 1 January 2022 making up accounts to 31 March 2023.

He made profit of £15,000  for the year ended 31/3/2023



He made profit of £24,000  for the year ended 31/3/2024

1. The first tax year (TY1 =  2021/22)

1/1/2022 - 5/4/2022

Profit = £15,000 x 3/15 = £3,000

2. The second tax year TY2 = 2022/23 (06/04/2022 – 05/04/2023)

Does the accounting date (31/3/2023) end in TY2?

YES

How long is this accounting period?

> 12 months long    1/1/2022 - 31/3/2023

Calculate profits for the 12 months to the accounting date ending in Y2

1/4/2022 - 31/3/2023

Profit = £15,000 x 12/15 = £12,000

3. The third tax year (TY3 = 2023/24)

Use the current 12 months accounting period



1/4/2023 - 31/3/2024

Profit = £24,000

4. Comment on Overlap profits

Trading profits £15,000 + £24,000 = £39,000

Less profits assessed (£3,000 + £12,000 + £24,000) = (£39,000)

Overlap profits £Nil

71
Illustration 2: < 12 months long

Draco commences business on 01/01/2022. He prepares his accounts to 31/07

• Below are the trading profits for the accounting periods:

£10,500 for the 7 months to 31/07/2022

£33,600 for the year ended 31/07/2023

• What trading profits will be assessed for 21/22, 22/23 and 23/24?

Solution:

1. The first tax year (TY1 =  2021/22)

1/1/2022 - 5/4/2022

Profits 3/7 * £10,500 = £4,500

2. The second tax year TY2 = 2022/23 (06/04/2022 – 05/04/2023)

Does the accounting date (31/7/2022) end in TY2?

YES

How long is this accounting period? (1/1/2022 - 31/7/2022)

< 12 months long                                           

Calculate profits for the first 12 months of trading - 01/01/2022 - 31/12/2022

Profits £10,500 + (5/12 * £33,600) = £24,500

3. The third tax year (TY3 = 2023/24)

Use the current 12 months accounting period:



01/08/2022 - 31/07/2023

Profits £33,600

4. Overlap profits

Trading profits £10,500 + £33,600 = £44,100

Less profits assessed (£4,500 + £24,500 + £33,600) = (£62,600)

Overlap profits £18,500

72
Illustration 3 - when the accounting date doesn't fall in Tax Y2

Lachmi commenced self employment on 01/01/2021. 

She had a profit of £10,000 for the period ending 30/04/2022.

Calculate the tax adjusted trading profit for the period ended 30/04/2022.

Solution

1 The first tax year (TY1 =  2020/21)



Assessed period 01/01/21 - 05/04/21 =  3/16 * £10,000 = £1,875

2 The second tax year TY2 = 2021/22 (06/04/2021 – 05/04/2022)


Does the accounting date (30/4/2022) end in TY2? 



NO


Therefore, profit will be calculated as follows:

Assessed period 06/04/21 - 05/04/22 = £10,000  * 12/16 = £7,500

3 The third tax year (TY3 = 2022/23)


Does the accounting date (30/4/2022) end in TY3? 



YES

How long is this accounting period? (1/1/2021 - 30/4/2022)

> 12 months long therefore tax the last 12 months of the long period

01/05/21 - 30/04/22 = £10,000  * 12/16 = £7,500

4 Overlap profits

Overlap profits created: £1,875 + £7,500 + £7,500 - £10,000 = £6,875


73
Assessable profits on cessation

Cessation of a business

The penultimate tax year will be taxed as normal.

The basis of assessment for the final tax year is as follows:

1. Actual trading profits from the end of the previous accounting period to the date
of cessation.

2. Deduct any overlap profits to find the trading profit assessment of the final year.

Illustration:

Barry has been trading for many years and making up his accounts to 31/01. He ceases
to trade on 31/05/2022, with trading profits as follows:

Year ended 31/01/2022 £55,000

Year ended 31/05/2022 £6,000

He has overlap profits of  £2,000

What is his assessment in the final tax year of trading?

74
Solution:

Final tax year

Tax year 22/23

Accounting period  01/02/2022 - 31/05/2022

Profits  £6,000

Less overlap profits  (£2,000)

Trading profit assessable  £4,000

Illustration:

In the final period of trading, 01/02/2022 - 31/05/2022, Barry had a tax written down
value on his main pool of £1,000 and sold the main pool assets for £600.

What would his profit assessment be in 22/23?

Solution:

Trading profit £6,000



Less:

Overlap profits (£2,000)



Balancing allowance (£400)

Trading profit assessable £3,600

75
Capital allowances

Plant and machinery(P&M) for capital allowances purposes

Capital Allowances (Tax depreciation) are deducted from Operating profits

• CA are given for P&M used in the business only

• CA are given for a period of account eg for a year ended 31/12/22, and are
deducted in the adjustment of profits calculation to reach the Trading Profits
figure

Plant is defined as assets that perform an active function in the business

e.g.  office furniture and equipment including moveable office partitioning.

Machinery will include motor vehicles and computers, including building alterations


necessary for the installation of plant and machinery.

Rates of allowance %

Main pool assets 18

Special Rate Pool assets 6

Capital allowances are now also available on integral features of a building including lifts
and escalators, electrical systems, heating and air cooling system.

Main pool

1. Computers, equipment, shelving, vans and lorries

2. Movable office partitioning

3. Alterations to building incidental to the installation of plant and machinery

4. Tables and chairs

5. Fire regulation expenditure

Special Rate Pool

The following asset acquisitions should be allocated to the special rate pool:

1. Integral features of a building

– these include all major systems in a building.

76
For example, electrical, thermal, cooling systems.

2. Long life assets

These are assets, when new, with an expected economic working life of 25 years or
more when total expenditure based on a 12-month accounting period exceeds
£100,000

Writing down allowances

W.D.A.’s are given on main pool assets and special rate pool assets.

For main pool assets, the W.D.A. is 18% for a 12 month period

For example Assets in the main pool had a brought forward value of £100,000 at
01/01/2022

The writing down allowance on these assets will be £18,000 (£100,000*18%) in the year
ending 31/12/2022.

 

Note if the above period was for 6 months, then the WDA for the main pool would be
£9,000 (£100,000*18%*6/12) in the period ending 31/12/2022.

For special rate pool assets, the W.D.A. is 6% for a 12 month period.

For example Assets in the special rate pool had a brought forward value of £100,000 at
06/04/2022

The writing down allowance on these assets will be £6,000 (£100,000*6%) in the year
ending 05/04/2023.

Note if the above period was for 6 months, then the WDA would be £3,000
(£100,000*6%*6/12) in the period ending 05/04/2023.

First year allowances

These are given for new motor cars with zero CO2 emissions.

This is a 100% allowance on the cost of the car and it is given in the period of
acquisition.

The F.Y.A. is not time apportioned for a period of less than 12 months.

For example, a car was purchased on 01/05/2022 for £100,000.

It had zero emissions.

The first year allowance for this car will be £100,000 ( £100,000*100%).

Note if the above period was for 6 months, then the FYA would still be £100,000 - it is
not reduced for a period of less than 12 months.

77
Annual investment allowance

From 1 January 2019, the annual investment allowance is £1,000,000.

This is given to an individual for a 12 month period and is time apportioned if the period
is below 12 months.

Ideally, this A.I.A should be allocated to special rate pool assets purchased first
because the allowances on these assets are only 6% per year, therefore tax relief on
these assets is received over a longer period.

Once allocated to special rate pool assets purchased in the tax year, then if any of the
allowance is remaining, it can be allocated to main pool assets purchased in the year.

The A.I.A cannot be given to motor cars purchased in the tax year.

(Note: prior to 1 January 2019, the AIA was £200,000. You will not be tested on this in
the ATX exam)

For example a business purchased equipment worth £1,300,000 in their year ending
31/03/2023.

The annual investment allowance is £1,000,000 (maximum available).

For the remaining £300,000 (£1,300,000- £1,000,000), a writing down allowance will be
available.

As equipment is a main pool asset, the writing down allowance will be £54,000
(£300,000*18%).

The total capital allowances available will be AIA + WDA = £1,054,000 (£1,000,000
+  £54,000)

Note if the above purchase was made in a 6 months period, then the AIA would be
(£1,000,000*6/12) = £500,000 + WDA ((£1,300,000 - 500,000)*18%*6/12) = £72,000.
This would total to £572,000 of capital allowances for the 6 month period.

78
Illustration:

Buzzy Ltd. in the year ended 05/04/2023 made the following transactions..

Date Item Price

01/05/2022 Ventilation system and lift for his freehold office £1,278,000
building
26/06/2022 Machinery purchased and alterations made to £29,300
office building to install the machinery

08/08/2022 Computers £22,900

The tax written down value on the main pool was £87,800 on 06/04/2022.

What are Buzzy Ltd. capital allowances?

Solution:

Main Special Capital


Particular AIA
pool rate pool allowances

Tax written down


value brought £87,800
forward
Additions:

Ventilation
£1,278,000
system and lift

AIA  (£1,000,000) £278,000 £1,000,000


Machinery
purchased and £29,300
alterations 
Computers £22,900
Total £140,000 £278,000

WDA 18%/6% (£25,200) (£16,680) £41,880


Tax written down
value carried £114,800 £261,320 £1,041,880
foward

79
Notice how the AIA was first allocated to special rate pool assets.

The capital allowances are £1,041,880.

This is the total of:



WDA 18% on the main pool of £25,200

+

WDA 6% on the special pool of £16,680

+

AIA of £1,000,000

= £1,041,880

Illustration:

Shivani commenced trading on 1 July 2022 and prepared accounts to 31 December


2022 thereafter.

Shivani made the following acquisitions of main pool assets:

Accounting Period to 31 December 2022 £

1 July 2022 Plant 70,000

Computer
20 October 2022 580,000
equipment

Accounting Year ended 31 December 2023

19 October 2023 Machinery 300,000

What capital allowances will be allowed for both periods?


80
Solution:

Capital Allowance Computations

Main
6 month period to 31 December 2022 Allowances
Pool

Additions (AIA):

1 July 2022 Plant 70,000

20 October 2022 Computers 580,000

650,000

AIA (max 6/12 x 1,000,000) (500,000) 150,000 500,000

WDA (max 6/12 x 18% x 150,000) (13,500) 13,500

Total Allowances 513,500

Tax Written Down Value (TWDV) c/f 136,500

Year Ended 31 December 2023

TWDV b/f 136,500

Additions (AIA)

19 October 2023 300,000

AIA (300,000) 300,000

WDA (18%) (24,570) 24,570

Total Allowances 324,570

TWDV c/f 111,930

81
Assets with private use

• A company

Companies do not have assets used privately. 

This is because all of the people who work in the company are considered to be
employees of the company. 

Therefore, the capital allowances given are not reduced by the % of private usage by an
employee of a company.

• A Sole trader

If an asset is used privately by the owner of the business, the capital allowance given
must be reduced by the % of private usage. 

If an asset is used privately by an employee of the business, the capital allowance given
is not reduced by the % of private usage.

Illustration (a company)

Cow Ltd. is a trading company.

The company bought computer for £3,000 which is used by the sales manager  30%
privately.

Cow Ltd. has already used the AIA in this year.

Calculate the capital allowances.

Solution:

WDA = £3,000 x 18% = £540

Note: The private use of the computer by the employee is not relevant for capital allowance
purposes. 

No adjustment is ever made to a company's capital allowances to reflect the private use of
an asset.

82
Illustration (a sole trader)

Mia has been in a business as a sole trader.

She bought computer for £3,000 which she uses 70% in her business and 30% privately.

She has already used the AIA in this tax year.

Calculate the capital allowances.

Solution:

WDA = £3,000 x 18% = £540

Capital Allowances (business use only) £540 x 70% = £378

Compute capital allowances for motor cars

The F.Y.A is given to new motor cars purchased that have zero CO2 emissions.

For cars with a CO2 emission less than or equal to 50g, an 18% W.D.A. is given,
therefore these are considered to be main pool assets.

For cars with a CO2 emission of more than 50g, an 6% W.D.A. is given, therefore these
are considered to be special rate pool assets.

Illustration (a Company)

Cow Ltd.:

06/04/2022 Tax written down value on main pool of £16,800

Purchase of car for £10,600. The car had CO2 emissions of 46g/
25/06/2022
km.
Purchase of car for £18,000. The car had CO2 emissions of
16/02/2023
142g/km.

14/03/2023 Purchase of car for £22,000. The car had zero emissions.

What are Cow Ltd. capital allowances?

83
Solution:

Special
Main Capital
Particulars F.Y.A. rate
Pool allowances
pool

Tax written down


£16,800
value brought forward

Additions:

Zero CO2

(£22,000) £22,000
Car £22,000

Car 46g/km £10,600 £10,600

Car 142g/km £18,000 £18,000

Total (£22,000) £27,400 £18,000

WDA (18%/6%) (£4,932) (£1,080) £6,012

Tax written down


22,468 16,920
value carried forward

Total capital allowances for the year £28,012 (£22,000 + £6,012)

Illustration (ANNA - a Soletrader )

06/04/2022 Tax written down value on main pool of £16,800

Purchase of car for £10,600.

The car had CO2 emissions of 46g/km. 



25/06/2022
This car is 60% privately used by Anna’s husband who is an
employee of the business.
Purchase of car for £18,000.

16/02/2023 The car had CO2 emissions of 142g/km.

This car is 30% used privately by Anna.


Purchase of car for £22,000.

14/03/2023 The car had zero CO2 emissions.

This car is 25% privately used by Anna’s assistant.

What are Anna’s capital allowances?

Solution:

84
Capital
Main Special
Particulars F.Y.A. allowan
Pool rate pool
ces

Tax written down


£16,800
value brought forward

Additions:

Zero CO2

(£22,000) £22,000
Car £22,000

Car 46g/km £10,600 £10,600

Car 142g/km £18,000 £18,000

Total (£22,000) 27,400 £18,000


(£1,080) *
70%
business £5,688
WDA (18%/6%) (£4,932)
use = (W1)
Capital
allowance

Tax written down


22,468 £16,920
value carried forward

W1:

The capital allowance is reduced by % of private usage

£4,932 + (£1,080 * 70%) = £5,688

W2:

The tax written down value carried forward is calculated using the entire W.D.A.

£18,000 - £1,080 = £16,920

Total capital allowances for the year £27,688 (£22,000 + £5,688)

85
Disposal of the assets

Use LOWER OF

1. Proceeds

2. Original cost

When an item of plant or machinery is sold - the lower of the sale proceeds received or the
original cost of the asset is deducted from the written down value of the relevant pool.

For example, if the written down value is 100 and sale proceeds received are 120 but the
original cost of the asset is 110, then 110 will be deducted from the pool to give a balancing
charge of 10. The difference between proceeds and original cost will be treated as a capital
gain.

Compute balancing allowances and balancing charges


In the final year of trading, the A.I.A., W.D.A., F.Y.A. are not given.

Instead, balancing allowances and balancing charges are computed on each pool.

Balancing adjustments on the pools can only occur on cessation of trade.

A balancing allowance will be deducted from trading profit to find tax adjusted trading profit
and a balancing charge will be added to trading profit to find tax adjusted trading profit.

Illustration:

Karen Ltd. prepares accounts to 05/04.

The company ceased to trade on 05/04/2023 on which all of its plant and machinery was
sold for £8,000.

The written down value on its main pool at 06/04/2022 was £11,000.

The company purchased machinery for £4,000 during the year.

Solution:

Particulars Main pool Capital allowances


TWDV b/f £11,000

Additions £4,000

Total £15,000

Disposals (£8,000)

Balancing allowance £7,000 £7,000

Karen Ltd.’s balancing allowance in her final year of trading is £7,000.

86
Structural and Buildings Allowance

The SBA is is a new type of capital allowance available when a building (or a structure) has
been constructed / purchased for use in the trade. For example, offices, retail and
wholesale premises, factories and warehouses all qualify for the SBA.

This allowance is also available if an unused building/structure has been renovated for use
in the trade.

The rate of the allowance is 3% per annum and is given for a period of 33 years and 4
months.

To note about the SBA:

- The value of land does not qualify for the SBA

- Expenditure which qualifies as plant and machinery (and therefore will get the AIA) cannot
also qualify for the SBA and vice versa.

- The SBA can only be claimed from when the building / structure is brought into use in the
trade. This means that the SBA will be time apportioned for the period when it is first
brought into use, this is unlike capital allowances for plant and machinery which are given
the full allowance in the period of purchase.

- A separate SBA is given for each building / structure

- When the building / structure is sold, this will not result in a balancing allowance or
balancing charge. For the seller, he allowances already given at the date of sale will be
added to the sale proceeds when calculating the chargeable gain / capital loss for capital
gains tax. For the buyer, the 3% p.a. will continue to be given for the period remaining out
of the 33 years and 4 months.

Illustration

Anaya Ltd prepares accounts to 31/3/2023.

On 1/7/2022 a newly constructed factory was purchased from a builder for £500,000
(including land cost of £130,000).

The factory was brought into use on 1/9/2022.

What is the SBA available on this factory?

Solution

Purchase price £500,000

Less land cost (£130,000)

Qualifying expenditure for SBA £370,000

SBA £370,000 x 3% x 7/12 = £6,475

The allowance will be given from September 2022 (date it was brought into use).

Illustration

87
Anaya Ltd sold the factory above on 31/3/2023 for £600,000.

What will the SBA be for the year ended 31/3/2024 for the buyer?

What will the capital gain be on the sale?

Solution

The SBA will be given normally for the year ended 31/3/2024 to the buyer:

£370,000 x 3% = £11,100

The capital gain on the sale for Anaya Ltd

Sale proceeds £600,000 + SBA £6,475

= £606,475

Less cost (£500,000)

Capital gain £106,475

Recognise the treatment of short life assets

Short life assets are main pool assets that have an expected life of 8 years or less.

A de-pooling election can be made so that the asset gets its own W.D.A.’s and on sale of
the asset, a balancing allowance or balancing charge can arise.

The benefit of this election is that a balancing adjustment will arise within 8 years, which
would not have arisen, if this de-pooling did not take place.


If the asset is not sold within the 8 years of acquiring the asset, then the written down value
is added back to the main pool.

This happens on the 8th anniversary of the end of the accounting period in which the asset
was acquired.

88
Illustration:

Aadi prepares accounts to 05/04 each year.

At 06/04/2022 the WDV of the main pool was £14,000.

On 01/07/2022 Aadi purchased machinery for £1,020,000.

On 01/09/2022 Aadi purchased a printer for £8,000 and made a short life asset election.

On 01/07/2023, the printer was sold for £4,000.

Calculate the capital allowances for the two years ending 05/04/2024

Solution:

Year ended
05/04/2023
Particulars AIA Main Short Capital
pool life allowances
asset

Tax written down £14,000


value brought
forward

Additions:

Machinery £1,020,000

AIA  (£1,000,000) £20,000

Printer £8,000

Total £34,000 £8,000 £1,000,000

WDA 18% (£6,120) (£1,440) £7,560

Tax written down £27,880 £6,560 £1,007,560


value carried
forward

89
Year ended 05/04/2024

Particulars Main Short Capital


pool life allowances
asset

Tax written down value brought £27,880 £6,560


forward

Disposal proceeds (£4,000)

Balancing allowance £2,560 £2,560

WDA 18% (£5,018) £5,018

Tax written down value carried £22,862 Nil £7,578


forward 

Assets on hire purchase or lease

Any asset (including a car) bought on hire purchase (HP) is treated as if purchased
outright for the cash price. Therefore:

The buyer normally obtains capital allowances on the cash price when the agreement
begins

He may write off the finance charge as a trade expense over the term of the HP contract

Long-term leases (those with a term of five or more years) are treated in the same way
as HP.


90
Relief for trading losses - for individuals

Trading losses can be:

1. Carried forward against the Trading income of the same trade of future years

2. Relieved against Current year total income plus capital gains

3. Carried back against 12 months of total income plus capital gains

1) Trading losses carried forward against the Trading income of future years

Illustration:

Peter had a Trading loss of £50,000

Next year, Peter made the following income:

Trading income £20,000

Property income £10,000

Interest income (gross) £5,000

How can the trading loss be carried forward?

Trading loss of (£50,000) will be relieved against the trading income generated next year.

Trading income £20,000

Less c/f trading loss (£20,000)

Trading income Nil

Property income £10,000

Interest income (gross) £5,000

Total income £15,000

Loss memo:

Trading loss (£50,000)

91
c/f loss relief £20,000

Loss to be carried forward in the future (£30,000) 

2) Trading losses relieved against Current year total income plus capital gains

Trading losses can be relieved against the total income of the current year and the total
income of the previous 12 months.

If the total income of the year has been used, then the chargeable gains of that year can
also be used to relieve the loss remaining.

Total income consists of:

1. Trading income

2. Property income

3. Interest income

4. Employment income

Other Income

is any income other than Trading income.

For example:

1. Property income

2. Interest income

3. Employment income

Other Income - maximum limit

For using the loss against other income (not including trading income), there is a
maximum limit which applies, this is the greater of £50,000 or 25% of total income
(including trading income).

92
For example: 

if you have a trading loss of £300,000 and employment income (other income) is
£250,000, the amount that can be relieved is the greater of £50,000 or (25% x £250,000)
= £62,500.

Therefore, the amount of loss that would be relieved against employment income that
year is £62,500. 

Note, this applies to the carryback claim against total income also. 

However, the previous year's trading profit can be entirely used, it only applies to other
income. 

For example:

This year, you have a trading loss of £300,000 and no other income.

Last year, there was a trading profit of £10,000 and other income for the year was
 £250,000, the amount that can be relieved is the greater of £50,000 or (25%* £260,000)
=  £65,000 PLUS £10,000 from trading profit, therefore £75,000 of loss can be relieved.

Illustration

This year, John had a trading loss of (£100,000).

Last year, He had a trading income of £2,000 and other income of £300,000.

How much of his trading loss can he relieve using the carry back total income claim?

Solution


Trading loss (£100,000)

Trading profit £2,000

Other income Restriction 

The higher of:



25% x £302,000 = £75,500 0r £50,000

Therefore, he can relieve £2,000 + £75,500 = £77,500 of his trading loss using the carry
back total income claim.

93
Chargeable gains of the year

Once the total income of a year has been relieved against, and there is still trading loss
remaining, then the loss can be used against the chargeable gains of the year.

The amount of trade loss available to offset against chargeable gains is the lower of:

- trade loss left

- current year capital gains less current year capital losses less the full amount of
capital losses brought forward.

Chargeable gains do not have to be utilised in the loss claim but if the taxpayer chooses
to use the trade loss against capital gains of the same year then the loss is treated as a
current year capital loss and so it cannot be restricted to preserve the annual
exemption.

The only times you can restrict a capital loss to preserve the annual exemption are on
capital losses b/f and capital losses in the year of death.

For example Kathy had a capital gain of £44,000. She has a capital loss brought
forward of £4,000 and has trading losses available of £24,000 - after a claim against the
total income of this tax year

Gain 44,000

Current year trade loss (24,000)

Annual exemption (12,300)

Chargeable gain 7,700

Loss b/f (4,000)

Taxable gain 3,700

Illustration:

Jane had a trading loss of £100,000 and uses £45,000 against her current year total
income claim.

She had chargeable gains of £50,000 and a loss b/f of £1,000. She wants to use the
loss against chargeable gains.

How much of the trading loss will she relieve against the chargeable gains?

94
Solution:

Trade loss available: lower of

- trade loss left £55,000 (100,000 - 45,000)

- capital gain less losses cy and bf £49,000 (50,000 - 1,000)

Therefore £49,000 is available to offset against current year gains.

Gain 50,000

Current year trade loss (49,000)

Chargeable gain 1,000

Annual exemption (12,300)

Loss b/f (NIL*)

Taxable gain NIL

*the loss b.f of £1,000 does not need to be used this year as the gain that is left after
using the cy trade loss is covered by the annual exemption. The £1,000 capital loss and
the remaining trade loss of £6,000 (55,000-49,000) will be carried forward.

95
Illustration:

Peter made the following income for the year ended 05/04/2022:

Trading income £40,000

Property income £20,000

Interest income (gross) £5,000

Capital gains £15,000

Peter made the following income for the year ended 05/04/2023:

Trading income (£75,000)

Property income £20,000

Interest income (gross) £5,000

Capital gains £15,000

Peter made the following income for the year ended 05/04/2024:

Trading income £20,000

Property income £20,000

Interest income (gross) £5,000

How can the trading loss of the year ended 05/04/2023 be relieved against the
current year total income and carried back against total income for 12 months?

Solution:

£65,000 of the trading loss of (£75,000) incurred in the year ended 05/04/2023 will be
carried back against the total income generated in 05/04/2022 and Peter will receive a
refund of any tax paid for 2021/22. This wastes the personal allowance unfortunately.

The remaining loss of £10,000 will be used against total income of the current year
2022/23. This leaves total income of £15,000 of which will be mostly covered by the
personal allowance.

96
It would not be advisable to use the £10,000 remaining loss against the gains of
£15,000 in either year as most of the gains would be covered by the annual exemptions.

In the year ended 05/04/2023

Trading income Nil

Property income £20,000

Interest income  £5,000

Trading income total income claim (£10,000)

Total income £ 15,000

The personal allowance has not been wasted.

In the year ended 05/04/2023 and 05/04/22

Capital gains £15,000

Trading loss relief Nil

Annual exemption (£12,300)

Taxable gains £2,700

In the year ended 05/04/2022

The carry back total income claim for 12 months:

Trading income £40,000

Property income £20,000

Interest income £5,000

Trading loss relief carry back claim (£65,000)

Total income £Nil

The personal allowance is wasted in 21/22.

97
Notice here that the capital gains could not be used because there was not enough trading
loss left after making the carry back claim against total income.

Loss memo:

Trading loss of 05/04/2023 (£75,000)

Carry back total income claim £65,000

Current year total income claim £10,000

Loss to be carried forward Nil 

Note carefully that it is a carry back claim against total income for 12 months.

Therefore, if there is a shorter chargeable accounting period before the loss making year,
then the claim extends back for a full 12 months.

Opening years’ relief

If a loss is made within the first 4 tax years of trading (after applying the opening year
rules) …

… then the loss can be relieved against total income of the individual for the previous 3
tax years on a FIFO basis.

The loss cannot be restricted to save personal allowances.

Illustration:

Mary started trading in 2019/20. She had never worked before she opened her
business.

She made the following trading profits/losses in the following tax years.

2019/20 Trading profit £2,000



2020/21 Trading profit £17,000 

2021/22 Trading profit of £12,000 

2022/23 Trading loss of (£10,000)

How can Mary apply the opening years relief for trading losses?

98
Solution:

Trading loss of (£10,000) in 2022/23

Relieve first against:



2019/20 Trading profit £2,000 

2022/23 Trading loss (£2,000)

Trading profit reduced to £0 in 2019/20

Trading loss of (£8,000) in 2022/23 remaining

Relieve second against:

2020/21 Trading profit of £17,000 



2022/23 Trading loss (£8,000)

Trading profit reduced to £9,000 in 2020/21

2022/23 loss to carry forward - £nil

Note 

That the opening years relief has to be applied on a FIFO basis, therefore the personal
allowance for 19/20 was wasted.

Terminal loss relief

If a trading loss occurs in the final 12 months of trading, then this trading loss can be offset
against any trading profits of the final tax year of trade and then carried back for 3 tax years
against the trading profits of the company on a LIFO basis.

Once again, the loss cannot be restricted to save any personal allowances.

Additionally, for the years in which tax has already been paid, this will result in a repayment
of tax.

There is a specific way to compute a the terminal loss

99
This is:

First, calculate from 06/04 to the date of cessation    x

Second, calculate from 12 months before date of cessation to 05/04 before


date of cessation NOTE: if this calculation produces a profit figure, ignore it. If  x
this calculation produces a loss then include it in the terminal loss figure.

Plus: any overlap profits  x

Terminal loss that relief can be claimed for  x

Illustration:

• Mr. Unlucky, ceased trading on 30/09/2022 and incurred a loss for the 9 months of
(£13,500).

• The trading profits for the year ended 31/12/2021 were £22,500.

• He had overlap profits of £2,000.

• What is the terminal loss that Mr. Unlucky can claim terminal loss relief on?

Solution:

06/04/22-30/09/22  = 6 months/9 months * (£13,500)  = (£9,000)

01/10/21-05/04/22 = 3 months / 12 months * £22,500  = £5,625

= 3 months / 9 months * (£13,500)  = (£4,500)

The net is a profit and so the figure is


Total in penultimate
ignored for the purposes of the terminal 1,125
tax year
loss calculation
Overlap profits = (£2,000)

Terminal loss  (£9,000 + £2,000) (£11,000)

100
Illustration:

How would Mr. Unlucky obtain terminal loss relief for this loss?

Solution:

He had profits of £22,500 in the year ended 31/12/2021.

• Terminal loss relief states that the terminal loss must be relieved against trading
profits from the same trade of the last 3 tax years on a LIFO basis.

Therefore,

Trading profits £22,500

Terminal loss relief (£11,000)

Trading profits £11,500

The factors that will influence the choice of loss relief claims are:

1. Loss relief is as soon as possible.

2. Loss relief is obtained at the highest tax rate.

3. Personal allowances are saved when the claims are made

Illustration:
If Mr Unlucky had trading profits of £4,000 in the year ended 31/12/21 instead of
£22,500, the terminal loss calculation would have been as follows:

Loss from 6/4 to cessation (6/9 x £13,500) £(9,000)

Loss from 1/10 to 5/4 (3/9 x £13,500) £(4,500)

3/12 x £4,000) £1,000

Net loss in previous tax year £(3,500)

Overlap profits £(2,000)

Terminal loss £(14,500)

The key difference to note between the two illustrations is the figure prior to 5/4. In the
first illustration it produced a profit and so it was ignored. In the second illustration it
produced a loss and as such it was included in the terminal loss figure.

101
Partnerships and limited liability partnerships

What is a partnership?

A partnership is a single trading entity, but for taxation purposes each partner is treated
individually.

Allocation of the trading profit or trading loss

1. The trading income or trading loss is divided between the partners according to their
profit sharing arrangements.

2. Partners may firstly be entitled to salaries and interest on capital.

The balance of any trading profit (or loss) will then be allocated in the profit sharing ratio
(PSR).

Illustration 1

Peter has been in partnership with Paul for many years.

The partnership's tax adjusted trading profit is £120,000.

The partners share profits equally.

• Required:

What will Peter's and Paul's share of tax adjusted trading profit be?

Solution:

£120,000 x 1/2 = £60,000

• Peter and Paul will both have £60,000 profit.

102
A change in the profit sharing agreement
If the profit sharing agreement is changed during a period of account, the profit must be
time apportioned before allocation under the different agreements.

Illustration 2

Peter has been in partnership with Paul and Claire.

Paul resigned as a partner on 1 January 2023.

The partnership's tax adjusted trading profit for the year ended 5 April 2023 is
£120,000.

The partners have always shared profits equally, and continued to do so after Paul
resigned.

All partners have overlap profits of £5,000, which they incurred on the start of trading.

• Required:

What will Peter's share of tax adjusted trading profit be for the year ending 05/04/2023?

What will Paul's share of tax adjusted trading profit be for the year ending 05/04/2023?

Solution:

Tax adjusted trading profit for Paul:

6/4/2022 - 31/12/2022: £120,000 x 9/12 x 1/3 = £30,000



Less:

Overlap profits (£5,000)

Tax adjusted trading profit for Paul £25,000

Note as seen in Topic Assessable profits on commencement, Assessable profits on


cessation - overlap profits are deducted from total profits when a person ceases to
trade.

• Tax adjusted trading profit for Peter:

6/4 2022 - 31/12/2022

£120,000 x 9/12 x 1/3 = £30,000

• 1/1/2023 - 5/4/2023

£120,000 x 3/12 x 1/2 = £15,000

• £30,000 + £15,000 = £45,000 profit for Peter


103
Illustration 3

Canda and Panda are in partnership.

The trading income was £18,000

Profits are shared between Canda and Panda in this ratio 3:2, after paying salary of
£3,000 to Canda.

Calculate Canda’s share of residual trading profits.

Solution:

£18,000 - £3,000 = £15,000*3/5 = £9,000

If the question had asked for her total profit share then she would have been entitled to
her salary of £3,000 plus the residual profit share of £9,000, giving a total income from
the partnership of £12,000.

104
Illustration 4

Doug and Rob are in partnership.

The trading income for the year ended 30 September 2022 was £18,000

• Up to 30 June 2022 profits were shared between Doug and Rob 3:2, after paying
salaries of £3,000 and £2,000 per annum.

• From 1 July 2022 profits were shared 2:1 after paying salaries of £6,000 and £4,000
per annum.

• Required:

Show the allocation of trading profits for the Accounting Period ended 30 September
2022.

Solution:

Total Doug Rob

£ £ £

1/10/2021 to 30/6/2022

(Income £18,000 × 9⁄12


= 13,500)

5,000 x 9/12 = 3,000 x 9/12 2,000 x 9/12


Salaries (9/12)
3,750 = 2,250 = 1,500

13,500 - 3,750 9,750 x 3/5 = 9,750 x 2/5 =


Profit shared (3:2)
= 9,750 5,850 3,900

Profit + salary 13,500 8,100 5,400

1/7/2022 to 30/9/2022

(Income £18,000 × 3/12


= 4,500

Salaries (3/12) 2,500 1,500 1,000

Profit shared (2:1) 2,000 1,333 667

Profit + Salary 4,500 2,833 1,667

Total allocation 18,000 10,933 7,067

105
Partnership capital allowances

1. Capital allowances are deducted as an expense in calculating trading profit.

2. If assets are used privately, the business proportion is included in the


partnership’s capital allowances computation.

Illustration 5

Peter has been in partnership with Paul.

The partnership's tax adjusted trading profit is £120,000. this figure is before taking
account of capital allowances.

Capital allowances for the period are £20,000.

The partners share profits equally.

• Required:

What will Peter's and Paul's share of tax adjusted trading profit be?

Solution:

£120,000 - £20,000 = £100,000



£100,000 x 1/2 = £50,000

• Peter and Paul will both have £50,000 profit.

106
Commencement and cessation

1. The rules for commencement and cessation are the same as for a sole trader.

2. The profit is allocated between the partners for accounting periods and then the
assessment rules are applied.

3. Each partner is effectively taxed as a sole trader on his/her share of the adjusted
trading profit

4. Continuing partners will be assessed using CYB

5. When a new partner joins a partnership, he is treated as commencing a new


trade and hence the opening years rules apply

6. When an old partner leaves a partnership he is treated as ceasing a trade and


hence the closing years rules apply

7. Each partner has his own overlap profit available for relief.

As long as there is at least one partner common to the business before and after
the change, the partnership continues.

Illustration 6

Ann and Beryl have been in partnership since 1 July 2020 making up their accounts to
30 June each year. On 1 July 2022 Clair joins the partnership.

The partnership’s trading profit is as follows:

Year ended 30 June 2021 12,000

Year ended 30 June 2022 14,000

Year ended 30 June 2023 24,000

Profits are shared equally.

107
1) Show the amounts assessed on the individual partners for all relevant tax years
of assessment.

Profits will be allocated between


the partners as follows:

Total Ann Beryl Clair

£ £ £

y/e 30/6/2021 12,000 6,000 6,000 -

y/e 30/6/2022 14,000 7,000 7,000 -

y/e 30/6/2023 24,000 8,000 8,000 8,000

2) Compute each partner’s trading income as though they were a sole trader

Ann and Beryl will both be assessed as follows,


based upon a commencement on 1 July 2020:
£
1 July 2020 to 5
2020/21 (Actual) April 2021 4,500
£6,000 × 9/12
Year ended 30
2021/22 (CYB) 6,000
June 2021
Year ended 30
2022/23 (CYB) 7,000
June 2022
Year ended 30
2023/24 (CYB) 8,000
June 2023

They will both carry forward overlap profits of £4,500.

Clair will be treated as commencing on 1 July 2022, and will be assessed on her
share of the partnership profits as follow:

2022/23 (Actual) 1 July 2022 to 5 April 2023 £8,000 × 9/12 6,000

2023/24 (CYB) Year ended 30 June 2023 8,000

She will carry forward overlap profits of £6,000

108
Partnership losses

1. Losses are allocated between partners in the same way as profits.

2. Loss relief claims available are the same as for sole traders.

3. A partner joining the partnership may claim under opening years loss relief, for
losses in the first four tax years of his membership of the partnership.

This relief is not available to existing partners.

4. A partner leaving a partnership may claim under terminal loss relief. This relief is not
available to partners remaining in the partnership.

Illustration 7

John, James and Paul are in partnership making up their accounts to 5 April.

During the year Paul left the partnership and George joined in his place.

The partnership made a trading loss of £40,000.

• Required:

State the loss relief claims that will be available to the partners.

Solution:

Paul will be entitled to terminal loss relief since he has actually ceased trading.

George will be entitled to claim opening years relief since he has actually
commenced trading.

John and James will not be entitled to either of the above reliefs. 

All the partners will be entitled to relief against total income of the current or
previous tax year and against gains. 

All the partners except Paul will be entitled to carry forward relief.

109
Syllabus A1a. TX - UK Recap Property and investment
income

The contents of the Paper TX - UK study guide for income tax and national insurance, under
headings:
- Property and investment income

Computation of property business profits

How to pay income tax on property business profits?

What does property business profit consist of?

Income from land and buildings in the UK will be liable to assessment under property
income.

This includes: rent received/receivable under any lease or tenancy agreement and the
premium received on the grant of a short lease

Property income is assessed in the following manner:

Rent received/receivable in the tax year   x

Plus: premiums received in the tax year  x

Less: capital element of the premium received in the tax year (x)

Less: allowable property expenses paid/payable (x)

Property business profit/loss x/(x)

From 2019/20 the default method for the calculation of property income is the cash
basis - rental income received less allowable expenses paid. This gives automatic bad
debt relief as rental income is not taxed unless it is received.

110
Rental income and allowable expenses will be assessed on an accruals basis
when:

• Property income receipts for the tax year exceed £150,000

• The property business is carried on by a company

• An election is made for the accruals basis to apply (elect by 31 January 2025 for
2022/23 tax year

Under the accruals method, whatever income is allocated to the tax year and whatever
expenses are allocated to the tax year will be taxable in that tax year.

When this income is actually received in hand or when the expenditure is actually paid
out does not matter.

For premiums received due to grant of a short lease, the entire income element of the
premium will be assessed in the tax year that it is received.

This calculation will be illustrated in Topic Premiums granted for short leases.

NOTE: in your exam you should assume that the cash basis applies unless told
otherwise.

Illustration:

Penny owned two properties which were let out unfurnished until both properties were
sold on 31 December 2022.

The following information is available in respect of the two properties:

Property one Property two

Rent received in the year  3,500  7,300 

Allowable revenue expenditure paid in


(4,800) (2,500) 
the year

What is her property business profit?

Solution:

Revenue 3500 + 7300 = 10,800

Allowable cost (4,800 + 2500) = (7,300)

Property business profit 3,500

111
Illustration:

Anne bought a property and rented it out for the first time on 01/07/2022.

The rent of £1,000 is paid in arrears on the last day of each month. The payment for
March 2023 was not received until 10 April 2023.

She paid allowable expenses of £300 in November 2022 and £500 in May 2023 for
repairs that were completed in March 2023.

• Required:

Calculate the property income for 2022/23 using (i) the cash basis and (ii) the accruals
basis.

Solution:

(i) cash basis (ii) accruals basis

8 x £1,000 = 9 x £1,000 = £9,000

£8,000
(rent is accrued for
Rent received / receivable (1 July -
(March rent was the whole period
5 April)
not received until from 1 July 2022 to
after 5 April 2023). 5 April 2023).

Allowable expenses  (£300) (£300)

(£500)

Property income assessable £7,700 £8,200

Note

On the cash basis the March 2023 rent was not received before the tax year end and so
it is not taxed in 2022/23. The expense of £500 was not paid until after the tax year end
and so it is excluded from the calculation. Both of these items will be dealt with in the
tax year 2023/24.

112
Allowable expenses:

These are expenses incurred by the landlord and reduce the taxable property business
profits.

To be allowable, an expense must have been incurred wholly and exclusively in


connection with the business, for example:

1. Insurance

2. Agent’s fees

3. Other management expenses

e.g. cleaning expenses

4. Repairs

Capital expenditure is NOT allowed, therefore repairs are allowable, however capital
expenditure to improve the property are not allowed.

This differentiation can be made simpler by asking yourself whether the expenditure
improved the income earning capacity of the property, if it did, it is likely to be capital
expenditure.

Capital allowances may be claimed for expenditure on plant and machinery used for the
maintenance of the property.

5. Interest on a loan to purchase a non residential property - allowable.

6. Interest on a loan to purchase a non residential property (100% restriction for


residential property)

7. Decorating

8. Impairment losses

e.g. A tenant left owing 1 month’s rent which you were unable to recover.

9. Advertising costs

10.Cost of replacing windows, doors and boilers

11.Motor expenses

If a landlord uses their own vehicle to travel to and from the property they can either
deduct the actual motoring costs or use the approved mileage allowance which we saw
in the Topic The authorised mileage allowances.

12.Replacement furniture allowance (this replaces the old wear and tear
allowance and is available for all properties (except furnished holiday lets)
whether fully or part furnished)

113
Individuals and companies deduct the actual cost of replacing furniture and furnishings
when calculating the property income from renting out a residential property.

Furnishings include items such as beds, televisions, fridges and freezers, carpets and
floor coverings, curtains, and crockery and cutlery.

There is no relief for the initial cost of furniture and furnishings, there is only relief when
assets are replaced.

The amount of relief is reduced by any proceeds from selling the old asset which has
been replaced (Replacement cost - sale proceeds = replacement furniture relief).

Also, relief is not given for any cost which represents an improvement, for example, if a
washing machine is replaced with a washer-dryer, only the cost of an equivalent
washing machine qualifies for relief.

Example, during April 2022, Fred furnished a residential property with a cooker costing
£440, a washing machine costing £330, and floor coverings costing £2,200.

The cooker was sold during December 2022 for £110, and replaced with a similar model
costing £460.

The washing machine was scrapped, with nil proceeds, during March 2023.

It was replaced by a washer-dryer costing £670, although the cost of a similar washing
machine would have been £360.

What would the replacement furniture allowance be?

Replacement furniture relief:



Cooker (£460 – £110) = (£350)

Washing machine    =  (£360)

Note:

No relief is available for the initial cost of the cooker, washing machine and floor
coverings.

Relief for the replacement cooker is reduced by the proceeds of £110 from the sale of
the original cooker.

No relief is given for that part of the cost of the washer-dryer which represents an
improvement over the original washing machine.

Illustration - Impairment losses under the accruals basis

Howard had an unfurnished property and charged rent of £800 per month payable at
the end of each month.

The property was let from 06/04/2022 - 31/12/2022 when the tenant left owing 1
month’s rent which Howard was unable to recover. Allowable expenses paid in the
period amounted to £500.

• What is Howard’s property income assessable for 2022/23 using the accruals basis?

114
Solution:

Rent receivable 9 months * £800 =  £7,200

Impairment losses 1 month * £800 = (£800)

Allowable expenses =  (£500)

Property income assessable =  £5,900

Note: impairment losses do not exist under the cash basis as property income is based
on rental income actually received.

Pre-trading expenditure

Relief is available for revenue expenditure incurred before letting commenced.

• This means that it must be incurred within 7 years of renting

• It will be treated as though it is incurred on day 1 of renting

Illustration:

Hailey owned a furnished flat that she acquired on 01/06/2022.

She paid mortgage interest of £700 on the loan taken out to acquire the property.

On 01/06/2022 she incurred advertising fees of £500 and paid an insurance premium of
£300 for the year to 31/05/2023.

He paid decorating costs of £900 on 15/06/2022.

The flat remained empty until 01/12/2022 when it was rented for £500 payable monthly
in advance.

Solution:

Rent received ( 1 Dec 22 - 5 April 23) 5 months * £500  = £2,500

Allowable expenses:

Decorating costs paid (£900)

Advertising fees paid (£500)

Insurance premium paid (£300)

Mortgage interest (100% restricted) 0

115
Property income £800

Note: under the cash basis the whole of the insurance premium is accounted for in the
tax year because it was paid in the tax year. Under the accruals basis, when calculating
the insurance premium payable, the premium has been paid for 12 months to
31/05/2023, but we only need the premium applicable until 05/04/2023, this is why
10/12 months are taken.

Note if there was furniture that was replaced for this property, then this would also be
deducted when calculating the property loss.

Note from 2017/18 there is a restriction on the amount of mortgage interest relating to a
loan on a residential property that can be deducted from property income. In 2022/23,
none of interest can be deducted from property income. The entire interest amount is
taken off the income tax liability at the basic rate of 20%. This means that any higher
rate or additional rate taxpayers will not get full relief for the interest expense.

So in this illustration, Hailey deducts no interest from property income and then would
deduct £140 (100% x £700 x 20%) from her income tax liability.

This is explored in more detail in Topic Property Income Finance Costs

116
Furnished holiday lettings

What is a furnished holiday letting?

Advantages of being classified as a furnished holiday letting are:

1. Capital allowances are claimed on the cost of furniture instead of claiming


replacement furniture relief if the accruals basis is used (Refer to Topic Capital
allowances). If the cash basis is used then deduction is available for the capital costs
of the furniture when paid.

2. Annual investment allowance is 100%. (This is a part of capital allowances and can
be seen in Topic Capital allowances)

3. Relevant earnings when calculating the maximum amount that can be invested in a
registered pension scheme includes income from a furnished holiday letting. (Refer
to Topic Pensions)

4. Rollover relief is available if the owner invests in another furnished holiday letting.
(Refer to Topic Rollover relief, Holdover relief)

5. Gift relief is available on the gift of a furnished holiday letting. (Refer to Topic Rollover
relief, Holdover relief)

6. Entrepreneur’s relief/Business Asset Disposal Relief is available on the disposal of a


furnished holiday letting. (Refer to Topic Rollover relief, Holdover relief)

Note - the restriction to mortgage interest does not apply to furnished holiday lets.

In order to qualify to be a furnished holiday letting, the following conditions need


to be satisfied:

1. The accommodation must be situated in the European Economic Area.

For example in Malta.

2. The accommodation must be available for letting for at least 210 days in the tax
year.

3. The accommodation must actually be let for 105 days in the tax year.

4. The accommodation must be let on a commercial basis.

This means that no one person should occupy the letting for more than 31 consecutive
days in the tax year.

For example if the letting is let for 105 days in the tax year, it cannot be let by one
person for more than 31 days at a stretch.


117
Rent a room relief

What is rent a room relief?

Two methods to calculate the relief

This relief is based upon letting a room out in your main residence where you live.

There are 2 methods under which the income from letting this room can be assessed.

One of the two methods below can be chosen

Method 1:

If an individual lets a room, furnished, in their main residence – the gross rent up to
£7,500 is exempt.

Alternative – rent a room relief calculation:

Gross rent x
Less: rent a room relief  (£7,500)
Property income  x

Method 2:

This exemption may be ignored if under normal treatment (rental-allowable expenses)


the tax payer is able to generate a lower assessable income, that is where the allowable
expenses exceed £7,500.

Ordinary calculation:

Gross rent x

Less: rent a room relief  (x)

Wear and tear allowance (x)

Property income  x

The election for 2022/23 must be made by 31/01/2025 and stays in force until it is
revoked.

118
Illustration:

Sunder rents a room in his main residence. Gross rents are £145 per week and
expenses amount to £120 per year.

• What is his property income assessable and when does the relevant election need to
be made?

Solution:

Ordinary calculation:

Rent receivable 52 weeks * £145 =  £7,540

Allowable expenses (£120)

Property income £7,420

Alternative – rent a room relief – calculation:

Rent receivable  £7,540

Rent a room relief exemption (£7,500)

Property income £40

Sunder will decide to elect the rent a room relief exemption as this produces the lower
property income assessable. He will have to make this election by 31/01/2025 for
income receivable in 2022/23.


119
Premiums granted for short leases

How are premiums paid for short leases taxed?

The grant of a short lease

When a tenant takes on a new lease, he may pay a one off premium to the landlord in
addition to the annual rent.

• This premium is paid to the landlord for the lease to secure the property space for a
number of years by the person renting the space.

• If this lease is for less than 50 years, then it is considered to be a short lease and a
part of it will be taxable in the year that it is received

• This taxable part is known as the income element of the premium.

• The part that is not taxable is known as the capital element of the premium received.

Calculating the income element and capital elements of the premium:

• Capital element:

Premium received * ((number of years of lease-1)*2%)

• Income element:

Premium received-capital element = income element

Illustration:

Amanda was granted a 22 year lease of a property on 01/05/2022. She paid the
landlord a premium of £6,900 and also pays rent of £2,100 per month.

• What will the property income assessable be for the landlord in 2022/23?

Solution:

Calculation of income element of lease that will be taxable

£6,900*((22-1)*2%) = £2,898 is the capital element of the lease


120
Therefore,

£6,900-£2,898 = £4,002 is the income element that will be taxed

Rent received 11 months * £2,100= £23,100

Income element of premium received £4,002

Property income £27,102

Trading profit deduction for traders

If a trader paid a premium for a short lease he may deduct the following annual amount
against his trading profit in each of the year’s of the lease in which the property is used
in the trade.

This deduction per year is calculated as:

• Income element of premium / number of years of lease

This is in addition to any rent paid.

Illustration:

Amanda is using the property for her trade.

• What will be the allowable deduction from property income in 2022/23 for Amanda?

Solution:

Rent paid (as shown above) £23,100

Lease payment (£4,002/22) £182

Property payments £23,282

121
Property income finance costs

If a loan is taken out to either purchase or repair a residential property, there is a restriction
on the amount of interest expense that will be allowable.

How does the restriction work?

1) No interest can be deducted from property income (100% restriction)

2) The entire interest cost will be use to create a tax credit (a deduction from the income
tax liability) at 20%.

Who/What does this restriction NOT apply to?

1) Companies

2) Furnished Holiday Lettings (FHL)

3) Non residential property

The restriction has no impact on basic rate taxpayers but it still applies to them.

Illustration:

Freddie purchased a freehold house.

The property was then let throughout the tax year at a monthly rent of £1,000 (all rent was
received in the year).

Freddie partly financed the purchase of the property with a repayment mortgage, paying
mortgage interest of £4,000.

The other expenditure on the property amounted to £1,300, and this is all allowable.

Freddie has a salary of £80,000.

Solution:
Freddie’s property income is:

Rent received (£1,000*12) = £12,000

122
Less:

Mortgage interest (100% restricted)= £0

Other expenses (£1,300)

Property income £10,700

His income tax liability is:

Employment income £80,000

Property income £10,700

Total £90,700

Less P.A. (£12,570)

Taxable Income £78,130

Income tax
£37,700 * 20% = £7,540

£40,430*40% = £16,172

Total £23,712

Interest relief (£4,000*20%) ( £800)

I.T. Liability £22,912

123
Property business loss relief

How is a property business loss given tax relief?

Relief is only given against future property business profits

If total expenses exceed total income, the property income assessable is NIL and the
excess property loss is carried forward and offset against future property income profits
ONLY.

Illustration:

• In 2021/22 a trading income of £6,000 was generated and a property loss of (£1,000)
was generated.

• In 2022/23 a trading income of £1000 was generated and property income of £800
was generated.

• What property income will be assessable to income tax in the years 21/22 and
22/23?

Solution:

2021/22 – Nil (as if property loss is incurred, the income assessed is Nil and can only be
used against future property profits)

2022/23

Property income £800

Property loss b/f (£800)

Property income assessed Nil

Property loss carried forward to 2023/24 = (£200)

Note that for a furnished holiday letting loss, this can only be carried forward against
future furnished holiday letting profits, it cannot be relieved against other property
income.

For example, if there was a FHL loss of £1,000 in 21/22 and property income of £1,000
(from a different property) in 22/23 - the FHL loss would not be relieved against the
other property income in 22/23.


124
Tax payable on savings and dividend income/Income
tax computation/Income tax payable

What is included in taxable income?

Computation of Taxable Income

An Income Tax Computation is prepared for each taxpayer and records the income to
be taxed for that individual for a tax year.

The tax year runs from April 6 to following April 5.

The tax year 2022/23 runs from April 6, 2022 to April 5, 2023.

Therefore each source of income requires its own basis of assessment to determine
how much income is to be assessed to tax in each such tax year.

125
Proforma income tax computation

Non- Savings Divide Tot


savings income nds al
income
£ £ £ £

Trading Profit X X

Less Trading Loss relief – (X) (X)


brought forward
Employment Income X X

Property Income X X
Dividends from UK companies X X

Building society interest X X

Bank deposit interest X X


Other interest X X

TOTAL INCOME X X X X
Less
Qualifying interest (X) (X)

Other Trading Loss reliefs (X) (X)

NET INCOME X X X X
Less: Personal Allowance (X) (X)

TAXABLE INCOME X X X X

126
Non savings income

This includes income from employment, income from self employment and property
income.

Savings income

Savings income is all types of interest income, and for the exam, it is received gross -
therefor you just need to include the received amount in the income tax computation.

Savings income now benefits from a 0% rate, because there is a savings income nil
rate band.

For basic rate taxpayers, the savings income nil rate band for the tax year 2022/23 is
£1,000, and for higher rate taxpayers it is £500. Additional rate taxpayers do not benefit
from any savings income nil rate band.

Do not confuse this with the personal allowance, this savings nil rate band is only
for savings income, if there is no savings income then it cannot be used against
any other types of income.

Also do not confuse this with the savings income starting rate of £5,000 which
applies if there is taxable non savings income of less than £5,000.

Savings income in excess of the savings income nil rate band is taxed at the basic rate
of 20% if it falls below the higher rate threshold of £37,700, at the higher rate of 40% if
it falls between the higher rate threshold of £37,700 and the additional rate threshold of
£150,000, and at the additional rate of 45% if it exceeds the additional rate threshold of
£150,000.

127
• Example of a higher rate payer

For the tax year 2022/23, Jina has a salary of £48,500 and savings income of £1,800.

Income tax on:  

Non savings income



Employment income £48,500

Personal allowance (£12,570)

Taxable income £35,930

£35,930 at 20% = £7,186


Savings income

Savings income £1,800


£500 at 0% = £0

£1,270 at 20% (£37,700 - £35,930 - £500) = £254

£30 at 40% (£1,800 – £500 - £1,270) = £12



 

Tax liability  (7,186 + 0 + 254 + 12) = £7,452

Note if she was a basic rate taxpayer, then the savings nil rate band would have been
£1,000 and if she was an additional rate taxpayer, then there would be no nil rate band
available.

Also notice that the nil rate band uses up some of the basic rate band.

128
Dividend income

This includes dividends received from UK companies.

The first £2,000 of dividend income for the tax year 2022/23 benefits from a 0% rate.

This £2,000 nil rate band is available to all taxpayers, regardless of whether they pay tax
at the basic, higher or additional rate.

• Example of dividend nil rate band

For the tax year 2022/23, Eesha has a salary of £56,000 and dividend income of £6,800.

Income tax on:  

Non savings income



Employment income £56,000

Personal allowance (£12,570)

Taxable income £43,430

37,700 at 20% = £7,540



5,730 (43,500 – 37,700) at 40% = £2,292

Dividend income

Dividend income £6,800

2,000 at 0% =£Nil

4,800 (6,800 – 2,000) at 33.75% = £1,620

 

Tax liability (7,540 + 2,292 + 1,620) = £11,452

Carefully note

These nil rate bands are not deductions, they just allow some savings and dividend
income to be taxed at 0%.

They also use up the bands so, if there is taxable non savings income of £23,000 and
taxable savings income of £15,000, even though £500 of the savings income is at 0% it
still uses the band and so £38,000 is taxable in total meaning that this taxpayer is higher
rate.

Only the personal allowance is a deduction which must be first given to non savings
income, then savings income and then dividend income.

This also applies for other reliefs.

129
Income that is exempt from income tax

1. Interest or bonuses on National Savings & Investment Certificates

2. Interest and dividends within an Individual Savings Account [ISA]

3. Gaming, lottery and premium bond winnings

The difference between an income tax liability and income tax payable

Income tax liability is a taxpayers total tax liability for the year.

Tax payable is the amount of tax that is still owing at the end of the year.

For example, if you are an employee with no other income, you are unlikely to have any
tax to pay at the end of the year as it has all been deducted at source by your employer
via PAYE.

You would still calculate your tax liability, then deduct any tax paid at source via PAYE to
leave you with a tax payable figure.

Therefore,

Income tax liability – tax deducted at source = Income tax payable

Non-savings income is taxed at the following rates:

£1-£37,700 at 20% (basic rate band)

£37,701-£150,000 at 40% (higher rate band)

£150,001 -  onwards at 45% (additional rate band)

Savings income is taxed at the following rates:

£1-£37,700 at 20% (basic rate band) (unless the starting rate is available and then it is
£0 - £5,000 at 0% and £5,001 to £37,700 at 20%)

£37,701-£150,000 at 40% (higher rate band)

£150,001 -  onwards at 45% (additional rate band)

Remember to use your savings nil rate band!

Dividend income is taxed at the following rates:

£1-£37,700 at 8.75% (basic rate band)

£37,701-£150,000 at 33.75% (higher rate band)

£150,001 -  onwards at 39.35% (additional rate band)

130
Remember to use your dividend nil rate band!

Illustration:

For the tax year 2022/23, Joe has a salary of £40,000, savings income of £2,000 and
dividend income of £9,000.

During the year, he paid interest of £300 which was for a qualifying purpose.

Joe’s employer deducted £5,800 in PAYE from his earnings.

What is the income tax payable by Joe for 2022/23?

Solution:

Type of income £
Employment income 40,000

Savings income 2,000

Dividend income 9,000

Total income 51,000

Interest paid (300)

Personal allowance (12,570)

Taxable income 38,130

Income tax:  

27,130 (40,000 – 300 – 12,570) x 20% = £5,426

500 at 0% = £0

1,500 (2,000 – 500) x 20% = £300



2,000 at 0% = £0

6,570 x 8.75% = £575

430 (9,000 – 2,000 – 6,570) x 33.75% = £145


Tax liability (5,426 + 0 + 300 + 0 + 575 + 145) = £6,446

Less PAYE (£5,800)


Income tax payable £646

Note

Joe is a higher rate taxpayer, so his savings income nil rate band is £500.

The dividend 0% nil rate band used up some of the basic rate band leaving £6,570 of
the basic rate band for some of the dividends with the remainder of the dividends being
taxed at the higher rate.

131
Savings income starting rate band

There is a tax rate of 0% for savings income within the savings income starting rate
band (£5,000) (don't confuse this with the Savings income nil rate band, that you have
seen in the previous topic)

The savings income starting rate only applies where the savings income falls wholly or
partly below the starting rate limit.

Keep in mind that income tax is charged first on Non-savings income.

The savings income starting rate band counts towards the basic rate limit of £37,700.

Example (using the Saving starting rate band)

Peter has a trading income of £13,000 and savings income of £9,000.

Calculate Peter's tax liability.

• Solution

NSI

13,000 - PA 12,570 = £430 x 20% = £86


SI

Savings income starting rate band: £5,000 - £430 = £4,570 x 0% = £0

Savings nil rate band: basic rate taxpayer = £1,000 x 0% = £0

(£9,000 - £4,570 - £1,000) = £3,430 x 20% = £686


Tax liability = £86 + £686 = £772


Note: We used the Savings starting rate band here because the Non-savings taxable
income was below £5,000. In fact, it was £430, therefore we could still use £4,570
(5,000 - 430) savings starting rate band and use 0% rate.


132
Example (where you can't use the Saving starting rate band)

Peter has a trading income of £46,350 and savings income of £9,000.

Calculate Peter's tax liability.

• Solution


NSI

46,350 - PA 12,570 = £33,780 x 20% = £6,756


Peter is a higher rate taxpayer since his Total taxable income is more than £37,700
(£33,780 + £9,000 = £42,780).


SI

Savings nil rate band: higher rate taxpayer = £500 x 0% = £0

(£37,700 - £33,780 - £500) = £3,420 x 20% =£684

(£9,000 - 500 - 3,420) = 5,080 x 40% = 2,032


Tax liability = £6,756 + £0 + £684 + 2,032 = £9,472


Note: We could NOT use the Savings starting rate band here because the Non-
savings taxable income was more than £5,000. In fact, it was £33,780


133
Individual Savings Accounts and other tax exempt
investments

What is an Individual Savings Account

Individual Savings Accounts (ISA’s) have for many years been the most common form
of tax efficient investment.

The individual savings account (ISA) investment limit for the tax year 2022/23 is
£20,000.

The £20,000 limit is completely flexible, so a person can invest £20,000 in a cash ISA, or
they can invest £20,000 in a stocks and shares ISA, or in any combination of the two –
such as £10,000 in a cash ISA and £10,000 in a stocks and shares ISA.

The main advantages of ISAs are:

1. Income is free of income tax (since the introduction of the savings income nil rate
band for basic and higher rate taxpayers this is less likely to be an advantage for
most individuals. However, it would still be an advantage to an additional rate
taxpayer and other taxpayers who have already used their nil rate band)

2. Disposals of investments within an ISA are free from capital gains tax.

Components of an ISA

1. Cash - for example in a bank account

2. Stocks and shares listed anywhere in the world

National savings

These offer a variety of products some of which are tax free, e.g. National Savings
Certificates

However, some National Savings & Investments (NS&I) products are taxable, namely:

• NS&I Easy Access account / NS&I Direct Saver Account

• NS&I Investments accounts

• The income is received gross without deduction of tax at source.

The nature of the investments are historically risk free.


134
Syllabus A1a. TX - UK Recap The Comprehensive
computation of taxable income

The contents of the Paper TX - UK study guide for income tax and national insurance, under
headings:
- The comprehensive computation of taxable income and the income tax liability

Personal allowance

What is a personal allowance?

Personal allowance

is an amount on which income tax will not be charged.

If an individual makes income above this allowance amount, then income tax will be
charged on that additional income and the relevant rates.

Calculation of the personal allowance

For the tax year 2022/23 the personal allowance is £12,570 but it is reduced if the
taxpayer has adjusted net income for the year in excess of £100,000.

If the adjusted net income exceeds £100,000 then the personal allowance is reduced by
½ of the excess of £100,000.

Therefore, the personal allowance is reduced to Nil if the adjusted net income is
£125,140. (£125,140-£100,000)/2 = £12,570.

How to calculate Net income and Adjusted net income?

Net income = Total income – qualifying interest payments – trading loss reliefs.

• Adjusted net income = Net income – gross personal pension contributions -


gross gift aid contributions.

135
How does this all look?

TOTAL INCOME X
Less:

Trading loss reliefs (X)

Qualifying interest    (X)

NET INCOME X
Less:

Personal allowance (X)

TAXABLE INCOME X

Illustration:

Bubble has net income of £103,150 and has a gross personal pension contribution of
£2,000.

• How much personal allowance will she be entitled to?

• What is her taxable income?

Solution:

Adjusted net income = £103,150 - £2,000 = £101,150

Personal allowance reduction

£101,150 - £100,000 = £1,150 / 2 = £575

• £12,570

• (£575)

• £11,995 is the personal allowance available to Bubble

Total income

Net income £103,150

Personal allowance (£11,995)

Taxable income £91,125

136
Transferable Personal Allowance

What is a transferable P.A.?

For an unused personal allowance to be transferrable between spouses/civil


partners:

One individual must be a non-taxpayer and have unused P.A.

The other individual must be a basic rate tax payer.

• The maximum amount that can be transferred from the non taxpaying spouse is:
£1,260. This reduces their available personal allowance to £11,310 (£12,570-£1,260)

• (This must be available to the non-taxpayer to actually transfer)

• This transfer will be given in the form of a 20% tax credit to the spouse who is taking
it: 20% * (£1,260) = £252 is the maximum tax credit that can be given to the spouse
who is paying tax.

• Thus, this amount will be deducted from their income tax liability to reduce their
income tax payable.

• The election to transfer must be made within 4 years of the end of the tax year to
which it should apply, and remains automatically effective until it is withdrawn.

Illustration:

A husband has trading income of £30,000. His wife only has employment income of
£8,000.

• Does she have unused personal allowance? YES

• How much of her unused personal allowance can she transfer to her husband?
£1,260

• How will this reduce his income tax payable? TAX REDUCER OF £252 (£1,260*20%)

137
Solution:

Wife

Employment income £8,000

Personal allowance  (12,570)

Taxable income Nil 

£4,570 – the maximum that can be transferred is


Unused P.A. =
£1,260.

Husband

Trading income £30,000

Personal allowance (£12,570)

Taxable income £17,430

I.T. liability = £17,430 * 20% = £3,486

Tax credit of unused


(£252)
P.A. (£1,260 * 20%)

I.T. payable     £3,234

138
Qualifying loans

Trading income and property business income

Certain interest payments made on loans taken are deducted from trading income and
property business income.

These include taking out a loan for trading purposes and taking out a loan to purchase
an investment property.

The interest payments here will be deducted from their respective headings that they
relate to.

• Here, we will look at the interest payments which will reduce a taxpayer’s total
income.

• Interest paid on certain loans are deductible from a taxpayer’s total income is
known as interest paid on qualifying loans.

The main types of eligible loans are:

1. Loan to purchase plant and machinery which is necessarily acquired for the use in
the employment of the taxpayer

Illustration:

Purchasing a computer to use for employment, if a loan was taken out to make this
purchase, then the interest payable is deducted from total income.

2. Loan to purchase plant and machinery for the use in the business of a partnership, in
which the taxpayer is a partner.

Illustration:

A partner would have taken out a personal loan to purchase a computer for use in the
partnership, here interest payable would be deducted from total income

139
3. Loan to purchase an interest in a partnership.

Illustration:

Partner A puts in £20,000 into the partnership bank account to fund the business.

If he has borrowed this £20,000 from a bank at 7% p.a., then he can deduct the £1,400
payable from his total income.

4. Loan to purchase ordinary shares in a close company

This is allowable as long as the taxpayer owns at least 5% of the ordinary share capital
or works for the greater part of his time in the management of the company.

140
Gift aid donations

There are 3 tax benefits available for making personal gift aid
donations:

Let's say that an individual makes a gift aid donation of £1,000

1. Pay net of 20%.

For example, if an individual want to make a gift aid donation of £1,000, he needs to
pay 80% and HMRC will make the remaining 20% donation on his behalf.

Therefore, he will pay £800 and HMRC will pay £200 to the fund.

2. Increase the basic and higher rate bands by the gross gift aid donation.

Therefore, this same individual will increase his basic rate band to 37,700 + 1,000 =
£38,700 and his higher rate band to 150,000 + 1,000 = £151,000.

This will result in an additional £1,000 being taxed at the lower rate of 20% instead of
40%, and an additional £1,000 being taxed at the higher rate of 40% instead of 45%.

3. Gross gift aid donation are deducted from net income to arrive at adjusted net
income.

Adjusted net income is used to determine the amount of personal allowance available.
(Topic Personal allowance)

141
Illustration:

Eli has a trading profit of £60,000 and he paid £2,400 to charity under the gift aid
system.

• Show the tax benefits of this donation.

Calculate Eli’s income tax liability.

Solution:

Benefit 1:

• Eli paid £2,400 (80%)

HMRC paid £600 (20%)

Benefit 2:

• Basic band extension: £37,700 + £3,000 = £40,700

Higher band extension: £150,000 + £3,000 = £153,000

Benefit 3:

Adjusted net income = £60,000 - £3,000 = £57,000

Income tax liability

Total income £60,000

Personal allowance £(12,570)

Taxable income £47,430

£40,700 * 20% =  £8,140

(£47,430 - £40,700) = £6,730 * 40% =  £2,692

Total income tax liability £10,832

142
Syllabus A1a. TX - UK Recap National insurance
contributions

The contents of the Paper TX - UK) study guide for income tax and national insurance, under
headings:
- National insurance contributions for employed and self-employed persons

National insurance contributions (NICs)

We have the following classes of NIC:

1. Employee’s Class 1 



Paid by employees

2. Employer’s Class 1

Paid by employers

3. Class 1A

Paid by employers

4. Class 2

 

Paid by the self-employed

5. Class 4

Paid by the self-employed

143
NIC for the employed

There are 3 types of contributions that are payable for those who are employed:

1. Employee’s Class 1

2. Employer’s Class 1

3. Class 1A

Class 1 Primary is paid by employees

For the tax year 22/23 the rates of employee class 1 NIC is 13.25% and 3.25%.

NIC Paid by Paid on behalf Limits Rates


of 
Employee’s Employees Employee cash £0- 0%

Class 1 earnings  £12,570


£12,571- 13.25%
£50,270
£50,271 - 3.25%
onwards

144
Illustration - Employee’s Class 1

Cow plc has one employee who is paid £57,000 per year.

Calculate the Class 1 NIC primary payable by the employee.

Solution:

Employee’s Class 1 payable:

12,570 * 0% = £0

£50,270-£12,570 = £37,700 * 13.25% = £4,995

£57,000 – £50,270 = £6,730 * 3.25% = £219

Total Employee’s Class 1 NIC payable = £5,214

Note that when you are calculating the NIC payable, you need to start paying from
£12,570.Therefore, you will subtract £12,570 from £50,270 and so forth.

Employer’s Class 1 NIC is paid by employers

Employer’s Class 1 NIC is paid by employers on the employee earnings.

There is an employment allowance of £5,000 (given in the exam) available per employer
to reduce the employer’s Class 1 Secondary NIC payable.

Note: this allowance is not available to companies where the only employee is the
Director.

• The rate of employer class 1 NIC is 15.05% and is paid on all earnings over £9,100
in the tax year.

There is no higher limit for the earnings.

NIC Paid by Paid on behalf of  Limits Rates

Employer’s Employee cash £0-


Employer 0%
Class 1 earnings  £9,100

£9,101
15.05%
onwards

145
Illustration - Employer’s Class 1

Cow plc has three employees who are each paid £55,000 per year.

Calculate the Class 1 secondary payable by the employer in 22/23.

Solution:

£9,100 * 0% = £0

£55,000 – £9,100 = £45,900 * 15.05% = £6,908

£6,908 x 3 employees = £20,724



Less employment allowance = (£5,000)

Total Class 1 Secondary payable = £15,724

Class 1A is paid by employers

Class 1A NIC is paid by employers on behalf of the benefits provided to employees.

Note that you may have to calculate the monetary value of the benefits, and then
calculate the Class 1 A NIC payable, based on the benefit's monetary value.

These explanations can be found in topics The statutory approved



mileage allowances and RTI reporting

The amount of benefits assessable

The rate of employer Class 1 A NIC is 15.05%.

NIC Paid by Paid on behalf of  Limits Rates

Benefits provided to
Class I A Employer No limits 15.05%
employee

146
Illustration Class 1A NIC

Jane is employed by Cow plc and earns £25,000 per year.

During 22/23 she received the following taxable benefits:

Car benefit £4,500



Fuel benefit £2,222

Medical Insurance £1,800

Calculate the Class 1A NIC liability in 22/23.

Solution:

Car benefit £4,500



Fuel benefit £2,222

Medical insurance £1,800

Total benefits = £8,522

Class 1 A NIC payable = £8,522 * 15.05% = £2,135

Illustration - Calculating monetary value of car benefit

John has been given a petrol car to use for private purposes during the tax year
2022/23.

The car has a list price of £10,000 and it has a CO2 emission of 135g.

What is the monetary value of this benefit?



What will the Class 1 A NIC payable be upon this benefit?

Solution

Calculating the car benefit:

135g-55g = 80g/5g = 16% + 16% = 32%

32% * £10,000 = £3,200 is the monetary value of the car benefit

Class 1 A NIC payable = £3,200*15.05% = £482

147
Class 2 and 4 NIC

Contributions for the self employed

Class 2 NIC is £3.15 per week for 22/23.

The small earnings exemption is £6,725 which means that if the trading profits are
below this figure no Class 2 NIC is payable.

 

If trading profits are above this amount then £3.15 is paid per week.

Profits £5,000 Profits £7,500

No Class 2 NIC payable because of small Class 2 NIC payable = 52 weeks


profits exemption * £3.15 = £164

Class 4 NIC is payable by the self employed on behalf of their earnings

The rates of Class 4 NIC are 10.25% and 3.25%.

The rate of 0% is paid on profits below £11,908.



The rate of 10.25% is paid on profits between £11,909-£50,270 

The rate of 3.25% is paid on all profits over £50,270

Paid on behalf
NIC Paid by Limits Rates
of 
£0-£11,908
 0%

Self Self employed
Class 4 £11,909-£50,270 
 10.25% 

employed cash earnings
£50,271 -onwards  3.25%

148
Illustration:

Calculate the NIC payable by Shobha if she is self-employed in 22/23 has self-
employed income of £57,000.

Solution:

Class 2 NIC payable = £3.15 * 52 weeks = £164

Class 4 NIC payable:



£11,908 * 0% = £0

£50,270-£11,909 = £38,361 * 10.25 % = £3,932

£57,000 - £50,270 = £6,730 * 3.25% = £219

Total Class 4 NIC payable = £4,151

Total NIC payable £4,315

149
The annual employment allowance

Employment allowance

Annual employment allowance

There is an employment allowance of £5,000 available per employer to reduce the


employer’s Class 1 Secondary NIC payable.

• This allowance cannot be used to reduce the employer's Class 1 A NIC payable or


the employee's Class 1 NIC payable. It is only available to reduce the employer's
Class 1 NIC payable.

• This will reduce the liability that results from the calculation of Employer’s Class 1
NIC by £5,000.

Do not mistake this for a lower limit.

Illustration:

Arya has a salary of £52,000 per annum.

What will her:

1) Employee’s Class 1 NIC payable be?

What will her employer's:

1) Class 1 NIC payable be?

2) Class 1 A NIC payable be?

150
Solution:

Employee’s Class 1 NIC:

£12,570 * 0% = £0

(£50,270-£12,570) = £37,700 * 13.25% = £4,995

(£52,000-£50,270) = £1,730 * 3.25% = £56

Total Employee’s Class 1 NIC payable = £5,051

Employer’s Class 1 NIC: £9,100 * 0% = £0

(£52,000-£9,100) = £42,900 * 15.05% = £6,456

Employer’s Class 1 NIC liability £6,456

Less annual employment allowance (£5,000)

Employer’s Class 1 NIC Payable £1,456

Class 1 A NIC payable= Nil 

There is no Class 1 A because the employee was not given any employment benefits.

151
Syllabus A1a. TX - UK Recap Exemptions and reliefs

The contents of the Paper TX - UK study guide for income tax and national insurance, under
headings:
- The use of exemptions and reliefs in deferring and minimising income tax liabilities

Pensions

Types of pension schemes

Occupational pension schemes

These are pension schemes that are run by an employer.

An employee can contribute into the scheme and an employer can contribute into the
scheme on behalf of the employee.

If an employee contributes into the scheme, tax relief is given as follows:

• The contribution made by the employee is deducted from their salary in arriving at
taxable income.

It is basically treated as an allowable expense.

Illustration 1:

David has a salary of £20,000 for the year ended 05/04/2023.

During the year he has contributed £1,000 into his occupational pension scheme.

• What is his taxable income for 22/23?

152
Solution:

Salary £20,000

Less:

Pension contribution (£1,000)

Net income     £19,000

Less:

Personal allowance (£12,570)

Taxable income £6,430

Registered personal pension schemes

There are 3 tax benefits available for making personal pension contributions into a
registered scheme.

They are exactly the same as the tax benefits available for making gift aid donations.


These are:

1. Pay net of 20%.

For example, if an individual wants to make a personal pension contribution of £1,000,


he needs to pay 80% and HMRC will make the remaining 20% contribution on his
behalf.

Therefore, he will pay £800 and HMRC will pay £200 to the fund.

2. Increase the basic and higher rate bands by the gross personal pension
contribution.

Therefore, this same individual will increase his basic rate band to £38,700 and his
higher rate band to £151,000.

This will result in an additional £1,000 being taxed at the lower rate of 20%, and an
additional £1,000 being taxed at the higher rate of 40%.

3. Gross personal pension contributions are deducted from net income to arrive at
adjusted net income.

Adjusted net income is used to determine the amount of personal allowance available.

153
Illustration 2:

Eli has a trading profit of £55,000 and he paid £2,400 (net) to a registered personal
pension scheme in the tax year 22/23.

• Show the tax benefits of this contribution.

Calculate Eli’s income tax liability for 22/23.

Solution:

Benefit 1:

Eli paid £2,400 (80%)

HMRC paid £600 (20%)

Benefit 2:

Basic band extension:  £37,700 + £3,000= £40,700

Higher band extension:  £150,000 + £3,000 = £153,000

154
Benefit 3:

Adjusted net income = £55,000 - £3,000 £52,000

Income tax liability

Total income £55,000

Personal allowance (£12,570)

Taxable income £42,430

£40,700 * 20% =  £8,140

(£42,430 - £40,700) * 40% =  £692

Total income tax liability £8,832

Limitations of the tax relief available

Pensions do have the taxable benefits mentioned above.

However, there are 2 limitations under which contributions must be to qualify for the tax
relief outlined.

These are:

1. They must be within the relevant earnings of the individual. If not, a certain amount
of the contribution will be taxable.

2. If they are within the relevant earnings, they must be also within the annual
allowances of the individual. If they are not, a certain amount of the contribution will
be taxable.

155
What are relevant earnings?

These are the greater of

• £3,600 and

100% of:

• Trading income (e.g) profits from a business

• Employment income (e.g.) salary

• Income from furnished holiday lettings (e.g.) rental income from a FHLA

For example

if an individual has trading profits of £50,000, then the greater of £3,600 and £50,000
will be chosen as relevant earnings, £50,000 will be the relevant earnings.

if an individual has trading profits of £3,000, then the greater of £3,600 and £3,000 will
be chosen as relevant earnings, £3,600 will be the relevant earnings.

What is the annual allowance?

This is an allowance given to individuals every year.

The individual can use the allowance yearly, and the amount unused is carried forward for 3
years but only if they are a member of a pension scheme in those years.

• Therefore, at any particular time, an individual can use their current year allowance plus
3 years’ b/f unused annual allowances on a FIFO basis. 

The gross contributions are deducted from the annual allowances.

2019/20 £40,000

2020/21 £40,000

2021/22 £40,000

2022/23 £ 40,000

156
Illustration 3:

Sally's trading income for the year ended 05/04/2023 were £60,000.

Sally made contributions of £56,000 (gross) into a personal pension scheme during the tax
year 22/23.

She has made gross pension contributions of £30,000 per annum for the last 10 years.

How much of the pension contribution qualifies for relief?


What is the income tax liability of Sally?

Solution:

Sally’s relevant earnings are the higher of £3,600 and £60,000:

Therefore, Relevant earnings is £60,000.

Therefore, the pension contribution is within 100% of relevant earnings.

However, is the contribution within the annual allowance?

Current year annual allowance £40,000


19/20 b/f annual allowance 40,000 - 30,000 £10,000
20/21 b/f annual allowance 40,000 - 30,000 £10,000
21/22 b/f annual allowance 40,000 - 30,000 £10,000
Total allowance £70,000

Gross contribution    £56,000


Total allowance         (£70,000)
The contribution is within the annual allowance, therefore £56k qualifies for
the tax relief

What happens if the gross contributions are above the relevant earnings or annual
allowance available?

The additional amount is added to the total income, on top of other income, therefore it is
chargeable to income tax at the highest rate that the individual pays.

This is called the annual allowance charge.

Annual allowances only start to accumulate in the first year that an individual makes a
contribution.

157
Illustration 4:

Jenny is self employed.

Her trading income for the year ended 05/04/2023 were £95,000.

Jenny made contributions of £56,000 (gross) into a personal pension scheme during the tax
year 22/23.

She has made gross pension contributions of £39,000 per annum for the last 10 years.

• How much of the pension contribution qualifies for relief?

• How much will result in an annual allowance charge?

• What is the income tax liability of Sally?

Solution:

Sally’s relevant earnings are the higher of £3,600 and £95,000:

• Relevant earnings: £95,000

• Therefore, the pension contribution is within 100% of relevant earnings.

• However, is the contribution within the annual allowance?

Current year annual allowance £40,000


19/20 b/f annual allowance 40,000 - 39,000 £1,000
20/21 b/f annual allowance 40,000 - 39,000 £1,000
21/22 b/f annual allowance 40,000 - 39,000 £1,000
Total allowance £43,000

Gross contribution    £56,000


Total allowance         (£43,000)

Annual allowance charge  £13,000

Total income
Trading income £95,000
Annual allowance charge   £13,000
Total income £108,000

158
Basic Band extension:  £37,700 + £56,000 = £93,700

Income tax liability:

Total income £108,000



Less P.A. (£12,570)

Taxable income £95,430

£93,700 * 20% = £18,740

£1,730 * 40% = £692

Total income tax liability £19,432

159
Tapered annual allowance

• If AI (Adjusted income) is more than £240,000 then the CURRENT year Allowance is a
tapered allowance

• It means that the normal annual allowance of £40,000 is reduced by £1 for every £2 by
which a person’s adjusted income exceeds £240,000, down to a minimum tapered
annual allowance of £4,000.

This is similar to how personal allowances are reduced, except the adjusted net
income in this case needs to be £240,000 to reduce, not £100,000.

Therefore, a person with adjusted income of £312,000 or more, will only be entitled to
an annual allowance of £4,000 (£40,000 – ((£312,000 – £240,000)/2) = £4,000).

Tapering applies on a tax year basis, so a taxpayer with variable income might find
themselves entitled to the full £40,000 annual allowance for some years, and a tapered
annual allowance in other years.

For example

• if the taxpayer only has adjusted income of £100,000

then they will be entitled to the full £40,000 but  

• if the taxpayer has adjusted income of £260,000

then they will be entitled to (£260,000 - £240,000 = £20,000/2 = £10,000,

£40,000 - £10,000 = £30,000 annual allowance available.

A.I. For the self employed= Net income

A.I. For the employed = Net income + employer contributions to the pension
scheme + employee contributions to the pension scheme

Remember that it is only the 2016/17, 2017/18, 2018/19, 2019/20, 2020/21, 2021/22
and 2022/23 annual allowances that you will apply this tapering to, the other
allowances are always given in full

160
• If the annual allowance is not fully used in any tax year, then it is possible to carry
forward any unused allowance for up to three years on a FIFO basis

For this exam, you will be carrying forward annual allowances from 2019/20 onwards
based on the £40,000 that was applicable in that year.

If there is any annual allowance remaining from 2022/23, after the tapering has been
done to the allowance, this can also be carried forward in the normal way.

General rule carry forward is only possible if a person is a member of a pension


scheme for a particular tax year.

Therefore, for any year in which a person is not a member of a pension scheme the
annual allowance is lost.

Illustration AI < £240,000

Peter has made the following gross personal pension contributions:

2019/20 £32,000 

2020/21 £31,000 

2021/22 £19,000 

2022/23 £48,000

Peter's adjusted income for the tax year 2022/23 is £140,000.

Will Peter be subject to an annual allowance charge?

Solution:

No, Peter will not be subject to an annual allowance charge.

The pension contribution of £48,000 for 2022/23 has used all of Peter’s annual allowance of
£40,000 for 2022/23 and £8,000 (48,000 – 40,000) of the unused allowance of £8,000
(40,000 – 32,000) from 2019/20.

Unused allowances to carry forward to 2023/24: 



£9,000 (40,000 – 31,000) from 2020/21

£21,000 (40,000 – 19,000) from 2021/22.

161
Illustration AI>£240,000

Chirag has made the following gross personal pension contributions:

2019/20 £32,000 

2020/21 £31,000 

2021/22 £19,000 

2022/23 £3,000

His adjusted income for the year is £350,000. This is the first time that his AI has been
above £240,000.

Will Chirag be subject to an annual allowance charge?

Solution:

No.

Chirag’s tapered annual allowance for 2022/23 is the minimum of £4,000 because his
adjusted income exceeds £312,000.

His contribution this year is only £3,000 - therefore it is within the tapered annual allowance
of £4,000 and he will have £1,000 to carry forward to 2023/24.

Unused allowances to carry forward to 2023/24:



of £9,000 (40,000 – 31,000) from 2020/21, 

£21,000 (40,000 – 19,000) from 2021/22

£1,000 (£4,000 – £3,000) from 2022/23.

Lifetime allowance

In 2022/23 the lifetime allowance is £1,073,100. This is the total amount of funds that can
be built up within a person’s pension scheme.

If a pension fund grows above this amount the excess will be subject to a tax charge of
either 55% if it is taken out of the pension fund as a lump sum or 25% if it is taken out in
any other way eg pension payments or cash withdrawals.

162
Tax planning

When we dealt with jointly owned assets, we illustrated the tax advantage to be gained
from transferring ownership of an income producing asset, such as a rental property
from a higher rate taxpayer to a spouse who was only a basic rate taxpayer, or even
greater tax savings to be had when the transferee spouse was not even a basic rate
taxpayer and was not therefore using some or all of their personal allowance.

This would allow income that would have been taxed at 40% to now be taxed at 20%
or indeed not taxed at all if the income fell within the available personal allowance of the
transferee spouse.

The introduction of nil rate bands for savings income and dividend income has created
opportunities for spouses to reduce their overall charge to income tax and may even
give an advantage to transferring such income from a basic rate taxpayer spouse to a
higher rate taxpayer spouse!

Example

Donald and Theresa are a married couple and have regular annual income as follows:

Donald

Salary £60,000

Theresa 

Salary  £18,000

Interest  £2,000

Dividends £9,000

It is clear from the above information that Donald is a higher rate taxpayer with taxable
income of £47,430 (60,000 – 12,570) and Theresa is a basic rate taxpayer with taxable
income of £16,430 ((18,000 + 2,000 + 9,000) – 12,570).


In this situation it would normally be the case that for tax planning purposes it would be
advisable to see if any investment income could be moved from the higher rate taxpayer
to the basic rate taxpayer.

This, however is not possible as Donald’s only income is his salary.

The introduction of the nil rate bands, however, means that in the above example tax
savings can be achieved if firstly, £500 of the interest income could be made by Donald
and therefore use his savings income nil rate band of £500 that is currently being
wasted.

This income is being taxed at 20% on Theresa as she has savings income in excess of
her nil rate band of £1,000.

Theresa is £2,000 - £1,000 = £1,000 * 20% = £200

163
If she transferred £500 to her husband, he would use his nil rate band and she would
save: £500 * 20% = £100

The second transfer would be of sufficient shares to move £2,000 of dividend income
from Theresa to Donald in order that both may use their dividend income nil rate bands
of up to £2,000.

Currently Theresa is being taxed at 8.75% on £2,000 of her dividend income, so a tax
saving of £175 (£2,000 * 8.75%) would be possible here.

Clearly in practice choosing the right amount of interest bearing securities and shares to
transfer to Donald to allow usage of the available nil rate bands may be a little difficult to
precisely achieve.

Carefully keep in mind that it is now not necessary to transfer from a higher rate payer
to a basic rate payer to save tax, it can be the other way round.

164
Syllabus A1b. The Scope of Income Tax

i) Explain and apply the concepts of residence, domicile and deemed domicile and advise on
the relevance to income tax

ii) Advise on the availability of the remittance basis to UK resident individuals

Residence, Domicile and Deemed Domicile for


Income Tax

What is UK Income Tax paid on?

• If an individual is UK resident – he will pay UK income tax on his worldwide income

• If an individual is NOT UK resident, he will only pay UK Income Tax on his UK


Income ONLY

Illustration – UK resident

John is UK resident and earns a trading profit in the UK of £60,000 p.a. and he earns
rental income from a villa in Spain of £10,000. 

How much UK Income tax will he pay?


165
Solution


Trading profit £60,000



Overseas income £10,000

Total income £70,000

Less:

P.A. (£12,570)

Taxable Income £57,430

£37,700 * 20% = £7,540

(£57,430 - £37,700) * 40% = £7,892

UK Income tax payable £15,432

Illustration - Non-UK resident

John is not UK resident and earns a trading profit in the UK of £63,000 p.a. and he
earns rental income from a holiday home in New Zealand of £10,000. He is not a citizen
of the EEA.

How much UK Income tax will he pay?

Solution


Trading profit £63,000



Less:

P.A. (£nil)

Taxable Income £63,000

£37,700 * 20% = £7,540

(£63,000 - £37,700) * 40% = £10,120

UK Income tax payable £17,660

Note: as John is not a UK resident, nor a citizen of the EEA, it is unlikely that he will be
able to claim a personal allowance.

166
Domicile and deemed domicile

Domicile

An individual’s domicile can be determined in one of three ways:

1. Domicile of origin - inherited from father at birth

2. Domicile of dependency - if, whilst under 16, father’s domicile changes then domicile
changes with that of the father

3. Domicile of choice - once 16, can sever ties with old country and move to settle
permanently in another country

Deemed domicile

An individual can be deemed domicile in the UK for income tax and capital gains tax if
they satisfy one or both of the following conditions:

1. The individual is a formerly UK domiciled resident who:

• Was born in the UK; and

• Has a UK domicile of origin; and

• Is UK resident in the relevant tax year.

2. The individual is a long-term UK resident who has been UK resident for at least 15 of
the 20 years immediately before the relevant tax year.

Illustration

Jake was born in the UK and his father was domiciled in the UK. At the age of 3 Jake
acquired a domicile of dependency when is father became domiciled in Spain. Jake
returned to the UK on 6 April 2022 and intends to stay for the foreseeable future.

Jake passes the statutory residence test and so will be treated as UK resident in
2022/23.

He also passes the deemed domicile condition of a formerly UK domiciled resident.

Jake will therefore be UK resident and domicile for 2022/23.

UK Resident but not UK Domiciled/deemed domiciled

If an individual is UK resident but not UK Domiciled/deemed domiciled, there are 2


options for taxing income that arises overseas (Overseas Income):

167
1) Remittance Basis 

– whatever overseas income/gain exists, you only pay UK income tax on the amount
of income that you send back to the UK.

If the remittance basis is claimed, the taxpayer will not be entitled to a Personal
Allowance (PA) for Income Tax or Annual Exempt Amount (AEA) for Capital Gains.


The remittance basis will automatically apply if unremitted income (income that has not
been sent back to the UK) is below £2,000. 


In this situation, there is no need to elect for the remittance basis and no RBC will be
charged. The taxpayer also gets to keep their PA and AEA


For example 

John is UK resident but not UK domiciled/deemed domiciled. 

He has investment income arising in Barbados of £10,000. 

He remits £9,000 of this income back to the UK. 

Will the remittance basis automatically apply? 


Solution

Yes it will automatically apply as unremitted income is below £2,000. 

There will be no Remittance Basis Charge (see below).

168
2) Arising Basis 

– whatever overseas income/gain exists, UK income tax is paid on it entirely.

Illustration – Remittance basis

John is UK resident but not UK domiciled/deemed domiciled and earns a trading profit
in the UK of £63,000 p.a. and he earns rental income from a villa in Spain of £10,000. 

He sends £3,000 of the rental income back to the UK. 

How much UK Income tax will he pay if he chooses the remittance basis (RB)?

Solution

Trading profit £63,000

Overseas income £3,000

Total income £66,000

Less:

P.A. (£Nil) - he is not entitled to the PA if he chooses the RB

Taxable Income £66,000

£37,700 * 20% = £7,540

(£66,000 - £37,700) * 40% = £11,320

UK Income tax payable £18,860

Illustration – Arising basis

John is UK resident but not UK domiciled/deemed domiciled and earns a trading profit
in the UK of £63,000 p.a. and he earns rental income from a villa in Spain of £10,000. 

He sends £3,000 of the rental income back to the UK. 

How much UK Income tax will he pay If he chooses the arising basis?

Solution

Trading profit £63,000

Overseas income £10,000

Total income £73,000

Less:

P.A. (£12,570)

Taxable Income £60,430

£37,700 * 20% = £7,540

(£60,430 - £37,700) * 40% = £9,092

UK Income tax payable £16,632

Conclusion:

The Remittance basis looks like the expensive option because John loses his
entitlement to the Personal Allowance. The decision whether or not to claim the

169
remittance basis should be considered year by year as, depending on the level of
unremitted income, sometimes it will be the cheaper option.

Consequences of choosing the remittance basis

• If income not sent back to the UK is LESS than £2,000 then the remittance basis is
automatic, otherwise it must be elected.

If the remittance basis is automatic, then there is no remittance basis charge.

• Remittance Basis Charge (see below)

• No personal allowance available for income tax.

Remittance Basis Charge

If prior to the current tax year an individual (over the age of 18) has been UK resident for
at least 7 of the last 9 tax years then by making the remittance basis election they must
pay HMRC a remittance basis charge. This charge increases once they have been in the
UK for at least 12 of the last 14 years.

This is similar to paying tax on their unremitted income, except that it’s just a flat
charge.

If prior to the current tax year a person has not been UK resident for at least 7 tax years
then if they make the remittance basis election, there is no remittance basis charge.

• Prior to the current tax year the person was UK resident for at least 7 out of the last 9
tax years 

RBC £30,000

• Prior to the current tax year the person was UK resident for at least 12 out of the last
14 tax years

RBC £60,000

Illustration

Benny is domiciled in India.

She has been resident in the UK since 06/04/2015 and earns an annual salary of
£80,000.

She has a property in India from which she earns rental income of £24,000 and from this
remits £15,000 to the UK annually. 


170
Which basis should she choose to tax the overseas rental income in 2022/23?

Solution

Remittance basis

She has been resident in the UK from 06/04/2015 – 05/04/2023 = 8 out of the last 9 tax
years. 

Therefore the R.B.C. will be £30,000. 

Income tax computation:

Salary £80,000

Remitted Income £15,000

Total income £95,000

(No P.A.)

£37,700 * 20% = £7,540

(£95,000 - £37,700) * 40% = £22,920


Total amount payable to HMRC = £7,540 + £22,920 + £30,000 = £60,460

Arising basis

Salary £80,000

Overseas Income £24,000

Total income £104,000

Less:

P.A. (W1) (£10,570)

Taxable income £93,430


Income tax

£37,700 * 20% = £7,540

(£93,430 - £37,700) * 40% = £22,292

Total I.T. payable £29,832

W1

Income>£100,000

(£104,000 - £100,000)/2 = £2,000 – reduce P.A.

P.A. £12,570

Less (£2,000)

Available P.A. £10,570

Conclusion

The remittance basis results in a payment to HMRC of £60,460

The arising basis results in a payment to HMRC of £29,832

The arising basis should be chosen as it saves £30,628

171
Syllabus A1b.
iii) Advise on the tax position of individuals coming to and leaving the UK

Coming to and leaving the UK

Splitting the tax year

Normally, the tax status of an individual is fixed for a whole year.

However, there are some circumstances where a tax year can be split and an individual is
deemed to be UK resident for only part of the tax year (UK part) and not UK resident for a
part of the year (overseas part).

For the split year basis to apply, the individual must be UK resident in the tax year under the
automatic tests or sufficient ties tests. 

Leaving the UK

The split year basis applies in the tax year if the individual:

- Is UK resident in the previous year and



- Is UK resident in the current tax year and

- Is not UK resident in the following tax year and

- Leaves the UK part way through the current tax year for one of three reasons below:

172
Reason 1

They begin to work abroad full time.

• Conditions

They do not spend more than a permitted number of days in the UK after they leave
(less than 91 days per tax year, reduced proportionately in the year of leaving).

• Date of start of overseas part



The overseas resident part will start from the date that they start the overseas work.

Illustration

Seeta has been living in the UK since she was born and is UK resident for tax purposes.
She has taken up a job in India as a teacher and signed a 2 year contract to work there. 

One week after she moved to India, she started her work there on 01/11/2022.

She visited her family in the UK for 2 weeks for Christmas. 

Will the split year basis apply? When will it be applicable from?

• Solution

Yes it will apply because, she has been UK resident in the current and previous tax years
and she is ceasing to be UK resident in the following tax year. 

She has left the UK for one of the acceptable reasons for the split year treatment to apply –
working abroad full time and she spends a limited amount of time in the UK.

It will be applicable from the date she starts the overseas work 01/11/2022. 

UK Part 06/04/22-31/10/22

Overseas Part 01/11/22-05/04/23

173
Reason 2

They accompany or later join their partner to continue to live with them.

• Conditions

Their partner must satisfy the first test above of working full time abroad.

The partner must be a spouse/civil partner with whom they have lived with at some point
during the tax year.

They have no home in the UK or if they do, they spend the greater part of their time in the
overseas home.

• Date of start of overseas part



The overseas resident part will start from the date they join their partner.

Illustration

As Seeta left the UK to start working in India (seen above), Seeta’s husband Vishan has
been a UK resident since birth, for tax purposes. 

He sold their home in the UK and moved to India to live with her on 01/02/2023. 

Will the split year basis apply? When will it be applicable from?

• Solution

Yes it will apply because, he has been UK resident in the current and previous tax years and
he is ceasing to be UK resident in the following tax year. 

He has left the UK for one of the acceptable reasons for the split year treatment to apply –
joining his wife to continue to live with her, and they have sold their home in the UK.

It will be applicable from the date he moves to live with her on 01/02/2023. 

UK Part 06/04/22-31/01/23

Overseas Part 01/02/23-05/04/23

174
Reason 3

They cease to have any home in the UK.

• Conditions

They spend minimal time in the UK (less than16 days) and they establish ties with
the overseas country, for example, they buy a home there.

• Date of start of overseas part



The overseas resident part will start from the date that they cease to have a UK
home.

Illustration

Shane and Sophie have been UK residents for the last 5 years. 

They are moving to India on 01/02/2023, they sell their home in the UK and purchase one in
India on 01/03/2023. 

Will the split year basis apply? When will it be applicable from?

• Solution

Yes it will apply because, they have been UK resident in the current and previous tax years
and they are ceasing to be UK resident in the following tax year. 

They have sold their home in the UK and formed a tie with India by purchasing a home
there. 

It will be applicable from the date the home is purchased on 01/03/2023. 

UK Part 06/04/22-28/02/23

Overseas Part 01/03/23-05/04/23

175
Arriving in the UK

The split year basis applies in the tax year if the individual:

- Is not UK resident in the previous year and



- Is UK resident in the current tax year and

- Arrives in the UK part way through the current tax year for one of three reasons below

Reason 1

They begin to work in the UK full time.

Conditions

For >=365 continuous days and did not have sufficient ties in the UK to be UK resident
prior to entry.

Date of start of overseas part



The UK resident part will start from the date that they start the UK work.

Reason 2

They accompany or later join their partner to continue to live with them in the UK.

Conditions

Their partner must satisfy the first test above.

The partner must be a spouse/civil partner with whom they have lived with at some point
during the tax year.

They are resident in the following tax year. 

Date of start of overseas part



The UK resident part will start from the date they join their partner.

Reason 3

They buy a home in the UK. 

Conditions

They did not have sufficient ties in the UK to be UK resident prior to purchasing a home in
the UK. 

Date of start of overseas part



The UK resident part will start from the date that they buy a home in the UK.

176
Illustration

Brenda has been working in Spain for many years but on 01/07/2022, she came to the UK
in search of other employment. 

She has no ties in the UK and has not been UK resident in the past. 

She secured herself a three year contract of employment and started work on 22/08/2022. 

Will the split year basis apply? When will it be applicable from?

• Solution

Yes it will apply. This is because she is not resident in the previous tax year and she is
resident in the current tax year (she will spend at least 183 days in the UK). 

The condition for working full time in the UK has been satisfied. 

It will be applicable from the date she starts the overseas work 22/08/2022. 

Overseas Part 06/04/22-21/08/22



UK Part 22/08/22-05/04/23

177
Syllabus A1biv/v/vi. Determine the income tax treatment of overseas income

and

Understand the relevance of the OECD model double tax treaty to given situations

and

Calculate and advise on the double taxation relief available to individual

What is double taxation relief?

Under UK tax law an individual who is resident in the UK must pay UK income tax on his
worldwide income.

In the case of income arising in another country, that income may also be taxed in the
foreign country, and will be taxed in the UK, if the individual is UK resident. 

The rules that apply to the taxing of overseas income are set out in the Organisation for
Economic Co-operation and Development Model (OECD). 

This model states that if there is no double taxation treaty between 2 countries, then double
taxation relief is available. (There will never be a treaty in your exam, you will always have to
calculate DTR) 

Therefore, in order to avoid being taxed on the same income two times, double taxation
relief (DTR) is available, usually as a tax credit against the UK income tax liability. 

The DTR tax credit is the lower of: 



(i) UK tax on overseas source 

(ii) Overseas tax suffered. 

Where there is more than one source of overseas income, DTR on each source must be
considered separately.

178
Illustration

John owns a home in Barbados and rents it out for £30,000 p.a.

He is UK resident and has employment income of £60,000.

The income tax rate in Barbados is 45%.

How much double tax relief will be available to John?

• Solution

He is a higher rate tax payer.

Therefore, he will pay 40% UK IT, however, in Barbados, the tax rate is 45% - which is
higher than the UK.

Therefore, DTR: £30,000 * 40% = £12,000.

Illustration

Jeremy is resident and domiciled in the UK and has the following income.

Salary from UK employment (gross) £98,000 



Bank interest received £3,000 

Dividend income received £7,000 

Barbados bank interest £1,100 (gross) (42% I.T. in Barbados)

India rent £600 (gross) (15% I.T. in India)

You should assume that no double tax treaty exists between the UK, Barbados and India. 

What is the income tax liability after considering all available reliefs?

• Solution

Income tax computation:



Employment income £98,000 

Dividend income £7,000 

Indian rent £600 

Barbados bank interest £1,100 

Bank interest £3,000 

Net income £109,700 



Less: Personal allowance £12,570- £4,850 [½ (£109,700 – £100,000)] (£7,720)

 

Taxable income £101,980

179
Analysis of income: 

Non-savings income £90,880 (£98,000 + £600 - £7,720) 



Savings income £4,100 

Dividend £7,000

Taxable income £101,980

• Income tax:

Non-savings income 

£37,700 × 20% = £7,540

£53,180 × 40% = £21,272

Savings income: 

£500 × 0% =£Nil 

£3,600 x 40% =£1,440

Dividend income: 

£2,000 x 0% =£Nil 

£5,000 x 33.75% =£1,688

I.T. Liability £31,940



Less: 

DTR Relief for Barbados tax (W1) (£440) 

Relief for India tax (W2) (£90) 

Income tax payable: £31,410

• W1

UK tax on £1,100 (£1,100 × 40%) 440



Foreign tax on £1,100 (1,100 × 42%) 462 

DTR = lower £440

• W2

UK tax attributable = £600 × 40% =£240 



Foreign tax on £600 (600 × 15%) =£90

DTR = lower £90

180
Syllabus A1c. Income from employment

Syllabus A1ci) Advise on the tax treatment of share option and share incentive schemes.

Incentive Schemes

Share related income from employment

An employee incentive scheme provides financial incentives for employees to improve their
work performance. 

Types of share schemes:

1. Share incentive plans (SIP)

2. Company share option plans (CSOP)

3. Enterprise management incentive scheme (EMIs)

4. Savings-related share option schemes (SAYE)

Share incentive plans (SIP)

1. Income tax - when you receive them

Pay No income tax or NIC, (If you hold the shares for more than 5 years)

2. Capital gains tax - when you sell them

Pay No I.T. or NIC (If you hold the shares for more than 5 years)

How to get the tax advantaged treatment?

1. Shares must be offered to all employees who have been working in the company for
more than 18 months (It can NOT be selective)

2. Maximum value of shares that the employer can give to the employee each tax
year can not exceed £3,600

3. Shares must be held for 5 years

181
Consequences of withdrawing the shares before 5 years from SIP

• Withdrawing shares in < 3 years

I.T. and NIC consequences:



I.T. and class 1 primary NIC are payable in the tax year of withdrawal based on the market
value when withdrawn.

• Withdrawing shares in more than 3 years but  less than 5 years

I.T. and NIC consequences:



I.T. and class 1 primary NIC are payable in the tax year of withdrawal based on the lower of:

1) MV when first awarded and



2) MV when withdrawn from the SIP

• Capital gains tax the base cost of the share is the market value at the time they are
withdrawn from the share incentive plan.

Illustration

Jake receives 100 shares valued at £2 from an approved SIP from his employer. 

He wants to know the tax implications if he sells them in:

1) 2 years (M.V. £4)



2) 4 years (M.V. £6)

3) 6 years (M.V. £8)

• Solution

No income tax will be payable in the year that the shares are received as this is an
approved plan. 

Selling in 2 years

Income tax and class 1 primary national insurance is payable in the tax year of withdrawal
based on the market value when withdrawn.

Benefit: 

M.V. £4*100 = £400 



Less cost (£Nil)

Taxable benefit £400

182
• Selling in 4 years

Income tax and class 1 primary national insurance is payable in the tax year of withdrawal
based on the lower of:

1) MV when first awarded and



2) MV when withdrawn from the SIP

Therefore, M.V. when first awarded is lower at £2

M.V. £2*100 = £200



Less cost (£Nil)

Taxable benefit £200

• Selling in 6 years

As the shares are held for more than 5 years in the approved plan, there is no income tax or
NIC payable.

Capital gains tax implications

The cost of the shares are the market value of the shares when they leave the plan.

Therefore if withdrawn in 2 years (M.V. £4 is cost), 4 years (M.V. £6 is cost) and 6 years (M.V.
£8 is cost)

Share options

A share option is an offer to an employee of a right to purchase shares at a future date at a


pre-determined fixed price which is set at the time the offer is made. 

The pre-determined fixed price is usually below the market value of the shares at that time.

The taxation consequences of share options depends on whether or not they are approved
by HMRC as follows. 

The tax advantaged share option schemes are the company share option plan (CSOP), the
enterprise management incentive share option scheme (EMIs) and the Save As You Earn
(SAYE) share option scheme.

183
Share option plans

1. Income tax implications

2. CGT - When you sell the shares

Pay CGT:  (Market value @ sale date - Market value @ grant date) x CGT tax rate

Illustration

Jake wants to offer share options to 5 of his employees under the EMI scheme. 

On grant: 10,000 shares/employee when the shares have a market value of £2 and the
exercise price is £1.75.

They can be exercised in 6 years when they have a market value of £5.

What conditions need to be satisfied to qualify as an approved EMI scheme? 

What are the tax implications?

• Solution

Conditions to be satisfied

1) The company must have less than 250 employees.



2) Each employee must work for at least 25 hours/week.

3) The company must have less than £30 Million in gross assets. 

Income tax implications

Market value on grant £2



Less Ex. Price (£1.75)

Benefit £0.25*10,000 shares = £2,500

Capital gains tax implications

The cost of shares will be the market value at the grant date. 

Therefore,

S.P  £5

Less cost (£2)

Capital gain £3*10,000 shares = £30,000 

Entrepreneur's relief/Business asset disposal relief will be available at 10% as the


conditions are satisfied.


184
Approved EMI scheme conditions

There are conditions that the company and the employee must satisfy for the share options
to get the tax advantaged treatment

• Conditions that the company needs to satisfy

1) Gross assets must not exceed £30 million

2) Employees in the company must not exceed 250

3) Maximum value of share options issued must not exceed £3,000,000

• Conditions that the employee needs to satisfy

1) Employees must work for at least 25 hours per week

2) Employee must own less than 30% of the shares in the company

3) Maximum value of share options per employee £250,000

Approved CSOP scheme conditions

There are conditions that the company and the employee must satisfy for the share options
to get the tax advantaged treatment

• Conditions that the company needs to satisfy

1) Gross assets must not exceed £30 million

2) Employees in the company must not exceed 250

3) Maximum value of share options issued must not exceed £3,000,000

• Conditions that the employee needs to satisfy

1) Maximum value of share options that can be issued is £30,000 per employee

2) Each employee must own less than 30% of the shares

185
SAYE scheme

Employees are granted an option to buy shares and then save through a tax-free savings
scheme in order to raise funds to exercise the option.

There is favourable tax treatment for share option schemes that are linked to a SAYE (Save
As You Earn) contract.

• How does the scheme operate?

Each employee pays a minimum of £10 per month and a maximum of £500 per month into
a SAYE scheme, for a period of 3 or 5 years. 

Interest on the scheme is exempt from income tax. 

At the end of the scheme the money can be used to exercise the share options or the
employee may just withdraw the money on their own.

Income tax and NIC implications

No income tax or national insurance will be charged on the grant of the option. 

No income tax or national insurance will be charged on the exercise of the option.

• Capital gains tax implications

On the subsequent disposal of the shares, a capital gain may arise.

Conditions for the SAYE scheme

1) The amount saved must be at least £10 per month but cannot exceed £500 per month.

2) The savings contract must last for three or five years.

3) The scheme must be available to all employees (full and part-time) who have worked for
a specified qualifying period (which cannot exceed five years).

4) The exercise price must be at least 80% of the shares’ market value at the time that the
option is granted.

186
Illustration

B plc. has a proposed SAYE scheme. 

The duration of the scheme is 5 years.



The maximum monthly deposit into the scheme is £500. 

The scheme is available to all employees who have worked for at least 3 years. 

Exercise price £2.48



Market value at grant date £3

Will this qualify to be an approved SAYE scheme?

• Solution

Yes it will be.

The conditions that have been satisfied are:

1) The amount saved per month does not exceed £500.

2) The savings contract lasts for 5 years.

3) The scheme is available to all employees who have worked for 3 years.

4) The exercise price is 82% (£3-£2.48/£3*100%) of the market value at the grant date.


187
Syllabus A1cii) Advise on the tax treatment of lump sum receipts

Lump sum receipt

Payments on termination of employment

Payments on termination of employment may be entirely exempt or partially exempt.

• Exempt payments

1) Payments on account of injury, disability or accidental death.

2) Lump sum payments from approved pension schemes.

• Partially exempt payments

On the loss of office, and individual is entitled to £30,000 statutory redundancy pay. 

Any amount given above this will be taxable. This can be known as non-statutory pay/
compensation for loss of office/ex gratia payments. 

Whatever it is called, any amount above £30,000 is taxable as employment income for the
employee, and the employer will pay Class 1 A NIC on the excess.

• Payments in lieu of notice (PILON)

These are payments made when an individual is asked to leave office immediately, without
giving them the proper notice period.

If these payments are contractual, then the employee will pay income tax on them, and
Class 1 NIC will be payable on them by both the employee and employer.

If these payments are non contractual and it is not custom to give such payments by the
employer, then any amount paid as a PILON that would have normally been paid to work
during the notice period will be subject to income tax (employee) and Class 1 NIC
(employee and employer).

Any additional amount given can be qualifying expenditure and if it falls within the £30,000 -
they will be tax free, but any excess will be subject to income tax (employee) and Class 1 A
NIC (employer). 


188
Illustration

John was given a £22,000 statutory redundancy payment. 

He was also given £48,000 ex gratia. 

His annual salary was £120,000 and he received 3 months salary as payment in lieu of
notice, as part of his contract of employment.

What are the tax implications of these payments?

• Solution

Statutory redundancy = exempt (but uses up some of the £30,000 exempt amount)

Taxable ex gratia payment:



£48,000 - (£30,000 - £22,000) = £40,000 taxable as the top slice of income. This will be
subject to income tax (employee) and Class 1 A NIC (employer)

Payment in lieu of notice:



3/12* £120,000 = £30,000 taxable as employment income. This will be subject to income
tax (employee) and Class 1 NIC (employee and employer)

Lump sum pension receipts

When lump sum pension receipts are received by an individual, the entire receipt is not
taxable, subject to a total tax free lump sum amount of 25% of the fund value.

The lump sum amount is limited to 25% of the lifetime allowance limit of £1,073,100 for
2022/23.

189
Syllabus:
Syllabus A1ciii) Identify personal service companies and advise on the tax consequences of
providing services via a personal service company

and

A4vi) Identify personal service companies and advise on the tax consequences of services
being provided via a personal service company

Personal Service Companies

What is a personal service company?

An individual offering services to a client, therefore being employed by the client will not get
the income tax and national insurance advantages that a company would get from offering
the same services to the client. 

One way in which an individual might seek to avoid being classed as an employee is to
form a limited company (a personal service company) and then to hire out his or her
services in the name of the company. 

This would allow the individual to get the tax and national insurance advantages that are
available to a company but not an individual.

Personal service companies have been coming under a lot of scrutiny recently as HMRC
are trying to crack down on arrangements that are created purely to avoid income tax and
national insurance. For example, someone leaving their employer, setting up a limited
company and then working for the previous employer though the new limited company.
HMRC will deem this person to be an employee of the previous employer and tax them as
such. 


190
Illustration

Mary works for Jake Ltd. and earns a salary of £60,000 per annum. 

What are her NIC payments?

• Solution

Employee Class 1: 



50,270 – 12,570 * 13.25% = £4,995

60,000 – 50,270 * 3.25% = £316

Total £5,311

Illustration

Mary owns 100% of the share capital of Mary Ltd. and is employed full time by Mary Ltd. 

Mary Ltd. offers services to Jake Ltd and earns £60,000 per annum from this contract. 

What are the NIC payments of Mary Ltd?

If Mary does not take a salary from Mary Ltd then there will be no NIC implications for Mary
or Mary Ltd.

Anti-avoidance legislation (the “IR 35” legislation) was created to stop this kind of disguised
employment but it seems that amendments need to be made to this legislation as personal
service companies still exist and are still created.

The legislation applies to relevant engagements, where a worker provides services to a


client through an intermediary (usually a company) – wherein, if the intermediary company
did not exist, this individual providing services to a client – would be treated as getting
income from employment. 

For example, if Mary Ltd. did not exist, Mary would be employed by Jake Ltd directly, and
therefore receive employment income from Jake Ltd and pay income tax and national
insurance on that income.

It is only because Mary Ltd. exists that the services are being offered through the
intermediary to save tax. 

If an intermediary company receives income from a relevant engagement during a tax year
and this income (less allowable expenses) is greater than the worker’s employment income
received from the intermediary in that year, then the excess is treated as a deemed salary
payment made on the last day of the tax year. 

The deemed payment is subject to both income tax and national insurance contributions.

191
Conditions to be classified as a personal service company:
• The company enters into a contract to provide services to the client

• The services are carried out by the individual

• If the services were carried out under a contract between the individual and the
client, then the individual would be regarded as an employee of the client

• The individual must own >=5% of the share capital in the intermediary company

As a result of the changes introduced by Finance Act 2021, the rules for off payroll working
in the public sector (introduced in 2017) will now apply to workers in the private sector
where the client is a medium or large sized organization.

Services provided via a PSC to a small organisation

In this situation, the PSC (ie the intermediary company) is required to determine whether or
not the IR35 rules apply.

Where the rules do apply, the PSC is required to treat the income from relevant
engagements as if it were a salary paid to the employee, and to account for income tax and
class 1 national insurance contributions (NIC) on the deemed employment payment.

Calculation of the deemed salary

Income from relevant engagements £A

Less: 

Statutory deduction (5% × £A) (X) 

NICs paid by employer (X) 

Expenses paid by the employer which would be deductible under employment income rules
(X)

Pension contributions by employer (X) 

Salary paid by employer (X) 

Deemed salary including employer’s NICs £B

Less: Employer’s NICs (£B × 15.05/115.05) (X)

Deemed salary £C

Ignore any dividends paid by personal service company

192
Illustration:

Jake has formed a limited company which is a PSC. Jake is the only employee of the
PSC. During the year ending 31 March 2023, Jake will perform services via the PSC
for a client which is classified as a small organisation for the purposes of the IR35
legislation. The budgeted fee income of the PSC for the year ending 31 March 2023
in respect of relevant engagements is £80,000. The PSC will pay Jake a gross salary
of £35,000 for this period.

Solution:

The deemed employment payment will be calculated as follows:

Income in respect of relevant engagements 80,000

Less: 5% deduction (4,000)

76,000

Less: Salary (35,000)

Employer’s NIC on salary

(35,000 – 9,100) x 15.05% (3,898)

37,102

Less: Employer’s NIC on deemed payment

(15.05/115.05 x 37,102) (4,853)

Deemed employment payment 32,249

193
The PSC has the obligation to calculate and pay income tax and NIC on the deemed
employment payment.

In order to prevent double taxation, dividends paid by the PSC to the worker out of
this income, are exempt from income tax.

Services provided via a PSC to a medium or large sized organisation



In this situation it is the client, rather than the PSC, which is responsible for
determining the status of the individual. The client will issue a Status Determination
Statement to the individual.

Where it is determined that the IR35 rules apply, the client is then required to
calculate and pay the income tax and NIC on the deemed direct payment (DDP).
The DDP is calculated as follows:

Payment in respect of services provided(Net of VAT) X

Less: Direct cost of materials incurred by the PSC (X)

Less: Deductible employee expenses incurred by PSC (X)

DDP X

Illustration:
Maria provides services via her PSC to a client which is classified as a medium or
large sized organisation. The client has issued her with a Status Determination
Statement stating that her services fall within the IR35 rules.

Maria sends her client an invoice for £10,000 (net of VAT). The PSC incurred
deductible expenses of £750 and the direct cost of materials in respect of the
services provided was £500.

The DDP is therefore £8,750 (£10,000 – £750 – £500). This payment will be
chargeable to tax and NICs (employee and employer) in the same way as if Maria
was a direct employee of the client.

In order to prevent double taxation, the DDP is deducted from any payment made by
the PSC to the worker before calculating the tax and NICs due in respect of such
payment. 


194
Syllabus A1d. Income from self employment

Syllabus: A1di) Advise on a change of accounting date

Choice of accounting date

What accounting date should you choose?

The choice of accounting date will affect:


• The level of profits to be taxed in that year

• The overlap profits created

• The timing and amount of tax payments

Advantages of tax year ending on 05/04

1. No overlap profits created

2. Minimise the final year’s liability

3. Simplicity

Advantages of tax year ending on 30/04

1. Time to prepare accounts and calculate tax.

2. Time lag between earning income and paying tax.

Conditions to change an accounting date

An individual must meet certain conditions in order to qualify to change an


accounting date:

• The change of accounting date must be notified to HMRC by the 31/01 following the
tax year in which the change was made.

• The first accounts to the new accounting date must not exceed 18 months in length.

• A change of accounting date must not have occurred within the previous 5 years

195
When an accounting date is changed, if the accounts are prepared to a period that is longer
than 12 months, then overlap profits can be deducted.

(Note: overlap profits cannot be deducted to the extent that they create a period of less
than 12 months).

However, if the accounts prepared to a period are shorter than 12 months, then 12 months
to the new accounting date must be taxed, which will create more overlap profits.

Illustration for change of accounting date (Period longer than 12 months):

Jake has always had an accounting date of 31/12.

His trading profits for the year ending 31/12/2021 are £15,000.

He wants to change his accounting date to 31/3 and will prepare accounts to 31/3/2023.

His profits for the 15 month period to 31/3/2023 are £18,000.

He has 4 months of overlap profits totalling £4,000.

What profits will be assessable in 22/23?

Solution:

22/23

15 month profits to 31/3/2023 £18,000

Less overlap profits (£3,000)

Taxable profits £15,000

Note: you cannot reduce a long period to less than 12 months so in this case we could only
use 3 of the 4 months of overlap. The remaining overlap profits of £4,000 will be carried
forward for use on cessation or if there is another change of accounting date in the future.

196
Illustration for change of accounting date (Period shorter than 12 months):

Jake has always had an accounting date of 31/12.

His trading profits for the year ending 31/12/2021 are £15,000.

He wants to change his accounting date to 30/4 and will prepare accounts to 30/4/2022.

His profits for the 4 month period to 30/4/2022 are £8,000.

What profits will be assessable in 22/23?

Solution:
22/23

Tax 12 months to the new accounting date

12 month profits to 30/4/2022 £8,000 + (8/12 * £15,000)

Taxable profits £18,000

Therefore, overlap profits of £10,000 were created.

197
Syllabus: A1dii) Advise on the relief available for trading losses following the transfer of a
business to a company

Transfer of a business to a company

Trading loss relief available

If the owner of a business transfers that business to a limited company, there is a change in
the legal ownership of the business and the seller is deemed to have stopped
trading. 

Any trading loss which the seller still has before the date of transfer cannot be carried
forward and set against the company’s trading profits.

However, there is a relief that the sole trader can use if their business is transferred to a
limited company, provided that the following conditions are met:

1. The consideration is wholly or mainly in exchange for shares in that company (≥


80% of the consideration must be in shares).

2. The seller of the business continues to hold those shares throughout the tax year in
which the relief is given.

3. The company continues to carry on the transferred business.

Then the seller may set unrelieved trading losses against the first available income that he
or she receives from the company in the most tax efficient manner. 

For example, if the seller receives dividends from the company, then they can set the
unrelieved trading loss off against the first dividend received.

198
Illustration

Jake transferred his manufacturing business to Jim Ltd. for 10,000 shares in the company
which he intends to hold for many years. 

When he transferred his business, there were unrelieved trading losses of (£20,000).

Jim Ltd. is continuing the manufacturing business. 

Jake received dividends of £8,000 from the company. 

Can he relieve the trading loss?

• Solution

Yes, he can as the following conditions are met:

The consideration is wholly for shares. 

Jake continues to hold those shares. 

Jim Ltd. continues to carry on the transferred manufacturing business. 

Loss Relief

Dividends received £8,000

C/f loss used (£8,000)

Loss to be carried forward £12,000 (£20,000-£8,000)

199
Syllabus:
A1diii) Advise on the allocation of the annual investment allowance between related
businesses

and

A4ei) Advise on the allocation of the annual investment allowance between group or related
companies

 Related companies - Annual Investment Allowance

Related companies

The AIA must be split between related companies.

Companies owned by the same individual will be regarded as related where they are:

a) Engaged in the same activities or

b) Share the same premises

This could be the case if an individual runs two companies from home, the AIA will be split
between the two companies. 

In such circumstances, the owner of the companies can choose to how to share the single
AIA between.

• Unrelated companies owned by the same individual will be entitled to their own


AIAs so long as they do not share the same premises and are engaged in different
activities

200
Groups of companies
Only one AIA is available to a group of companies.

Note that

- A group for this purposes is where the parent company holds a majority shareholding in the
subsidiary.

When allocating the AIA

- The group members can allocate a maximum of £1,000,000 across the group in any way
wanted.

- The AIA does not need to be divided equally between the companies

- All of the allowance can be given to one company, or any amount can be given to any
number of companies within the group.

Illustration
Jane owns Jake Ltd.

Jane also wants to know whether she should purchase Jill Ltd. (Jake Ltd. will then purchase
all of the components from Jill Ltd.)

Which arrangement will result in the AIA being shared?

• Solution
If Jake Ltd. purchases Jill Ltd. directly, then Jake Ltd. will own 100% of the share capital of
Jill Ltd. and there will be one AIA for the group.

If Jane purchases Jill Ltd. and:

1) Jake Ltd. and Jill Ltd. are run from the same premises or,
2) Jake Ltd. and Jill Ltd. are engaged in the same activities,

Then, Jake Ltd. and Jill Ltd. will share one AIA – otherwise they will not.

201
Syllabus A1e. Property and investment income

Syllabus: A1ei) Advise on the tax implications of jointly held assets

Jointly owned property by a married couple/civil


partners

Understand the treatment of property owned jointly by a married couple, or by a couple in a


civil partnership.

• If assets are owned jointly then the rule is that any income generated from the asset must
be split 50:50.

• It is possible to make a declaration of beneficial interest in order that the joint income is
split in order to the actual entitlement.

• If one spouse does not own any shares in the property, shares can be transferred to that
spouse to result in actual entitlement. 

Transferring just 5% of shares can result in actual entitlement of 50% to income. If more
shares are transferred, then more income can be legally transferred.

• Ideally, to be tax efficient, the declaration should assign more income to the individual
who is a lower rate tax payer and potentially has some unused personal allowance. 

The overall objective is to save tax for the family as a whole.


202
Illustration:

A couple has a joint property of which generates annual income of £100,000. 

The husband contributed nothing towards the purchase of the house and the wife
contributed 100% towards the purchase of the house. 

How will this income be split if no declaration is made?

Solution:

If no declaration is made, then the income will be split in the following manner:

Husband  £50,000
Wife £50,000

Illustration:

For the same couple above, the following information relates to their yearly income aside
from the property income.

• Husband: £100,000 salary per annum.

Wife: Not earning

If the declaration is made to split the income according to actual entitlement, how
much income tax will be saved as a couple?

Solution:

Current situation

Husband 
Salary £100,000
Property income  £50,000
Total income  £150,000
P.A. Nil (Income above £125,000)
Taxable income  £150,000

203
Wife £

Property Income 50,000

Total income 50,000

P.A. -12,570

Taxable income 37,430

Income tax liability:

Husband
£37,700 * 20% =  £7,540
(£150,000 - £37,700) * 40% = £44,920
Total £52,460

Wife
£37,430 * 20% = £7,486

Total £7,486

• The husband is already paying tax at a higher rate and with the £50,000 property
income, he will not have any personal allowance remaining.

• Therefore, the husband and wife should make a declaration so that the husband is
able to use his personal allowance as well.

• A declaration should be made to transfer 100% of the property income to the wife,
so that the both individuals can utilise their personal allowances fully.

• This will save them tax at £12,570 x 40% = £5,028 (At ATX - UK this kind of short
cut calculation is expected. The long version is shown below for completeness)

204
After declaration:

Husband 

Salary £100,000

Property income  £Nil

Total income  £100,000

P.A. (12,570)

Taxable income  £87,430

Wife
Property income £100,000

Total income £100,000


P.A.    (£12,570)
Taxable income  £87,430

Income tax liability (same calculation for both husband and wife as they both now
have income of £100,000 less the personal allowance)

Income tax liability

£
£37,700 * 20% =  7,540
(£87,430 - £37,700) * 40% = 19,892
Total 27,432

Income tax liability

Total liability of husband and wife before declaration (£52,460 +


£59,946
£7,486)
Total liability of husband and wife after declaration (£27,432 +
(£54,864)
£27,432)

Tax saving £5,082

205
Note:

A joint bank account will always be taxed 50:50 regardless of who contributions what
amount

If shares are owned, dividends are always divided according to the exact proportion to
which each is actually entitled, it is never assumed that it is in equal proportions.


206
Syllabus: A1eii) Recognise the tax treatment of savings income paid net of tax

Savings income paid net of tax

Although most interest is now paid gross, companies are still required to deduct 20%
income tax from interest paid to individuals (unless the interest is in respect of a quoted
Eurobond). 

Interest paid net will need to be grossed up by 100/80 for inclusion in the income tax
computation and there will then be a tax credit equal to the tax deducted. 

This credit is deducted from the income tax liability (together with any PAYE) in arriving at
income tax payable.

Illustration

A Ltd. pays Bob £16,000 interest.

Bob also has salary income of £60,000. (PAYE £13,200)

What is Bob’s income tax payable?

Solution

Non savings:

Salary £60,000

Less P.A. (£12,570)

Taxable income £47,430

Savings:

Interest Income (£16,000 * 100/80) = £20,000

Less: (NRB £500)

Taxable income: £19,500

Income tax payable:

Income tax Liability (Salary) (£37,700 20%) + (£9,730 *40%) = £11,432



Income tax liability (Interest) £19,500*40% = £7,800

Less Tax credits:



PAYE (£13,200)

TDS (£4,000)

Income tax payable £2,032


207
Syllabus: A1eiii) Income from trusts and settlements: Understand the income tax position of
trust beneficiaries

Income tax position of trusts

What is a trust?

A trust is an arrangement whereby:

1. Property is transferred by a settlor

2. To the trustees

3. To be held for the benefit of one or more specified beneficiaries

4. On specified terms in the trust deed

5. Therefore:

Settlor --> Property passes into a trust -->  Trustees are given the legal title to the property

What is interest in possession?

IIP can be the legal right to receive income generated by the trust assets and/or use the
trust asset or live in a property owned by a trust.

Types of trusts

1. Discretionary trusts

2. Interest in possession trusts

Discretionary trusts
• No interest in possession exists

• The beneficiaries have no legal right to benefit from the income or capital of the trust

• The trustees decide how the trust assets are invested and managed

208
• Any distribution of income or capital out of the trust is at the complete discretion of
the trustees

Interest in possession trust


• Interest in possession exists

• The beneficiary is known as the life tenant

• The life tenant has a legal right to benefit from the income of the trust

• The trustees will distribute the life tenant’s full entitlement every year

Income tax position of trust beneficiaries

The trust is a separate legal entity for income tax purposes. 

The body of trustees is a separate taxable person. 

The trustees are subject to income tax on the income arising in respect of trust assets each
tax year and they distribute income to the beneficiaries. 

(You will not have to calculate IT payable by trustees).

The taxation of trust income

• The trustees account for income tax on the receipt of income by the trust each tax
year under self assessment.

• Trustees are taxed at different rates depending on the type of trust.

• Trustees distribute income to the beneficiaries according to the terms of the trust.

Interest in possession trusts


• The life tenant of and IIP trust must be distributed his full entitlement to income each
tax year.

• The life tenant is assessed in the tax year of entitlement (not receipt).

• The income of an IIP trust is received by the beneficiary net of 20% tax.

209
Discretionary trusts

How are the beneficiaries taxed?

They are taxed on the gross trust income in their personal income tax computations and
they can deduct from their income tax liability, any tax deducted at source by

• The beneficiary of a discretionary trust only received income at the discretion of the
trustees.

• Any income distributed from a discretionary trust is assessed on the beneficiary in


the tax year of receipt.

• Discretionary trust income is always deemed to be received by the beneficiary net of


45% tax.

Illustration

John receives £10,000 income from a beneficiary trust.

He is an additional rate taxpayer (45%).

How much income tax will John have to pay on this trust income?

• Solution

Income tax computation

£10,000 * 100/55 = £18,182

I.T. liability £18,182*45%  = £8,181



Less Tax credit (45%) (£8,181)

I.T. payable £Nil

210
Illustration

Jake has put a house and some cash into an I.I.P. trust.

His wife is the beneficiary. She is a higher rate taxpayer.

She lives in the house and the cash has been invested in shares which generate dividends
of £5,000/year. 

What amount of income tax is payable by his wife on the dividends?

• Solution

As she is the life tenant, she will be taxed on the dividend fully each year. 

Income tax computation



Dividend £5,000

Tax

£2,000 * 0% = 0 (Dividend NRB)

£3,000 * 33.75% = 1013

211
Syllabus A1f. Income tax computation and income tax
liability

Syllabus: A1fi) Understand the allocation of the personal allowance to different categories of
income.

Allocation of the personal allowance

Which income should the P.A. be given to?

The personal allowance to be offset against income in the most tax-beneficial manner.

This may require the personal allowance to be offset against dividend income before it is
offset against savings income. 

This will be easier to understand with the illustrations below!

Illustration

Able has pension income of £8,000, savings income of £4,500 and dividend income of
£9,000. 

What is his income tax liability?

• Solution

Pension income £8,000



Savings income £4,500

Dividend income £9,000

Total income £21,500

Less: P.A. (£12,570)

Taxable income £8,930

212
Pension income

£8,000 covered by the P.A.

Savings income

£4,500 covered by the 0% starting rate 

Dividend income

£4,570 covered by the P.A.

£2,000 at 0% (Dividend NRB)

£2,430 * 8.75% = £213

Income tax liability £213

• Note

The personal allowance has been offset against the dividend income in priority to the
savings income in order to maximise the tax saved.

If the personal allowance had been offset against the savings income, there would have
been an additional £4,570 of dividend income which would have been subject to income
tax at 8.75%.

There is a 0% starting rate for savings income which falls within the first £5,000 of taxable
income, and, possibly, a savings income nil rate band of either £500 or £1,000. These must
be taken advantage of if at all possible.

Accordingly, it will not be tax-efficient for savings income to be relieved by the personal
allowance if it would otherwise be taxable at 0%.

213
Syllabus: A1fii) Advise on the income tax position of the income of minor children

Tax position of minor children

Do minor children pay income tax?

Children are taxed on their own income from birth, this means that they will have their own
income tax computation. 

They will have their own personal allowance and reliefs. If needed, tax returns will be
completed by the parent/guardian.

For minor children, if income comes from a source set up by a parent and exceeds £100
p.a. that income is taxed on the parent, and not on the child, provided the child is < 18
years and unmarried. 

The capital can be provided by setting up a formal trust or can be a gift of money, for
example, opening a bank account in the child’s name. 

A child receiving investment income from capital that has been provided by the parent, this
income will be treated as belonging to the parent.

Illustration

Jake sets up a bank account in Jim's name (Jim is his two year old son).

Jake provides the capital and Jim received £2,000 interest income per annum. 

Will Jake or Jim be taxed on this income?

Solution

Jake will be taxed on this income because:

1) He is the parent and the source of the capital that provides the income 

2) It is more than £100 per annum

3) Jim is under 18 years of age

The income will be covered by the personal allowance, so there will be no taxable income,
but his income tax return must be completed.


214
Syllabus A1g. Exemptions and Reliefs for I.T.

Syllabus: A1g) The use of exemptions and reliefs in deferring and minimising income tax
liabilities: i) Understand and apply the rules relating to investments in the seed enterprise
investment scheme and the enterprise

investment scheme 
C1. Identify and advise on the types of investment and other expenditure that will result in a
reduction in tax liabilities for an individual and/or a business.
C2. Advise on legitimate tax planning measures, by which the tax liabilities arising from a
particular situation or course of action can be mitigated.
C3. Advise on the appropriateness of such investment, expenditure or measures given a
particular taxpayer’s circumstances or stated

objectives.
C4. Advise on the mitigation of tax in the manner recommended by reference to numerical
analysis and/or reasoned argument.

EIS and SEIS Investment Relief

What are EIS and SEIS Investment Relief?

If a qualifying individual invests in qualifying unquoted EIS/SEIS company shares, the


amount invested can be used to reduce the individual's income tax liability.

There are conditions to be a qualifying investor, qualifying company and for the maximum
amount of relief available.

Note: the conditions that a company must meet in order to qualify as an EIS / SEIS
company are not examinable.

215
EIS Relief

Conditions to be a qualifying investor

1. Individual at least 18 years and must subscribe for newly issued shares.

2. Must not own shares before the investment.

3. Must own less than 30% of the shares.

4. Must not be an employee of the company before making the investment, but can
become a paid director of the company after making the investment.

Income tax implications

1. In the tax year in which investor subscribes for the shares he can claim EIS relief at
30%. 

This means that he can reduce his income tax liability by: (Amount invested *30%). EIS
relief cannot create an Income Tax repayment it can only bring the bill down to £0.

The maximum relief that can be given is £300,000, therefore if more than £1,000,000 is
invested, only £300,000 EIS relief can be claimed.

2. Dividends received by investors from the EIS company are subject to income tax at
8.75%, 33.75% and 39.35%.

3. This relief can be claimed in the current or previous tax year.

4. If the EIS shares are sold within 3 years of ownership, the relief given will need to be
paid back to HMRC.

Illustration

Tom is not an employee of A Ltd (an unquoted company) and owns < 30% of shares in A
Ltd. 

Tom subscribes for 10,000 new ordinary shares in A Ltd for £30,000 on 30 June 2022. 

A Ltd. qualifies as an EIS company.

How much can Tom reduce his income tax liability by?

Solution

Tom can reduce his income tax in the tax year in which he buys the Enterprise Investment
Scheme shares (or the prior year) by EIS relief at 30%. 

Tom can reduce his income tax by £9,000 (30% * 30,000) in 2022/23 or 2021/22.

216
SEIS Relief

An SEIS is similar to the EIS but is intended to promote investment in smaller early stage
trading companies.

Conditions for investors in SEIS Companies

1. Individual at least 18 years and must subscribe for newly issued shares.

2. Must not own shares before the investment.

3. Must own less than 30% of the shares.

4. Must not be an employee of the SEIS company before making the investment but
can become a paid director of the company after making the investment.

Income tax implications for SEIS investment

• In the tax year in which investor subscribes for the shares he can claim SEIS relief at
50% (tax reducer).

• Dividends received by investors from the SEIS company are subject to income tax
at 8.75%, 33.75% and 39.35%.

• If investor sells the shares within three years income tax relief is withdrawn - an
adjustment will need to be made in the assessment for the year in which the relief
was originally claimed.

• The upper limit on SEIS relief is £50,000 (50% x 100,000) each tax year.

• This relief can be claimed in the current or previous tax year.

Illustration

Tommy is not an employee of A Ltd (an unquoted company) and owns no shares in A Ltd. 

Tommy subscribes for 10,000 new ordinary shares in A Ltd for £30,000 on 30 June 2022
under the SEIS.

Solution

Tommy can reduce his income tax in the tax year he buys the SEIS shares by SEIS relief at
50%. 

Tommy can reduce his income tax by £15,000 maximum (50% x 30,000) in 2022/23 and /or
2021/22. 

Tommy must repay the income tax saving to HMRC is he sells the shares within three years.


217
Syllabus: A1gii) Understand and apply the rules relating to investments in venture capital trusts

VCT Investment Relief

What is VCT Investment Relief?

If an individual invests in Venture Capital Trust shares, they can reduce their tax liability by a
% of their investment. 

There are conditions to be a qualifying company and on the reduction of the I.T. Liability

Note: the conditions that a company must meet in order to qualify as a VCT company are
not examinable.

Income tax implications


• In the tax year in which investor subscribes for the new issue of shares he can claim
to reduce his income tax by VCT relief at 30% (tax reducer). 

• The maximum VCT relief available is £60,000 as a tax deduction, therefore even if
more than (£200,000*30%=£60,000) was invested - the maximum of £60,000 would
be relief.

• Dividends received by investor from VCT are exempt from income tax if they relate
to shares acquired within the £200,000 permitted maximum.

• If investor sells the shares within five years he must repay this VCT relief to HMRC.

218
Illustration

Tom subscribes for 10,000 new ordinary shares in a venture capital trust (VCT) on 30 June
2022 for £30,000. 

How much can he reduce his income tax liability by?

• Solution

Tom can reduce his income tax in 2022/23 (the tax year he buys the VCT shares) by VCT
relief at 30%. 

Tom can reduce his income tax by £9,000 (30% * 30,000) in 2022/23. 

Tom must repay the income tax saving to HMRC if he sells the VCT shares within five years.

219
Syllabus A2: Chargeable Gains For Individuals

Syllabus A2a. TX - UK Recap The scope of the taxation of


capital gains

The contents of the Paper TX - UK study guide for chargeable gains for individuals under
headings:
- The scope of the taxation of capital gains

The scope of capital gains tax

You should pay CGT on:

1. The sale or gift of the whole or part of an asset.

For example out of a 10 hectare plot of freehold land, 5 hectares are sold.

Capital gains tax will be paid on the disposal of the 5 hectare part.

2. The loss or destruction of an asset.

For example a painting costing £5,000 was destroyed in a fire.

This destruction will be considered to be a capital disposal.

The disposal proceeds will be Nil and therefore a capital loss of (£5,000) will be realised
on destruction.

3. Compensation in connection with an asset.

For example a painting costing £5,000 was destroyed completely in a fire.

The painting was insured and insurance proceeds of £10,000 were received because of
the loss of the painting.

220
Capital gains tax should be paid on (£10,000-£5,000) = £5,000.

Chargeable assets

All assets are chargeable unless specifically exempted.

This list is provided in C1b.

Chargeable person

An individual who is resident in the UK is a chargeable person and is therefore subject


to UK CGT on their worldwide assets.

Note the differences between companies and individuals

1. Companies pay corporation tax on their capital gains whereas individuals pay capital
gains tax.

2. Companies have an indexation allowance up to December 2017 which allows for the
adjustment of the cost of an asset for inflation, whereas individuals have an annual
exemption.

3. Companies can use rollover and holdover relief with respect to their chargeable
gains whereas individuals have a much wider array of reliefs available to them.

4. Companies can be part of 75% gains groups whereas individuals cannot.

5. Loss relief is dealt with differently between both companies and individuals.

221
Assets which are exempt

Exempt assets include:

• Motor cars suitable for private use

• Animals (wasting chattels that do not qualify for capital allowances)

• Debtors

• Cash

• Chattels bought and sold for less than £6,000

• Corporate bonds

• Government securities

• Trading stock

• Shares in individual savings accounts ISA

• Shares in a VCT

• Foreign currency for private use (Cash)

• Works of art given for national use

• Damages for personal injury

• Life insurance policies (Cash)

• National Savings and Investment certificates

222
Syllabus A2a. TX - UK Recap The basic principles of
computing gains and losses

The contents of the Paper TX - UK study guide for chargeable gains for individuals under
headings:

- The basic principles of computing gains and losses

The treatment of capital gains

The capital gain is calculated as follows:

Disposal proceeds X

Less: Incidental cost of disposal (X)

Less: Acquisition cost (X)

Capital Gain (Chargeable gain) / (Capital loss) X / (X)

Incidental costs of disposal may be:

Valuation fees

Estate agency fee

Legal costs

Advertising costs

Allowable costs include:

Original cost of acquisition

Incidental costs of acquisition

Similar to incidental costs of disposal

223
Capital expenditure incurred in enhancing the asset

Enhancement expenditure is capital expenditure which enhances the value of the asset.

Excluding:

- Cost of repairs

- Cost of insurance

- Any expenditure met by public funds (e.g. council grants)

Illustration

Mia bought a piece of land as an investment for £30,000.

The legal costs of purchase were £300.

She spent £1,200 draining the land.

Mia sold the land on 31 December 2022 for £40,000.

She incurred estate agency fees of £500 and legal costs of £200 on the sale.

Calculate Mia's gain on sale.

Solution:

Proceeds of sale £40,000

Less costs of disposal £(500 + 200) = (£700)

Less costs of acquisition £(30,300)

Less enhancement expenditure £(1,200)

Gain £7,800


224
Illustration:

A chargeable asset was disposed of for £10,000.

On disposal, £1,000 of legal cost was incurred.

The asset originally cost £5,000.

Which of these costs will be classified as incidental cost to disposal?

Which of these costs will be classified as acquisition cost?

Solution:

The legal cost of £1,000 is classified as the incidental cost to disposal.

This is because it has been incurred because of the disposal.


225
Incidental cost to disposal can also include advertising cost and agency fees.

• The original cost of the asset of £5,000 is classified as the acquisition cost.

This is because it is the amount spent to acquire the asset originally.

Capital gain calculation:

Disposal proceeds  £10,000

Incidental cost to disposal (legal cost) (£1,000)

Acquisition cost  (£5,000)

Capital gain (Chargeable gain) £4,000

Capital gains tax is calculated as follows:

Capital Gains (Chargeable gain) X 

Less: Annual exempt amount (12,300)

Taxable Gains X

CGT × 10%, 20% or 18%, 28% X

Annual exempt amount

This is an amount of capital gain that will not be subject to capital gains tax.

It is similar to the personal allowance that individuals get.

The amount is £12,300 for 2022/23.

If this amount is not used in a particular tax year, then it is wasted.

226
The rates of capital gains tax are:

• Rate 10%

After considering a person's taxable income, any remaining amount falling within the
basic rate band is charged at 10%

• Rate 20%

Once the entire basic rate band has been used, then a rate of 20% is applied.

• For a residential property only

The same treatment applies as explained above, except that the 10% rate is replaced
with 18% and the 20% rate is replaced with 28%.

• Rate 10%

This rate is used for capital gains that qualify for entrepreneur's relief/business asset
disposal relief.

There are conditions that need to be met in order to be able to use this rate.

They are discussed in Topic Entrepreneur's relief/Business asset disposal relief.

Illustration 1

Peter sold a capital asset and this resulted in a taxable gain of £40,000.

Peter has taxable income of £20,000.  (Basic rate band: £37,700)


Calculate Peter's capital gain tax.

Solution:

Taxable gain 40,000

Basic band remaining (£37,700 - £20,000) = £17,700

Capital Gains Tax (£17,700 * 10%) = £1,770

(£40,000 - £17,700) * 20% = £4,460

Total capital gains tax payable = 1,770 + 4,460 = £6,230

227
Illustration 2

Peter sold a residential property and this resulted in a chargeable gain of £40,000.

Peter has taxable income of £20,000.  (Basic rate band: £37,700)

Required:

Calculate Peter's capital gain tax.

Solution:

Chargeable gain 40,000


Annual exempt amount (12,300)
Taxable gain 27,700
Basic rate band remaining (£37,700 - £20,000) = £17,700
Capital gains tax (£17,700 * 18%) = £3,186
(£27,700 - £17,700) * 28% = £2,800

Total capital gains tax payable = 3,186 + 2,800 = £5,986

Illustration 3

Katherine has a trading profit of £35,600 in 2022/23. (Basic rate band: £37,700)

Additionally, she sold a capital asset giving a rise to a taxable gain of £15,000.

• What is her capital gains tax payable for 2022/23?

Note A taxable gain is after the deduction of the annual exempt amount.

Solution:

Trading income £35,600

Personal allowance  (£12,570)

Taxable income £23,030

Basic rate band left £37,700 - £23,030 = £14,670

Capital gains tax £14,670 * 10% = £1,467

(£15,000 - £14,670) * 20% = £66

Total capital gains tax payable 1,467 + 66 = £1,533

228
Illustration 4

What if Katherine had capital gain (Chargeable gain) of £15,000?

Solution:

Basic rate band left = (£37,700 - £23,030) = £14,670

Chargeable gain £15,000

Less:

A/E (£12,300)

Taxable gain £2,700

CGT £2,700 * 10% = £270

The rate used is 10% because the taxable gain of £2,700 fell entirely into the remaining
basic rate band of £14,670 that remained.

When is capital gains tax payable?

Capital gains tax is payable on the 31 January following the tax year.

For 22/23, it is payable on 31/1/24.

Payment on account when a residential property is disposed:

When a residential property is disposed of, a payment on account of CGT must be made
within 30 days of the disposal for this gain.

Calculation of payment on account:

Residential property capital gain

(Current year capital losses before sale of residential property)

(Brought forward capital losses)

(Annual exemption)

= Taxable gain for residential property

Tax on this taxable gain will be paid at 18% or 28% (Depending on whether the individual is
a basic or higher rate taxpayer for the year).

Calculation of final capital gains tax payable on 31/1/24:

CGT Liability

229
(POA)

= x (payable) / (x) (refund from HMRC)

This POA on the disposal of a residential property has nothing to do with the payments on
account that are made for income tax.

Illustration

Anaya is a higher rate taxpayer for 2022/23.

She has the following chargeable gains/loses for the year:

11/4/2022 Capital loss on disposal of shares (£5,000)

1/6/2022 Capital gain on disposal of shares £28,400

31/8/2022 Capital gain on disposal of residential property £90,000

11/3/2023 Capital loss on disposal of shares (£15,000)

What is the payment on account for the residential property and the final capital gains tax
payment?

Solution

POA:

Residential capital gain £90,000

Less current year capital loss (£5,000)

Less brought forward capital loss (£0)

Less annual exemption (£12,300)

Taxable gain = £72,700 x 28% (Higher rate taxpayer)

= £20,356 is the POA to be made on 30/9/2022 (30 days from the sale)

On 31/1/24 (Final payment)

Residential property gain £90,000

Less current year losses (£20,000)

Less annual exemption (£12,300)

Taxable gain £57,700

CGT at 28% = £16,156

230
Gain on disposal of shares £28,400

At 20% = £5,680

Total CGT payable = £16,156 + £5,680 = £21,836

Less POA (£20,356)

CGT payable on 31/1/24 = £1,480

Note that the capital losses and annual exemption are given to the residential property gain
first as that gain pays tax at 28%, whereas the other gain pays tax at 20%

231
The treatment of capital losses

Capital losses

1. Current year capital losses are set against current year capital gains in the same tax
year.

2. The set off is made to the maximum possible extent and cannot be restricted to
avoid wasting the annual exemption.

3. If there are insufficient gains to set off the capital losses in the year they arise, the
unrelieved capital losses may be carried forward.

4. The capital losses brought forward are offset after the deduction of the annual
exemption and therefore do not waste the annual exemption.

5. Any unrelieved capital losses brought forward are carried forward to the next year to
be set off against capital gains.

Illustration:

Fiona and Jane made capital gains and capital losses for the years 2021/22 and
2022/23 as set out below:

Fiona Jane

2021/22
Capital gains 15,000 7,000
Capital losses 10,000 10,000

2022/23
Capital gains 17,500 15,000
Capital losses 5,200 2,000

Calculate the taxable gains for Fiona and Jane for both 2021/22 and 2022/23 and the
amount of any losses carried forward at the end of 2022/23.

Solution:

Fiona 21/22 22/23

Capital gains £15,000 £17,500

Capital losses (£10,000) (£5,200)

232
Net capital
£5,000 £12,300
gains/loss

Annual
(£5,000) (£12,300)
exemption

Taxable gain Nil £Nil

Jane 21/22 22/23

Capital gains £7,000 15,000

Capital losses (£10,000) (2,000)

(£3,000) will be carried


Net capital
forward against c. gains £13,000
gains/loss
of 21/22

Annual
Wasted -£ 12,300
exemption

Capital losses
brought forward (£700)

Taxable gain Nil Nil

Explanation:

Jane had a loss of 3,000 in 21/22 which was carried forward to 22/23.

After deducting the annual exemption in 22/23, 700 of the loss brought forward was
used, therefore 2,300 is carried forward to 23/24.

Note that the current year losses must be set off to a maximum without any restriction
and thus wastage of the annual exemption.

However, capital losses brought forward will only be offset if a gain remains after
deduction of the annual exemption.


233
Allowable expenditure on a part disposal

Allowable cost for part disposal

A part disposal

If an individual owns a chargeable asset and disposes of only part of it, a capital gain
will arise.

For example, if an individual owns a large piece of land and decides to only dispose of a
part of it, this is known as a part disposal.

Or, if an individual owns 5 antique vases, bought as a set, then disposing of only 2 from
the set is considered to be a part disposal of the whole set.

A cost from the entire asset cost must be given to the part of the asset being disposed
of.

This is known as the allowable cost. This allowable cost is then deducted from the
proceeds received for the part disposal, to arrive at a capital gain.

How much cost can you deduct from the disposal proceeds?

= Original purchase cost * [A / (A+B)]

Where:

A – Disposal proceeds received



B – Market value of the remainder of the asset (Given in question)

Illustration:

Peter buys a house for £26,000 in October, 1997.

He has never lived in the house as his main residence.

He sells the garden for £100,000 in August 2022, incurring selling costs of £1,000.

The value of the remaining house in August 2022 is £160,000.

• What capital gain will arise on this sale?

Solution:

Allowable cost for part sold:

• Original purchase cost = £26,000

234
Proceeds received (A) = £100,000

Market value for the remainder of the house (B) = £160,000

• Original purchase cost * [A / (A+B)]

£26,000 * [£100,000 / (£100,000 + £160,000)] = £10,000

• Notice that only £10,000 is considered to be the allowable cost.

This is because, this is the amount of cost that relates only to the part of the asset being
disposed of.

The other £16,000 is known as the base cost.

It relates to cost that will be used to calculate the capital gain when the remainder of the
asset is disposed of.

Chargeable gain on sale:

Disposal proceeds £100,000

Incidental cost to sell (£1,000)

Net proceeds £99,000

Allowable cost (£10,000)

Chargeable gain £89,000

Less annual exemption (£12,300)

Taxable gain £76,700

Illustration:

The base cost of remaining house is: Original purchase cost – Allowable costs used.

£26,000 - £10,000 = £16,000

If the remainder of the house was disposed of for £120,000 after one year, what
taxable gain would arise then?

Disposal proceeds £120,000

Allowable cost (£16,000)

Chargeable gain £104,000

Less annual exemption (£12,300)

Taxable gain £91,700


235
Syllabus A2a. TX - UK Recap Gains and losses on the
disposal of property

The contents of the Paper TX - UK study guide for chargeable gains for individuals under
headings:

- Gains and losses on the disposal of movable and immovable property.

Chattels and wasting assets

Exempt chattels and wasting assets.

Chattels

A chattel is a piece of tangible, movable property (something that you can touch and
move).

Your personal possessions will normally be chattels.

For example, items of household furniture, paintings, cars, items of plant and machinery
fixed to a building.

Some chattels are exempt and some are chargeable to capital gains tax.

A wasting chattel is exempt from capital gains tax.

A wasting chattel is one with a life of 50 years or less.  

For example, racehorses, boats.

Exception to the exemption of wasting chattels:

Plant and machinery (with a life of less than 50 years) on which capital allowances have
been claimed are treated as non wasting chattels.

A capital gain needs to be calculated on their disposal, but a capital loss will not be
allowable on their disposal. It is possible that they are exempt under the non-wasting
chattel exemption (being bought and sold for less than or equal to £6,000).

236
Non wasting chattels:

Non wasting chattels with a life of more than 50 years are chargeable to capital gains
tax in the usual way.

However, if both the proceeds and the cost are less than £6,000, the chattel will be
exempt from capital gains tax.

Note: the detailed calculations for chattels where the cost or proceeds are less than
£6,000 are not examinable in ATX.

Non wasting chattel capital gains calculation:

Cost Proceeds Treatment

<=£6,000 <=£6,000 Exempt

Normal calculation but the gain is restricted to


<=£6,000 >£6,000
5/3*(Gross proceeds-£6,000)

>£6,000 <£6,000 Deemed gross proceeds = £6,000

>£6,000 >£6,000 Normal calculation

Illustration:

Maria sold the following assets in December:

1. An antique table which had cost £3,000 and was sold for £5,000

2. A painting which had cost £2,000 and was sold for £10,000

3. An antique vase which had cost £8,000 and was sold for £3,000

4. A set of china which had cost £7,000 and was sold for £8,000.

Solution:

1. The table is exempt as it was bought and sold for less than £6,000

237
2. The painting:

Proceeds £10,000

Cost (£2,000)

Capital gain £8,000

Maximum gain assessable = 5/3*(£10,000-£6,000) = £6,667

3. The vase:

Deemed proceeds £6,000

Cost (£8,000)

Capital loss (£2,000)

The proceeds are deemed to be £6,000 as it was bought for more than £6,000 and
sold for less than £6,000.

4. The china:

Proceeds £8,000

Cost (£7,000)

Capital gain £1,000

This is the normal calculation as the set of china had been bought and sold for more
than £6,000.


238
Principal private residence relief

Principal Private Residence Relief

What is it?

Simply, don't pay any tax if you sell your house.

• But you will have to if you didn't live there all the time or used it for business
purposes.

How much capital gain is exempt with PPR Relief?

• FULL exemption

If you occupied the property throughout the entire period of ownership.

• Partial exemption

If you stay there only for part of the period.

This is calculated as:

Capital gain * Period of occupation/Period of ownership

There are however periods of absence which are deemed to be full occupation

1. Last 9 months - if the property was the individuals main residence at some point
in time.

For example An individual purchased a house on 31/03/2006, he lived in it for 2 months


and then travelled the world, living in hotels until he sold it on 31/03/2023.

The last 9 months of ownership of the house, from 01/07/2022-31/03/2023 will be


considered to be occupied by the individual, even though he did not live there at the
time.

2. Any periods during which the individual was required by his employment to live
abroad.

The person must come back to live in the house after this period in order for this time to
be considered to be deemed occupation.

For example An individual purchased a house in London on 31/03/2006, he lived in it


for 2 months and then moved to Barbados for employment for 4 years, he then returned
to live in the house until he sold it on 31/03/2023.

239
For capital gains tax purposes the 4 years during which the individual lived abroad will
be considered to be deemed occupation by the individual.

This is because the reason for living abroad was employment purposes and he moved
back to the house when he returned.

3. Any period up to four years during which the individual is required to live
elsewhere in the UK due to employment.

The person must come back to live in the house after this period in order for this time to
be considered to be deemed occupation.

For example An individual purchased a house in London on 31/03/2006, he lived in it


for 2 months and then moved to Newcastle for employment for 4 years, he then
returned to live in the house until he sold it on 31/03/2023.

For capital gains tax purposes the 4 years during which the individual lived elsewhere in
the UK will be considered to be deemed occupation by the individual.

This is because the reason for living elsewhere in the UK was employment purposes
and it was for 4 years only, and he moved back to the house when he returned.

4. Up to three years for any reason.

The person must come back to live in the house after this period in order for this time to
be considered to be deemed occupation.

For example An individual purchased a house on 31/03/2006, he lived in it for 2 months


and then traveled the world until 31/03/2009, he then moved back to the house and
lived in it until he sold it on 31/03/2023.

For capital gains tax purposes the 3 years during which the individual was travelling will
be considered to be deemed occupation by the individual.

This is because the reason up to 3 years for any reason is allowable and he lived in the
house when he returned.

For points 2 and 3, where an individual is not living in their main residence due to work,
if they do not return to their house to live in it after because of another work
engagement immediately after the first one, this will still be considered deemed
occupation.

240
Illustration:

On 30 September 2022, Jane sold a house for £400,000.

The house had been purchased on 1 October 2002 for £167,500.

Jane occupied the house as her main residence from the date of purchase until 31
March 2006.

The house was then unoccupied between 1 April 2006 and 31 December 2009 due to
Jane moving to Chicago for work.

From 1 January 2010 until 31 December 2016, Jane again occupied the house as her
main residence.

The house was then unoccupied until it was sold on 30 September 2022.

What capital gain arises on this sale?

Solution:

Principal private residence exemption £174,375 (232,500 x 180/240).

The total period of ownership of the house is 240 months (180 + 60), of which 180
months qualify for exemption as follows because the unoccupied period from 1 January
2017 to 31 December 2021 is not a period of deemed occupation because it was not
followed by a period of actual occupation.

Disposal proceeds 400,000

Acquisition cost (167,500)

232,500

PPR Exemption  (174,375)

Capital gain 58,125

Exempt months Chargeable months 

1 October 2002 to



42
31 March 2006 (occupied)

1 April 2006 to



31 December 2009
 45
(working overseas)

241
1 January 2010 to

31 December 2016
 84
(occupied)

1 January 2017 to



31 December 2021 60
(unoccupied)

1 January 2022 to



30 September 2022
 9
(final 9 months)

180 60

Illustration:

Dolly bought a house on 1 April 1993 for £10,000.

She lived in it for 3 months.

Then she worked abroad for 24 months.

She came back and lived in the house for another 174 months.

Then she lived and worked elsewhere in UK for 48 months.

Dolly never returned to the house and it was sold 108 months later in December 2022
for £150,000.

Calculate the chargeable gain arising.

Solution:

The total period of ownership of the house is 357 months, out of which 210 months
qualify for the PPR exemption.

The 4 years of working elsewhere in the UK cannot be classified as deemed occupation


because she never returned to the house to live in it after that.

The exemption is 210/357 * £140,000 = £82,353

Disposal proceeds 150,000

Less cost (10,000)

Capital Gain 140,000

242
Less PPR relief (W1) (82,353)

Chargeable Gain 57,647

Actual & Deemed


Absent
(W1) Occupation
(months)
(months)

Actual 3

Working overseas 24

Actual 174

4 years work in UK 48

Living elsewhere 108 - 9 = 99

Last 9 months 9

210 147

Business use

Where part of a residence is used exclusively for business purposes throughout the
period of ownership, the gain in relation to that part is not covered by relief.

Illustration:

On 30 September 2022, Henry sold a house for £155,000.

The house had been purchased on 1 October 2013 for £100,000.

Throughout the period of ownership, the house was occupied by Henry as his main
residence, but one of the house’s five rooms was always used as Henry's office
premises.

What capital gain arises on this disposal?

243
Solution:

The principal private residence exemption is restricted to £44,000 (55,000 x 4/5).

This is because 1 out of 5 rooms of the house has always been used only for business
purposes.

The capital gain arising on the sale is £11,000

Disposal proceeds 155,000

Acquisition cost (100,000)

PPR exemption (44,000)

Capital gain 11,000

Letting relief

If an individual lives in a property as their main residence and while living in the property
lets part of the residence for residential purposes;

on the disposal of this property, in addition to claiming PPR relief, the letting relief is
also available to reduce the capital gain.

This relief is the lower of:

• PPR relief given

• £40,000

• Gain attributable to letting

Illustration

Susan purchased a house for £150,000 in 2000, selling it for £300,000 in 2022.

Throughout that time, she lived in the house as her only residence but let out two
spare rooms amounting to 25% of the property to tenants who had exclusive use
of their rooms.

What is the chargeable gain after PPR and letting relief?

244
Solution:

Sale proceeds £300,000

Less cost (£150,000)

Chargeable gain £150,000

PPR (75% x £150,000) (£112,500)

Letting (25% x £150,000) (£37,500) (W1)

Chargeable gain Nil

W1:

Letting relief is due on the lower of:

1) The amount of PPR - £112,500

2) £40,000

3) Gain attributable to letting £37,500

245
Syllabus A2a. TX - UK Recap: Gains and losses on the
disposal of shares and securities

The contents of the Paper TX - UK study guide for chargeable gains for individuals under
headings:

- Gains and losses on the disposal of shares and securities

Gift of quoted shares

If you give away shares as a gift

You will have to pay Capital gains tax on it

Step by step approach:

1. Step 1

Value the shares using:

Lower quoted price + 1/2 (higher quoted price - lower quoted price)

Note: the share valuation rules are different for IHT so be careful not to confuse them

2. Step 2

Calculated Disposal proceeds 

= Number of shares given * value per share (step 1)

246
Illustration:

Megha gifted 1,000 shares in N plc when they were quoted at 400-408 pence per share,
with marked bargains on that day of 398p, 402p, and 407p.

• Calculate the value to be used for capital gains disposal proceeds.

Solution:

1. Step 1 - Value the shares




400 + 1/2 (408 - 400) = 404p

Note: the marked bargains are not relevant for CGT life gifts but they would be relevant
for IHT

2. Step 2 - Calculated Disposal proceeds

= Number of shares given * value per share (step 1)



= 1,000 shares * £4.04 = £4,040

247
Share matching rules for individuals

Share matching rules

The problem

When shares are disposed of, a problem arises in finding their allowable cost, if the
shares were acquired over a long period of time. 


The solution

To make this simpler, HMRC uses a set of rules to determine the acquisition date and
cost of the shares being disposed of.

These rules are called the matching rules.

Disposals of shares are matched with acquisitions in the following order:

1. Shares acquired on the same day of disposal.

2. Shares acquired in the next 30 days following the disposal (FIFO).

3. Shares from the share pool.

This would be much easier to understand if we did an example!

Illustration:

Benazir owns shares in L plc.

She acquired 1,500 shares in the company on 31/05/2020 for £20,000, and 500 shares
on 30/06/2021 for £10,000.

On 07/03/2023 Benazir bought a further 200 shares in L plc. for £4,000.

• Benazir sold 1,000 shares in L. plc for £25,000 on 28/02/2023.

• Calculate Benazir’s capital gain on the disposal of the shares in February 2023.

Solution:

We need to dispose of 1,000 shares.

Let us apply our matching rules to see which shares we are disposing of.

248
FIRST MATCH –
SECOND MATCH – 30 days THIRD MATCH –
same day
following disposal acquisition share pool 
acquisition

800 shares
07/03/2023 – 200 shares for
None. needed from
£4,000.
share pool.

Share pool:

Description Number Cost

31/05/2020 purchase 1,500 20,000

30/06/2021 purchase 500 £10,000

2,000
Total £30,000
shares
(800
Disposal from share pool (800/2000) * £30,000 = (£12,000)
shares)
1,200
Remaining in share pool £18,000
shares

Specially note how the cost of the shares from the share pool is calculated.

(No. of shares to be disposed from pool/Total shares in pool) * Total cost in pool =
Average cost that we apply to our disposal

Calculating capital gain:

Disposal proceeds £25,000

Acquisition cost:

07/03/23 (£4,000)

Share pool (£12,000)

Capital gain £9,000

• You also might want to try to draw a timeline to ensure that you do not miss any
acquisition dates!

249
Exemptions available for gilt-edged securities and
qualifying corporate bonds

Exemptions

Disposals of gilt edged securities and qualifying corporate bonds are exempt from
capital gains tax.

A qualifying corporate bond is:

• A normal commercial loan

• In sterling

• Cannot be converted into other shares, securities or currencies.

• Issued on or after 13/03/1984

250
Syllabus A2a. TX - UK Recap Entrepreneurs’ relief/
Business Asset Disposal Relief

The contents of the Paper TX - UK study guide for chargeable gains for individuals under
headings:

- The use of exemptions and reliefs in deferring and minimising tax liabilities arising on the
disposal of capital assets

Entrepreneurs' relief/Business Asset Disposal Relief

Entrepreneurs’ relief/Business Asset disposal relief

covers the first £1,000,000 of qualifying chargeable gains that a person makes in their
lifetime.

• It operates by charging CGT at 10% for the disposals on which it is claimed


regardless of an individual’s taxable income.

Conditions to get the relief:

1. The asset must have been owned for at least two years prior to the disposal.

2. The election for the relief must be made by the anniversary of the 31/01 following the
tax year of the disposal.

Therefore, if the tax year of disposal is 22/23, then the election must be made by
31/01/25.

3. It must be a disposal of a qualifying asset.

251
Qualifying assets include:

1. The disposal of a whole business run by a sole trader or by partners in a partnership.

The assets must have been used in the trade to qualify for the relief.

Also, the entire business must be disposed of, if a single trading asset is disposed of it,
it will not qualify for the relief.

2. Individual business assets of the individual’s or partnership’s trading business that


has now ceased.

Note the disposal of assets must take place within three years of cessation of trade.

The difference here is that the entire business is not being sold, it is being shut down.

Therefore, no trading activity will continue and this is why the assets can be disposed of
within 3 years of cessation

3. The disposal of shares in a trading company, where the individual has 5%


shareholding and is also an employee of the company, for 2 years prior to the
disposal.

Entrepreneurs’ relief/Business Asset Disposal relief will be available on the entire


disposal, regardless of whether the trading company owns assets for investment or not.

Illustration:

On 14 October 2022, a shareholder of Numbers Ltd, an unquoted trading company,


sold his entire shareholding in the company.

He had been the advertising director of Numbers Ltd since the company’s incorporation
on 1 December 2021.

He had 40% shares in the company since its incorporation on 1 December 2021.

Will this disposal qualify for entrepreneurs’ relief/business asset disposal relief?

Solution:

This disposal will not qualify for entrepreneurs’ relief/business asset disposal relief
because:

• The shares were owned for less than two years.

252
Illustration:

Sunder disposed of his business to an unconnected person. The business had the
following asset values:

• Goodwill £150,000

• Freehold office £200,000

• Inventory stock £20,000

• Debtors £30,000

• Investment property   £100,000

• Cash £50,000

• Which of the assets will qualify for entrepreneurs’ relief/business asset disposal
relief on disposal of the entire business?

Solution:

The investment property does not qualify for entrepreneurs' relief/business asset
disposal relief as only assets that are used in the trade can qualify.

An investment property is just held for investment, not used in the trade.

Market
Asset Capital gains tax treatment
Values

 Entrepreneurs’ relief/Business asset


Goodwill £150,000
disposal relief available. Taxed at 10%

 Entrepreneurs’ relief/Business asset


Freehold office £200,000
disposal relief available. Taxed at 10%

Inventory stock £20,000 Exempt


Debtors   £30,000 Exempt

Cash £50,000 Exempt

Investment
£100,000 Taxed normally at 10% or 20%.
property

253
Illustration:

In March 2023, Sunder also disposed of a 20% shareholding in Cow Ltd.

He had been an employee of Cow Ltd. since January 2021, when he acquired the
shares.

The gain arising on disposal was £200,000.

• Will this gain be eligible for entrepreneurs’ relief/business asset disposal


relief?

Solution:

Yes it will be.

This is because he has owned the shares and worked in the company for more than two
years.

Illustration:

On 30 October 2022, Bhumi sold a business that she had run as a sole trader since 1
February 2015 to an unconnected person. The disposal resulted in the following
chargeable gains:

Goodwill 150,000

Freehold office building 400,000

Freehold warehouse 180,000

730,000

The warehouse had never been used by Bhumi for business purposes.

Bhumi has taxable income of £6,000 for the tax year 2022/23.

She has unused capital losses of £30,000 brought forward from the tax year 2021/22.

What is Bhumi's capital gains tax liability for the year?

254
Solution:

Gains qualifying for entrepreneurs’ relief/business asset £


disposal relief
Goodwill 150,000
Freehold office building 400,000
550,000

Other gains

Freehold warehouse           180,000


Annual exempt amount (12,300)
167,700

Capital losses brought forward (30,000)


137,700

Capital gains tax:
 55,000


550,000 at 10%
137,700 at 20% 27,540
Tax liability 82,540

Explanation:

• The capital losses and the annual exemption are set against the gains that do not
qualify for entrepreneur’s relief/business asset disposal relief.

• This is because it saves more tax to set the losses and exemptions against gains
that are taxed at a higher rate of 20%.

• £31,700 (37,700 – 6,000) of Mika’s basic rate tax band is unused, but this remaining
band limit is first set against the gains qualifying for entrepreneurs’ relief/business
asset disposal relief of £550,000 even though this has no effect on the 10% tax rate.

• If there is any basic rate band remaining, then it will be used for gains that do not
qualify for entrepreneurs’ relief/business asset disposal relief.

Things to note:

• a) Gains that qualify for entrepreneurs’ relief/business asset disposal relief will take
priority in using up the basic rate band limit first.

Therefore, it is likely that other capital gains will normally fall into the higher band
and pay CGT at 20%.

255
• b) The annual exemption and relief for losses is not automatically given to the gains
which qualify for entrepreneur’s relief/business asset disposal relief.

Therefore 2 separate calculations should be made and gains which do not qualify
should be given the annual exemption and losses carried forward first, in order to
save CGT at a higher rate.

Note: From 6 April 2019, where an unincorporated business has been sold to a
company wholly or partly in exchange for shares, and incorporation relief has been
applied, the period when the individual owned the unincorporated business now
counts towards the qualifying two year period.

256
Rollover relief

Rollover relief (The replacement of business assets)

If you sell your warehouse and buy a new one, you can decrease the Capital gain by
deducting the new warehouse's purchase costs.

Conditions:

1. The new and old assets must be used for business purpose.

2. You have to replace the asset 12 months prior to the sale or 36 months post the sale.

3. No Rollover relief is available if the amount not reinvested exceeds the chargeable gain.
(See below)

Qualifying assets:

• Land and buildings.

• Fixed plant and machinery.

257
Step by step approach

1. Step 1 - Get the information

About the OLD asset:



- Disposal proceeds of the OLD asset

- Original purchase costs of the OLD asset

- Any costs relating to the sale or the purchase of the asset (e.g. legal fees)

About the NEW asset:



- Purchase costs of the NEW asset

2. Step 2 - Calculate the Chargeable gain

Disposal proceeds                       X

The Original Purchase costs       (X)

Legal fees                                    (X)

Chargeable gain                           X

3. Step 3 - Calculate how much is NOT reinvested

Disposal proceeds - Purchase costs of the NEW asset

How much you get for the OLD asset        X



How much you pay for the NEW asset      (X)

Amount NOT reinvested                              X

4. Step 4 - Check whether the amount NOT reinvested (Step 3) exceeds the
Chargeable gain (Step 2)

No Rollover relief is available if the amount NOT reinvested exceeds the chargeable
gain.

5. Step 5 - Calculate the new Chargeable gain

Disposal proceed                               X

The Original Purchase costs              (X)

Chargeable gain                                  X

Rollover relief (Balancing figure)         (X)

The new Chargeable gain (Step 3)      X (proceeds not reinvested)

6. Step 6 - Calculate Base cost

Basically, the Purchase costs of the NEW asset less the Rollover relief

This base cost will be used as the cost of the new office when it is disposed of in the
future.

258
Illustration 1

Peter sold a freehold warehouse for £200,000 on 1 January 2023.

The warehouse had been purchased for £150,000.

Peter incurred legal fees of £10,000 in connection with the purchase.

On 1 February 2023, he bought another freehold factory for £100,000.

Required:

Calculate the chargeable gain.

Step by step answer

1. Step 1 - Get the information

About the OLD asset:



- Disposal proceeds of the OLD asset = £200,000

- Original purchase costs of the OLD asset = £150,000

- Any costs relating to the sale or the purchase of the asset (e.g. legal fees) = £10,000

About the NEW asset:



- Purchase costs of the NEW asset = £100,000

2. Step 2 - Calculate the Chargeable gain

Disposal proceed                       200,000

The Original Purchase costs       (150,000)

Legal fees                                   (10,000)

Chargeable gain                          40,000

3. Step 3 - Calculate how much is NOT reinvested

How much you get for the OLD asset          200,000



How much you pay for the NEW asset       (100,000)

Amount NOT reinvested                              100,000

4. Step 4 - Check whether the amount NOT reinvested (Step 3) exceeds the
Chargeable gain (Step 2)

Amount NOT reinvested (Step 3) = £100,000 > Chargeable gain (Step 2) = £40,000

No Rollover relief is available if the amount NOT reinvested exceeds the chargeable
gain.

Therefore the chargeable gain will be £40,000.

259
Illustration 2

Peter sold a freehold warehouse for £300,000 on 1 January 2023.

The warehouse had been purchased for £150,000.

On 1 February 2023, he bought a freehold factory for £200,000.

Required:

Calculate the chargeable gain.

Step by step answer

1. Step 1 - Get the information

About the OLD asset:



- Disposal proceeds of the OLD asset = £300,000

- Original purchase costs of the OLD asset = £150,000

About the NEW asset:



- Purchase costs of the NEW asset = £200,000

2. Step 2 - Calculate the Chargeable gain

Disposal proceed                        300,000



The Original Purchase costs       (150,000)

Chargeable gain                           150,000

3. Step 3 - Calculate how much is NOT reinvested

How much you get for the OLD asset          300,000



How much you pay for the NEW asset       (200,000)

Amount NOT reinvested                              100,000

4. Step 4 - Check whether the amount NOT reinvested (Step 3) exceeds the
Chargeable gain (Step 2)

Amount NOT reinvested (Step 3) = £100,000 < Chargeable gain (Step 2) = £150,000

5. Step 5 - Calculate the new Chargeable gain

Disposal proceed                                                             300,000



The Original Purchase costs                                            (150,000)

Chargeable gain                                                                150,000

Rollover relief (balancing figure) 150,000 - 100,000 =      (50,000)

The new Chargeable gain  (Step 3)                                    100,000

260
Illustration 3

Jeremy sold his business office on 30/06/2022 for £350,000.

This office cost him £100,000.

He bought another business office for £250,000 on 31/12/2022.

• How much of his capital gain can be rolled over?

• What is the base cost of his new business office?

Solution:

Disposal proceeds £350,000

Acquisition cost (£100,000)

Chargeable gain £250,000

Rollover relief  (balancing figure) (250,000 - 100,000) (£150,000)

Capital gain now (w1) (proceeds not reinvested) £100,000

W1: Proceeds not reinvested

Old office sale proceeds £350,000

New office costs (£250,000)

Capital gain to be realised now £100,000

Base cost of new business office:

Cost of new office £250,000

Rollover relief (£150,000)

Base cost of new office £100,000

• This base cost will be used as the cost of the new office when it is disposed of in
the future.

Qualifying assets that are not fully used in the business

Assets that are not used in the business entirely will have restrictions for roll over relief
on sale.

The amount of gain that cannot be rolled over, and must be realised now is:

% of asset not used in business * Chargeable gain.

261
Illustration:

Jeremy had another property purchased for use in his business.

However, he did not require the entire property for his business and rented out 20% of
the property.

The property cost him £400,000 on 06/06/2012 and he sold it for £800,000 on
06/06/2022.

He bought another property for use in his business on 12/12/2022 for £900,000.

He will use 100% of this property for his business.

• How much of his capital gain can be rolled over?

• What is the base cost of his new business office?

Solution:

Disposal proceeds          £800,000

Acquisition cost         (£400,000)

Chargeable gain          £400,000

Rollover relief (400,000 - 80,000) (£320,000)

Capital gain now (W1)          £80,000

W1:

Chargeable gain £400,000

All proceeds relating to the business element of the property were reinvested
(80%*£800,000) BUT 20% of the property was not used in business (£80,000 = 20% *
£400,000)

Therefore, Rollover relief is restricted to £320,000   (£400,000 - £80,000)

Base cost of new business office:

Cost of office £900,000

Gain to be rolled over (£320,000)

Base cost of new office £580,000

262
Holdover relief

Holdover relief

If the new asset purchased is a depreciating asset (an asset with an expected life of 60
years or less at the time of acquisition) for example, leasehold land and buildings or
fixed plant and machinery the gain arising on the disposal of the old asset is not rolled
over and cannot be deducted from the cost of the new asset.

Instead, the gain is to be temporarily frozen or “held over” until it becomes


chargeable on the earliest of the 3 following dates:

1. Date on which the new asset is disposed of.

2. Date on which the new asset ceases to be used in the trade.

3. 10th anniversary of acquisition of the new asset.

Illustration:

Craig purchased a freehold factory in 2013 for £100,000.

In June 2021 he sold it for £300,000 and purchased a leasehold factory with a 55-year
lease for £350,000 in December 2021.

Craig then sold the leasehold factory in October 2022 for £400,000.

• What capital gain will be chargeable in the tax year 21/22 and in 22/23?

Solution:

• 21/22 capital gain:

Disposal proceeds £300,000

Acquisition cost (£100,000)

Capital gain £200,000

Gain held over (W1) (£200,000)

Chargeable gain Nil


263
• 22/23 Capital gain:

Disposal proceeds £400,000

Acquisition cost (£350,000)

Capital gain £50,000



+

Capital gain held over £200,000

Total capital gain £250,000

W1:

Disposal proceeds received £300,000

Disposal proceeds reinvested (£350,000)

Capital gain now Nil

• Therefore, the entire gain (£200,000) will be held over as all of the disposal
proceeds are reinvested.

Note:

The £200,000 capital gain held over becomes chargeable in the tax year 2022/23
because the asset against which it was held over has been sold.

Also note that the gain held over was not deducted from the cost of the new asset, it
was held over in its own right.

No holdover relief for assets that do not qualify.

Holdover relief is available for qualifying business assets (chargeable business


assets).

Qualifying business assets are basically assets that are used in the business, not assets
held for investment (chargeable assets).

Therefore, if a gift is made of unquoted shares (a qualifying business asset), holdover


relief is available for the capital gain, however it will not be available for the proportion of
assets held by the business for investment (non-business) purposes.

Illustration:

On 5 October 2022, Tina made a gift of her entire holding of 20,000 £1 ordinary shares
in Banana Ltd, a personal company, to her daughter.

The market value of the shares on that date was £200,000.


264
The shares had been purchased on 1 January 2020 for £140,000.

On 5 October 2022, the market value of Banana Ltd’s chargeable assets was £150,000,
of which £120,000 was in respect of chargeable business assets.

Tina and her daughter have elected to hold over the gain as a gift of a business asset.

What chargeable gain will arise on this gift?

Solution:

Tina’s chargeable gain for 2022/23 is:

Deemed proceeds   £200,000   



Cost                         (£140,000)  

                                £60,000  

Holdover relief        (£48,000)   W1

                                 £12,000

WI

Holdover relief is restricted to £48,000 (60,000 x 120,000/150,000), the proportion of


chargeable assets to chargeable business assets.

265
Investors’ Relief

Investors’ relief effectively extends entrepreneurs’ relief/business asset disposal relief to


external investors in unquoted trading companies.

To qualify for investors’ relief shares must be:

• Newly issued and acquired by subscription;

• Owned for at least 3 years after 17 March 2016.

£10m lifetime limit (in addition to the entrepreneurs’ relief/business asset disposal relief
limit)

Gains taxed at 10%

Investor must not be an employee or director of the company whilst holding the shares
in that company.

Illustration

On 6 November 2019 Elise subscribed for 100,000 £1 ordinary shares (a 3% holding) in


Oz Ltd, an unquoted trading company, at their par value.

She has never been an employee or director of the company.

On 22 November 2022 Elise sold the shares for £700,000.

Elise’s shareholding does not qualify for Entrepreneurs’ Relief/Business asset disposal
relief as she was not an employee and did not hold 5% or more of the share capital of
Oz Ltd.

However, she does qualify for Investors’ Relief - newly issued shares acquired by
subscription and owned for more than 3 years.

Elise’s capital gains tax liability would be:

Chargeable gain £600,000

AE £(12,300)

Taxable gain £587,700

CGT: £587,700 x 10% = £58,770

266
Capital gains tax planning

How to plan to minimise the capital gains tax liability?

Gifting or selling assets has 2 results for tax - inheritance tax and capital gains tax,
therefore the choice to gift must be made carefully, in order to avoid both taxes!

Example

A owns 100% of the shares in A Ltd an unquoted trading company.

A Ltd has 100,000 £1 ordinary shares in issue all of which were subscribed for at par by
A in 2005, from which date A has been the managing director of the company.

Share valuations have now been agreed as follows:



20% £10 per share

40% £15 per share 

80% £20 per share 

100% £25 per share

What are the tax implications of gifting 20,000 of his shares to his daughter?

Inheritance tax implications

For IHT purposes the gift would be a potentially exempt transfer (PET) and have no
immediate tax implications.

If A died within 7 years of the transfer the PET would become chargeable at either nil
rate and / or 40% rate depending upon what other transfers had been made by A prior
to this gift.

If A survived for at least 3 years then any IHT computed would be reduced by taper
relief.

Any such IHT payable would be payable by the donee, V and should be paid within 6
months of the end of the month in which the death occurred.

The value of the PET would be the fall in value of the estate of A.

Before the gift 100,000 shares @ £25ps £2,500,000

After the gift 80,000 shares @ £20ps £(1,600,000)

Transfer of value £900,000

267
Capital gains tax implications

The shares are £1 ordinary shares which were subscribed for at par, so the cost is £1
per share.

The gain that arises would be included in the net gains of the tax year from which the
annual exempt amount would be deducted to derive the taxable gain.

The question then arises as to what tax rate would apply?

Shares in unquoted trading companies are a qualifying business asset for purposes of
entrepreneurs’ relief/business asset disposal relief and as A owns the minimum required
5% shareholding and is an employee of the company, a claim for entrepreneurs’ relief/
business asset disposal relief is available and will result in a 10% tax rate being applied
to the taxable amount of the gain.

There is however another RELIEF that is available where such an asset is gifted!

Yes gift relief is available to be claimed, jointly by A and V, as shares in an unquoted


trading company are qualifying business assets for gift relief purposes.

This would allow the entire gain to be deferred, such that the donor, A, would not now
be chargeable and the daughter, V would be deemed to acquire the shares at the
original cost to the father of £20,000 instead of a cost of £200,000 (20,000 x £10ps).

Without gift relief the shares are deemed to be acquired by V at their open market value
of £200,000. With gift relief, that cost is reduced by the amount of the deferred gain
(£180,000) and thus a cost to V of £20,000 would then apply.

NOTICE how the value for IHT and CGT are different. For CGT you value the asset that
is given away, eg a 20% shareholding. For IHT you look at how much the gift reduces
the donor’s estate by. In this case it reduced it from a 100% holding of shares to an
80% holding of shares.

Conclusion:

For those taxpayers with both a capacity and a willingness to make gifts in lifetime and
not just on death, the further guidance that they may request from you is whether to
make such gifts in lifetime or wait and gift the assets upon their death.

It is again a consideration of the capital taxes that is the key issue.

If assets are gifted on death there will be no CGT and the beneficiaries will acquire
those assets at their then value, thus wiping out any accrued gains on those assets.

The assets, however at their then open market value (probate value) will then be
included within the chargeable estate at death, which being in excess of the available nil
rate band will be charged to IHT at a rate of 40%.

268

Therefore to avoid IHT it would be better to gift in lifetime as when a PET is made there
is no immediate charge to IHT and the PET will only become chargeable if the donor
dies within 7 years.

The further advantages for IHT of gifting in lifetime are that if the taxpayer at least
survives for 3 years then taper relief will reduce any IHT payable, plus the value of the
PET is “frozen” at the date of the transfer meaning that an appreciating asset will have a
lower value charged to IHT than if it had been kept until death.

Lifetime gifts will also benefit from annual exemptions.


The problem of course with gifting in lifetime as we have already seen is CGT, as a gift
in lifetime is a chargeable disposal and a gain must be computed using the open market
value of the asset.

This, however will only happen if the asset is a chargeable asset so that exempt assets
such as cash, chattels and cars could be gifted without any CGT arising.

If assets are chargeable assets then they may still be gifted if the gains arising each tax
year do not exceed the AEA, for example if the taxpayer gifts an asset valued at
£50,000 and it cost £40,000, there will be a chargeable gain of £10,000 which will be
covered by the AEA of the taxpayer.

This will have removed £50,000 of value from the taxpayer’s estate which at death may
have been charged to 40% IHT.

If chargeable assets will give rise to more substantial gains then as we have seen
above, if the asset is a qualifying asset for gift relief then the gain may be deferred by a
claim for gift relief.

If the asset was the principal private residence of the taxpayer then PPR relief would be
available to exempt any gain arising but there could be a potential IHT charge if the
Gifts with Reservation of Benefits rules apply (see later Topic)

You should keep all of these things in mind for written sections in the exam!


269
Syllabus A2b. Chargeable gains

Syllabus: A2bi) Determine the tax implications of independent taxation and transfers between
spouses

Transfers between husband and wife or civil partners

Don’t pay CGT if:

You transfer the assets to your husband or wife or civil partner.

• If a husband transfers an asset to his wife, she would be treated as though she
acquired the asset on the same date and at the same cost as husband did.

Illustration:

On 01/05/2009 a man acquires a piece of land for £10,000.

On 01/05/2022 he transfers it to his wife when the market value of the land is £80,000.

• Will a capital gain be assessable on the husband?

Solution:

The wife would be treated as though she acquired the asset on 01/05/2009 for £10,000.

Therefore, this transfer would have been made at no gain/no loss and no capital gain
would be assessable.

270
Illustration:

What if this man made the same transfer to his daughter?

• Will a capital gain be assessable on the father?

Solution:

A capital gain would arise on the father:

Sale proceeds (deemed to be market value) £80,000

Cost (£10,000)

Capital gain £70,000

Annual exemption (£12,300)

Taxable gain £57,700

• Note: As the father made a gift to the daughter and no sale proceeds were
actually received, the market value of the land will be considered to be the value
that the asset is sold for.

Why is this treatment beneficial?

This treatment is beneficial if one spouse does not have any capital gains and is a basic
rate taxpayer.

• It would be wise to transfer the chargeable asset to this spouse so that they can fully
utilise their annual exemption and pay capital gains tax at the lower rate of 10%
since their basic band is not fully being used (unless it is a residential property, then
the lower rate is 18%).

• IF the asset stays with the spouse who is a higher rate payer and already has capital
gains, then an annual exemption allowance would be wasted and capital gains tax
would be paid at 20% (unless it is a residential property, then the higher rate is
28%).

271
Illustration:

Greg owned a piece of land bought on 01/06/2013 for £40,000.

For the tax year 22/23, he has taxable income of £50,000 and already has net capital
gains of £20,000.

He wants to sell this land for sale proceeds of £65,000.

• Greg’s wife is a housewife and does not have any income or capital gains of her
own.

He thinks that it is wise to transfer the asset to his wife and let her sell it.

• What amount of tax will Greg save if he does this?

Solution:

Without transferring the asset to his wife:

• Net capital gains £20,000

• Gain on land (W1) £25,000

• Net capital gains £45,000

• Annual exemption (£12,300)

• Taxable gains £32,700

• Capital gains tax payable at 20% = £6,540

• The CGT payable specifically on the land is = 20% * £25,000 = £5,000

W1:

Disposal proceeds £65,000

Cost (£40,000)

Net capital gain £25,000

• Why is CGT payable at 20% and not 10%?

• This is because he has used his entire basic rate band up with his taxable
income. (Explained in Topic: The treatment of capital gains)

272
Transferred the asset to wife and she sold it:

Disposal proceeds £65,000

• Cost (£40,000)

• Net capital gain £25,000

• Annual exemption (£12,300)

• Taxable gain £12,700

Capital gains tax is payable at 10% because this taxable gain falls entirely into the basic
rate band = £12,700 * 10% = £1,270

Savings:

CGT paid by husband on the land: £5,000

CGT paid by wife on the land: £1,270

Saving: £3,730


273
Syllabus: A2bii/iii)
Identify the concepts of residence, domicile and deemed domicile and determine their
relevance to capital gains tax and
Advise on the availability of the remittance basis to non-UK domiciled individuals

Residence, Domicile and Deemed Domicile for CGT

What is UK CGT paid on?

UK Residence

Capital gains tax implications

An individual’s residence status must be determined because, if they are UK resident – they
will pay UK capital gains tax on their worldwide gains, but if they are not, they will only pay
UK capital gains tax on their UK situated land and buildings.

Illustration – UK resident

John is UK resident and has capital gains of assets situated in the UK of £60,000 and an
overseas gain from the sale of a villa in Spain of £10,000. 

How much UK capital gains tax will he pay? 

He is a higher rate tax payer.

Solution

UK gains£60,000

Overseas gains £10,000

Total gains £70,000

Less:

A.E. (£12,300)

Taxable Income £57,700

£57,700 * 20% = £11,540

UK capital gains tax payable £11,540

274
Non-UK resident

Domicile

An individual’s domicile is usually the country in which they have their permanent home. 

An individual acquires a domicile of origin at birth, which is the permanent home of the
father.

Individuals retain this domicile until they acquire a different domicile, either through
dependency if under 16 and their father changes his domicile, or by severing ties with the
old country and acquiring a domicile of choice.

Deemed Domicile

An individual may be deemed domicile if they are not domicile under general law but they
satisfy either one of two conditions.

First condition, which is relevant to formerly UK domiciled residents, is that the individual
will be deemed domicile if the individual:

• Was born in the UK; and

• Has a UK domicile of origin; and

• Is UK resident in the relevant tax year.

Second condition, which is relevant to long-term UK residents, is that the individual will be
deemed domicile if they have been resident in the UK for 15 of the 20 years immediately
preceding the relevant tax year.

Illustration – Deemed UK domiciled first condition

Clare was born in the UK and her father was domiciled in the UK until he and the family
moved to New Zealand when Clare was 10 years old. Her father became domiciled in New
Zealand and so Clare acquired a domicile of dependency in New Zealand. Clare is UK
resident in the tax year 2022/23.

Will Clare be deemed UK domiciled in the UK in 2022/23?

Solution

Clare will be deemed UK domiciled in the UK in the tax year 2022/23 because she satisfies
all three parts of the first condition.

Illustration – Deemed domicile second condition

Tom was born in Australia, his father had a UK domicile and so Tom had a UK domicile of
origin. Tom moved to the UK in 2015/16 and became UK resident in that tax year.

Will Tom be deemed UK domiciled in 2022/23

Solution

275
No. Tom only satisfies 2 of the 3 parts of condition one and he does not satisfy condition
two as he has not been UK resident for 15 years.

UK Resident but not UK Domiciled/Deemed Domiciled

If an individual is UK resident but not UK Domiciled/Deemed Domiciled, there are 2 options


for taxing income that arises overseas (Overseas gain).

• Remittance Basis – whatever overseas income/gain exists, you only pay UK


capital gains tax on the amount of income that you send back to the UK.

• Arising Basis – whatever overseas income/gain exists, UK capital gains tax is


paid on it entirely.

Illustration – Remittance basis

John has been resident in the UK for one year and has capital gains of assets (not
residential property) situated in the UK of £60,000 and an overseas gain from the sale of a
villa in Spain of £10,000. 

He sends £3,000 of the rental income back to the UK. 

He is a higher rate tax payer.

How much UK capital gains tax will he pay If he chooses the remittance basis?

Solution

UK gain £60,000

Overseas gain £3,000

Total gain £63,000

Less:

A.E. (£nil) - remember that the AE is not available to taxpayers using the remittance basis

Taxable Gains £63,000


£63,000 * 20% = 12,600 (higher rate tax payer and non residential property)

276
Illustration – Arising basis (same details as the previous scenario)

John has been resident in the UK for one year and has capital gains of assets (not
residential property) situated in the UK of £60,000 and an overseas gain from the sale of a
villa in Spain of £10,000. 

He sends £3,000 of the rental income back to the UK. 

He is a higher rate tax payer.

How much UK capital gains tax will he pay If he chooses the arising basis?

Solution

UK Gain £60,000

Overseas gain £10,000

Total gain £70,000

Less:

A.E. (£12,300)

Taxable Income £57,700


£57,700 * 20% = £11,540

Conclusion

The Remittance basis looks like the expensive option because John loses his entitlement to
the Annual Exemption.

The decision whether or not to claim the remittance basis should be considered year by
year as, depending on the level of unmerited income, sometimes it will be the cheaper
option.

Once the taxpayer has been resident in the UK for more than 7 years, the remittance basis
is likely to be the more expensive option due to the remittance basis charge.

Consequences of choosing the remittance basis

• UK Tax on remitted income

• Remittance Basis Charge

• No annual exemption available for capital gains tax.

277
Remittance Basis Charge

If prior to the current tax year a person has been UK resident for at least 7 tax years then by
making the remittance basis election they must pay HMRC a remittance basis charge. 

This is similar to paying tax on their unremitted income, except that it’s just a flat charge.

• Prior to the current tax year the person was UK resident for at least 7 out of
the last 9 tax years 

RBC £30,000

• Prior to the current tax year the person was UK resident for at least 12 out of
the last 14 tax years

RBC £60,000

Illustration

Kailash is domiciled in India. 

He has been UK resident since 01/04/2015 and earns a salary of £125,000 p.a.

He realises chargeable gains every year equal to his annual exempt amount.

He sold a property in India for £200,000 and realised a chargeable gain of £70,000.

He has remitted £20,000 of the gain back to the UK.

Which is more beneficial for him, the remittance or arising basis in 2022/23?

Solution

Remittance basis

UK resident 06/04/2015 - 5/4/2023 = 8 tax years

R.B.C £30,000

Capital gains tax computation:

Capital gain = £20,000 * 20% = £4,000 + £30,000 = £34,000 (no AE available)

Arising basis

Capital gain £70,000 * 20% = £14,000


Conclusion

He should choose the arising basis as this saves him (£34,000 - £14,000) = £20,000.


278
Syllabus: A2biv)
Determine the UK taxation of foreign gains, including double taxation relief

Double tax relief

Sale of foreign located assets

Double tax relief (Foreign gains)

If you are UK resident, you will pay UK CGT on your worldwide gains. 

However, if you have a property located outside of the UK – if you sell it, you will have to
pay CGT in the country of sale and in the UK. 

This means, that you are paying tax two times on the same capital gain.

The government offers relief for this in the form of double tax relief.

Double tax relief

Double tax relief is available to offset any double tax suffered on assets disposed of abroad.

The DTR given is the lower of:

1 Overseas tax suffered

2 UK tax on that gain

Illustration

Frances Bond is resident and domiciled in the UK. 

During 2022/23 he earns £50,000. He has a residential property in Spain which he sells in
October 2022 for £80,000, incurring Spanish taxes of £8,000. 

This is his only gain of 2022/23. 

He had bought the villa in June 1998 for £25,000, and uses it only for holidays 

Calculate the capital gains tax liability in 2022/23.


279
Solution

CGT payable on disposal of the villa in Spain 

Disposal of the villa 2022/23

Sale proceeds £80,000



Less: Cost (£25,000)

Capital gain £55,000

Less: Annual exempt amount (£12,300)

Taxable gain £42,700

Capital gains tax 28% x £42,700 = £11,956

Less: 

DTR Lower of: 

(1) Foreign tax suffered (£8,000)

(2) UK CGT on the villa 

CGT payable £3,956


280
Syllabus: A2bv)
Conclude on the capital gains tax position of individuals coming to and leaving the UK

Temporary Absence

Individuals coming to and leaving the UK

Temporary non-residence

If the individual leaves the UK for a period of less than five years and also were UK
resident for at least four out of the previous seven tax years, individuals have to pay UK
CGT in respect of assets acquired before leaving the UK.

The rules for temporary non-UK residents are:

1 Any gains made during the tax year of departure – chargeable in that year.

2 Any gains made in subsequent years – chargeable in the tax year the
individual becomes UK resident again. 

This is only for assets that were sold, which were acquired by the individual
before they left the UK.

3 It does not apply to the disposal of assets acquired after leaving the UK.

4 This makes it difficult for individuals to avoid UK CGT by selling assets when
they are not UK resident (in the period of temporary absence), and then
return to the UK.

If a person is overseas for more than 5 years they are exempt from CGT on all disposal of
UK located chargeable assets and overseas located chargeable assets, when they sell the
assets - even if they become UK resident after the 5 years of being non-resident. 

The exception is disposals of UK located residential and non residential properties, as UK


CGT will be payable on any UK located residential and non residential properties.

281
Illustration

James has always been UK resident. 

He intends to leave the UK on 06/04/2022 and become resident in Barbados.

If he does not like Barbados, he will return to the UK in 4 years. 

What will he pay UK CGT on?

Solution

As he has been UK resident for 4 out of the previous 7 tax years and he will return the UK
within 5 years, he will pay UK CGT on:

1) Any capital assets that he owned before leaving the UK and sold while he was in
Barbados.

2) Any UK residential and non residential properties owned.

3) Any disposals that he makes after returning to the UK.


282
Syllabus: A2bvi)
Advise on the UK taxation of gains on the disposal of UK land and buildings owned by non-
residents

UK Located Land and Buildings

Is UK CGT payable by a non resident?

Normally, UK CGT is only payable by UK residents.

However, from 6 April 2019, there is an exception to this.

Non UK residents selling:

• UK land and buildings (residential and non-residential);

• UK assets used in a trade based in the UK

will pay UK CGT on the disposals.

These rules apply to both individuals and companies although in ATX the rules that
apply to companies will only be relevant in the following circumstances:

• Where a non-resident company disposes of a UK property used by a UK permanent


establishment;

• Where entities that derive at least 75% of their value from UK property and the person
making the disposal has a substantial interest (25% or more) in the entity holding the
property.

Where the property was acquired prior to 6 April 2019 the amount that will be within the
scope of CGT will be either:

• t he gain/loss arrived at by deducting the market value of the property as at 5 April


2019 from the sale proceeds, or

• the whole of the gain calculated in the normal way. This alternative method requires
an election.

If the disposal is of a business asset, rollover relief may be available if the replacement
asset is UK land/buildings.

283
Gift relief is also available despite the individual being non-UK resident and regardless of
the residence status of the donee.

The non-resident individual must submit a non-resident CGT return to HMRC within 30
days of completion regardless of whether or not there is a taxable gain (eg where
chargeable gain is covered by the annual exemption).

Note: non-resident individuals are entitled to the annual exemption.

Note: In the ATX exam:

• UK land sold by a non-UK resident will always be acquired after 5 April 2015 (the
date when disposals of UK property by non-UK resident individuals became
taxable).

• Properties will be either wholly residential or wholly non-residential throughout the


period of ownership.

• As a result of these restrictions, for disposals by non-UK residents of:

• residential properties, the gain will be calculated in the normal way;

• non-residential properties owned on 5 April 2019, the gain will be calculated


by reference to the market value as at 5 April 2019, or in the normal way (by
election);

• other non-residential properties, the gain will be calculated in the normal


way.

Illustration:

Jake was not UK resident in 05/04/23 but sold a non residential property for £300,000
during the tax year.

He originally purchased it for £150,000 on 05/04/16 and at 05/04/19 it had a market


value of £200,000.

What capital gain will arise on this sale?

Solution:

Method one:

Sale proceeds £300,000

Less MV at 05/04/19 (£200,000)

Capital gain £100,000

284
Method two (by election):

Sale proceeds £300,000

Less cost (£150,000)

Gain £150,000

Jake should not elect for NORMAL method one should be chosen to calculate the
capital gain. 


285
Syllabus: A2bvii)
Identify the occasions when a capital gain would arise on a partner in a partnership on the
disposal of a partnership asset

Partnership Disposals

Partnership CGT

Disposal of a partnership asset to a third party

Each partner is deemed to own a fractional share of the partnership assets. 

This is based on the agreed capital profit sharing ratio in the partnership agreement.

• On disposal of a partnership asset to a third party, a capital gain should be


calculated normally, then this gain is distributed to each partner based on the
profit sharing ratio. 

• Each partner should include their share of the gain in their own capital gains
computation.


Illustration

In January 2011, Paul and Phil commenced a partnership. 

They introduced capital into the business of £30,000 and £20,000 respectively and agreed
to share profits 60%:40%. 

The partnership purchased a freehold premises for £125,000 in January 2011. 

In September 2022 the partnership sold the premises for £495,000 and continued to trade
in a rented premises. 

What are the chargeable gains arising on Paul and Phil in 2022/23 in respect of the
partnership disposal?

Solution

Sale proceeds £495,000

Cost (£125,000)

Capital gain £370,000

Paul Capital Gain:  £370,000 * 60% = £222,000

Phil Capital Gain:  £370,000 * 40% = £148,000 


286
Syllabus A2c. Trusts
Syllabus: A2ci/ii) Advise on the capital gains tax implications of transfers of property into trust
and
Advise on the capital gains tax implications of property passing absolutely from a trust to a
beneficiary.

Capital gains tax and trusts

Trusts

CGT Implications

Any type of gift into a trust will be treated as a sale for market value. 

But remember, that any gift into and out of a trust is eligible for gift holdover relief, therefore
no capital gains tax will arise as the capital gain will be held over.

Recap of gift holdover relief



Jake makes a gift of shares into a trust when they have a market value of £50,000. 

They cost £10,000.

Therefore, the capital gain:

Sale proceeds £50,000

Cost (£10,000)

Capital gain £40,000


As gift holdover relief applies here, the capital gain will be held over, what will the
base cost of the shares be for the trustee?

Market value £50,000

Less: gain deferred (£40,000)

Base cost £10,000

Therefore, if the trustee decides to sell the shares, they will use a cost of £10,000 when
calculating the capital gain. 

Remember that this capital gain will only arise if the shares are sold while they are in the
trust, because gift holdover relief will apply for assets going into or coming out of the trust.


287
Selling assets while they are in the trust

The trustees can dispose of chargeable assets outside of the trust, however they will only:

1 Get 1/2 of the Annual Exemption (£12,300/2 = £6,150)

2 Be taxed at the higher rate of CGT (20%)

Illustration

For the example used above, the trustees sold the shares for £60,000. 

What capital gain will arise?

Solution

Sale proceeds £60,000

Base cost (£10,000)

Capital gain £50,000

Less A/E (£6,150)

Taxable gain £43,850

CGT £43,850 *20% = £8,770

Illustration

For the example above, the shares passed to the beneficiary when their market value was
£75,000. 

What will the base cost of the shares be for the beneficiary?

Solution

A gift passing out of a trust will be a sale at market value, however, it is eligible for gift
holdover relief.

Sale proceeds £75,000

Base cost (£10,000)

Capital gain £65,000

Base cost

Market value £75,000

Less capital gain deferred (£65,000)

Base cost £10,000

This cost will be used when the beneficiary wants to sell the shares.

Recap

1) Gifting into an out of trusts are eligible for gifts holdover relief

2) An asset sold while it is in a trust will give rise to a chargeable gain, the trustees will get
1/2 of the annual exemption and pay CGT at 20%.


288
Syllabus A2d. Principles of computing gains and losses
Syllabus: A2di) Identify connected persons for capital gains tax purposes and advise on the tax
implications of transfers between connected persons

Connected Persons

CGT Implications

Who are connected persons for CGT?

Connected persons include:

• Relatives (brothers, sisters, parents, grandparents, children)

• Spouse's Relatives

• Business Associates (partner, partner's spouse, partner's relatives)

Disposals to connected persons

Disposals to connected persons will be assumed to be at market value, regardless of the


consideration that the connected person has paid. 

This excludes disposals to spouse/civil partner, as these are exempt for CGT.

• If a capital loss arises on the disposal to a connected person, this capital


loss can only be relieved against a capital gain arising from a disposal to the same
connected person.

289
Illustration

Jane is planning on ceasing her business, and passing the assets to her daughter.

Assets:

Trading premises

Cost £100,000

M.V. £200,000

Inventory

Cost £10,000

M.V. £11,000

If Jane gifts the business to her daughter, what capital gain will arise?

Solution

Sale proceeds (M.V.) £200,000

Less cost (£100,000)

Capital gain £100,000

Inventory is an exempt asset for capital gains tax.


Note, the daughter is a connected person - therefore the sale is deemed to take place at
market value, even though no consideration has been given.

Successions to trade between connected persons - capital allowances

If the business is being transferred:

1) As a going concern

2) To a connected person

THEN … An election is available to transfer the assets at their TWDV (instead of


market value) and therefore avoid any balancing charges or balancing
allowances.

290
Illustration:

Julie has been trading as a sole trader since 2010 and has always prepared her
accounts to 31 December.

She wants to retire on 31/12/2022 and either sell her business to an unconnected
person or give the business to her daughter so that she can continue to run the
business.

The values of her plant and machinery at 01/01/2023 are:

Main pool items:

TWDV £24,000

MV £37,000

What are the tax consequences if she sells the business to an unconnected
person?

What are the tax consequences if she gives the business to her daughter and she
continues to run it?

Solution:

Selling to an unconnected person:

Sale proceeds (MV) £37,000

Less TWDV (£24,000)

Balancing charge £13,000 (I.T. will be payable on this)

Giving business to her daughter (connected person):

The assets will be assumed to be transferred at TWDV, therefore no balancing charge


will arise and no income tax will be payable. 


291
Syllabus: A2dii)
Advise on the impact of dates of disposal

Date of disposal

A chargeable disposal occurs on the date of the contract, whether verbal or written. This
may not be the same as the date the asset is transferred to the new owner.

It is important to consider the timing of disposals - for example, have you already used your
annual exemption for the year? Are you a higher rate tax payer this year but expect to be a
basic rate taxpayer next year? In these situations it may be advisable to delay the disposal
if practical to do so. Does the taxpayer qualify for any reliefs? Eg entrepreneurs’ relief/
Business asset disposal relief? If they delay the sale would they then meet the conditions?

All of these things need to be considered when advising a tax payer on the date of disposal.

292
Syllabus: A2diii)
Evaluate the use of capital losses in the year of death

Capital losses in tax year of death

Capital losses can be offset against current or future capital gains without time limit. 

However, capital losses realised in the tax year of death, occurs where an individual makes
a disposal of a chargeable asset(s) in the period from 6 April up to the date of death and a
net capital loss is realised. 

Net capital losses incurred in the year of death cannot be carried forward in the usual
manner, in these circumstances only; the capital loss may instead be carried back and set
off against the chargeable gains of the previous three years on a LIFO basis. 

This may generate a refund of CGT previously paid. The amount carried back to a particular
tax year is restricted to preserve the annual exempt amount.

Illustration

Nemo died on 1 November 2022. Nemo had annual taxable income of around £20,000
since 2019/20. 

Before his death he made the following sales of quoted shares with the following results: 

1. Chargeable gains of £8,000 and capital losses of £17,100 in 2022/23. 

2. Chargeable gains of approximately £14,000 each tax year 2019/20 to 2021/22. 

How much of the capital losses can be relieved and when?

Solution

Date of Nemo death 1 November 2022; Tax year of death = 2022/2023

Net capital loss realised in tax year of death is (£9,100) (£17,100-£8,000)

The net capital loss realised in the tax year of death can be carried back for three years on
a LIFO basis. 

The amount carried back to a particular tax year is restricted so that the annual exempt
amount is preserved in all tax years. (assume 2022/23 tax rates apply throughout)

293
21/22 Capital gain £14,000

Less A/E (£12,300)

Taxable gain £1,700

Capital loss (£1,700)

20/21 Capital gain £14,000



Less A/E (£12,300)

Taxable gain £1,700

Capital loss (£1,700)


19/20 Capital gain £14,000



Less A/E (£12,300)

Taxable gain £1,700

Capital loss (£1,700)


£5,100 loss used therefore (£9,100 - £5,100) £4,000 Capital loss unrelieved.


294
Syllabus A2e. Disposal of movable and immovable
property
Syllabus: A2ei) Advise on the tax implications of a part disposal, including small part disposals
of land

Small part disposals of land

Exception to the part disposal rule

Small part disposals of land

Normally, if you are selling a part of an asset, you will use the formula (a/(a+b)*total cost) to
find the allowable cost for your part disposal and then calculate the capital gain.

There is one exception to this part disposal rule and it applies if it is a small part disposal of
land.

The main conditions to be met are:

1 The land cannot be a wasting asset

2 Sale proceeds received must not exceed 20% of the market value of the
land and the amount of the disposal must not exceed £20,000

3 total amount of all disposals of land in the year does not exceed £20,000

CGT Implications of a small part disposal

The sale proceeds are deducted from the original cost of the land, and then that reduced
cost will be used to calculate the capital gain when the remainder of the land is sold.

295
Illustration

Jake has a 100 hectare plot of land valued at £1,000,000.

The original cost of the land was £500,000.

He sold 1 hectare for £10,000. This was his only disposal of land in the year.

What capital gain will arise?

Solution

The following conditions are satisfied:

1) The land is not a wasting asset



2) The sale proceeds are less than 20% of the market value of the holding and less than
£20,000

3) the proceeds from all sales of land in the year are less than £20,000

Therefore, the sale proceeds will be deducted from the original cost of the land.

£500,000

(£10,000)

£490,000 - is the cost that will be used to calculate the capital gain when the remaining
land is sold.

296
Syllabus: A2eii)
Determine the gain on the disposal of leases and wasting assets

Disposal of wasting assets

What is a wasting asset?

A wasting asset is:

An asset that has a life of 50 years or less. For example,

1 Chattels (These are tangible, movable assets - and you know how to
calculate capital gains for these already)

2 Intangible wasting assets, for example, copyrights and leases.

How to calculate the capital gain arising on an intangible wasting asset?

Sale proceeds 

Less (Allowable cost) (W1)

Capital gain

Allowable cost (W1)



(Remaining years after the sale / predictable life of the asset) * (total cost - scrap value)

(Note: for leases you need to use the lease percentage tables rather than the years. These
will be provided to you in the exam)

297
Illustration

On 01/12/2020 Jake bought a copyright for £25,000.

It had a predictable life of 30 years and scrap value of £1,000.

It was sold on 01/12/22 for £38,000.

What capital gain will arise?

Solution

Allowable cost:

Jake owned it for 2 years. 



Therefore, the remaining number of years = 30 - 2 = 28 years

(28 years / 30 years) * (£25,000 - £1,000) = £22,400

Sale proceeds £38,000



Allowable cost (£22,400)

Capital gain £15,600

Disposal of short leases

If the intangible asset is a lease, the lease percentage tables will need to be used - these
will be provided in the exam.

Illustration

Jack bought a short lease in March 2001 for £50,000 and he sold it in March 2023 for
£30,000 when it had 10 years left to run.

The chargeable gain in disposal will be:

Proceeds £30,000

Less: Cost £50,000 x (46.695 (10yrs)/89.354 (32 years) £(26,129)

Gain £ 3,871


298
Syllabus: A2eiii)
C3 Gains and losses on the disposal of movable and immovable property and
Establish the tax effect of capital sums received in respect of the loss, damage or destruction
of an asset

Insurance proceeds received for a damaged/lost/


destroyed asset

Damaged/lost/destroyed chargeable asset

When a chargeable asset is destroyed/lost/damaged and the value of the asset has
become negligible (very small value), then a person can make a negligible value claim.

The asset will be treated as though it has been disposed of at its current, negligible
value, therefore the person can realise a capital loss.

The asset does not actually have to be disposed of, it is just a way of realising a loss.
When the asset is sold in the future, the negligible value will be used as its cost.

299
Illustration:

A painting was acquired for £10,000 and damaged in a fire.

It now is worth a negligible amount.

How will it be treated for capital gains tax?

Solution:

Disposal proceeds £ Nil

Acquisition cost (£10,000)

Capital loss (£10,000)

• This capital loss will be relieved against current year capital gains, and if it
cannot be relieved fully, it will be carried forward to be relieved against future
capital gains.

• Note the painting could be valued at any amount and this same treatment would
apply.

For example, if the painting was valued at £100,000 at the time it was damaged, then
the capital loss realised would be:

Disposal proceeds £ Nil

Acquisition cost (£100,000)

Capital loss (£100,000)

Insurance proceeds received for damaged/lost/destroyed chargeable asset

You have to pay CGT on the insurance proceeds.

The disposal proceeds are the amount of money received from the insurance company.

Capital gains calculation:

Proceeds received from insurance company   X



Cost of asset lost/destroyed                           (X)

Chargeable gain                                               X

The disposal is treated as though it occurred in the tax year that the insurance proceeds
are received.

300
Illustration:

Holly owned a vase which was destroyed on 06/04/2022, she had paid £28,000 for it on
01/05/2011.

The market value when it was destroyed was £80,000 however she only received
£68,000 of insurance proceeds.

What will the capital gains treatment be if she does not decide to reinvest the
proceeds?

Solution:

For the tax year 22/23

Insurance proceeds £68,000

Acquisition cost (£28,000)

Chargeable gain £40,000

Annual exemption (£12,300)

Taxable gain £27,700

Note the insurance proceeds received are £68,000, this will be used in the computation.

It does not matter that the market value at the time of disposal was £80,000 - the actual
insurance proceeds received will be used.

Insurance Rollover Relief (IRR)

You get IRR if the insurance proceeds received are reinvested into another replacement
asset within 12 months of the proceeds being received.

• However, if only some of the proceeds are reinvested, then the proceeds which
are not reinvested will be taxable immediately.

For example:

Insurance proceeds received £1,000

Asset costing £900 was destroyed

Reinvestment in a new asset £950

The capital gain that resulted was £100 (£1,000 - £900)

The £50 of insurance proceeds not reinvested (£1,000 - £950) will be taxable
immediately.

The remaining £50 of capital gain will be deferred to be taxed at a later date.

This is known as the gain rolled over

• How is this £50 of capital gain deferred to be taxed at a later date?

301
It is deducted from the cost of the replacement asset.

£950 - £50 = £900

This £900 is known as the base cost of the replacement asset.

This base cost will be used as the cost of the replacement asset when it is disposed.

Illustration:

What if Holly used the insurance proceeds to buy a replacement vase for £59,000 on
01/03/2023?

What capital gain will be realisable in this case?

Solution:

Proceeds received £68,000

Acquisition cost   (£28,000)

Chargeable gain    £40,000

IRR (40,000 - 9,000) balancing figure   £31,000

Capital gain realisable now (w1) £9,000

Working 1:

Proceeds received £68,000

Proceeds reinvested within 12 months of receipt (£59,000)

Capital gain realisable now (w1) (proceeds not reinvested) £9,000

Base cost of replacement vase:

Cost to acquire the new vase – Capital gain rolled over = Base cost of replacement vase

Capital gain rolled over:

Total capital gain £40,000



Gain realised immediately (£9,000)

Capital gain rolled over £31,000

302
Cost of new vase £59,000

IRR  (£31,000)

Base cost of vase £28,000

What if Holly disposes of the new vase after 10 years for £100,000?

Proceeds received £100,000

Base cost  (£28,000)

Chargeable gain £72,000

Annual exemption (£12,300)

Taxable gain £59,700

Insurance proceeds used in restoration

If an individual receives insurance due to the damage of an asset and spends the
insurance proceeds on restoring the asset plus an additional amount, the base cost of
the asset will be treated as:

Cost                                               £X

(Insurance proceeds received)     (£X)

+ Additional amount spent           £X

Base Cost                                     £X


303
Illustration:

On 15/01/2023, a timepiece owned by Kamal fell and was badly damaged.

The timepiece had been purchased for £99,000. Kamal received insurance proceeds of
£54,000 and he additionally spent a total of £45,000 on restoring the timepiece to
working condition again.

What is the base cost of the timepiece after restoring it?

Solution:

Cost £99,000

Proceeds (£54,000)

Additional spent £45,000

Base cost £90,000

304
Syllabus: A2eiv)
Advise on the tax effect of making negligible value claims

Negligible Value Claims

Assets falling in value

Negligible Value Claim

If an asset is acquired at market value and then later on the market value of the asset is
lower than when acquired, it is said that the asset has fallen in value.

If the asset which has fallen in value is disposed of then a capital loss would be realised. 

If it is evident that an asset has fallen in value (e.g. as a result of a company going into
liquidation) then the taxpayer can claim relief for a fall in value of the asset and will be
treated as if the asset has been disposed of for market value. 

A capital loss is then treated as being realised even although the taxpayer has not actually
disposed of the asset.

This loss can then be offset against total income of the current or previous year - it is not
restricted to being offset against capital gains. You cannot restrict the loss to preserve your
personal allowance.

Illustration

Bob has 5,000 shares in Willis Ltd, an unquoted company based in the UK. 

He subscribed for these shares in August 2006, paying £3 per share. On 1 December 2022,
Bob received a letter informing him that the company had gone into liquidation. 

As a result, his shares were almost worthless. 

305
The liquidators dealing with the company estimated that on the liquidation of the company,
he would receive no more than 10p per share for his shareholding.

Bob has taxable income of £54,000.

If Bob makes a negligible value claim, what capital loss will he realise and how can he
obtain loss relief for this?

Solution


Bob can make a negligible value claim as at 1 December 2022. 



This will give rise to a capital loss of £14,500 (£500 – £15,000) which will be deemed to
arise in the year 2022/23. 

By doing so, his taxable income for that year will be reduced to £39,500 (54,000 – 14,500). 

As the capital loss is realised on the disposal of unquoted shares, this allows the loss to be
relieved against the taxpayer total income for the year in which the loss arose, and/or
against the total income of the previous year.


This will give Bob income tax relief at 40% saving income tax of £5,800 (40% x 14,500). 

The alternative option is the carry the loss forward against capital gains of future years,
which will give him maximum relief at 20%.


306
Syllabus A2f. Disposals of shares and securities
Syllabus: A2fi) Extend the explanation of the treatment of rights issues to include the small part
disposal rules applicable to rights issues

Part disposal rules for rights issues

Small part disposals

Rights issues

If a shareholder who is offered the rights issue does not wish to purchase more shares in
the company they can sell the right to buy the new shares to another person.

This is known as a "sale of rights nil paid".

The capital gains tax treatment of a 'sale of rights nil paid' depends on the amount of sale
proceeds received.

1 If the sale proceeds received are >5% of the value of the shares on which
the rights are offered and >£3,000 then this is deemed to be a part
of the original shares held and a normal part disposal computation is required.

2 If the sale proceeds received less than 5% of the value of the shares on
which the rights are offered or  less than £3,000 then this will not be
considered to be a chargeable disposal and the sale proceeds received are
deducted from the original cost of the shares.

Illustration

Edward bought 12,000 shares in P. Plc. for £24,000.

On 13/08/2022 there was a 1:5 rights issue for £2.35 (M.V. £2.65).

Edward sold the rights for £1,500. 

What chargeable gain will arise?

307
Solution

The sale proceeds are below £3,000 and 5% value of his ownership is (£2.65*12,000*5%) =
£1,590.


Therefore this will be considered to be a small part disposal and the sale proceeds will be
deducted from the original cost of the shares leaving a base cost of:


Original cost £24,000



Less sale proceeds(£1,500)

Base cost £22,500 


308
Syllabus: A2fii)
Define a qualifying corporate bond (QCB), and understand what makes a corporate bond
non-qualifying. Understand the capital gains tax implications of the disposal of QCBs in
exchange for cash or shares

Qualifying Corporate Bonds

What is a qualifying corporate bond?

Qualifying corporate bonds are debt securities (loan notes) that are exempt for capital gains
tax purposes. 

This means that if they are sold they will not give rise to any capital gains, and no capital
loss will be allowable.

Conditions to qualify as a QCB

1 It is in sterling and has no rights of conversion into, or redemption in, a


currency other than sterling.

2 It has no rights of conversion into shares, it must remain a debt security.

3 It must be issued/acquired after 13/03/1984

4 Interest must be at a commercial rate


309
Syllabus: A2fiii)
C4 Gains and losses on the disposal of shares and securities and
Apply the rules relating to reorganisations, reconstructions and amalgamations and advise on
the most tax efficient options available in given circumstances

Bonus issues, rights issues, takeovers and


reorganisations

Bonus Issues

This is an issue of shares to existing shareholders in proportion to the number of shares


owned.

• For example, if you owned 500 shares and a 1:5 bonus issue was declared, you
would receive (500/5) *1 = 100 bonus shares.

• These shares are deemed to be acquired at the same date and at the same cost
as the original shares to which they relate.

They have no cost of their own.

Therefore, in your share pool, a bonus issue will only result in an increase in the number
of shares, and no increase in the cost of shares

Illustration:

Mina purchased shares in C Co.

The details of her purchases are below:

• May 2022 Purchased 3000 shares for £3,000

• Jan 2023 Purchased 1500 shares for £2,000

• March 2023 Bonus issue of 1:3 declared by the company

• How many shares will Mina receive under the bonus issue?

• What is the cost of these shares?

310
Solution:

• Total shares in company = 4,500

• New shares received = (4,500/3) * 1 = 1,500

• The bonus shares will have a cost of £0

A rights issue
occurs where a company offers its existing shareholders the right to buy extra shares.

Rights issues are similar to bonus issues in that the number of shares offered to each
shareholder is generally in proportion to his or her existing shareholding.

• The only difference is that a price is paid for these shares.

• The price for the shares is normally lower than current market value, in order for
the existing shareholders to be attracted to taking up the issue.

Illustration:

Jack is an employee in Jill Ltd.

He had the following transactions in the company’s shares:

• Jul 2022 Purchased 6,000 shares for £15,000

• Sep 2022 Purchased 900 shares for £2,700

• Dec 2022 Took up 1:5 rights issue for £2.00 per share

• What will the rights issue cost Jack if he decides to subscribe to the issue fully?

Solution:

• Total shares in company = 6,900

• New shares received = (6,900/5) * 1 = 1,380 shares

• The rights shares will have a cost of £2.00 * 1,380 shares = £2,760

Bonus issues and rights issue and disposal

• Note carefully that these bonus issues and rights issue will follow the same
matching rules for shares when they are disposed.

• The bonus issues will be included in the share pool at no cost and the rights
issue shares will be included in the share pool at their respective cost.

• Nothing changes with the matching rules.

311
Takeovers/Reorganisations

Takeovers or company reorganisations normally occur when a company is facing


financial difficulty.

They attempt to change the structure and ownership of a company by having another
company take over the individual company.

This will result in the identity and management changed of the individual company, in
the hope of better decisions being made for the company in the future, resulting in a
longer life for the company.  

Takeovers/Reorganisations can either be for a share for share exchange, or a takeover


can be for a cash exchange.

We will deal with both of these situations separately via the use of illustrations.

Takeovers (share for share exchange)

• If a takeover is for a share for share exchange, then no capital gains tax arises
immediately.

• The market value of the new holding provided will be used to apportion our initial
holding cost.

• Then when we ultimately dispose of this new holding, we will use the original
holding cost, and this will result in a capital gain assessable.

Illustration:

Jayna owned 2000 shares in A plc. which cost her £2,000 in 2012, and A plc was being
taken over by B plc in 2023.

• Jayna was offered by B. plc 1,500 ordinary shares with a market value of £3,000
and 500 preference shares with a market value of £1,000.

• Jayna takes up the offer.

• Will capital gains tax arise immediately?

• If not, when Jayna sells these new ordinary shares and new preference shares,
what cost would be attributed to each?

Solution:

There will be no immediate charge to CGT as this is a ‘paper for paper’ exchange -
there is no cash involved.

The original cost of the A Plc shares will just need to be apportioned between the new
ordinary and preference B Plc shares.

312
Total market value of new holding: £3,000 + £1,000 = £4,000

• Total cost of original holding: £2,000

Cost attributed to ordinary shares:

Market value of ordinary shares/Total market value of new holding * original cost

• = £3,000/£4,000 * £2,000 = £1,500

Cost attributed to preference shares:

Market value of preference shares/Total market value of new holding * original cost

• = £1,000/£4,000 * £2,000 = £500

• Jayna needs to use this “attributed costs” as the acquisition cost when she
decides to sell the shares in B. plc.

(She cannot use the market value of the shares when they were given to her).

Takeovers (share for cash exchange)

• If a takeover is for a share for cash exchange, capital gains tax will arise
immediately for the proportion of cash given compared to the total market value
of the new holding.

• The market value of the new holding provided will be used to apportion our initial
holding cost to be used.

Illustration:

Jayna owned 2000 shares in A plc. which cost her £2,000 in 2012, and A plc was being
taken over by B plc in 2022.

• Jayna was offered by B. plc 1,500 ordinary shares with a market value of £3,000
and cash of £1,000.

• Jayna takes up the offer.

• Will capital gains tax arise immediately?

Solution:

Yes - CGT will arise immediately because there is a cash element to the consideration
which implies that some of the shares have been disposed of.

313
Total market value of new holding: £3,000 + £1,000 = £4,000

• Total cost of original holding: £2,000

Cost attributed to ordinary shares:

Market value of ordinary shares/Total market value of new holding * original cost

• = £3,000 / £4,000 * £2,000 = £1,500

Cost attributed to cash given:

Cash received/Total market value of new holding * original

• = £1,000 / £4,000 * £2,000 = £500

• Jayna needs to use this £500 as the acquisition cost of the shares disposed of
for the cash received.

Capital gains:

Disposal proceeds £1,000

Acquisition cost (£500)

Capital gain £500

• No capital gain will arise on the share element, as described above 

314
Syllabus: A2fiv)
Establish the relief for capital losses on shares in unquoted trading companies

Loss on sale of shares of unquoted company

Capital losses

Relief

Capital losses are normally carried forward and used to reduce future chargeable gains. 

However, there is another use of the capital losses. 

Relief against total income is available if – the loss arises on the disposal of unquoted
trading company shares. 

With this claim, the capital loss can be set off against total income of the current year and
previous year, this allows the loss to attract tax relief at the higher rates of 45%.

Illustration

Drey subscribed for 5,000 shares in W Ltd., an unquoted trading company in August 2011
for £3 per share. 

On 1 December 2022, the company made major losses and that the shares were now
valued at 10p per share. 

He has other income of £45,000. 

If he sells the shares at the current market value, how can he claim relief for his capital
loss?

Solution

Capital loss

S.P. 5,000 * 0.1 = £500

Cost  5,000 * £3 = (£15,000)

Capital loss £14,500

Claim against total income of the current year

Total income £45,000

Less

Capital loss (£14,500)

Total income £30,500


315
Syllabus A2g. Exemptions and Reliefs for C.G.T.
Syllabus: A2gi) Understand and apply enterprise investment scheme reinvestment relief

EIS Reinvestment Relief

Investing in EIS shares

If an individual disposes of any chargeable asset and reinvests in unquoted shares in a


qualifying Enterprise Investment Scheme it is possible to defer some (or all) of the gain
arising on the asset by claiming EIS Reinvestment Relief. 

In order for EIS Reinvestment Relief to be claimed the individual must be resident in the UK
when the gain arises and the reinvestment is made. 

The reinvestment must also occur within a qualifying time period, between 12 months
before and up to 36 months after the gain arises. 

The reinvestment must be wholly for cash, in new shares in an unquoted trading company,
trading wholly or mainly in the UK. 

The amount of gain to defer by way of EIS Reinvestment Relief can be chosen by the
taxpayer in order to utilise losses and the annual exemption, but it cannot exceed the
amount invested in unquoted shares. 

The EIS Reinvestment Relief is applied to the gain with any balance not deferred being
reduced by the annual exempt amount. 

316
Any gain deferred is held over until the EIS shares are disposed of when the deferred gain
will again crystalise.

Calculating EIS Reinvestment Relief

Sale Proceeds £x

Less: Cost (£x)

Capital Gain

Less:

EIS Reinvestment Relief (£x)

Chargeable gain

Less:

A/E (£x)

Taxable gain

Note, you should work backwards and make sure that your capital gain uses the capital
losses and annual exempt amount entirely, and then use the remaining capital gain against
the EIS relief.

Sale of EIS Shares

Capital gains implications of sale of EIS shares

First gain

The gain that is held over by the EIS Reinvestment Relief will become chargeable once the
EIS shares are sold.

EIS Gain

If investor disposes of the EIS shares after three years = no CGT. 

If he sells them within three years = CGT. 

If he sells them at a loss within or after three years he gets relief for his capital loss. 

The capital loss is realised on the disposal of unquoted shares and can be relieved against
total income of the current and previous tax years.

Note - we have already seen this capital loss relief available on the sale of unquoted
shares.

317
Illustration

Grace sold a vase in November 2022 for £275,000 realising a capital gain of £150,000. 

Grace subscribes for qualifying EIS shares in W Ltd, a trading company, the following
month at a cost of £268,000. 

She has no other capital transactions in 2022/23 but Grace has £9,300 of capital losses
brought forward at 6 April 2022. 

Three years later in 2025/26 Grace sells the EIS shares making a profit of £175,000. 

Assume Grace is a higher rate taxpayer. 

What are the capital gains tax implications on the purchase and on the sale of the EIS
shares?

Solution

Capital gain on purchase of EIS shares

Grace can claim relief for any amount up to £150,000, because she has invested more than
this in EIS shares.

However, to claim this full amount will mean that she does not make full use of her annual
exempt amount for 2022/23.

The EIS relief claim should therefore be £128,400 as follows:


Capital gain £150,000



Less:

EIS Reinvestment Relief (£128,400)

£21,600

Less:

Annual exemption £(12,300)

Capital loss (£9,300)



Capital gain £Nil

Capital gain on sale of EIS Shares



When Grace disposes of the EIS shares in 2025/26, the deferred gain of £128,400 on the
vase will become chargeable to CGT, but can be reduced by the annual exempt amount of
2025/26. 

The CGT will be £23,220 (128,400 – 12,300) x 20%, due on 31 January 2027.


The gain on the EIS shares will be exempt from CGT as they are held for at least three
years.


318
Syllabus: A2gii)
Understand and apply seed enterprise investment scheme reinvestment relief

SEIS Reinvestment Relief

Investing in SEIS shares

If an individual disposes of any chargeable asset and reinvests in unquoted shares in a


qualifying Seed Enterprise Investment company it is possible to exempt some (or all) of the
gain arising on the asset by claiming SEIS Reinvestment Relief.

Conditions

To qualify for SEIS reinvestment relief, the individual must be resident in the UK when the
gain arises and the reinvestment is made.

 

The reinvestment must be made in the same tax year as the disposal against which relief is
claimed. 


Any gain exempted under SEIS reinvestment relief will become chargeable if there is a claw
back of SEIS income tax relief. This includes the following events 


1) The investor disposes of the shares within 3 years. 



2) The investor becomes non-resident within 3 years of the issue of the shares. 

3) The company in which the shares were purchased ceases to be a qualifying company.

Maximum SEIS Relief

Lower of: 

1 50% x the capital gain 

2 50% x the cost of the shares purchased in the SEIS (upper limit on the cost
of the shares is £100,000) 

3 Some smaller amount

319
Illustration

Shawna sold a holiday cottage in August 2022 for £75,000 realising a capital gain of
£35,000. 

Shawna subscribes for qualifying SEIS shares in Victoria Ltd (a trading company), in March
2023 costing £60,000. 

She has no other capital transactions in 2022/23 but Shawna has £8,800 of capital losses
brought forward at 6 April 2022. 

In 2023/24 Shawna sells the SEIS shares for £70,000. 

Assume Shawna is a higher rate taxpayer. 

What amount of SEIS relief is available to Shawna?

Solution

Shawna should claim SEIS reinvestment relief on £13,900 of her capital gain as this ensures
that she gets early relief for her capital loss brought forward and ensures her annual exempt
amount is preserved.

Capital gain £35,000

Less:

SEIS Relief (£13,900)

Chargeable gain £21,100

Less:

Annual exemption (£12,300)



Capital loss (£8,800)

Taxable gain £Nil

Disposal of EIS Shares



Shawna is disposing of all her SEIS shares within 3 years and therefore her income tax relief
is withdrawn and the capital gain of £13,900 on the cottage is now chargeable in 2023/24.

Shawna must also pay CGT on the chargeable gain realised on the disposal of the shares. 

The SEIS shares are only exempt from CGT on capital gains if the shares are kept for at
least 3 years.


320
Syllabus: A2giii)
Advise on the availability of entrepreneurs’ relief/business asset disposal relief in relation to
associated disposals

Associated disposals for Entrepreneurs’ Relief/


Business asset disposal relief

Entrepreneurs' Relief (E.R.)/Business asset disposal relief

Associated Disposals

Normally, to qualify for E.R.(Business asset disposal relief) an entire interest in a business or
partnership must be disposed of. 

Only disposing of one asset cannot qualify for E.R. (Business asset disposal relief) 

However, there is an exception if the asset qualified as an Associated Disposal.

If an individual owns a building personally and it is being used in their sole trader/
partnership for business purposes, and that individual is disposing of his entire interest in
the business as well as the building, the building will qualify as an associated disposal, and
both disposals will attract E.R.(Business asset disposal relief)

Entrepreneurs’ relief/Business asset disposal relief is available on the disposal of the


building as an associated disposal provided the following conditions are met:

1 Individual disposes of their interest in a partnership or shares in a personal


company.

2 The intention is to no longer participate in the business using the premises.

3 The premises and the shares have been owned for at least two years.

321
Illustration

Jake disposed of his entire interest in a partnership which he owned for 3 years and
realised a capital gain of £100,000.

He also disposed of the building premises which was used by the partnership to conduct
it's trade, he owned it for 3 years and realised a capital gain of £50,000.

What amount of capital gains tax will he pay?

Solution

Both of these disposals qualify for E.R./Business asset disposal relief as the following
conditions have been met:


1) Individual disposes of their interest in a partnership or shares in a personal company.



2) The intention is to no longer participate in the business using the premises.

3) The premises and the shares have been owned for at least two years.


Therefore,

Capital gain £100,000

Capital gain £50,000

Total £150,000

Less A/E (£12,300)

Taxable gain £137,700


CGT £137,700 * 10% = £13,770


322
Syllabus: A2giv)
Understand and apply the relief that is available on the transfer of an unincorporated business
to a limited company

Transferring a business to a company

Incorporation Relief

Where an individual transfers their unincorporated business (sole trader business/


partnership) to a company, the individual assets of the business are deemed to have been
disposed of at market value to the company. 

Incorporation relief is available to allow the gains arising on incorporation to be deferred


until the shares in the company are disposed of.

Conditions for the relief

All of the following conditions must be satisfied for the relief to be obtained.

When all of these conditions are met, the relief is automatic.

1 The unincorporated business is transferred as a going concern.

2 All of the assets of the business (other than cash) are transferred to a
company.

3 The consideration received for the transfer of the business must be received
wholly or partly in the form of shares in the company.

How to calculate incorporation relief?

If the consideration is fully in shares, then the whole capital gain is deferred by deducting it
from the cost of shares, producing a lower base cost, which will be used to calculate the
capital gain when the shares are disposed of.

If the consideration is only partly in shares, then the following formula is used to calculate
the amount of gain deferred:

Deferred gain = Total capital gain * (M.V. of the shares received/M.V of the total
consideration)

This deferred gain is deducted from the cost of the shares, to produce a lower base cost,
which will be used to calculate the capital gain when the shares are disposed of.

323
Illustration

Jake has been a sole trader for the last 5 years and now intends to sell his business to Jake
Ltd.

His business is valued at £540,000 and he will receive shares in Jake Ltd in respect of the
market value. 

This will result in a chargeable gain of £160,000 in respect of the business premises and
£30,000 on goodwill. 

He will sell the shares for £600,000 in 6 months. 

He is a higher rate tax payer and has uses his annual exempt amount in full.

Should he allow the automatic incorporation relief to apply or should he specially elect for it
not to apply?

Solution

With incorporation relief

He has received the consideration fully in shares with a market value of £540,000 -
therefore the entire gain can be deferred.

Market value £540,000

Less C. gain (£160,000 + £30,000)

Base cost of shares £350,000

Sale of shares

Sale proceeds £600,000

Less base cost of shares (£350,000)

Chargeable gain £250,000

CGT (£250,000 * 10%) = £25,000 (ER relief applies as he has held an interest in the
business (sole trade + shares) for more than 2 years.)

Note: remember that if incorporation relief has applied on the sale of an unincorporated
business to a company, the two year qualifying period on the disposal of the shares
includes the period for which the individual owned the unincorporated business.

Specially elect for Incorporation relief not to apply



He is disposing of his entire business which he has owned for more than 2 years, therefore
entrepreneurs’ relief/business asset disposal relief will apply. 

Remember that entrepreneurs’ relief/business asset disposal relief will not apply for the
disposal of goodwill, as this is a close company. 

CGT for premises £160,000 * 10% = £16,000

CGT for goodwill £30,000 * 20% = £6,000


324
CGT on disposal of the sole trade £22,000

However, in this situation, on the sale of the shares, the 2 year qualifying period would not
have been met and as such Entrepreneurs’ relief/business asset disposal relief will not
apply on the disposal of the shares.

CGT on sale of shares

Sale proceeds £600,000



Less base cost of shares (£540,000)

Chargeable gain £60,000

CGT (£60,000 * 20%) = £12,000

Total CGT due if incorporation relief is disapplied £34,000 (£22,000 + £12,000)

Conclusion

He should not elect to disapply incorporation relief as this results in a lower total payment
of CGT.


325
Syllabus A3: Inheritance Tax
Syllabus A3a. TX - UK Recap: Basic principles of
computing transfers of value

The contents of the Paper TX - UK study guide for inheritance tax, under headings: 

- Basic principles of computing transfers of value

Chargeable persons

Chargeable persons

A person who is domiciled in the UK is liable to IHT in respect of their worldwide assets.

Additionally, the only relevant chargeable person is an individual.

Married couples (and registered civil partnerships) are not chargeable persons because
each spouse (or civil partner) is taxed separately.

326
Diminution in value principle

Transfers of value may need to be calculated using the diminution in


value principle

Ordinary shares in unquoted company as a GIFT

Normally, as seen in Topic Transfer of value, Chargeable Transfer, Potentially Exempt


Transfer for most assets the transfer of value will be the same as the open market value of
the asset

e.g. gifting a property worth £250,000 or cash of £100,000, but for some assets, notably
shares in unquoted companies the transfer of value may be considerably higher than the
market value of the asset being gifted.

The transfer of value will be calculated as the difference in estate value before and after the
gift of the asset.

Illustration:

A owns 60% of the shares in A Ltd. A Ltd has 100,000 £1 ordinary shares in issue.

Share valuations have been agreed as follows:

20% £10 per share


40% £15 per share
60% £25 per share
80% £40 per share

Required:

Compute the transfer of value if A were to die leaving his shares to his daughter, or
alternatively if he were to make a lifetime gift of 20,000 shares to his daughter.

Solution:

• If A died owning his 60,000 shares, a 60% shareholding, they would be valued at
£25 per share i.e. 60,000 @ £25 = £1,500,000.

• If, however, he were to give 20,000 shares in lifetime the transfer of value would not
be based on the value of a 20% interest i.e. £10 per share, but would be computed
as the difference between the value of his estate before and after the transfer:

327
Before 60,000 shares (60%) @ £25 = 1,500,000

After 40,000 shares (40%) @ £15 = 600,000

Transfer of Value 900,000

A transfer of value will arise by the gift of an asset either in lifetime and / or on death.

For most taxpayers, as stated above, their only transfers of value will arise as a result of
their death.

Illustration:

B owns 80% of the shares in B Ltd. B Ltd has 100,000 £1 ordinary shares in issue.

Share valuations have been agreed as follows:

20% @ £10/share

40% @ £15/share

60% @ £25/share

80% @ £40/share

Compute the transfer of value and IHT payable if B were to die 2 years after leaving 20,000
shares to his daughter.

All exemptions and Nil Rate Band have been used up.

Solution:

If B died owning his 80,000 shares, an 80% shareholding, they would be valued at £40 per
share i.e. 80,000 @ £40 = £3,200,000.

If, however, he were to give 20,000 shares in lifetime the transfer of value would not be
based on the value of a 20% interest i.e. £10 per share, but would be computed as the
difference between the value of his estate before and after the transfer:

Before 80,000 shares * £40 = £3,200,000



After transfer 60,000 shares * £25 = £1,500,000

Value of PET

£3,200,000 - £1,500,000 = £1,700,000

IHT payable £1,700,000 * 40% = £680,000

328
7 year accumulation principle

What and how is inheritance tax paid on?

2 things to remember

1. Firstly, every individual receives a nil rate band. If their total chargeable transfers exceed
this nil rate band, only then is inheritance tax payable.

2. Secondly, if a transfer is made MORE than 7 years before an individual dies, then
inheritance tax on death will not be paid on that transfer.

The nil rate band is £325,000, and for previous years it has been:

2007-08 3,00,000

2008-09 3,12,000

2009-10 3,25,000

2010-11 325,000

2011-12 325,000

2012-13 325,000

2013-14 325,000

2014-15 325,000

2015-16 325,000

2016-17 325,000

2017-18 325,000

2018-19 325,000

2019-20 3,25,000

2020-21 3,25,000

2021-22 3,25,000

The rate of IHT payable as a result of a person’s death is 40%

329
This is the rate that is charged on:

• a person’s estate at death

• PETs that become chargeable as a result of death within seven years

• any additional tax payable on CLTs made within seven years of death

The rate of IHT payable on CLTs at the time they are made is 20% (half the death rate). This
is the lifetime rate.

The tax rates information that will be given in the tax rates and allowances section of
the exam in this period is:

£1 – £325,000 Nil

Excess – Death rate 40%

– Lifetime rate 20%

Where nil rate bands are required for previous years then these will be given to you within
the question.

Illustration 1:

Sophie died on 26 May 2022 leaving an estate valued at £600,000.

The IHT liability is as follows:

Death estate
£
Chargeable estate 600,000

IHT liability
325,000 at nil% 0

(600,000 - 325,000) = 275,000 at 40% 110,000


110,000

Illustration 2:

Ming died on 22 April 2022 leaving an estate valued at £300,000.

On 30 April 2020, she had made a gift of £240,000 to her son.

This figure is after deducting available exemptions.

330
331
Solution:

IHT liabilities are as follows:

Lifetime transfer – 30 April 2020 (Less than 7 years)

Potentially exempt transfer 240,000

The PET is initially ignored.

Additional liability arising on death

The IHT must be calculated for the gift £240,000, because Ming gave it to her son

Less than 7 years before she died.

Potentially exempt transfer 240,000

The PET utilises £240,000 of the nil rate band of £325,000. No IHT is payable.

Death estate

£
Chargeable estate 300,000
IHT liability
(325,000 - 240,000) = 85,000 at nil% 0
(300,000 - 85,000) = 215,000 at 40% 86,000
86,000

Only £85,000 (325,000 – 240,000) of the nil rate band is available against the death estate.

332
Illustration 3:

Joe died on 13 October 2022 leaving an estate valued at £750,000.

On 12 November 2019, he had made a gift of £400,000 to a trust.

This figure is after deducting available exemptions.

The trust paid the IHT arising from the gift.

The nil rate band for the tax year 2019/20 is £325,000.

Lifetime transfer – 12 November 2019

£
Chargeable transfer 400,000
IHT liability
325,000 at nil% 0
(400,000 - 325,000) = 75,000 at 20% 15,000
15,000

The gift to a trust is a CLT. The lifetime IHT liability is calculated using the nil rate band for
2019/20.

Additional liability arising on death


£
Chargeable transfer 400,000
IHT liability
325,000 at nil% 0
75,000 at 40% 30,000
IHT already paid (15,000)
Additional liability 15,000

The additional liability arising on death is calculated using the nil rate band for 2022/23.

333
Death estate

£
Chargeable estate 750,000
IHT liability   750,000 at 40% 300,000

The CLT made on 12 November 2019 has fully utilised the nil rate band of £325,000.

A NRB is given every year, therefore the CLT used the NRB of 19/20 when lifetime tax
was calculated

The NRB of 22/23 is used to calculate tax paid on death, it is first given to the CLT and
then the death estate because it is allocated in chronological order

Illustration 4:

Mr Wealthy made the following gifts during his lifetime.

01/11/2021 £333,000 into a trust for his son (the trustees paid any life tax)

14/11/2022 £50,000 cash to his daughter

He died in January 2023 with an estate valued at £500,000.

What is his IHT payable during his lifetime and on his death?

Ignore annual exemptions.

The NRB of 21/22 and 22/23 are £325,000.

Solution:

Gift to trust £333,000



Less NRB (£325,000)

£8,000 *20% = £1,600 lifetime tax.

On death:

NRB is £325,000 (New NRB for 22/23)

This is allocated on chronological order - first to the CLT, then PET, then death estate.

CLT £333,000 

NRB (£325,000)

£8,000 *40% = £3,200

IHT paid (£1,600)

IHT due £1,600

PET £50,000 * 40% = £20,000 IHT due

Death estate £500,000 *40% = £200,000 IHT due on death estate

334
The 7 year cumulation period

In most of the illustrations so far, all the lifetime transfers, both PET’s and CLT’s have taken
place within the 7 years prior to death and have all therefore been chargeable to IHT on the
death of the taxpayer.

Very important note The earliest / oldest transfers within this period are first to use the nil
rate band with the later transfers and / or the chargeable estate at death then being taxed
at 40%.

Basically, the nil rate band must be applied in chronological order - it is given to the gift
made earliest.

If PET’s have been made more than 7 years before the date of death

• they were neither chargeable when made nor chargeable on death, they are exempt
from IHT and are ignored.

A CLT made more than 7 years before death

• These transfers were chargeable when made using the nil rate band in force at that date
but are not chargeable on death as the taxpayer has survived for the required 7 years.

The 7 year cumulation period, however means that when computing the IHT on either a
PET or CLT made within the 7 years of death it is necessary to take account of any CLT
made within the 7 years prior to it, so as to determine how much nil rate band, if any,
remains to use against that transfer.

For example

If an individual dies in January 2023 having made a CLT in June 2012 of £255,000, this CLT
will not be taxable on the death as he survived for more than 7 years.

If he had also made a PET in August 2018 of £200,000 this will be taxable.

In computing the nil rate band available to go against the PET, however, the £325,000 will
be reduced by the amount of the June 2012 CLT as this was within the 7 years prior to the
PET.

Therefore, the NRB available to the PET would be (£325,000-£255,000) = £70,000

IHT payable on the PET



Value of PET £200,000

Less NRB (£70,000)

£130,000 *40%  = £52,000

Illustration:

Lachman made a CLT in 2012 of £200,000 and a PET in 2016 of £255,000

He died in 2022 with an estate value of £500,000.

335
NRB in 2022 is £325,000.

How will the NRB be shared?



How much IHT will be payable?

Solution:

He will have to pay death tax when he dies, his NRB of 2022 will first go to the PET BUT,
the PET must share it with the CLT because it was within 7 years of the PET.

Therefore, NRB available to PET is (£325,000-£200,000) = £125,000

IHT payable on PET



Value £255,000

Less NRB (£125,000)

£130,000 * 40% = £52,000

Note: the CLT is more than 7 years before death and as such it will not be subject to death
tax.


Then, for his death estate - the NRB will only be shared with the PET as that is within 7
years of the date of date.

NRB available on death = £325,000 - £255,000 (Value of PET) = £70,000

IHT on death estate:



Estate value £500,000

Less NRB (£70,000)

£430,000 * 40% = £172,000

Illustration:

Lachman made a PET in 2012 of £200,000 and a PET in 2016 of £255,000.

He died in 2022 with an estate valued at £500,000.

NRB is £325,000.

How will the NRB be shared?



How much IHT will be payable?

Solution:

He will have to pay death tax when he dies, his NRB of 2022 will first go to the 2016 PET of
£255,000 BUT, the PET will not share it with the 2012 PET of £200,000 because that was
given more than 7 years before death and therefore exempt from IHT.

IHT payable for PET



Value of PET £255,000


336
NRB (£255,000)

Nil payable

Remaining NRB for the death estate £325,000 - £255,000 = £70,000

The NRB for the death estate will only be shared with the 2016 PET of £255,000 as that is
within 7 years.

IHT payable on death estate



Value of estate £500,000

Less NRB (£70,000)

£430,000*40% = £172,000

Note on death, NRB is shared with PET's that occur within 7 year's of death.

Note

When a gift is made during lifetime, the value of the gift is frozen and will only become
chargeable to IHT when the person who made the gift dies.

Therefore, if a grandmother makes a gift to her grandchild - IHT will only be paid once by
the grandmother and the second time when the grandchild dies, this avoid's the additional
IHT payable if the grandmother made a gift directly to her child.

To avoid additional IHT - making gifts to grandchildren as opposed to children is a good


option.


337
Syllabus A3a. TX - UK Recap: IHT arising on lifetime
transfers and on death

The contents of the Paper TX - UK study guide for inheritance tax, under headings: 

- IHT arising on lifetime transfers and on death

Tax implications of lifetime transfers

What tax liability arises on lifetime transfers?

When calculating the tax liability on lifetime transfers, there are three aspects that are a bit
more difficult to understand and can therefore cause problems.

Chargeable lifetime transfer preceded by a potentially exempt transfer that becomes


chargeable

The situation where a chargeable lifetime transfer (CLT) is made before a potentially exempt
transfer (PET) is fairly straightforward, and was covered previously.

However, where the sequence of gifts is reversed, the IHT calculations are more
complicated because the PET will use some or all of the nil rate band previously given to
the CLT.

Illustration:

Ali died on 3 March 2023. He had made the following lifetime gifts:

• 1 August 2020 – A gift of £360,000 to his son

• 21 November 2021 – A gift of £240,000 to a trust

These figures are after deducting available exemptions.

The nil rate band for all the tax years is £325,000.

338
IHT liabilities are as follows:

Lifetime transfers £

1 August 2020

Potentially exempt transfer 360,000

21 November 2021

Chargeable transfer 240,000

No lifetime IHT is payable because the CLT is less than the nil rate band for 2021/22.

Additional liabilities arising on death

1 August 2020
Potentially exempt transfer 360,000

IHT liability 325,000 at nil% 0


35,000 at 40% 14,000
14,000
£
21 November 2021
Chargeable transfer 240,000

IHT liability 240,000 at 40% 96,000


IHT already paid (Nil)
Additional liability 96,000

The nil rate band for 2022/23 of £325,000 has been fully utilised by the PET made on 1
August 2020.

339
Grossing up

So far, in all of the examples concerning a CLT, the trust (the donee) has paid any lifetime
IHT at the rate of 20%.

However, when the donor of the gift is paying IHT on the gift into the trust the rate of 20/80
(25%) is used as the gift is deemed to be the net amount of money that is leaving the
donors estate. The estate value falls by the gift plus the tax in this case.

 

For the death tax calculations, the amount of the gift will need to be grossed up by the
amount of the tax. Any available annual exemptions are deducted prior to grossing up.

The annual exemptions are explained in Topic Exemptions - there is an annual exemption of
£3,000 each year that is allowed, and if the annual exemption of £3,000 of the previous year
has not been used, this can be brought forward and used in the current year, after
allocating the current year annual exemption

For example if a cash gift was made into a trust in February 2023 of £100,000 and no other
gifts had been made previously, then the annual exemptions of £3,000 (22/23) and £3,000
(21/22) would be deducted first.

£100,000 - £3,000-£3,000 = £94,000 would be the value of the transfer and IHT would be
calculated on the 94,000.

Illustration:

On 17 June 2019, Annie made a gift of £406,000 to a trust. She paid the IHT arising from
the gift.

Annie has not made any other gifts since 6 April 2019.

The nil rate band for the tax year 2019/20 is £325,000.

340
The lifetime IHT liability is calculated as follows:

£ £

Value transferred 406,000

Annual exemptions

2019/20 3,000

2018/19 3,000

(6,000)

Net chargeable transfer 400,000

IHT liability

325,000 at nil% 0

75,000 x 20/80 18,750

Gross chargeable transfer 418,750

The amount of lifetime IHT payable by Annie is £18,750. This figure can be checked by
calculating the IHT on the gross chargeable transfer of £418,750:

IHT liability

325,000 at nil% 0

93,750 at 20% 18,750

18,750

Once the gross chargeable transfer has been calculated, then this figure is used in all
subsequent calculations.

CLTs are never re-grossed up on death, even if the nil rate band is reallocated as a result of
a PET becoming chargeable.

341
Illustration:

Continuing with example 2, Annie died on 12 March 2023

Additional liability arising on death

12 March 2023 £

Gross chargeable transfer 418,750

IHT liability

325,000 at nil% 0

93,750 at 40% 37,500

Taper relief reduction – 20% (7,500)

30,000

IHT already paid (18,750)

Additional liability 11,250

When an IHT question involves a CLT, then make sure you know who is paying the IHT.
Grossing up is not necessary if the trust (the donee) pays.

Seven-year cumulation period

Jayne died on 18 March 2023 leaving an estate valued at £450,000. She had made the
following lifetime gifts:

• 1 August 2014 – A gift of £200,000 to a trust

• 1 November 2020 – A gift of £280,000 to a trust

These figures are after deducting available exemptions. In each case, the trust paid any IHT
arising from the gift.

Note: if the question in the exam does not say who pays the tax then you will always
assume that the donor pays the tax and use 20/80 (25%).

The nil rate band for the tax year 2014/15 is £325,000, and for the tax year 2020/21 it is
£325,000.

342
IHT liabilities are as follows:

Lifetime transfers - 1 August 2014 £

Chargeable transfer 200,000

No lifetime IHT is payable because the CLT is less than the nil rate band for 2014/15.

Lifetime transfers - 1 November 2020 £

Chargeable transfer 280,000


IHT liability

(325,000 - 200,000) = 125,000 at nil% 0


155,000 at 20% 31,000
31,000

The CLT made on 1 August 2014 is within seven years of 1 November 2020, so it utilises
£200,000 of the nil rate band for 2020/21.

Additional liabilities arising on death

1 August 2014 £
Chargeable transfer 200,000

There is no additional liability because this CLT was made more than seven years before the
date of Jayne’s death on 18 March 2023.

1 November 2020 £
Chargeable transfer 280,000
IHT liability
125,000 at nil% 0
155,000 at 40% 62,000
IHT already paid (31,000)
Additional liability 31,000

The CLT made on 1 August 2014 utilises £200,000 of the nil rate band for 2022/23 of
£325,000 because it was made 7 years before this CLT.

343
Death estate

£
Chargeable transfer 450,000

IHT liability
45,000 at nil% 0

405,000 at 40% 162,000

162,000

The CLT made on 1 August 2014 is not relevant when calculating the IHT on the death
estate because it was made more than seven years before the date of Jayne’s death on 18
March 2023.

Therefore, only the CLT made on 1 November 2020 is taken into account, and this utilises
£280,000 of the nil rate band of £325,000.

Total IHT: life tax £31,000 + death tax £31,000 + 162,000 = £224,000

Illustration:

The same situation as in example 4, except that on 1 November 2020 Jayne made a gift of
£280,000 to her daughter rather than to a trust.

IHT liabilities are as follows:

Lifetime transfers £

1 August 2014

Chargeable transfer 200,000

1 November 2020

Potentially exempt transfer 280,000

344
Additional liabilities arising on death

£
1 August 2014
Chargeable transfer 200,000

1 November 2020
Potentially exempt transfer 280,000

IHT liability
(325,000 - 200,000) = 125,000 x 0%
(280,000 - 125,000) = 155,000 x 40% 62,000

The CLT is entirely covered by the NRB, therefore there is no lifetime tax payable, and
it was made 7 years before death, therefore there will be no additional death tax
payable.

Notice that the PET does not pay lifetime tax, and on death it used NRB first,
therefore paying no death tax either.

Death estate

Chargeable estate 450,000


IHT liability
45,000 at nil% 0
405,000 at 40% 162,000
162,000

Total IHT: Life tax £0, death tax £162,000.


345
Transfer of unused NRB between spouses

What if a spouse does not use their entire nil rate band?

This is how the other spouse benefits:

Any unused nil rate band on a person’s death can be transferred to their surviving spouse
(or registered civil partner).

The nil rate band will often not be fully used on the death of the first spouse because any
assets left to the surviving spouse are exempt from IHT (see the following section on
transfers to spouses).

A claim for the transfer of any unused nil rate band is made by the personal representatives
who are looking after the estate of the second spouse to die.

The amount that can be claimed is based on the proportion of the nil rate band not used
when the first spouse died.

Even though the first spouse may have died several years ago when the nil rate band was
much lower, the amount that can be claimed on the death of the second spouse is
calculated using the current limit of £325,000.

Illustration:

Nun died on 29 March 2023.

None of her husband’s nil rate band was used when he died on 5 May 2008.

When calculating the IHT on Nun’s estate a nil rate band of £650,000 (325,000 + 325,000)
can be used because a claim can be made to transfer 100% of her husband’s nil rate band.

Note: the nil rate band in 2008/09 was not £325,000 but it is the unused percentage that is
carried forward and not the amount of unused nil band. Therefore, Nun can use 100% of
the current value of the nil band as well as her own nil band.

Illustration:

Win died on 24 February 2023 leaving an estate valued at £800,000. Only 60% of his wife’s
nil rate band was used when she died on 12 May 2009.

On 10 May 2018, Win had made a gift of £200,000 to his son. This figure is after deducting
available exemptions.

The nil rate band for the tax year 2018/19 is £325,000

IHT liabilities are as follows:


346
Lifetime transfer – 10 May 2018

Potentially exempt transfer 200,000

No life tax on a PET.

Additional liability arising on death – 10 May 2018

Potentially exempt transfer 200,000

Covered by the 2018/19 NRB

Death estate

Chargeable estate 800,000

IHT liability

255,000 at nil% (325,000 + (40% x 325,000) - 200,000) 0

545,000 at 40% 218,000


218,000

Win’s personal representatives can claim the wife’s unused nil rate band of £130,000
(325,000 x 40%).

The amount of nil rate band is therefore £455,000 (325,000 + 130,000), of which £200,000 is
utilised by the PET made on 10 May 2018.

347
Residence NRB

An additional nil rate band has been introduced where a main residence is inherited on
death by direct descendants (children and grandchildren).

For the tax year 2022/23, the residence nil rate band is £175,000.

The residence nil rate band is only available if:

1) The individual dies on or after 6 April 2017

2) Their estate includes a main residence

3) It is inherited by direct descendants

Illustration:

Sophie died on 26 May 2022 leaving an estate valued at £800,000. Under the terms of her
will, Sophie’s estate was left to her children. The estate included a main residence valued at
£250,000.

Solution:
The inheritance tax (IHT) liability is:

Chargeable estate £800,000

IHT liability:

£175,000 at 0% (residence NRB)

£325,000 at 0% (NRB)

£300,000 *40% = £120,000

The residence nil rate band of £175,000 is available because Sophie’s estate included a
main residence and this was left to her direct descendants.

Transferring Residence NRB


In the same way in which any unused normal nil rate band can be transferred to a surviving
spouse (or registered civil partner), the residence nil rate band is also transferable. It does
not matter when the first spouse died.

Illustration:

Trevor died on 19 June 2022 leaving an estate valued at £700,000. Under the terms of his
will, Trevor’s estate was left to his children. The estate included a main residence valued at
£300,000.

Trevor’s wife died on 5 May 2009. She used all of her nil rate band of £325,000.

348
Solution:
Trevor’s IHT liability is:

Chargeable estate £700,000

IHT liability:

£675,000 at 0% (£325,000 NRB + £175,000 Trevor Residence NRB + £175,000 Trevor’s


wife’s unused residence NRB)

£25,000 *40% = £10,000

Trevor’s personal representatives can claim the wife’s unused residence nil rate band of
£175,000 even thought the residence nil rate band did not exist when Trevor’s wife died.

The amount of residence nil rate band is therefore £350,000 (175,000 + 175,000).

Note:
1) The value of the main residence is after deducting any repayment mortgage or
interest-only mortgage secured on that property (as normal).

2) If a main residence is valued at less than the available residence nil rate band, then
the residence nil rate band is reduced to the value of the residence.

If an individual’s death estate is valued at more than £2 million then the residence
NRB is reduced by £1 for every £2 that the estate is over £2 million. The whole
residence NRB will be withdrawn once the value of the estate exceeds £2,350,000
million.


349
Syllabus A3a. TX - UK Recap: Exemptions to defer /
minimise IHT

The contents of the Paper TX - UK study guide for inheritance tax, under headings: 

- The use of exemptions in deferring and minimising inheritance tax liabilities

Exemptions

Which exemptions are available for IHT?

Transfers to spouses

Gifts to spouses (and registered civil partners) are exempt from IHT. This exemption applies
both to lifetime gifts and on death.

Illustration:

Sophie died on 25 June 2022.

On 12 April 2018, she had made a gift of £400,000 to her husband.

Sophie's estate on 25 June 2022 was valued at £900,000.

Under the terms of her will, Sophie divided her estate equally between her husband and her
daughter.

The nil rate band for the tax year 2018/19 is £325,000.

IHT liabilities are as follows:

Lifetime transfers

The gift on 12 April 2018 is exempt as it is to Sophie’s husband.

350
Death estate

£
Value of estate 900,000
Spouse exemption (900,000/2) (450,000)
Chargeable estate 450,000
IHT liability
325,000 at nil% 0
125,000 at 40% 50,000
50,000

There are a number of other exemptions that only apply to lifetime gifts.

Small gifts exemption

Gifts up to £250 per person in any one tax year are exempt.

This exemption can not be used to reduce the value of a greater gift.

For example, a gift of £249 to one person will be wholly exempt, but if the gift is £251 it will
be wholly taxable. It is possible to use the exemption any number of times by making gifts
to different donees.

Illustration:

During the tax year 2022/23, Peter made the following gifts:

• On 18 May 2022, he made a gift of £240 to his son.

• On 5 October 2022, he made a gift of £400 to his daughter.

• On 20 March 2023, he made a gift of £100 to a friend.

The gifts on 18 May 2022 and 20 March 2023 are both exempt because they do not exceed
£250.

The gift on 5 October 2022 for £400 does not qualify for the small gifts exemption because
it is more than £250.

The whole amount of £400 will be chargeable unless it can be covered by Peter’s annual
exemption for 2022/23 (see the next section).

351
Annual exemption

Each tax year a person has an annual exemption of £3,000.

If the whole of the annual exemption is not used in any tax year, then the balance is carried
forward to the following tax year.

However, the exemption for the current tax year must be used first, and any unused
brought forward exemption cannot be carried forward a second time.

Therefore, the maximum amount of annual exemptions available in any tax year is £6,000
(£3,000 x 2).

Illustration:

Simone made the following gifts:

• On 10 May 2021, she made a gift of £1,400 to her son.

• On 25 October 2022, she made a gift of £4,000 to her daughter.

The gift on 10 May 2021 utilises £1,400 of Simone’s annual exemption for 2021/22.

The balance of £1,600 (3,000 – 1,400) is carried forward to 2022/23.

The gift on 25 October 2022 utilises all of the £3,000 annual exemption for 2022/23 and
£1,000 (4,000 – 3,000) of the balance brought forward of £1,600.

Because the annual exemption for 2022/23 must be used first, the unused balance brought
forward of £600 (1,600 – 1,000) is lost.

The annual exemption is applied on a strict chronological basis, and is therefore given
against PETs even when they do not become chargeable.

Illustration:

Nigel made the following gifts:

• On 17 May 2021, he made a gift of £60,000 to his son

• On 25 June 2022, he made a gift of £100,000 to a trust.

The gift on 17 May 2021 utilises Nigel’s annual exemptions for 2021/22 and 2020/21.

The value of the PET is £54,000 (60,000 – 3,000 – 3,000).

The gift on 25 June 2022 utilises Nigel’s annual exemption for 2022/23. The 2021/22 annual
exemption is not available as it has been used by the PET.

The value of the CLT is £97,000 (100,000 – 3,000). No lifetime IHT liability is payable
because this is within the nil rate band for 2022/23.

Ideally the gift to the trust should have been made before the gift to the son.

352
Normal expenditure out of income

IHT is not intended to apply to gifts of income.

Therefore, a gift is exempt if it is made as part of a person’s normal expenditure, is made


out of income and that person is left with sufficient income to maintain their normal
standard of living.

To count as normal, gifts must be habitual.

Therefore, regular annual gifts of £2,500 made by a person with an annual income of
£100,000 would probably be exempt.

A one-off gift of £70,000 made by the same person would probably not be, and would
instead be a PET or a CLT.

Gifts in consideration of marriage

This exemption covers gifts made in consideration of a couple getting married or registering
a civil partnership.

The amount of exemption depends on the relationship of the donor to the donee (who must
be one of the two persons getting married):

• £5,000 if the gift if made by a parent.

• £2,500 if the gift is made by a grandparent or by one of the couple getting married to the
other.

• £1,000 if the gift is made by anyone else.

Illustration:

On 19 September 2022, William made a gift of £20,000 to his daughter when she got
married.

He has not made any other gifts since 6 April 2022.

The gift is a PET, but £5,000 will be exempt as a gift in consideration of marriage and
William’s annual exemptions for 2022/23 and 2021/22 are also available.

The value of the PET is therefore £9,000 (20,000 – 5,000 – 3,000 – 3,000).

This remaining £9,000 will only be chargeable to IHT if William dies before 19/09/2029
(within 7 years of making the gift)

Note: the marriage exemption is deducted from the value of the gift BEFORE the annual
exemptions.

353
Syllabus A3b. The scope of IHT

Syllabus: A3bi)
Explain the concepts of domicile and deemed domicile and understand the application of
these concepts to inheritance tax.

Domicile

Domicile

Who pays UK IHT?

• An individual who is UK domiciled is charged to UK IHT on his worldwide


assets.

• An individual who is not UK domiciled is charged to UK IHT only on assets


situated in the UK.

What is domicile?

Domicile means place of permanent home and an individual can only have one domicile at
a particular point in time.

Four types of domicile for IHT

• Domicile of origin – automatically taking the domicile of their father at birth. 



Therefore, this is the place of their father’s permanent home.

For example, if your father’s permanent home is London when you
are born, then you will be UK domiciled.

• Domicile of dependency – if your father changes his domicile before you are
16, then your domicile will change with his. 

For example, if your father moves to France from London before you are 16
years old, and that is his permanent home now, both of you will be
French domiciled

354
• Domicile of choice – an individual can change their domicile from one
country to another if they show permanent intention to change the country of

permanent home. 

Showing permanent intention means not retaining property, moving burial
arrangements and changing nationality/citizenship from one country to the
other. 

For example, if you were UK domiciled since birth but you emigrated to
France with the intention of remaining there permanently (you sold all
property in London, you changed your nationality and you created burial
arrangements in France), then you will be French domiciled.

• Deemed domicile - for an individual to be deemed domicile in the UK for


IHT purposes at the relevant time (ie at the time of a transfer of value) they
must satisfy any one of the following three conditions.

• 1. Deemed domicile – an individual who has been domiciled in the UK can


move abroad and make another country their permanent home but they will
still retain UK domicile for 3 years after they change their domicile. 

For example, you moved to France and became French domiciled, but you
will still be considered to be UK domiciled for 3 years after the move.

• 2. Deemed domicile – this also applies to an individual who was never UK


domiciled but has been resident in the UK for at least 15 years out of the
previous 20 tax years immediately preceding the relevant tax year, and for at
least 1 of the 4 tax years ending with the relevant tax year. 

For example, if you have been UK resident since 2002, but never UK
domiciled, and you made a gift of a home in France in 2022, this gift will be
chargeable to UK IHT, because you are deemed domicile, as you have been
resident in the UK for the last 20 tax years.

• 3. The individual is a formerly UK domiciled individual. This is an individual


who:

was born in the UK; and

has a UK domicile of origin; and

is UK resident in the relevant tax year; and

was UK resident in at least 1 of the 2 tax years immediately before the


relevant tax year.

For example, Tom was born in the UK in 1975 and his father was UK
domiciled. In 2003 he moved to Australia. He returned to the UK in August
2022. He will be deemed UK domicile in 2022/23.

355
UK Domicile election

A person who is domiciled overseas can make an election to be treated as UK domiciled


provided the necessary conditions are met.

1 The non-domiciled person must be/or was married to a person who is UK


domiciled. 

The election can also be made following the death of the UK domiciled
spouse or civil partner. 

In this case the election must be made within two years of the date of the
spouse or civil partner’s death.

2 The election must be made after 5 April 2013

3 The election can be backdated for up to 7 years, but not before 6 April 2013.

4 The election is irrevocable, but if a person makes the election and is non-UK
resident for four consecutive tax years, the election will lapse

Transfers between spouses

Both spouses UK domiciled:

Gifts to spouses (and registered civil partners) are exempt from IHT. This exemption applies
to both lifetime gifts and on death.

UK domiciled spouse transferring to Non UK Domiciled spouse -

There is a maximum exemption of £325,000

356
Syllabus: A3bii/iii)
Identify excluded property and
Identify and advise on the tax implications of the location of assets

Excluded Property

UK vs non-UK domiciled individuals

1 When a person is UK domiciled they must pay IHT on UK and overseas


located assets

2 A non-UK domiciled individual is only liable to IHT on his UK assets.



Therefore, property that will be excluded from the IHT computation is:

Property situated overseas where the owner is not UK domiciled.

This is because, someone who is UK domiciled will pay IHT on their
worldwide assets, but someone who is not UK domiciled will pay IHT on
their UK located assets.

Therefore, it is necessary to be able to identify where in the world an asset is deemed to be


located for IHT purposes, to see whether a non-UK domiciled person needs to pay UK IHT.


357
Illustration

Sam has lived in the UK for the last 6 years, however he is not UK domiciled. 

He owns the following property:

1 Freehold property situated in the UK

2 Leasehold property situated in the USA

3 Shares in USA Inc., a company quoted and registered on the US stock


exchange. 

Will Sam be liable to IHT if he makes transfers all of his property?

Solution

He will be chargeable to IHT on UK located assets.

• The freehold property is situated in the UK and so is a chargeable asset for


IHT.

• The leasehold property is situated in the USA, and therefore is not a


chargeable asset for IHT.

• The shares is USA Inc. are registered in the USA and are therefore not a
chargeable asset for IHT.


358
Syllabus: A3biv)
Identify and advise on gifts with reservation of benefit

Gifts with reservation

What is a gift with reservation?

Gifts with reservation

A gift with reservation is a lifetime gift where:

• The legal ownership of an asset is transferred but

• The donor retains some benefit in the asset gifted



For example, a donor gifting a house but continues to live in it or the gift of
shares but the donor retaining the rights to future dividends.

Why would the donor make the gift if they still want to use the asset?

Gifting appreciating assets during lifetime is beneficial because the value of the gift will be
frozen at the date of gift, therefore if the value increases when the donor dies, tax will only
be paid on the value at the time of gift.

Gift with reservation rules

HMRC has introduced special anti-avoidance rules for a GWR to ensure that they do not
escape the correct IHT payment.

For a GWR, HMRC will choose the value of the asset to use for taxation at the time of
death. If the value of the asset is higher at the time of death, then they will use the market
value at the time of death.

Exception to GWR rules

If the donor pays market value for the benefit of still using the asset, then the GWR rules will
be lifted and it will be treated as a normal lifetime gift.

359
For example, if a donor gifts a house and continues to use it, but pays full market rent to
the donee while living in the house, then he has paid for the benefit fully and this will not fall
under the GWR rules.

Illustration

Priya gave her house to her son Sushil on 01/04/2017 when it was worth £360,000. She
had made no previous lifetime gifts.

She continued to live in the house and paid no rent to Sushil. 

She died on 01/01/2023 when the house was valued £500,000. 

How much IHT will be payable on this gift?

Solution

Treating it as a lifetime transfer

01/04/2017 PET £360,000

16/17 Annual Exemption (£3,000)

15/16 Annual Exemption (£3,000)

Gross chargeable amount £354,000

NRB 22/23 (£325,000)

Taxable amount £29,000

IHT 40%* £29,000 = £11,600

Less:

Taper relief (5-6 years 60% * £11,600) = (£6,960) (see topic Taper Relief)

IHT payable £4,640

Treating it as a death gift



Market value at death £500,000

Less RNRB and NRB (£175,000+ £325,000)

Taxable amount £Nil

IHT 40% * £0 = £0

The higher amount of £4,640 will be payable and it will be treated as part of the death
estate.

Note: the RNRB is not available against life gifts. It is only available in the death estate,

360
Syllabus A3c. Computing transfers of value
Syllabus: A3ci) Advise on the principles of valuation

Principles of valuation

Valuation rules

Principles

Generally assets are valued at their open market value, but there are other rules that often
need to be applied.

Valuation rules for quoted shares and securities



Quoted shares and securities including investment trusts, open ended


investment companies (OEICs), gilts, venture capital trusts (VCTs) and real
estate investment trusts (REITs).


The market value at the time of the transfer is the lower of:


Method one – 1⁄4 up method



The lower quoted value plus one-quarter of the difference between the lower and the higher
value.


Lower value + 1⁄4 (Higher value - Lower value)



Method two – average method



Take the higher and add it to the lower and take the average value by
dividing by two.


Average Value = (Higher value + Lower value) / 2

361
Illustration

Harry died on 5 October 2022 he left his £1 ordinary shares in Peel plc to his two children.

Harry owned 100,000 £1 ordinary shares in Peel plc a quoted company with an issued
share capital of 10,000,000 £1 ordinary shares. 

On 5 October 2022 the price for these shares was quoted at 200–208 pence per share.

On 5 October 2022, the mid recorded bargains were 200p, 203p, 207p per share.

What value should be included in the death estate for the shares in Peel plc?

Solution

Lower of:

1/4 up method:

200 + 1⁄4 x (208-200) = 202p

Mid recorded bargain method:

200p+207p/2 = 203.5p

Take the lower value of 202p.


Death estate value £2.02*100,000 = £202,000.

Foreign property valuation

Individuals who are UK domiciled must pay IHT on their UK and foreign assets.

The foreign property is valued using professional valuation and then converted into sterling.

The value of the property can be reduced by administration expenses limited to a maximum
of 5% of the property's value.

362
Illustration

Jake owned a piece of land in Barbados and on his death it was valued at £45,000. 

Administration expenses amounted to £5,000.

What amount will be included in his death estate?

Solution

Value of land £45,000

Less admin expenses (5%*£45,000) = (£2,250)

Value £42,750


Note, even though the administration expenses amounted to £5,000, the maximum that
could be deducted was (5%*£45,000) = £2,250.

Related property rules

Who is classified as a related party?

Property is “related” to the donor’s property if it is part of the same asset owned by:

1) The donor’s spouse (or civil partner).

2) An exempt body as a result of a gift from that person or the spouse.

An exempt body means a qualifying charity and qualifying political party. 

Property held by the exempt body is deemed to be related:

1) For as long as the body owns the asset and 

2) For five years after they have disposed of it.

• When property is deemed to be related property, and it is being transferred,


then a formula must be used to calculate the value of the transfer:

((Value of individual share/Value of individual+Value of related property share)
* Total value of related parties combined share)

363
Illustration

Harriet owns three antique plates which are part of a set of six. 

Harriet’s husband owns two plates and her son owns one plate.

The plates are currently valued as follows:

1 plate £1,000

2 plates £2,200

3 plates £3,800

4 plates £6,000

5 plates £10,000

6 plates £20,000

If Harriet gives her son one plate on 1 January 2023 as a wedding present what is the value
of the PET before deducting relevant IHT reliefs.

Solution

In order to calculate the value of the PET, the diminution in value principle must be used. 

Before she makes the gift Harriet has 3 plates and afterwards she has 2 plates. 

When determining the value before and after we must also include the plates owned by her
husband a related party


The value of the PET is £3,333, calculated as follows:



Before transfer:

Harriet 3 plates

Related party (her husband) 2 plates

Total 5 plates

After transfer:

Harriet 2 plates

Related party (her husband) 2 plates

Total 4 plates

Value of Harriet’s three plates before she makes the gift:

((Value of 3 plates/Value of 3 plates + Value of 2 plates)) * Value of 5 plates

((£3,800/£3,800+£2,200)*£10,000) = £6,333

Less:

Value of Harriet’s two plates after she makes the gift:

((Value of 2 plates/Value of 2 plates + Value of 2 plates)) * Value of 4 plates

((£2,200/£2,200+£2,200)*£6,000) = £3,000


364
Therefore:

Value before £6,333

Less value after (£3,000)

Value of transfer £3,333

Related property rules for shares

In respect of shares, the valuation is determined by the number of shares the individual
holds in relation to the total number of shares (inclusive of related property)  multiplied by
the combined valuation. 

Remember the principle of value before less value after.

Illustration

On 15 June 2022 Joan gave 20,000 of her 40,000 ordinary shares in Rouen Ltd, an
unquoted trading company to her son Michael. 

Rouen Ltd has an issued share capital of 100,000 ordinary shares. 

Joan’s husband also owns 40,000 ordinary shares in the company. 

On 15 June 2022 the relevant values of Rouen Ltd shares were as follows:

100% £22.30

80% £17.10

60% £14.50

40% £9.20

20% £7.90

Joan purchased her 40,000 shares in Rouen Ltd during 1991 for £96,400.

What is the value of the PET for IHT of the gift made by Joan on 15 June 2022 before all
IHT reliefs?

Solution

The market value of the lifetime gift (PET) is computed by determining the diminution in
value of her estate.

Before

Joan 40,000

Husband 40,000

Total 80,000 shares at £17.10

After

Joan 20,000

Husband 40,000


365
Total 60,000 at £14.50

Value before

40,000 * 17.10 = £684,000

Value after

20,000 * £14.50 = (£290,00)

Value of PET = £394,000

Unit Trusts Valuation

Unit trusts are valued at the lowest bid price.

Do not use the 1/4 up method for valuation.

Illustration

Jimmy owns 5,000 units in Growing Unit Trust. At the time of his death, they are quoted at
125p-133p.

What is the value of the unit trust for IHT purposes?

Solution

5,000 units * 125p = £6,250


366
Syllabus: A3cii)
Advise on the availability of business property relief and agricultural property relief

Business property and agricultural property relief

Business Property Relief

BPR is a very important relief that significantly reduces the value of lifetime gifts and the
value of an individual’s death estate if certain conditions are satisfied.

What conditions need to be satisfied to obtain BPR?

1. The property must be relevant business property.

2. The property must have been owned for a continuous two years prior to the
transfer.

3. If the property has been transferred from a spouse the periods owned by
each spouse can be added together on the second transfer to see if BPR is
available. 

For example, if a husband owned relevant property for 1 year and transferred it to his
wife, then the wife owned it for another year and transferred it to their son, BPR will be
available on the transfer to the son because the total time of ownership for husband and
wife can be added up.

4. Where the property was a lifetime gift, the business property must either still be owned
by the donee or have been replaced with other relevant business property at the date of
the donor’s death in order to get BPR on the additional tax payable by the donee. 

For example,if a father gifted his son an unquoted shares holding during his
lifetime, this will be a P.E.T. and IHT will only be payable if the father dies
within 7 years of making the gift. If the son sells the shares, he must replace them with
relevant property for BPR in order to get the relief when the father dies.

5. Where relevant business property was inherited and was eligible for BPR at the time of
transfer one, there is no minimum ownership period for BPR on transfer two. 

This is called the successive transfers rule. 

For example, if a father gifted his son unquoted trading shares on his death
which he had owned for 4 years and were eligible for BPR, if the son dies within 1 year of

367
the gift and gives them to his brother on death, this second transfer will automatically be
eligible for BPR because the first transfer was eligible for BPR.

6. BPR is not available for excepted assets which are assets held for investment purposes.

7. Any business involved wholly or mainly in dealing in land or buildings and making or
holding investments is not entitled to BPR.

What is relevant business property for BPR?

There is no upper limit on the amount of the relief and it applies to assets situated anywhere
in the world.

Illustration

Which rate of BPR will be available in the following situations?

1 Partnership share in a firm of accountants

2 20% shareholding in a trading company listed on the Stock Exchange

3 28% shareholding in an unquoted trading company

Solution

1 100%

2 0% (Not a controlling interest)

3 100% (Ltd Co - any interest is allowable)


368
Illustration

On 31/05/2022 Wane gifted 40,000 shares in ABC Ltd, an unquoted trading company to his
nephew. 

Wane had owned the shares since 2004 and on the date of the gift, the shares were worth
£200,000. 

On this date, ABC Ltd. owned assets worth £500,000 which included an investment
property valued at £50,000.

• What will the value of the transfer be?

Solution

This is a gift to his nephew, therefore it is a P.E.T.

31/05/2022:

Transfer value £200,000

Less:

BPR £450,000/£500,000 * £200,000 = (£180,000)

22/23 Ann Ex. (£3,000)

21/22 Ann Ex. (£3,000)

Gross chargeable amount £14,000

Notice carefully that BPR is only available for trading assets, it is not available for assets
held for investment.

Agricultural Property Relief

APR is a 100% relief and is very similar to BPR, but gives relief for transfers of agricultural
property.

What conditions need to be satisfied to obtain APR?

1 The property must be relevant agricultural property situated in the UK, the
Channel Islands, The Isle of Man, or in the European Economic Area.

2 It must have been owned for the minimum period of ownership, two years for
owner occupied farms and seven years for tenanted farms. 

For example, if someone is farming the land themselves, this is a owner
occupied farm which has a minimum ownership period of 2 years, and if
someone has rented the farm to someone else, this is a tenanted farm which has a
minimum ownership period of 7 years.


369
What is relevant agricultural property for APR?

1 Cost of the land

2 Cost of the farmhouse

3 Cost of the barns

4 Cost of the shelter belts

5 This does NOT include cost of the stock, the cost of plant and machinery or
the cost of goodwill

Illustration

Greg is gifting his farming business on 30/06/2022 to his son. 

Greg has lived on the farm and worked the business for the last 10 years.

Agricultural value £500,000

Development value £400,000

Animals and inventory £150,000

Plant and machinery £80,000

Total £1,130,000

What is the chargeable amount of the gift to Greg’s son?

Solution

Transfer Value £1,130,000

Less

APR (100%*£500,000) = (£500,000)

BPR 100% *(£400,000+£150,000+£80,000) = (£630,000)

Chargeable amount Nil

Note carefully that BPR will be available for the developmental value, animals and inventory
and plant and machinery because these are assets of the trade that is being given away.


370
Syllabus: A3ciii)
Identify exempt transfers

Exempt transfers

What are exempt transfers?

Exempt transfers

These are transfers that will not result in IHT payable

• spouse

• civil partner

• charity

• political party


371
Syllabus A3d. IHT liabilities on lifetime transfers during life
and death
Syllabus: A3di) D1 The basic principles of computing transfers of value and
Advise on the tax implications of chargeable lifetime transfers

Transfer of value, Chargeable Transfer, Potentially


Exempt Transfer

What is a transfer of value?

It is the reduction in value of a person's estate because they made a gift

A gift made during a person’s lifetime may be either:

1. Potentially exempt transfer (PET)

2. Chargeable lifetime transfers (CLT)

1) Potentially exempt transfers

Any transfer that is made to another individual is a potentially exempt transfer (PET).

If the donor survives for seven years then the PET becomes exempt and can be
completely ignored.

A PET only becomes chargeable if the donor dies within seven years of making the gift.


So, remember:

• If the donor dies within seven years of making a PET then it becomes chargeable.

• Tax will be charged according to the rates and allowances applicable to the tax year
in which the donor dies.

• However, the value of a PET is fixed at the time that the gift is made.

372
Illustration:

Sophie died on 23 January 2023.

She had made the following lifetime gifts:

• 8 November 2015 – A gift of £450,000 to her son.

• 12 August 2020 – A gift of a house valued at £610,000 to her daughter.

By 23 January 2023, the value of the house had increased to £655,000. Sophie did not live
in the house after the gift.

Solution:

• The gift to Sophie’s son on 8 November 2015 is a PET for £450,000.

Because the PET was made more than seven years before the date of Sophie’s
death it is exempt from IHT.

• The gift to Sophie’s daughter on 12 August 2020 is a PET for £610,000 and is
initially ignored.

It becomes chargeable as a result of Sophie dying within seven years of making the
gift, and the transfer of £610,000 less annual exemptions of £3,000 pa for 2020/21
and 2019/20 will be charged to IHT based on the rates and allowances for 2022/23.
No taper relief will apply because Sophie dies within 3 years of making the gift.

2) Chargeable lifetime transfers

Any transfer that is made to a trust is a chargeable lifetime transfer (CLT).

• There is no legal definition of what a trust is, but essentially a trust arises where a
person transfers assets to people (the trustees) to hold for the benefit of other
people (the beneficiaries).

For example, parents may not want to make an outright gift of assets to their young
children.

Instead, assets can be put into a trust with the trust being controlled by trustees
until the children are older.

• Unlike a PET, a CLT is immediately charged to IHT based on the rates and
allowances applicable to the tax year in which the CLT is made.

An additional tax liability may then arise if the donor dies within seven years of
making the gift.

Just as for a PET, the value of a CLT is fixed at the time that the gift is made, but the
additional tax liability is calculated using the rates and allowances applicable to the
tax year in which the donor dies.

373
Illustration:

Lim died on 4 December 2022.

She had made the following lifetime gifts:

• 2 November 2015 – A gift of £420,000 to a trust.

• 21 August 2020 – A gift of a house valued at £615,000 to a trust. By 4 December


2022, the value of the house had increased to £650,000.

Solution:

• The gift to the trust on 2 November 2015 is a CLT for £420,000, and will be
immediately charged to IHT based on the rates and allowances for 2015/16.

There will be no additional tax liability as the gift was made more than seven years before
the date of Lim’s death.

• The gift to the trust on 21 August 2020 is a CLT for £615,000, and will be
immediately charged to IHT based on the rates and allowances for 2020/21.

Lim has died within seven years of making the gift so an additional tax liability may
arise based on the rates and allowances for 2022/23.

Saving IHT

It is beneficial for a grandparent to give their grandchildren gifts to save IHT.

This is because if the grandparent gave it to their child and then their child gave it to their
child - IHT would be paid 3 times as all 3 individuals would die, whereas if the grandparent
gave the gift directly to their grandchild, IHT would only be paid twice as there would only
be 2 death's involved for IHT - the gift never went to the third person.

374
Syllabus: A3dii)
D1 The basic principles of computing transfers of value and
Advise on the tax implications of transfers within seven years of death

Taper Relief

What is taper relief?

This is:

It would be somewhat unfair if a donor did not quite live for seven years after making a gift
with the result that the gift was fully chargeable to IHT.

Therefore, taper relief reduces the amount of tax payable where a donor lives for more than
three years, but less than seven years, after making a gift. The reduction is as follows:

Years before death Percentage reduction %

Over three years but less than four years 20

Over four years but less than five years 40

Over five years but less than six years 60

Over six years but less than seven years 80

Although taper relief reduces the amount of tax payable, it does not reduce the value of a
gift for cumulation purposes.

The taper relief table will be given in the tax rates and allowances section of the exam.

Illustration:

Winnie died on 9 January 2023. She had made the following lifetime gifts:

2 February 2016 – A gift of £473,000 to a trust. The trust paid the IHT arising from this gift.

16 August 2019 – A gift of £320,000 to her son.

375
These figures are after deducting available exemptions.

The nil rate band for the tax year 2015/16 is £325,000, and for the tax year 2019/20 it is
£325,000

IHT liabilities are as follows:

Lifetime transfers £
2 February 2016
Chargeable transfer 473,000
IHT liability
325,000 at nil% 0
148,000 at 20% 29,600
29,600
16 August 2019
Potentially exempt transfer 320,000

Additional liabilities arising on death

2 February 2016
Chargeable transfer 473,000
IHT liability
325,000 at nil% 0
148,000 at 40% 59,200
Taper relief reduction – 80% (47,360)
11,840
IHT already paid (29,600)
Additional liability 0

The taper relief reduction is 80% because the gift to the trust was made between six and
seven years of the date of Winnie’s death.

Although the final IHT liability of £11,840 is lower than the amount of IHT already paid of
£29,600, a refund is never made for life tax paid, it can only reduce the IHT liability to zero.

376
16 August 2019
£

Potentially exempt transfer 320,000

IHT liability 320,000 at 40% 128,000

Taper relief reduction – 20% (25,600)

102,400

The taper relief reduction is 20% because the gift to the son was made between three and
four years of the date of Winnie’s death.

377
Syllabus: A3diii)
Advise on the tax liability arising on a death estate

IHT liability on the death estate

Tax liability on the death estate

Until now, the examples have simply given a figure for the value of a person’s estate.

However, it may be necessary to calculate the value.

A person’s estate includes the value of everything that they own at the date of death such
as property, shares, motor vehicles, cash and other investments.

A person’s estate also includes the proceeds from life assurance policies even though the
proceeds will not be received until after the date of death.

The actual market value of a life assurance policy at the date of death is irrelevant.

The following deductions are permitted:

• Funeral expenses

• Debts due by the deceased provided they can be legally enforced. Therefore,
gambling debts cannot be deducted, nor can debts that are unenforceable because
there is no written evidence.

• Mortgages on property. This does not include endowment mortgages because


these are repaid upon death by the life assurance element of the mortgage.
Repayment mortgages and interest-only mortgages are deductible.

Illustration:

Andy died on 31 December 2022.

At the date of his death he owned the following assets:

• A main residence valued at £425,000 - - this was not left in his will to a direct
descendent and as such the residence nil rate band will not apply.

This had an outstanding interest-only mortgage of £180,000.

378
• Motor cars valued at £63,000.

• Ordinary shares in Herbert plc valued at £54,000.

• Building society deposits of £25,000.

• Investments in individual savings accounts valued at £22,000, savings certificates


from NS&I (National Savings and Investments) valued at £19,000 and government
securities (gilts) valued at £34,000.

• A life assurance policy on his own life. On 31 December 2022, the policy had an
open market value of £85,000 and proceeds of £100,000 were received following
Andy’s death.

On 31 December 2022, Andy owed £700 in respect of credit card debts and he had also
verbally promised to pay the £800 legal fee of a friend.

The cost of his funeral amounted to £4,300.

£ £

Property 425,000

Mortgage (180,000)

245,000

Motor cars 63,000

Ordinary shares in Herbert plc 54,000

Building society deposits 25,000

Other investments (22,000 + 19,000 + 34,000) 75,000

Proceeds of life assurance policy 100,000

562,000

Credit card debts 700

Funeral expenses 4,300

(5,000)

Chargeable estate 557,000

IHT liability

  325,000 at nil% 0

 232,000 at 40% 92,800

379
92,800

The promise to pay the friend’s legal fee is not deductible because it is not legally
enforceable.

Unlike capital gains tax, there is no exemption for motor cars, individual savings accounts,
saving certificates from NS&I or for government securities.

The IHT liability on the life assurance policy could have easily been avoided if the policy had
been written into trust for the beneficiaries of Andy’s estate.

The proceeds would have then been paid directly to the beneficiaries, and not form part of
Andy’s estate.

Illustration:

Joe Kerr died on April 6 2022, leaving £25,000 to his friend and the remainder to his
nephew.

At the date of his death Joe owned the following assets:

His principal private residence valued at £300,000 upon which the outstanding repayment
mortgage at the date of death was £80,000. The house was not left to a direct descendent
and as such the residence nil rate band does not apply.

A holiday home valued at £140,000

Bank and Building Society Deposits amounting to £230,000

12,000 Shares in Joe Ltd valued at £20 per share

A life assurance policy with an open market value on April 6 2022, of £125,000 from which
proceeds of £140,000 were received into trust following Joe’s death.

Joe had no outstanding expenses at the date of his death.

Joe's wife only used 50% of her NRB at the date of her death and Joe never made any gifts
previous to his death, so his NRB is fully available.

How much will his nephew inherit?

Solution:

Death estate

House £300,000

Less: Repayment mortgage (£80,000)

Holiday home £140,000

Bank deposits £230,000

Shares (12,000*£20) = £240,000

Total estate £830,000

380
NRB available (£325,000 + 0.5*£325,000) = (£487,500)

Chargeable estate = £342,500

IHT payable = 40%*£342,500 = £137,000

Therefore, his nephew will inherit:

£830,000 - £25,000 (gift to friend) - £137,000 (IHT paid from estate) = £668,000

Note: the life assurance policy was written into trust for the beneficiaries and so it does not
become chargeable to IHT in the death estate.

381
Syllabus: A3div)
Advise on the relief for the fall in value of lifetime gifts

Fall in value of lifetime gifts

What is the value of a gift for IHT?

Value of a gift

The chargeable amount of a lifetime gift (CLT and PET) is calculated and fixed at the time of
gift. 

If the gift becomes chargeable on death (donor dies within 7 years of making the gift):

1) An increase in value of the gift will be ignored

2) A decrease in value of the gift will get relief for the fall in value

Condition for the fall in value claim

The asset must still be owned by the done at the date of death, or if it has been sold, it
must have been sold in an arms length transaction.

Note

When applying the 7 year accumulation principle for nil rate band availability, the value that
will be used for this gift is the original gross value, not the value after the relief has been
given.

Illustration

Preeti’s aunt died in August 2022.

On 1 May 2021 Preeti’s aunt gave her a property worth £92,000 but now it is only worth
£40,000. 

Preeti’s aunt made other gifts, so no annual exemptions or nil rate band was available for
this gift.

382
What is the IHT payable for this gift?

Solution

Original transfer value £92,000

Fall in value claim (£52,000)

Value after claim £40,000

IHT: 40% * £40,000 = £16,000


383
Syllabus: A3dv)
Advise on the operation of quick succession relief

Quick succession relief

Quick succession relief

This is a relief that is only available on the death estate.

Quick succession relief applies where an individual dies and within the previous 5
years

• - they had inherited an asset on someone else’s death and IHT was charged
on the inheritance. 

For example, two members of a family dying within 5 years of each other,
and one having given a gift to the other

• - they received a lifetime gift and IHT was charged on the gift

How to calculate Quick Succession Relief?

IHT paid on first death * appropriate %

Appropriate Percentages


384
IHT paid on the first death

This is normally given in the question, but if it is not, it can be calculated as follows:

Total IHT paid on first death estate/Gross chargeable estate value of first death * Value of
the asset gifted out of the first estate

Note

QSR is given before DTR but after the deduction of taper relief and life tax, and it is not
necessary for the second person to still own the asset on death. 

It is deducted after the IHT liability has been calculated.

Illustration

Denny died on 31/07/2022 leaving an estate of £340,000. 

He had made no lifetime gifts. 

In June 2018, Denny had been left £28,000 from his brother’s estate. 

Inheritance of £75,000 was paid on a total chargeable estate of £450,000 because of his
brother’s death. 

How much IHT will be due as a result of Denny’s death?

Solution

Chargeable estate £340,000

Nil band £(325,000)

Taxable estate £15,000

IHT at 40% £6,000

Less QSR: June 2018 to July 2022 4-5 years

(£75,000/£450,000) * £28,000 * 20% = £(933)

IHT due on Denny’s death £5,067

385
Syllabus: A3dvi)
Advise on the operation of double tax relief for inheritance tax

Double tax relief for inheritance tax

If an individual is UK domiciled he must pay IHT on UK and overseas


located assets

This means

that IHT is payable in the UK on these overseas assets and possibly death duties were paid
overseas as well (two lots of tax (double)).

• When this is the case the individual is allowed to reduce the UK IHT payable
on these assets by something called Double Tax Relief (DTR)

Calculation of DTR:

This is the lower of:

1 The foreign tax suffered

2 UK IHT on the overseas asset

Calculation of the UK IHT on the overseas asset:

Estate rate * Value of the foreign asset

• Estate rate: IHT on the estate after QSR/Gross chargeable estate value

Note:

Remember from valuation of assets for IHT that the value of a foreign asset is the value
after deducting any additional expenses incurred in realising or managing the property
(which may be subject to a maximum of 5%)

• Also, DTR is deducted after QSR.


386
Illustration

Prem died on 15/08/2022 leaving an estate valued at £375,000. 

The estate included property situated overseas valued at £60,000. 

Overseas IHT of £18,000 was paid on this property. 

No lifetime transfers have been made and he left his entire estate to his son.

• How much IHT is payable on death?

Solution

IHT Payable

Gross chargeable estate value £375,000

Less:

NRB (£325,000)

Taxable amount £50,000

IHT 40%*£50,000 = £20,000

Less DTR (W1) (£3,200)

IHT payable £16,800

W1

DTR is the lower of:

1 Overseas IHT suffered £18,000

2 UK IHT payable = £60,000 * £20,000/£375,000 = £3,200


387
Syllabus: A2gv/A3dvii)
Understand the capital gains tax implications of the variation of wills and
Advise on the inheritance tax effects and advantages of the variation of wills

Deed of variation

Changing a person's will after death

It is possible to change a person’s will after they have died, but why?

• A variation may benefit the estate and its beneficiaries by reducing the IHT
on the estate. 

If the variation increases charitable legacies sufficiently the estate may
benefit from the reduced rate of IHT at 36%.

• If an estate is left to children, who have enough property of their own, then
the variation can leave property for grandchildren instead of the children,
therefore skipping one generation of IHT payable.

How can a will be changed?

1 By court order

2 By a deed of variation

Conditions to be satisfied to be able to execute a deed of variation:

1 The deed must be in writing and signed by all beneficiaries that are affected
by the deed.

2 Not be made for any payment.

3 Must be executed within two years of death.

4 State that it is intended to be effective for tax (IHT and/or CGT) purposes.

388
Illustration

Faisal died on 8 August 2022 leaving his entire estate to his son. 

The estate is valued at £500,000 after deducting exemptions and reliefs including a
donation to charity of £18,000. 

No lifetime gifts have been made.

a) What amount of charitable legacy must be made to benefit from the reduced rate of
36%?

b) What is the net increase in the estate if the charitable legacy is increased by executing a
deed of variation?

Solution

Net estate after charitable legacy £500,000

Less NRB (£325,000)

Taxable estate (this would be taxed at 40% = £70,000) £175,000

Add charitable legacy £18,000

Total £193,000

10% of total is £19,300

Therefore, the charitable legacy needs to be increased by £1,300 (£19,300 - £18,000) to


benefit from the reduced rate of 36%

Net increase in estate if charitable legacy is increased


IHT on original estate (£175,000 * 40%) = £70,000



Cost of extra charitable donation (£1,300)

IHT on revised estate ((£175,000 - £1,300) * 36%) = (£62,532)

Net increase in estate as a result of the reduced rate of IHT after increasing charitable
payment £6,168

How can a variation of will be beneficial for CGT?

Illustration

Jake dies on 01/01/23 and gifts his home to his son John. 

John intends to give the home to his daughter a few months later.

How will a variation in Jake's will be beneficial for CGT?

Solution

On Jake's death, any gifts made will not be liable to CGT.

However, when John gifts the house to his daughter - he will be liable to CGT.

Therefore, Jake should gift the house directly to John's daughter to avoid the payment of
CGT!


389
Syllabus A3e. Trusts
Syllabus: A3ei-ii)
i) Define a trust
ii) Distinguish between different types of trust

What are the different types of trusts?

What is a trust?

Definition of a trust

A trust is an arrangement whereby:

- Property is transferred by a settlor

- To the trustees

- To be held for the benefit of one or more specified beneficiaries 

- On specified terms in the trust deed

Therefore:

Settlor ->  Property passes into a trust -> Trustees are given the legal title to the property

What is interest in possession?

IIP can be the legal right to receive income generated by the trust assets and/or use the
trust asset or live in a property owned by a trust.

Types of trusts

• Discretionary trusts

• Interest in possession trusts

390
Discretionary trusts

1 No interest in possession exists

2 The beneficiaries have no legal right to benefit from the income or capital of
the trust

3 The trustees decide how the trust assets are invested and managed

4 Any distribution of income or capital out of the trust is at the complete


discretion of the trustees

Interest in possession trust

1 Interest in possession exists

2 The beneficiary is known as the life tenant

3 The life tenant has a legal right to benefit from the income of the trust

4 The trustees will distribute the life tenant’s full entitlement every year

It is now not possible to create an Interest In Possession Trust but they may still exist.

Most trusts that you will see in your exam will be Relevant Property Trusts which is a
generic name for most trusts and these are treated for tax purposes in the same way as
Discretionary Trusts.


391
Syllabus: A3eiii-v)
iii) Advise on the inheritance tax implications of transfers of property into trust
iv) Advise on the inheritance tax implications of property passing absolutely from a trust to a
beneficiary
v) Identify the occasions on which inheritance tax is payable by trustees

IHT implications of trusts

Trusts

IHT implications

There are 2 IHT charges when property passes into/out of a trust for inheritance tax

Principal Charge

This must be paid every 10th anniversary by the trustees.

It is 6% of the value of the property every 10th anniversary. 


For example, property passed into a trust in 2013 when it was valued at £1,000,000 and in
2023, it is valued at £1,500,000. 

The principal charge will be 6% * £1,500,000 = £90,000.

Exit Charge

This must be paid when the property leaves the trust.


For example, property passed into a trust in 2013 when it was valued at £1,000,000, the
donor died in 2022 and the property will be passed on to the beneficiary in 2022 when it is
valued at £1,200,000.

The exit charge will be 6% * £1,200,000 = £72,000.

This is all you need to know about these charges. You will not be asked to compute the
them in your exam.


392
Syllabus A3f. IHT planning
Syllabus: A3fi) Advise on the use of reliefs and exemptions to minimise inheritance tax liabilities,
as mentioned in the sections above.

IHT planning

How to pay as little IHT as possible?

IHT planning

The overall objective is to ensure that the HMRC get as little as possible and the next
generations get as much as possible.

 

The total inheritance payable can be reduced if a person makes lifetime gifts rather than
death gifts.

Lifetime exemptions can apply to reduce the total inheritance tax payable.

• Make lifetime gifts each year sufficient to use the Annual Exemption – £3,000.

• Make as many small gifts of £250 per donee per tax year.

• On the marriage of a son, daughter, grandchild, nephew and niece make gifts covered
by the marriage exemption.

From a parent £5,000 



From a grandparent £2,500 

From any other person £1,000

• Make lifetime gifts of appreciating assets and those which do not generate a significant
CGT liability.

• Lifetime gifts to other individuals can reduce IHT as they will not give rise to any IHT
when the gift is made and will be totally exempt from IHT if the donor lives for > 7 years.

• If the donor dies within 7 years then IHT may become payable on death but provided
the donor lives for > 3 years the IHT payable will be reduced by taper relief.

• There is no IHT saving by lifetime giving of assets that qualify for BPR or APR at 100%.

• Ensure that estates of husband and wife are shared so that each spouse will fully use
their nil bands in the event that they should die at the same time.


393
Syllabus A3g. IHT administration
Syllabus: A3gi) Identify the occasions on which inheritance tax may be paid by instalments.

Payment of IHT by instalments

When can IHT be paid in instalments?

IHT in respect of certain assets can be paid in instalments.

Qualifying assets are:

1 Land and buildings wherever they are located

2 A business or an interest in a business

3 A shareholding in a company where the transferor controlled the company


immediately before the transfer. 

For example, a company had 100,000 shares with a nominal value of £1
each, the transferor owned 60,000 shares before the transfer, this will qualify

4 Unquoted shares with a value in excess of £20,000, which represents 10%


or more of the company’s shares. 

For example, a company had 100,000 shares with a nominal value of £1
each. 

The transferor gave the transferee 20,000 shares with a nominal value of
£20,000 – this will qualify as both conditions are satisfied.

How are the instalments paid?

Payment by instalments is in 10 equal annual instalments, the first instalment being due 6
months after the end of the month of death, therefore the first payment is the normal due
date and the remaining 9 payments will be annual after that.

Is interest payable on the 9 instalments?

Interest will be due on the remaining 9 instalments for the following assets:

a) Land and buildings

394
b) Investment company shares 

Interest will only be due on the other assets if the actual instalment is paid late.

• For example, if instalments were being paid on the transfer of a business,


then the 10 annual instalments would not attract interest, but if the instalments
were being paid on land and buildings transfer, the first instalment would not
attract interest but the 9 instalments after that would attract interest.

Restriction to the instalment option

The instalment option is only available provided the instalment asset is still owned by the
donee, and if it is sold the whole of the outstanding IHT on that asset becomes payable
immediately.

• For example, if land and buildings were given to the donee, and the
instalment option was used, if the land and buildings were sold by the
donee, then all of the remaining IHT payable would be due immediately
because of the sale of the asset.


395
Syllabus: A3gii)
Advise on the due dates, interest and penalties for inheritance tax purposes.

Payment of inheritance tax and the due date

When does inheritance tax need to be paid?

For Chargeable lifetime transfers:

The donor is primarily responsible for any IHT that has to be paid in respect of a CLT.

However, a question may state that the donee is to instead pay the IHT.

Remember that grossing up is only necessary where the donor pays the tax.

The due date is the later of:

• 30 April following the end of the tax year in which the gift is made.

• Six months from the end of the month in which the gift is made.

Therefore, if a CLT is made between 6 April and 30 September in a tax year, then any IHT
will be due on the following 30 April.

If a CLT is made between 1 October and 5 April in a tax year, then any IHT will be due six
months from the end of the month in which the gift is made.

The donee is always responsible for any additional IHT that becomes payable as a result of
the death of the donor within seven years of making a CLT.

The due date is six months after the end of the month in which the donor died.

396
For potentially exempt transfers

The donee is always responsible for any additional IHT that becomes payable as a result of
the death of the donor within seven years of making a PET.

The due date is six months after the end of the month in which the donor died.

For death estate:

The personal representatives of the deceased’s estate are responsible for any IHT that is
payable.

The due date is six months after the end of the month in which death occurred.

However, the personal representatives are required to pay the IHT when they deliver their
account of the estate assets to HM Revenue and Customs, and this may be earlier than the
due date.

Where part of the estate is left to a spouse, then this part will be exempt and will not bear
any of the IHT liability.

Where a specific gift is left to a beneficiary, then this gift will not normally bear any IHT. The
IHT is therefore usually paid out of the non-exempt residue of the estate.

Illustration:

Alfred died on 15 December 2022. He had made the following lifetime gifts:

• 20 November 2020 – A gift of £420,000 to a trust. Alfred paid the IHT arising from
this gift.

• 8 August 2021 – A gift of £360,000 to his son.

These figures are after deducting available exemptions.

Alfred’s estate at 15 December 2022 was valued at £850,000. Under the terms of his will, he
left £250,000 to his wife, a specific legacy of £50,000 to his brother, and the residue of the
estate to his children. The residue of the estate did not include a residential property.

The nil rate band for the tax years 2020/21, 2021/22 and 2022/23 is £325,000.

397
IHT liabilities are as follows:

Lifetime transfers

20 November 2020

£
Net chargeable transfer 420,000
IHT liability
325,000 at nil% 0
95,000 x 20/80 23,750
Gross chargeable transfer 443,750

The due date for the IHT liability of £23,750 payable by Alfred was 31 May 2021.

8 August 2021

Potentially exempt transfer 360,000

The PET is initially ignored.

Additional liabilities arising on death

20 November 2020

Gross chargeable transfer 443,750

IHT liability

325,000 at nil% 0

118,750 at 40% 47,500

IHT already paid (23,750)

Additional liability 23,750

398
The due date for the additional IHT liability of £23,750 payable by the trust is 30 June 2023.

8 August 2021

Potentially exempt transfer 360,000

IHT liability 360,000 at 40% 144,000

The CLT made on 20 November 2020 has fully utilised the nil rate band.

The due date for the IHT liability of £144,000 payable by Alfred’s son is 30 June 2023.

Death estate

£
Value of estate 850,000

Spouse exemption (250,000)


Chargeable estate 600,000

IHT liability 600,000 at 40% 240,000

The due date for the IHT liability of £240,000 payable by the personal representatives of
Alfred’s estate is 30 June 2023.

Alfred’s wife will inherit £250,000, his brother will inherit £50,000, and the children will inherit
the residue of the estate of £310,000 (850,000 – 250,000 – 50,000 – 240,000).

399
If the death estate is distributed with:

1) Specific gifts left to specific chargeable persons, and

2) The remaining (residue) of the estate left to an exempt person

Then, there is a special way to calculate the inheritance tax payable.

How to calculate the IHT payable?

Step 1:

(Net chargeable estate – NRB available) * 40/60 = IHT payable

Step 2:

The gross chargeable estate is then calculated as:

Net chargeable estate + IHT payable (from above)

Step 3:

The amount allocated to the exempt residuary legatee will be:

Total estate – gross chargeable estate (from above)

This is much easier to understand with an illustration!

Illustration:

Eddy dies on 06/06/2022 leaving an estate valued at £900,000.

He made no lifetime gifts.

He left £400,000 in cash to his son, and the remainder of his estate to his wife.

What is the gross chargeable estate value, the IHT payable and the amount of the
estate that is left to his wife?

400
Solution:

IHT liability on estate:

Net chargeable estate £400,000

Less NRB (£325,000)

Taxable amount £75,000

IHT on death £75,000 * 40/60 = £50,000

Gross chargeable estate

£400,000 + £50,000 = £450,000

Remainder of estate left to his wife:

£900,000 - £450,000 = £450,000

401
Syllabus A4: Corporation Tax
Syllabus A4a. TX - UK Recap: The scope of corporation tax

The contents of the Paper TX - UK study guide for corporation tax under headings: 

- The scope of corporation tax

Period of account and CAP

Period of account

A period of account is the period for which a company prepares its accounts.

Normally, a period of account is for 12 months, however it may be longer or shorter than
this.

This will normally occur when the company starts to trade, ceases to trade or changes its
accounting date.

The maximum length of a period of account is 18 months.

Accounting period / Chargeable accounting period

Normally, a company’s chargeable accounting period is the same as it’s period of account.

The difference between both is that a chargeable accounting period must be equal to or
less than 12 months.

402
However, a period of account can exceed 12 months.

• In the case where a company’s period of account exceeds 12 months, then 2


chargeable accounting periods will be created.

The first one for the first 12 months and the second one for the remaining months in
the period of account.

• It is the chargeable accounting period for which a corporation tax computation is


prepared.

Therefore, if a period of account is split into 2 chargeable accounting periods


because it is longer than 12 months, then 2 corporation tax computations will result.

When does a chargeable accounting period start or end?

A CAP will normally start immediately after the end of a previous CAP.

• A CAP will also start when a company commences to trade, or when its profits
become liable to corporation tax.

• A CAP will normally finish 12 months after the beginning of the period or at the end
of a company’s period of account.

• A CAP will also finish when a company ceases to trade, or when its profits otherwise
cease being liable to corporation tax.

Financial Year (FY)

Financial years run from 01 April – 31 March.

• FY 22 = 01/04/2022 - 31/03/2023

• Financial years determine which corporation tax rates to use.

• There is a single rate of corporation tax of 19% regardless of the size of the
company.

• For the ATX - UK examination, students will be required to calculate a hybrid rate of
corporation tax when a chargeable accounting period spans an earlier financial year
although this is unlikely as the corporation tax rate has been 19% since FY 17 (20%
FY 16).

403
Illustration:

A company prepares accounts for the 15 month period from 01/01/2022 - 31/03/2023.

• What is the period of account?

• What are the chargeable accounting periods?

• How many corporation tax computations will be prepared?

Solution:

Period of account:

01/01/2022 – 31/03/2023

Chargeable accounting periods:

01/01/2022-31/12/2022 (First 12 months)

01/01/2023-31/03/2023 (Last 3 months)

2 corporation tax computations will be prepared, one for each chargeable accounting
period.

Note: in the case of the first chargeable accounting period, 3 months fall in FY21 and 9
months fall in FY22 but as the rates of corporation tax haven’t changed we can go a
straightforward TTP x 19%.

Illustration:

Smarty Ltd. became incorporated on 26/05/2022.

It commenced trading on 01/11/2022.

It prepared its accounts to 31/03/2023.

When does Smarty Ltd,’s accounting period start?

Solution:

It starts on 01/11/2022, this is because the company commenced trading on this date.

Note normally the accounting period starts after the previous one has finished, however as
Smarty Ltd. does not have a previous accounting period, their accounting period starts
when they commenced to trade.

404
Residency of a company

Who pays UK corporation tax?

How is UK residency determined for companies?

A company is resident in the UK if:

1. It has been incorporated in the UK, for example. K Ltd. or B plc.

2. It is centrally managed and controlled in the UK for example, M. Inc. which was
incorporated overseas has majority of its board meetings held in the UK, and most
of its directors are resident in the UK.

Illustration:

B Inc. was incorporated in Barbados.

All of the company's board meetings are held in the UK.

Will the company be considered to be UK resident and therefore have to pay UK


Corporation Tax?

Solution:

Yes, the company will be considered to be UK resident and therefore pay UK Corporation
Tax.

This is because, even though it has been incorporated overseas, it is centrally managed
from the UK.

405
Syllabus A4a. TX - UK Recap: Taxable total profits

The contents of the Paper TX - UK study guide for corporation tax under headings: 

- Taxable total profits

Allowable expenditure in calculating tax adjusted


trading profit

How to calculate Tax adjusted trading profit?

Profit before tax "PBT" (Operating profit) X


+ Disallowed expenses   X

Allowable expenses 0*
- Non trading income (X)

- Capital allowances   (X)


Tax adjusted trading profit   X

* At the exam you will start the adjustment with the profit before taxation of £X and deal with
all the items listed and you will indicate with a zero (0) any items which do not require
adjustment.

Allowable expenses

Are expenses which should be included in taxable profits.

Therefore we have to keep them in Profit before tax.

You will indicate these expenses with 0 in the exam, because these items do not require
adjustment.

• Patent royalties’ receivable/payable

• Staff costs

406
• Impairment losses

• Legal fees:

- in connection with the Issue of loan notes (for trading activities)



- in connection with an action brought against a supplier for breach of
contract

- in connection with  the registration of trade marks

• Loan arrangement fee for trading

• Income element of premium paid for grant of short lease for trading premises

• Gifts to customers if:

- they cost LESS than £50 per recipient per year



- are not of food, drink, tobacco

 - are not vouchers for exchangeable goods

- carry an advertisement for the company making the gift (e.g. pens costing
£30 each displaying company's name)

• Repairs

For example:

- Repairs to warehouse following a flood

- Repainting the exterior of the company's office building

• Accountancy

• Entertaining employees

The only exception to the non-deductibility of entertaining expenditure is when it is


in respect of employees.

• Non-qualifying charitable donations

Illustration 1:

Operating profit is £100,000.

Expenses included in Operating profit:



Repairs to warehouse following a flood £100

Entertaining employees £30

Accountancy £15

Legal fees in connection with  the registration of trade marks £5

Gifts to customers - pens costing £30 each displaying company's name = £60

Required:

Calculate Tax adjusted trading profit.

407
Solution:

Profit before tax "PBT" (Operating profit) 100,000


Allowable expenses:
Repairs 0
Entertaining employees 0
Accountancy 0
Legal fees 0
Gifts 0
Tax adjusted trading profit   100,000

Note: All expenses were indicated with 0, because these items do not require adjustment,
they were all allowable.

Disallowable expenses

Are adjustments which INCREASE taxable profits.

Meaning these items were included in PBT, but should not have been there, therefore we
have to take them away by ADDING them to PBT.

An Example:

The expense of £10 is included in PBT (e.g. £100) but should not have been there, so you
need to take the expense away from PBT and therefore you add the expense to PBT (100 +
10 = £110) and therefore you will increase Profit.

• Entertaining customers and suppliers UK and overseas

Note: 

The only exception to the non-deductibility of entertaining expenditure is when it is
in respect of employees.

408
• Depreciation / Amortisation (usually given in the question)

• Legal costs:

- of acquiring a short lease



- for issue of preference shares

- for renewal of long lease

• Issue cost of shares

• Dividends

• Issue cost and interest for non-trading loan

• Capital expenditure

e.g. 

- costs of new computers

- Extending the office building in order to create a new reception area

- Improvement of the building rather than repair

• Donations:

- to political parties

- paid under the gift aid scheme

• Gifts to customers if:

- they cost MORE than £50 per recipient per year  (e.g. pens costing £60)

- are of food, drink, tobacco (e.g. food hampers)

- are vouchers for exchangeable goods

- don't carry an advertisement for the company making the gift (e.g pens not
displaying company's name)

Remember that the above is just a summary of the important points


mentioned in Topic: Allowable expenditure.

Illustration 2:

Operating profit is £100,000



Depreciation £500

Capital Allowances £400

Expenses included in Operating profit:



Donations to political parties £25

Entertaining employees £30

Accountancy £15

Legal fees in connection with the registration of trade marks £5

Legal fees for issue of preference shares £20

Gifts to customers - pens costing £30 each displaying company's name = £60

Gifts to customers - watches costing £60 each = £120

409
Required:

Calculate Tax adjusted trading profit.

Solution:

Profit before tax "PBT" (Operating profit) 100,000

+ Disallowed expenses:

+ Donations 25

+ Legal fees for issue of preference shares 20

+ Gifts to customers - watches costing £60 eac 120

+ Depreciation 500

Allowable expenses:

Entertaining employees 0

Accountancy 0

Legal fees in connection with the registration of trade marks 0


Gifts to customers - pens costing £30 each displaying company's name 0

- Capital allowances   (400)

Tax adjusted trading profit   100,265

Illustration:

Alpha Ltd. had an accounting profit of £59,850.

The following items are included in the accounting profit figure:

Income from sales          £20,000


Cost of 4 computers £5,000
Interest paid on a loan for working capital requirements £3,000
Depreciation £1,250

What is the tax adjusted accounting profit?


410
Solution:

Accounting profit      £59,850


Allowable expenses (Interest paid ) 0
Add: disallowed expenditure
Capital expenditure on computers            £5,000
Depreciation £1,250
Deduct: capital allowances  (£5,000)
Tax adjusted accounting profit £61,100 

Note:

1. A company is allowed an annual investment allowance of £1,000,000 per year.

This means that a company can spend up to £1,000,000 on capital items, for
example computers and be allowed the full expenditure to be an allowable expense
in the tax year.

Therefore, as the capital expenditure on computers is only £5,000 – the annual


investment allowance will take care of this.

2. The loan interest is allowable as it is a loan for trading requirements

As companies will have shareholders (people who own the company) and directors/
employees (people who work for the company) separated, there are some
differences between the rules for unincorporated traders and companies:

These include:

• No private element of expenses added back

• Drawings (cash and goods) are not relevant for companies.

Dividends are paid out of post-tax profits.

• Family salaries are not relevant for companies.

Basically, there will not be any personal use of expenses because everyone who uses
the company’s money or facilities will be an employee, not an owner.

411
Relief for pre-trading expenditure

When does trading commence?

Trading commences on the first day on which a trader makes a sale.

However, the trader would have incurred expenditure before this date, for example,
advertising expenditure and/or rent paid in advance.

• This expenditure incurred before trading has commenced is known as “pre-


trading expenditure”.

• Pre-trading expenditure will get tax relief by being treated as though it was
incurred on the first day that a sale is made, if the following conditions are
satisfied.

Conditions for pre-trading expenditure to be allowable

• 1) It is incurred within 7 years of the commencement of the trade.

• 2) It is an allowable expense.

• For example, if goods were purchased for sale for the business 4 years
before the business had its first sale; this purchase price will be deducted
from the first profits also.

412
Illustration:

Manny Ltd. made their first sale in his packaging business on 04/05/2022.

Before this they incurred the material expenses of £3,000 on 31/12/2021.

• Will this expenditure be deducted from the sales revenue to arrive at tax
adjusted trading profit?

Solution:

Yes, this expenditure will be deducted from their sales revenue to arrive at the tax
adjusted trading profit.

It will be treated as though the expenditure was incurred on 04/05/2022.

This is because money spent on materials used in the business are an allowable
expense and it was incurred within 5 months of the trade starting.

413
Capital allowances

Plant and machinery(P&M) for capital allowances purposes

Capital Allowances (Tax depreciation) are deducted from Operating profits

• CA are given for P&M used in the business only

• CA are given for a period of account eg for a year ended 31/12/22, and
are deducted in the adjustment of profits calculation to reach the Trading
Profits figure

• Plant is defined as assets that perform an active function in the business

• e.g.  office furniture and equipment including moveable office partitioning

• Machinery will include motor vehicles and computers, including building


alterations necessary for the installation of plant and machinery

Rates of allowance %

Main pool assets 18

Special Rate Pool assets 6

Capital allowances are now also available on integral features of a building including
lifts and escalators, electrical systems, heating and air cooling system.

Main pool

1. Computers, equipment, shelving, vans and lorries

2. Movable office partitioning

3. Alterations to building incidental to the installation of plant and machinery

4. Tables and chairs

5. Fire regulation expenditure

Special Rate Pool

414
The following asset acquisitions should be allocated to the special rate pool:

1. Integral features of a building

– these include all major systems in a building.

For example, electrical, thermal, cooling systems.

2. Long life assets

These are assets, when new, with an expected economic working life of 25
years or more when total expenditure based on a 12-month accounting
period exceeds £100,000

Writing down allowances

W.D.A.’s are given on main pool assets and special rate pool assets.

For main pool assets, the W.D.A. is 18% for a 12 month period

For example Assets in the main pool had a brought forward value of £100,000 at
01/01/2022

The writing down allowance on these assets will be £18,000 (£100,000*18%) in


the year ending 31/12/2022.

 

Note if the above period was for 6 months, then the WDA for the main pool
would be £9,000 (£100,000*18%*6/12) in the period ending 31/12/2022.

For special rate pool assets, the W.D.A. is 6% for a 12 month period.

For example Assets in the special rate pool had a brought forward value of
£100,000 at 06/04/2022

The writing down allowance on these assets will be £6,000 (£100,000*6%) in the
year ending 05/04/2023.

Note if the above period was for 6 months, then the WDA would be £3,000
(£100,000*6%*6/12) in the period ending 05/04/2023.

Annual investment allowance

From 1 January 2019, the annual investment allowance is £1,000,000.

This is given to an individual for a 12 month period and is time apportioned if the
period is below 12 months.

415
Ideally, this A.I.A should be allocated to special rate pool assets purchased first
because the allowances on these assets are only 6% per year, therefore tax
relief on these assets is received over a longer period.

Once allocated to special rate pool assets purchased in the tax year, then if any
of the allowance is remaining, it can be allocated to main pool assets purchased
in the year.

The A.I.A cannot be given to motor cars purchased in the tax year.

For example a business purchased equipment worth £1,300,000 in their year


ending 31/03/2023.

The annual investment allowance is £1,000,000 (maximum available).

For the remaining £300,000 (£1,300,000- £1,000,000), a writing down allowance


will be available.

As equipment is a main pool asset, the writing down allowance will be £54,000
(£300,000*18%).

The total capital allowances available will be AIA + WDA = £1,054,000


(£1,000,000 +  £54,000)

Note: if the above purchase was made in a 6 months period, then the AIA would be
(£1,000,000*6/12) = £500,000 + WDA ((£1,300,000 - 500,000)*18%*6/12) = £72,000.
This would total to £572,000 of capital allowances for the 6 month period.


416
Enhanced capital allowances for companies only

For a two-year period from 1 April 2021 to 31 March 2023, companies can benefit
from enhanced capital allowances when they purchase new plant and machinery:

• For expenditure which would fall into the main pool, there is a 130% super
deduction. This means that for every £100 of expenditure, a first year
allowance of £130 is available.
• For expenditure which would fall into the special rate pool, there is a 50% first
year allowance.

For expenditure falling into the main pool, the 130% super deduction should be
claimed rather than the 100% annual investment allowance.

However, for expenditure falling into the special rate pool, the 100% annual
investment allowance should be claimed in preference to the 50% first year
allowance.

Enhanced capital allowances are not available to sole traders or partnerships. Only
expenditure on new plant and machinery qualifies, not expenditure on second-hand
assets. Motor cars do not qualify.

Illustration:

Anaya Ltd has an accounting reference date of 31 March each year.


On 1 April 2022, the tax written down value of plant and machinery in the company’s
main pool and special rate pool is £0.

During the year ended 31 March 2023, Anaya Ltd purchased new equipment for
£1,650,000, of which £350,000 is main pool expenditure and £1,300,000 is special
rate pool expenditure.

What is Anaya Ltd’s capital allowance claim for the year ended 31 March 2023?

Solution:

Allowance £

Super deduction (£350,000 x 130%) 4,55,000

Annual investment allowance 10,00,000

First year allowance (£1,300,000 - 1,000,000) x 50% 1,50,000

Total allowances 16,05,000

417
Illustration:

Buzzy Ltd. in the year ended 31/03/2023 made the following transactions.

Date Item Price

01/05/2022 Ventilation system and lift for his freehold office £1,278,000
building
26/06/2022 Machinery purchased and alterations made to £29,300
office building to install the machinery

08/08/2022 Movable partition walls £22,900

11/03/2023 New decorative wall constructed £41,200

The tax written down value on the main pool was £87,800 on 01/04/2022.

What are Buzzy Ltd. capital allowances?

Solution:

Allowance £

Super deduction (£52,200 x 130%) 67,860

Annual investment allowance 10,00,000

First year allowance (£1,278,000 - 1,000,000) x 50% 1,39,000

Total allowances 12,06,860

Notice how the AIA was first allocated to special rate pool assets.

Also notice that the expenditure on the decorative wall is not eligible for capital
allowances as this does not qualify as plant and machinery.

The capital allowances are £1,206,860.

418
Compute capital allowances for motor cars

First year allowances

These are given for zero emission motor cars.

This is a 100% allowance on the cost of the car and it is given in the period of
acquisition.

The F.Y.A. is not time apportioned for a period of less than 12 months.

For example, a car was purchased on 01/05/2022 for £100,000.

It had a zero CO2 emissions.

The first year allowance for this car will be £100,000 ( £100,000*100%).

Note if the above period was for 6 months, then the FYA would still be £100,000
- it is not reduced for a period of less than 12 months.

The F.Y.A is given to motor cars purchased that have zero CO2 emissions

For cars with a CO2 emission of less than or equal to 50g CO2 emissions, an 18%
W.D.A. is given, therefore these are considered to be main pool assets.

For cars with a CO2 emission of more than 50g, an 6% W.D.A. is given, therefore
these are considered to be special rate pool assets.

Illustration (a Company): (changes in table @mustafa)

Cow Ltd.:

06/04/2022 Tax written down value on main pool of £16,800

Purchase of car for £10,600. The car had CO2 emissions of 46g/
25/06/2022
km.
Purchase of car for £18,000. The car had CO2 emissions of
16/02/2023
142g/km.

14/03/2023 Purchase of car for £22,000. The car had zero CO2 emissions.

What are Cow Ltd. capital allowances?

419
Solution:

Special
Main Capital
Particulars F.Y.A. rate
Pool allowances
pool

Tax written down


£16,800
value brought forward

Additions:

Zero CO2 car £22,000 (£22,000) £22,000

Car 46g/km £10,600 £10,600

Car 142g/km £18,000 £18,000

Total (£22,000) £27,400 £18,000

WDA (18%/6%) (£4,932) (£1,080) £6,012

Tax written down


22,468 16,920
value carried forward

Total capital allowances for the year £28,012 (£22,000 + £6,012)

Illustration (ANNA - a Soletrader ) (changes in table @mus)

06/04/2022 Tax written down value on main pool of £16,800

Purchase of car for £10,600.

The car had CO2 emissions of 46g/km. 



25/06/2022
This car is 60% privately used by Anna’s husband who is an
employee of the business.
Purchase of car for £18,000.

16/02/2023 The car had CO2 emissions of 142g/km.

This car is 30% used privately by Anna.


Purchase of car for £22,000.

14/03/2023 The car had zero CO2 emissions.

This car is 25% privately used by Anna’s assistant.

What are Anna’s capital allowances?

Solution:

420
Capital
Main Special
Particulars F.Y.A. allowan
Pool rate pool
ces

Tax written down


£16,800
value brought forward

Additions:

Zero CO2 car £22,000 (£22,000) £22,000

Car 46g/km £10,600 £10,600

Car 142g/km £18,000 £18,000

Total (£22,000) 27,400 £18,000

(£1,080) *
70%
business £5,688
WDA (18%/6%) (£4,932)
use = (W1)
Capital
allowance

Tax written down


22,468 £16,920
value carried forward

W1:

The capital allowance is reduced by % of private usage

£4,932 + (£1,080 * 70%) = £5,688

W2:

The tax written down value carried forward is calculated using the entire W.D.A.

£18,000 - £1,080 = £16,920

Total capital allowances for the year £27,688 (£22,000 + £5,688)

421
Assets with private use

A company

Companies do not have assets used privately. 

This is because all of the people who work in the company are considered to be
employees of the company. 

Therefore, the capital allowances given are not reduced by the % of private usage
by an employee of a company.

A Sole trader

If an asset is used privately by the owner of the business, the capital allowance
given must be reduced by the % of private usage. 

If an asset is used privately by an employee of the business, the capital


allowance given is not reduced by the % of private usage.

Illustration (a sole trader)

Mia has been in a business as a sole trader.

She bought computer for £3,000 which she uses 70% in her business and 30%
privately.

She has already used the AIA in this tax year.

Calculate the capital allowances.

Solution:

WDA = £3,000 x 18% = £540

Capital Allowances (business use only) £540 x 70% = £378

Illustration (a company)

Cow Ltd. is a trading company.

The company bought computer for £3,000 which is used by the sales manager
 30% privately.

Cow Ltd. has already used the AIA in this year.

422
Calculate the capital allowances.

Solution:

WDA = £3,000 x 18% = £540

Note: The private use of the computer by the employee is not relevant for capital
allowance purposes. 

No adjustment is ever made to a company's capital allowances to reflect the private
use of an asset.

423
Disposal of the assets

Use LOWER OF

1. Proceeds

2. Original cost

When an item of plant or machinery is sold - the lower of the sale proceeds received or
the original cost of the asset is deducted from the written down value of the relevant
pool.

For example, if the written down value is 100 and sale proceeds received are 120 but
the original cost of the asset is 110, then 110 will be deducted from the pool to give a
balancing charge of 10. The difference between proceeds and original cost will be
treated as a capital gain.

Compute balancing allowances and balancing charges


In the final year of trading, the A.I.A., W.D.A., F.Y.A. are not given.

Instead, balancing allowances and balancing charges are computed on each pool.

Balancing adjustments on the pools can only occur on cessation of trade.

A balancing allowance will be deducted from trading profit to find tax adjusted
trading profit and a balancing charge will be added to trading profit to find tax
adjusted trading profit.

Illustration:

Karen Ltd. prepares accounts to 05/04.

The company ceased to trade on 05/04/2023 on which all of its plant and
machinery was sold for £8,000.

The written down value on its main pool at 06/04/2022 was £11,000.

The company purchased machinery for £4,000 during the year.

424
Solution:

Particulars Main pool Capital allowances


TWDV b/f £11,000
Additions £4,000
Total £15,000
Disposals (£8,000)
Balancing allowance £7,000 £7,000

Karen Ltd.’s balancing allowance in her final year of trading is £7,000.

Structural and Buildings Allowance


The SBA is is a new type of capital allowance available when a building (or a
structure) has been constructed / purchased for use in the trade. For example,
offices, retail and wholesale premises, factories and warehouses all qualify for the
SBA.

This allowance is also available if an unused building/structure has been renovated


for use in the trade.

The rate of the allowance is 3% per annum and is given for a period of 33 years and
4 months.

To note about the SBA:

- The value of land does not qualify for the SBA

- Expenditure which qualifies as plant and machinery (and therefore will get the AIA)
cannot also qualify for the SBA and vice versa.

- The SBA can only be claimed from when the building / structure is brought into
use in the trade. This means that the SBA will be time apportioned for the period
when it is first brought into use, this is unlike capital allowances for plant and
machinery which are given the full allowance in the period of purchase.

- A separate SBA is given for each building / structure

- When the building / structure is sold, this will not result in a balancing allowance or
balancing charge, the 3% p.a. will continue to be given for the period remaining
out of the 33 years and 4 months. However, the allowances already given at the

425
date of sale will be added to the sale proceeds when calculating the chargeable
gain / capital loss for capital gains tax.

Illustration

Anaya Ltd prepares accounts to 31/3/2023.

On 1/7/2022 a newly constructed factory was purchased from a builder for


£500,000 (including land cost of £130,000).

The factory was brought into use on 1/9/2022.

What is the SBA available on this factory?

Solution

Purchase price £500,000

Less land cost (£130,000)

Qualifying expenditure for SBA £370,000

SBA £370,000 x 3% x 7/12 = £6,475

The allowance will be given from September 2022 (date it was brought into use).

Illustration

Anaya Ltd sold the factory above on 31/3/2023 for £600,000.

What will the SBA be for the year ended 31/3/2024?

What will the capital gain be on the sale?

Solution

The SBA will be given normally for the year ended 31/3/2024:

£370,000 x 3% = £11,100

The capital gain on the sale:

Sale proceeds £600,000 + SBA £6,475

= £606,475

Less cost (£500,000)

Capital gain £106,475

426
Recognise the treatment of short life assets

Short life assets are main pool assets that have an expected life of 8 years or less.

A de-pooling election can be made so that the asset gets its own W.D.A.’s and on
sale of the asset, a balancing allowance or balancing charge can arise.

The benefit of this election is that a balancing adjustment will arise within 8 years,
which would not have arisen, if this de-pooling did not take place.


If the asset is not sold within the 8 years of acquiring the asset, then the written
down value is added back to the main pool.

This happens on the 8th anniversary of the end of the accounting period in which
the asset was acquired.

Assets on hire purchase or lease

Any asset (including a car) bought on hire purchase (HP) is treated as if purchased
outright for the cash price. Therefore:

• The buyer normally obtains capital allowances on the cash price when the agreement
begins

• He may write off the finance charge as a trade expense over the term of the HP
contract

Long-term leases (those with a term of five or more years) are treated in the same way
as HP.


427
Property business profits/losses - for Companies

Compute property business profits

The calculation of property business profits is exactly the same as that for individuals
with 4 exceptions:

1. Interest payable on a loan to buy an investment property is deducted from “Interest


income” under the loan relationship rules as opposed to “property business
profits”.

The 100% restriction to interest expenses that we saw in the income tax topic does
not apply to companies.

2. There is no rent a room relief for companies as a company will not have a main
residence.

3. Property losses for a company are entirely relieved against Total Income of the:

1) current year or

2) carried forward to future years.

4. Property business profits are calculated using the accruals basis for companies - NOT
the cash basis.

Note:

Property losses CANNOT be carried back 12 months. ONLY Trading losses CAN

Qualifying charitable donations (QCD) CANNOT be saved!

LOSS must be deducted first and if any income remains - then the QCD can be
deducted

Please refer to Topics Computation of property business profits, Furnished holiday


lettings, Rent a room relief, Premiums granted for short leases, Property business
loss relief to review how property business profits are calculated.

428
Illustration:

For the year ended 31/03/2023 Theta Ltd. has:


Trading income  £100,000 
Property loss (£20,000)
Qualifying charitable donation  £85,000

What will Theta Ltd. taxable total profits be?

Solution:

Trading income £100,000

Property loss (£20,000)

Net income £80,000

Qualifying charitable donation  (£80,000)

Taxable total profit Nil 

Note that the property loss is relieved before the qualifying charitable donation against total
income.

Additionally, this has resulted in £5,000 of the qualifying charitable donation being wasted.

429
Relief for Trading losses - for Companies

1) Relieved against Current year Total income


You HAVE TO deduct the LOSS from Total Income FIRST and then deduct Qualifying
charitable donations (QCD)


The LOSS can be given to 75% group companies for relief against total income (Without
relieving loss against their own income first)

2) Carried back against 12 months of Total income - AFTER the current year total
income has been fully used for the trading loss


You HAVE TO deduct the LOSS from Total Income FIRST and then deduct Qualifying
charitable donations (QCD)

3) Carried forward against Future Total income (Trading, Property, Interest)


Qualifying charitable donations can be saved (ie) the QCD is deducted before the LOSS


The LOSS can be given to 75% group companies for relief against total income (The loss
must be relieved against their own income first)

Note: The current year total income relief and carry back total income relief are normally
used before the carry forward relief.

This is because companies want loss relief as soon as possible.

430
1) Current year relief of trading losses

Trading losses can be deducted from a Current year Total income

Illustration 1

In 2023, Cow plc. made a trading loss of (£100,000)

It also had property income of £75,000 and chargeable gains of £35,000.

Cow plc can relieve the trading loss by deducting it from it's total income:

Property income £75,000



Chargeable gains £35,000

Total Income £110,000


Less:

Current year trading loss (£100,000)


Total profits £10,000 - this is the amount that corporation tax will be paid on.

431
Remember:
Qualifying charitable donations (QCD) CANNOT be saved, loss must be deducted first and
if any income remains - then the QCD can be deducted

Illustration 2

In 2023: 

Trading loss of (£100,000)

Property income of £75,000 

Chargeable gains of £35,000

Qualifying charitable donations of £35,000

Calculate the Total Taxable Profit.

Solution: 

Property income £75,000


Chargeable gains £35,000


Total Income £110,000


Less: 

Current year trading loss (£100,000)


Total Income £10,000


Less: QCD £10,000


Total Taxable Profit £Nil - no corporation tax will be paid.

Note £25,000 (£35,000-£10,000) of the qualifying charitable donation was wasted because
the claim for loss relief must be made in full.

432
3) Carry back relief of trading losses

A trading loss can be deducted from the previous 12 month's Total income

Illustration 1

In 2023 - a trading loss of (£100,000), it had no other income in that year.

In 2022 - property income of £75,000 and chargeable gains of £35,000

Calculate Total taxable profits.

Solution:

Property income £75,000



Chargeable gains £35,000

Total income £110,000


Less:

Trading loss carried back (£100,000)


Total Taxable Profit £10,000 - this is the amount that corporation tax will be paid on.

433
The current year total income claim must be made FIRST

Unlike for individuals, the current year total income claim must be made before the claim
against total income for the previous 12 months.

Illustration 2

In 2023 - a trading loss of (£100,000) and property income of £25,000

In 2022 - Total income of £200,000

The trading loss must first be deducted from 2023 total income, and can then be carried
back to the total income (2022)

Current year relief:

Property income £25,000



Less

Current year trading loss (£25,000)

Total income £Nil - no corporation tax will be paid in the year ending 31/03/2023

Carry back relief:

Total income £200,000



Less:

Trading loss carried back (£75,000)

Total income £125,000 - corporation tax will be paid on this amount in the tax year ended
31/03/2022.

434
Illustration 3:

Pulkit Ltd. made the following income for the year ended 31/03/2023:

Trading income (£30,000)

Property income £20,000

Interest income £5,000

Qualifying charitable donations £5,000

Pulkit Ltd. made the following income for the year ended 31/03/2022:

Trading income £20,000

Property income £20,000

Interest income £5,000

Qualifying charitable donations £5,000

How can the trading loss of the year ended 31/03/2023 be relieved?

Solution:

The trading loss of 31/03/2023 can be relieved against:

1. Current year total income (£20,000 + £5,000)

2. Carry back 12 months of total income (£20,000 + £20,000 + £5,000)

3. Total profits of future years (future total profits = unknown)

435
Here we will illustrate only the current year total income claim and the carry back claim
against total income for 12 months.

Trading loss of (£30,000) incurred in the year ended 31/03/2023 will be relieved against the
total income generated in 31/03/2023 as shown below.

In the year ended 31/03/2023

Property income £20,000

Interest income £5,000

Total income £25,000

Current year trading loss (£25,000)

£Nil - no corporation
Total taxable profit
tax will be paid.

The qualifying charitable donations for the year ended 31/03/2023 have been wasted.

In the year ended 31/03/2022: 

The carry back total income claim for 12 months:

Trading income £20,000

Property income £20,000

Interest income £5,000

Total income £45,000

Trading loss relief carry back claim (£5,000)

Qualifying charitable donations (£5,000)

Taxable total profits £35,000

Notice here that the qualifying charitable donation has not been wasted, as there was
enough income remaining for it to be deducted.

436
Loss memo:

Trading loss of 31/03/2023 (£30,000)

Current year claim against total income £25,000

Carry back claim against total income £5,000

Loss to be carried forward Nil 

437
The carry back of a loss - periods of less than 12
months

If the period before your loss making period is less than 12 months

Your carry back relief claim is for 12 months

Therefore you will have to go one further period back for the remaining months.

For example:

2023 - Loss

2022 - the period was 8 months long only

2021 - the period was 1 year long

Then you will take the full 8 months from 2022 and 4 from 2021 total income - to make sure
that you have gone back a full 12 months.

You must compare the same months of loss and profit.

For example

Above, I must compare 4 month's of profit with 4 months of loss, and take the lower of
them as the amount of loss I can deduct.

You'll see this clearly in Lina Ltd. Illustration below.

438
Illustration

Lina Ltd. had the following results:

Year ended 8 months ended Year ended


30.04.2022 31.12.2022 31.12.23

Trading profits/
£60,000 £2,000 £(66,000)
loss

Chargeable gains £2,000 £10,000 £10,000

Total income £62,000 £12,000 £10,000

What amount of loss relief will the company get if they use the current year and carry
back total income claims for loss relief?

Solution

Note the company's previous accounting period is only for 8 months.  

Therefore the carry back 12 month claim will use these 8 months and go further back 4
months to make up an entire 12 month claim.

8 months ended Year ended


Year ended 30.04.2022
31.12.2022 31.12.2023

Trading profits/
£60,000 £2,000 Nil
loss

Chargeable gains £2,000 £10,000 £10,000

Total income £62,000 £12,000 £10,000

Total income
(£10,000)
current year claim

Total income
carry back 12
months claim (set £(12,000)
off in full in this 8
month period)

439
Total income
Carry back 12 Lower of: 4/12*£66,000 =
months claim £22,000 (Loss for the 4 months)

(restrict set off to                   or

lower of 4 months of 4/12*£62,000 = £20,667 (total


profit or 4 months of income for 4 months)
loss)

(20,667)
Total taxable
£41,333 £Nil £Nil
profts

Loss memo:

Trading loss of 31/12/2023 (£66,000)

Current year claim against total income £10,000

Carry back claim against total income (8 months) £12,000

Carry back claim against total income (4 months) £ 20,667


Loss to be carried forward £23,333

Notice how the loss is restricted in the second carry back period. Look out for this in the
exam, don’t just allocate the remaining loss in this second period.

440
Carry forward relief of trading losses

If a company makes a trading loss, then it can relieve the loss by carrying it forward and
deducting it from it's Future total Income.  

This rule came into force for losses from 1 April 2017.

In your ATX - UK exam you will not be tested on losses that arose before 1 April 2017.

You have to start claiming it within 2 years

Claims for carried forward loss relief must be made within 2 years of the end of the
accounting period in which the loss is relieved.

BUT once you start claiming it, then Trading losses can be carried forward for any amount
of time, until the full amount of the loss has been relieved.

For example (Cow plc ) 

In 2022 - a trading loss of (£30,000)

In 2023 - a trading profit of £50,000 and investment income of £20,000.

Calculate Total taxable profit:

Trading profit £50,000

Investment income £20,000

Total Income £70,000

Less:

Trading loss carried forward (£30,000)

Total taxable profit £40,000 - this is the amount that will be taxed, after the carried forward
loss has been deducted.

441
REMEMBER:

Unlike current year and carry back loss relief, under carry forward loss relief a company
can choose the amount of trading loss to use in order to save its charitable donations.

For example

Cow plc made a trading loss of (£30,000) in the tax year ending 31/03/2022.

Cow plc made a trading profit of £50,000 in the tax year ending 31/03/2023.

Cow plc also made a qualifying charitable donation of £35,000 in the tax year
ending 31/03/2023.

In the tax year ending 31/03/2023 Cow plc will:

Trading profit £50,000



Less:

Trading loss carried forward (£15,000) (restricted to save charitable donation)

Trading profit £35,000 



Less:

Qualifying charitable donation (£35,000)

Trading profit £Nil - no trading profit will be taxed in the tax year ending
31/03/2023.

The remaining trading loss of £15,000 (£30,000 - £15,000) will be carried forward to future
years

Trading losses can be carried forward for any amount of time, until the full amount of the
loss has been relieved.

442
Illustration:

Pulkit Ltd. made the following income for the year ended 31/03/2022:

Trading LOSS (£50,000)

Property income £20,000

Interest income £5,000

Qualifying charitable donations £5,000

Pulkit Ltd. made the following income for the year ended 31/03/2023:

Trading income £20,000

Property income £20,000

Interest income £5,000

Qualifying charitable donations £5,000

How can the trading loss of the year ended 31/03/2022 be relieved against future Total
Income?

Solution:

Trading loss of (£50,000) incurred in the year ended 31/03/2022 will be relieved against the
total profits generated in 31/03/2023.

BUT only £40,000 of the loss needs to be used, the remaining £5,000 of total profits is
covered by the qualifying charitable donations.

The remaining £10,000 of the loss will be carried forward to offset against future total
profits.

Trading income £20,000


Property income £20,000
Interest income £5,000
Total income £ 45,000
Trading loss carried forward (£ 40,000)
Qualifying charitable donations (£5,000)
Taxable total profits £NIL

443
Loss memo:

Trading loss incurred in 31/03/2022 (£50,000)

C/F loss relief in 31/03/2023 £40,000

Loss to be carried forward to 31/03/2024 (£10,000)

Note: an alternative would be to use the loss in the current year and offset £25,000 against
other income. The downside of this is that £5,000 of donations would be wasted.

A4cvi) Identify the restriction on carried forward trading and capital losses for companies
with profits over £5 million.

Restrictions on carried forward loss relief

Companies are entitled to a deductions allowance of £5m for a 12 month period for brought
forward trading and capital losses.

Companies can choose how the allowance will be allocated between the brought forward
trading and capital losses.

The maximum relief for the carried forward trading loss will be the amount of deduction
allowance available plus 50% of the company’s profits after deduction of current period
loss reliefs (including group reliefs) and the deductions allowance.

For example, if total income was £12,000,000 and brought forward trading losses were
£15,000,000 - the allowable deduction would be:

Total income 12,000,000

B/f loss (allowable deduction) (5,000,000)

Total income 7,000,000

Additional deduction (7,000,000 * 50% = 3,500,0000)

Taxable total profit 3,500,000

Notice here that the entire deductions allowance was given to the carried forward trading
loss, so none will be available for the carried forward capital loss.

The maximum relief for carried forward capital losses will be the amount of deduction
allowance available plus 50% of the excess of capital gains above this amount.

Companies in a group will only be entitled to one £5m deduction allowance, it can be
allocated to any company/companies in the group.

You only need to have an awareness of this restriction for the ATX - UK paper.

444
Terminal loss relief

If a trading loss occurs in the final 12 months of trading, then this trading loss can be
carried back for 36 months against the total income of the company, on a LIFO (last in first
out) basis.

Once again, the loss cannot be restricted to save qualifying charitable donations.

For example Creamy plc. made a trading loss of (£100,000) in its final year of trading.

It had the following total income:

Year ended 31/03/2023 - £40,000



Year ended 31/03/2022 - £20,000

Year ended 31/03/2021 - £55,000

The trading loss of (£100,000) will first be relieved against the total income of 31/03/2023:

Total income £40,000



Less:

Terminal loss relief (£40,000)

Total income £Nil - whatever corporation tax has been paid will be repaid to the company
by HMRC


445
Then,

The trading loss of (£60,000) (£100,000-£40,000) will second be relieved against the total
income of 31/03/2022:

Total income £20,000



Less:

Terminal loss relief (£20,000)

Total income £Nil - whatever corporation tax has been paid will be repaid to the company
by HMRC

Then,

The trading loss of (£40,000) (£100,000-£40,000-£20,000) will second be relieved against


the total income of 31/03/2021:

Total income £55,000



Less:

Terminal loss relief (£40,000)

Total income £15,000- whatever corporation tax has been paid, part of it will be repaid to
the company by HMRC

Note for the years in which tax has already been paid, this will result in a repayment of tax.

446
Factors that influence choice of loss relief claim

Influencing loss relief claims

There are 3 factors that will be relevant in the ATX - UK exam that will influence the
choice of the loss relief claim:

1. Relief as soon as possible

Therefore, the current year total income and and carry back 12 months’ total income
claim are much more likely to be used before the carry forward claim against trading
profits

2. The rate of corporation tax

The rate of corporation tax fell from 20% in FY16 to 19% in FY17 and has remained
at 19% for FY18, FY19, FY20, FY21 and FY22. However, the rate is expected to fall
in the future and so it is best to try and utilise losses as soon as possible to obtain
relief at the highest rate

3. Making a large company a small company for corporation tax purposes

If a loss relief claim can reduce the size of the company, then this will avoid the
company having to make quarterly instalments of corporation tax.

447
Loan relationship rules

The loan relationship rules

Basis of assessment of “Interest Income”

There is an “interest element” in the corporation tax computation for companies.

All interest is received gross for companies and the basis of assessment for interest income
is the accruals basis.

Operation of “Interest Income”

Any interest payable or receivable by companies will be deducted from or added to


“interest income”

• For example, if a loan was taken out to purchase an investment property, the
interest payable would be deducted from this area, not property income.

• However, there is one exception to this rule, that is that any loan taken or received
for trading purposes will have its interest payable or receivable adjusted within
“Trading profits”.

• Otherwise, any non trading loans will be adjusted within “Interest income”.

Summary of similarities and differences between individuals and companies

Particular              Individuals Companies

Gross/Net Gross and net Gross

Accruals Accruals Accruals

Trading/Non trading Deducted from their Deducted from Interest income,


loans respective areas except for trading loans

Simple proforma:

Bank and building society interest receivable x

Gilt interest receivable x

Loan note interest receivable x

Repayment interest receivable from HMRC x

448
Less:

Loan arrangement fee (x)

Interest payable on loan to buy inv. Prop. (x)

Late payment interest on overdue tax (x)

Interest surplus/deficit x/ (x)

Illustration:

Seeta Ltd. took out a £190,000 loan on 01/07/22 in the year ended 31 March 2023.

The arrangement fee for this loan amounted to £1,400.

The interest on this loan is £7.25% per annum. The loan is used for various activities:

1. £130,000 to buy an investment property.

2. £45,000 to repair an office building that is rented out.

3. £15,000 to fund working capital requirements.

How much of the interest payable will be taken under the Interest income?

Solution:

The year ends 31/03/2023. Therefore 9 months of interest will be payable.

9/12 * (190,000 * 7.25%) = £10,331

Interest Income:

1. Loan interest to buy an investment property is allowable

2. Loan interest to repair an investment property is allowable



= (£130,000+£45,000)/£190,000 * £10,331 = £9,515

Trading income:

1. Loan interest to fund working capital requirements will be treated as a trading


expense. 

£10,331 - £9,515 = £816

449
Qualifying charitable donations

Tax relief is available for qualifying charitable donations

You can deduct charitable donations from the taxable total profits.

Also note the difference between how qualifying charitable donations are treated
between individuals and companies:

1. Companies deduct these payments, whereas individuals cannot deduct the


payments from their income.

2. A company makes the payment gross, whereas an individual makes the payment
net.

Illustration:

Satya Ltd. has the following income and expenses for the year ending
31/03/2023:

Tax adjusted trading profit £200,000

Property income £50,000

Interest receivable £20,000

Chargeable gains £10,000

Qualifying charitable donation £15,000

Compute the taxable total profits for the year.

450
Solution:

Tax adjusted trading profit £200,000

Property income £50,000

Interest receivable £20,000

Chargeable gains £10,000

Qualifying charitable donations (£15,000)

Taxable total profits £265,000

451
Computation of taxable total profits

How to calculate taxable total profits?

Taxable total profits include:

Trading income x

Other income and gains:

Property income x
Interest income x
Capital gains x

Less:

Loss relief claims (x)


Qualifying charitable donations (x)
= Taxable total profits x

Each of these areas are discussed in detail in their respective sections.

Remember that dividends received are not subject to corporation tax and are therefore not
included in taxable total profits.

Illustration:

Lachmi Ltd. has the following income for the year:

Trading profits £310,000

Property income £200,000

Interest income £50,000

Capital gains £20,000


Qualifying charitable donations £50,000

Dividend received from a non-associated company £18,000.

452
What are Lachmi Ltd. taxable total profits for the year?

Solution:

Taxable total profits for the year

Trading profits £310,000

Property income £200,000

Interest income £50,000

Capital gains £20,000

Less:

Qualifying charitable donations (£ 50,000)

Taxable total profits £530,000

Notice that dividends received are not included. They are exempt and only used for the
calculation of Augmented profits for payment of corporation tax.

453
Little trick!

Interest income is taxed on an accruals basis.

This means that the interest which is taxed is the interest which is receivable during the
year, not the interest which is actually received during the year.

For example, bank interest receivable of £2,000 was accrued at 31 March 2022 and £1,000
was accrued at 31 March 2023 respectively.

This means that £2,000 should have been paid in the year ended 31 March 2022 but was
not, and £1,000 should have been paid at 31 March 2023, but has not been paid yet.

The interest actually received during the year ended 31 March 2023 was £6,000.

Therefore, how do we figure out what the amount is that should have actually been received
for the year ended 31 March 2023?

Amount paid - Amount due for previous year + Amount still due at this year end = Amount
that should have been paid for this year.

£6,000 was paid 



(£2,000 was paid towards the amount due at 31 March 2022)

Therefore, £4,000 paid was actually due for the year ended 31 March 2023

+

£1,000 is still to be paid for the year ended 31 March 2023

Therefore, £6,000-£2,000+£1,000 = £5,000 is actually due for the year ended 31 March
2023 - this amount will be used in the proforma.

Illustration:

Kamal Ltd. has the following results for the year ended 31/03/2023

£
Trading profits/loss before capital allowance 30,000
Chargeable gains 2,000

Interest income 62,000


Property income 50,000

Dividends received 100,000


Capital allowances for the year 3,000
Qualifying charitable donations 15,000

Kamal Ltd. had £2,000 interest accrued at 31/03/2022 and £3,000 of interest accrued at
31/03/2023.


454
What will Kamal Ltd.'s taxable total profits be for the year ended 31/03/2023?

Particular £

Trading profits/loss before capital


30,000
allowance

Less: capital allowances (3,000)


Tax adjusted trading profits 27,000
Interest receivable (62,000-2,000+3,000) 63,000
Chargeable gains 2,000
Property income 50,000
Total income 142,000
Less:
Qualifying charitable donations (15,000)
Taxable Total Profits 127,000

Taxable Total Profits 127,000

Exempt dividends  100,000

227,000 (This is below the upper limit, so the


Augmented Profits company is a small company and would not
have to pay its corporation tax in instalments)

455
Syllabus A4a. TX - UK Recap: Chargeable gains for
companies

The contents of the Paper TX - UK study guide for corporation tax under headings: 

- Chargeable gains for companies

Capital gains computation

How to calculate capital gains?

Capital gains and losses are netted off for each tax year

Corporation tax is paid upon this net gain.

For a company’s capital gain, the following computation can be used:

Disposal proceeds X

Less: Incidental cost of disposal (X)

Net proceeds X

Less: Acquisition Costs (X)

Capital Gain / (Capital loss) X / (X)

Less: Indexation allowance (X)

Taxable gain X

After all individual indexed gains and losses have been computed, then they must be
aggregated and the following computation can be used. 

Capital Gains in tax year X

456
Less: Capital losses in tax year (X)

Net Capital Gains in tax year X

Less: Capital losses brought forward

Taxable Gains X

This final figure is then taken to the TTP computation if it is a gain and carried forward if it is
a loss.

What is the indexation allowance?

The indexation allowance is an allowance given to companies to remove the part of the gain
that has been produced by increases in inflation rather than genuine increases in the value
of the asset. Indexation therefore reduces the chargeable gain.

This allowance is given to companies, instead of the annual exemption.

Note: indexation was frozen in December 2017 so even if the disposal is in 2022,
indexation will only be calculated up to December 2017.

How do we calculate the indexation allowance?

Prices increase due to inflation, therefore to avoid a company paying tax due to the
increases in inflation, an indexation allowance is calculated based on retail price indexes to
remove the effects of inflationary increases in the capital gain.

Total cost of asset *

(R.P.I disposal date or Dec 17 – R.P.I. acquisition date)/R.P.I. = Indexation


acquisition date allowance 

Note: you will not be expected to calculate indexation allowances in your exam. You will be
given the correct figure to use.

Other things regarding the Indexation allowance:

1. The indexation allowance can only reduce a capital gain to Nil, it cannot create a
capital loss or increase a capital loss.

2. If an asset has been enhanced, therefore capital expenditure has been incurred to
improve the earning capacity of the asset, then another indexation allowance must
be calculated for this enhancement expenditure.

The same calculation is used, replacing “cost” with the “enhancement expenditure”
and the “R.P.I acquisition” with R.P.I at enhancement date.

457
Total enhancement expenditure of asset * (R.P.I disposal date or Dec 17 – R.P.I.
enhancement date)/R.P.I. enhancement date = Indexation allowance for
enhancement expenditure

3. If there are incidental costs to acquisition or enhancement, for example, legal costs
incurred on the date of purchase, this cost also needs to be included in the “total
cost” and indexed along with it.

4. If the R.P.I factor has fallen from the month of acquisition to the month of disposal,
the indexation allowance is Nil.

5. Indexation was frozen in December 2017 so any inflation element of a gain from
January 2018 will be taxable.

Illustration:

Greenwood Ltd. disposed of an investment property on 31/12/2022.

They received disposal proceeds of £115,000 for the property and incurred legal fees on
disposal of £5,000.

They had initially purchased the property for £15,000 and incurred incidental costs on
acquisition of £1,500 on 31/12/2011.

They had spent £25,000 to extend the property on 31/12/2013.

Relevant indexation factors are:

On cost 0.306

On enhancement 0.218

What is the capital gain for the FY22?

Solution:

Disposal proceeds £115,000

Incidental costs to dispose (£5,000)

Net sale proceeds £110,000

Acquisition cost (£15,000)

Incidental costs to acquire (1,500)

Extension cost (£25,000)

Unindexed gain £68,500

Indexation allowance for acquisition (£5,049)  (W1)

458
Indexation allowance for extension (£5,450)  (W2)

Capital gain £58,001

The unindexed gain is a capital gain from which indexation allowance has not yet been
deducted.

W1:

I.A. for acquisition:

0.306 * £16,500 =  £5,049 

Note that the incidental costs to acquire are included (15,000 + 1,500) = 16,500.

W2:

I.A. for enhancement:

0.218 * £25,000 =  £5,450

Note: even though the asset was not sold until December 2022, indexation is only
calculated to December 2017.


459
Capital losses

How to get relief for capital losses?

When a company has a capital loss:

1. It is first set off against any Capital gains arising in the same accounting period.

2. Any remaining capital loss is then carried forward and set off against future Capital
gains.

Illustration:

Kruti Ltd. sold an office building on 06/06/2021 for £400,000, the unindexed cost of the
asset was £420,000.

There were no other chargeable asset sales in the FY21.

In FY22, Kruti Ltd. realised a capital gain of £25,000 on the sale of a small piece of land that
the company owned.

What is the capital income to be assessed to corporation tax in FY 21 and FY 22?

Solution:

• FY21 (1/4/21 to 31/3/22)

Disposal proceeds £400,000



Acquisition cost (£420,000)

Capital loss (£20,000)

• The capital income to be assessed to corporation tax in FY21 is Nil.

The loss of (£20,000) will be carried forward and set off against future capital gains.

• FY22 (1/4/22 to 31/3/23):

Net capital gain £25,000



Capital loss b/f (£20,000)

Chargeable gain £5,000

460
Disposals of shares by companies, with share
identification rules

Matching rules
Disposals of shares for individuals and for companies are extremely similar.

There are 2 differences, these are that we index the cost of the shares and when looking at
shares to be sold, and we do not look 30 days after the sale, we look 9 days previous to the
sale.

• There is no other difference between the two.

• For this reason, the same illustrations and quizzes have been used to explain this
are so that you can compare for yourself both applications.

• When shares are disposed of, a problem arises in finding their allowable cost, if the
shares were acquired over a long period of time.

• To make this simpler, HMRC uses a set of rules to determine the acquisition date
and cost of the shares being disposed of.

• These rules are called the matching rules.

Disposals of shares are matched with acquisitions in the following order:

1. Shares acquired on the same day of disposal.

2. Shares acquired within 9 days before disposal date (there is no indexation


calculation required for this match)

3. Shares from the share pool.

This would be much easier to understand if we did an example!

Illustration:

Benazir Ltd owns shares in L plc.

They acquired 1,500 shares in the company on 31/03/2016 for £20,000, and 500 shares on
30/06/2017 for £10,000.

On 21/02/2023 Benazir Ltd bought a further 200 shares in L plc. For £4,000.

• Benazir Ltd sold 1,000 shares in L. plc for £25,000 on 28/02/2023.

• Calculate Benazir’s capital gain on the disposal of the shares in February 2023.

461
Solution:

We need to dispose of 1,000 shares.

Let us apply our matching rules to see which shares we are disposing of.

FIRST MATCH – same SECOND MATCH – 9 days previous THIRD MATCH –


day acquisition to disposal acquisition share pool 

800 shares needed


None. 21/02/2023 – 200 shares for £4,000.
from share pool.

NOTE: you do not round indexation in the share pool

Indexation factors:

To June 2017 £356

To Dec 2017 £1,731


462
Share pool:

Description Number Cost Indexed cost


31/03/2016 purchase 1,500 £20,000 £ 20,000

Indexing to June 2017:



£ 356

Indexed cost of March 16


£ 20,356
purchase
30/06/2017 purchase 500 £10,000 £ 10,000

Total £ 30,356

Index to Dec 17: £ 1,731

Total 2,000 £ 30,000 £ 32,087

(800/2000) * £30,000 = (800/2000)*£32,087 =

Disposal from share pool (800)


(£12,000) (£12,835)

1,200
Remaining in share pool £18,000 £ 19,252
shares

Specifically note how each purchase will be indexed to the next EVENT date (an event
being either a purchase, sale or rights issue).

463
Calculating capital gain:

Disposal proceeds £25,000

Acquisition cost:

21/02/23 (£4,000)

Share pool (£12,835)

Capital gain £8,165

Note: The share pool figure in the above calculation is the indexed cost figure. This could
be shown separately as cost £12,000 and indexation (12,835 - 12,000) £835. This is useful
to be aware of because indexation cannot create or increase a loss so if the proceeds had
been £11,000 and the cost £12,000 there would have been an allowable loss of £1,000.
But if you had not separated out the cost and indexation you would have calculated a loss
of (11,000 - 12,835) £1,835 which would have been incorrect.

• You also might want to try to draw a timeline to ensure that you do not miss any
acquisition dates!

• From this illustration, you have learnt how to index shares.

• Shares issued through a bonus issue will not be indexed as no money has been
paid for them.

• It will be assumed as though they have been acquired on the last purchase date.

• Shares that have been issued via a rights issue will be indexed as normal, as money
has been paid for them.

464
Bonus issues, rights issues, takeovers and
reorganisations

Share issues

Bonus issues, rights issues, takeovers and reorganisations.

Once again, the treatment of bonus issues, rights issues, takeovers and reorganisations are
exactly the same for companies and individuals.

The only difference is that a company will index its cost, whereas an individual will get an
annual exemption.

For this reason, very similar illustrations and quizzes have been used so that the difference
can be highlighted to you.

Bonus Issues

This is an issue of shares to existing shareholders in proportion to the number of shares


owned at the date of the bonus issue.

• For example, if you owned 500 shares in a company and a 1:5 bonus issue was
declared, you would receive (500/5) *1 = 100 bonus shares.

• These shares are deemed to be acquired at the same date and at the same cost as
the original shares to which they relate.

• They have no cost of their own.

• Therefore, in your share pool, a bonus issue will only result in an increase in the
number of shares, and no increase in the indexed cost of shares.

465
Illustration:

Mina Ltd purchased shares in C Co.

The details of their purchases are below:

• May 2022 Purchased 3000 shares for £3,000

• Jan 2023 Purchased 1500 shares for £2,000

• March 2023 Bonus issue of 1:3 declared by the company.

• How many shares will Mina Ltd receive under the bonus issue?

• What is the cost of these shares?

Solution:

Total shares in company = 4,500

• Bonus shares received = (4,500/3) * 1 = 1,500 shares

• New total of shares at March 2023 = 4,500+1,500 = 6,000 shares

• The bonus shares will have a Nil cost.

• When they are included in the share pool, the shares purchased previously will not
be indexed to the bonus issue date.

• The indexation of all of the shares will only happen once the next monetary
purchase happens.

Rights Issues

A rights issue occurs where a company offers its existing shareholders the right to buy
extra shares.

Rights issues are similar to bonus issues in that the number of shares offered to each
shareholder is generally in proportion to his or her existing shareholding.

• The only difference is that a price is paid for these shares.

• The price for the shares is normally lower than current market value, in order for the
the existing shareholders to be attracted to taking up the issue.

466
Illustration:

Jack Ltd purchased share in Jill Ltd.

He had the following transactions in the company’s shares:

• Jul 2022 Purchased 6,000 shares for £15,000

• Sep 2022 Purchased 900 shares for £2,700

• Dec 2022 Took up 1:5 rights issue for £2.00 per share

• What will the rights issue cost Jack Ltd if they decide to subscribe to the issue fully?

Solution:

Total shares in company = 6,900

• Bonus shares received = (6,900/5) * 1 = 1,380 shares

• The rights shares will have a cost of £2.00*1,380 shares = £2,760

• Note carefully that these bonus issues and rights issue will follow the same
matching rules for shares when they are disposed.

• The bonus issues will be included in the share pool at no cost and the rights issue
shares will be included in the share pool at their respective cost.

• The rights issue share purchase will cause indexation of the previous purchases until
this date as this is a monetary purchase.

• Nothing changes with the matching rules.

467
Takeovers

Takeovers can either be for a share for share exchange, or a takeover can be for a cash
exchange.

We will deal with both of these situations separately via the use of illustrations.

• Takeovers (share for share exchange)

• If a takeover is for a share for share exchange, then no capital gains tax arises
immediately.

• The market value of the new holding provided will be used to apportion our initial
holding cost.  

• Then when we ultimately dispose of this new holding, we will use the original
holding cost, and this will result in a capital gain assessable.

Illustration:

Jayna Ltd owned 2000 shares in A plc which cost them £2,000 in 2010, and A plc was
being taken over by B plc in 2023

• Jayna Ltd was offered by B. plc 1,500 ordinary shares with a market value of £3,000
and 500 preference shares with a market value of £1,000.

• Jayna Ltd takes up the offer.

• Will capital gains tax arise immediately?

• If not, when Jayna Ltd sells these new ordinary shares and new preference shares,
what cost would be attributed to each?

468
Solution:

Total market value of new holding: £3,000+£1,000 = £4,000

Total cost of original holding: £2,000

Cost attributed to ordinary shares:

Market value of ordinary shares/Total market value of new holding * original cost

= £3,000/£4,000 * £2,000 = £1,500

Cost attributed to preference shares:

Market value of preference shares/Total market value of new holding * original cost

= £1,000/£4,000 * £2,000 = £500

• Jayna Ltd needs to use these costs as the acquisition cost when they decides to
sell the shares in B. plc.

• (They cannot use the market value of the shares when they were given to them).

• Takeovers (share for cash exchange)

• If a takeover is for a share for cash exchange, capital gains tax will arise immediately
for the proportion of cash given compared to the total market value of the new
holding.

• The market value of the new holding provided will be used to apportion our initial
holding cost to be used.

469
Illustration:

Jayna Ltd owned 2000 shares in A plc. which cost them £2,000 in 2010, and A plc was
being taken over by B plc in 2023.

• Jayna Ltd was offered by B. plc 1,500 ordinary shares with a market value of £3,000
and cash of £1,000.

• Jayna Ltd takes up the offer.

• Will capital gains tax arise immediately?

Solution:

Total market value of new holding: £3,000+£1,000 = £4,000

Total cost of original holding: £2,000

Cost attributed to ordinary shares:

Market value of ordinary shares/Total market value of new holding * original cost

= £3,000/£4,000 * £2,000 = £1,500

Cost attributed to cash given:

Cash received/Total market value of new holding * original cost

= £1,000/£4,000 * £2,000 = £500

Jayna Ltd needs to use this £500 as the acquisition cost of the shares that they are deemed
to have disposed of for the cash received.

Capital gains:

Disposal proceeds £1,000

Acquisition cost (£500)

Capital gain £500 (taxable immediately)

• No capital gain will arise on the share element, as described above.

470
Rollover relief

Capital gain reliefs for companies

Rollover relief for companies

Rollover relief for companies is the same as rollover relief for individuals.

The only difference between the two is that the indexed gain is rolled over for companies,
whereas individuals do not index the gain.

Explanation

Subject to certain conditions a company may claim that the gain arising on the disposal of
a business asset may be rolled over against the cost of acquiring a replacement business
asset.

Main effects:

1. Disposal of the old asset will arise in neither a gain nor a loss.

2. Cost of the new asset is reduced by the indexed gain that would have been
chargeable on the disposal of the old asset if the claim for roll over relief had not
been made.

Conditions:

1. The disposal must have been of a qualifying business asset and the reinvestment
must be in a qualifying business asset.

2. The reinvestment must be made 12 months prior to the sale or 36 months post the
sale.

3. All of the sale proceeds received on the sale must be reinvested for qualification of
full roll over relief. If only some of the sale proceeds are reinvested, then:

Total sale proceeds received-sale proceeds reinvested = indexed capital gain


realised NOW.

Total indexed capital gain-indexed capital gain realised now = indexed capital gain
to be rolled over.


471
Qualifying assets:

1. Land and buildings.

2. Fixed plant and machinery.

Both of these assets must be used in the business.

Illustration:

Jeremy Ltd. sold its business office on 30/06/2022 for £350,000.

This office cost the company £100,000 on 29/09/2003. Jeremy Ltd. bought another
business office for £250,000 on 31/12/2022. Indexation factor 0.915

• How much of the indexed capital gain can be rolled over?

• What is the base cost of new business office?

Solution:

Disposal proceeds £350,000

Acquisition cost (£100,000)

Unindexed capital gain £250,000

Indexation allowance (W1) (£91,500)

Indexed capital gain £158,500

Gain deferred (£58,500)

Capital gain now (W2) £100,000

Base cost of new business office:

• Cost of office £250,000

• Gain to be rolled over (£58,500)

• Base cost of new office £191,500

• This base cost will be used as the cost against the disposal of the new office.

W1:

0.915 * £100,000 = £91,500

W2:

Disposal proceeds received £350,000

472
Disposal proceeds reinvested (£250,000)

Capital gain to be realised now £100,000

473
Syllabus A4a. TX - UK Recap: The comprehensive
computation of corporation tax liability

The contents of the Paper TX - UK study guide for corporation tax under headings: 

- The comprehensive computation of corporation tax liability

Compute the corporation tax liability

How to calculate the corporation tax liability?

Corporation tax liability

A company will pay corporation tax at the rate of 19% for FY20, FY21 and FY22.

If a company’s CAP falls into a financial year prior to FY17 a hybrid rate will need to be
calculated (it is unlikely that this will be required in the ATX exam as the tax rates and
allowances section of the exam only includes rates for FY 18, FY19, FY20, FY21 and FY22).

As you know, dividends received by a company from non-associated companies are not
charged to corporation tax. However, they do determine whether a company is small or
large.

How?

You will need to add the dividends figure it to the taxable total profits.

If the total of this exceeds the upper limit, then the company will be deemed to be large.

Do not forget that dividends are just used to determine whether a company is small or
large, they are never subject to corporation tax!

474
Illustration:

A company has taxable total profits of £1,450,000.

They have received a dividend from a non-associated company of £250,000.

• Will they be considered to be a large company?

Illustration:

T.T.P    £1,450,000
Dividend     : £250,000   £250,000
Augmented profits   £1,700,000

Yes, the total has crossed £1,500,000 and therefore the company will be considered to be a
large company.

They will pay corporation tax at 19% like small companies but the difference is that they will
have to pay their corporation tax in quarterly instalments.

Corporation tax due 19% * £1,450,000 = £275,500

Note: the quarterly instalment dates for very large companies (profits above £20m) have
been brought forward by 4 months but they are not examinable in ATX - UK.

Related 51% group companies

Companies count as related 51% group companies if:

1. One company owns more than 51% of the other

2. Both companies are owned more than 51% by the same company

Which companies can/cannot be included in the group?

1. An individual is not a company, therefore if an individual controls 2 companies, these


companies will NOT be related 51% companies.

2. Dormant companies are not considered to be related companies.

3. Companies resident overseas are considered to be related companies.

What is control?

The parent company needs to own more than 50% of the share capital of the subsidiary at
the end of the previous chargeable accounting period. The 50% needs to be both direct
and effective interest. For example if A Ltd owns 51% of B Ltd and B Ltd owns 51% of C
Ltd, the situation would be as follows:

475
A Ltd is related to B Ltd so A would divide the limit by 2

B Ltd is related to A Ltd and C Ltd so B would divide the limit by 3

C Ltd is related to B Ltd so C would divide the limit by 2

What are the tax implications of related 51% group companies?

1. One annual investment allowance is given to the entire group. The group can decide
which companies get the allowance. Therefore, it is tax efficient to allocate the
allowance to large companies and companies which have purchased special rate
pool assets.

2. The upper limit of £1,500,000 is divided by the number of related 51% group
companies to determine an upper limit for each company in the group. If the
individual company’s profits exceed the upper limit, then they are deemed to be a
large company and must pay quarterly instalments of their corporation tax.

Illustration 1:

Q Ltd owns 51% of Z Ltd. and 65% of A Ltd. 

Z Ltd. owns 100% of Z Inc. (overseas company).

A.  Ltd. owns 100% of B Ltd. and 100% of C. Ltd. (dormant company)

Which companies are related 51% group companies?

Solution:

There are 5 related companies in this group.

• Q Ltd, Z Ltd, Z Inc, A Ltd and B Ltd.

• C Ltd. is not considered as it is a dormant company.

• The upper limit would be: £1,500,000/5 = £300,000

Illustration 2:

Q Ltd owns 51% of Z Ltd. and 65% of A Ltd. 

Z Ltd. owns 50% of Z Inc. (overseas company).

A.  Ltd. owns 60% of B Ltd. and 100% of C. Ltd. (dormant company)

476
Which companies are related 51% group companies of Q Ltd?

Solution:

Q Ltd is only related to Z Ltd and A Ltd so the limit would be divided by 3

C Ltd is excluded because it is dormant

B Ltd is excluded because the effective interest is less than 51% (65% x 60% = 39%)

Z Inc is excluded because the effective interest is less than 51% (51% x 50% = 25.5%)

Note:

A Ltd would include Q Ltd and B Ltd as related 51% companies (limit/3)

B Ltd would include A Ltd as a related 51% company (limit/2)

Z Ltd would include Q Ltd as a related 51% company (limit/2)

Hopefully you can see from these illustrations that there is not just one answer for a group.
It depends from which company you are looking from. The limit could be different for each
company.

When do large companies pay their C.T. instalments?

A large company will pay corporation tax in installments in the second year that they are
large on:

1. First installment = 3/CAP x Estimated C.T. liability on 14th of 7th month in CAP

2. Second installment = 3/CAP x Estimated C.T. liability on 14th of 10th month in CAP

3. Third installment = 3/CAP x Estimated C.T. liability on 14th of 13th month in CAP

4. Fourth balancing payment = (Final C.T. liability - payments already made) on 14th of 16th
month in CAP

Illustration

A Ltd. is a large company and has a final C.T. liability of £600,000 for the year ended
30/04/2022.

What/when are installments and the balancing payment made?

477
Solution

First installment of 1/4 x £600,000 = £150,000 on 14th November 2021

Second installment of 1/4 x £600,000 = £150,000 on 14th February 2022

Third installment of 1/4 x £600,000 = £150,000 on 14th May 2022

Balancing payment of (£600,000 - £450,000) = £150,000 on 14th August 2022


478
Syllabus A4a. TX - UK Recap: Group corporate structure
for C.T.

The contents of the Paper TX - UK study guide for corporation tax under headings: 

- The effect of a group corporate structure for corporation tax purposes

75% loss group

A group of companies is like a family, they can share their losses and
gains

There are 4 types of groups that you need to know:

A – Related 51% group companies

B – VAT groups

C – 75% loss groups

D – 75% gains groups

We have already dealt with related 51% group companies and VAT groups.

75% loss groups

There are 2 conditions that need to be satisfied for a company to be a part of a 75% loss
group.

These are:

1. The parent company must own (directly or indirectly) an effective interest of 75% of
the ordinary share capital all member companies.

479
Illustration:

A Ltd. owns 90% of B Ltd.

B Ltd. owns 90% of C. Ltd.

Which companies are members of this 75% loss group?

Solution:

All 3 companies are members.

This is because A Ltd. owns a direct interest of 90% in B Ltd. and an indirect interest of
81% (90% * 90%) in C Ltd.

Therefore, the parent effective interest is satisfied.

Illustration:

• A Ltd. owns 100% of B Ltd.

• B Ltd. owns 75% of C. Ltd.

• C. Ltd. owns 100% of D Ltd.

• Which companies are members of A Ltd’s 75% loss group?

Solution:

All 3 companies are members of A Ltd’s group

This is because A Ltd. owns a direct interest of 100% in B Ltd., an indirect interest of 75%
(100% * 75%) in C Ltd, and an indirect interest of 75%(100% * 75% * 100%) in D Ltd.

Therefore, the parent company condition is satisfied.

The effect of 75% loss groups

UK members of a 75% group can surrender losses to other UK members.

480
What losses can they surrender?

1. Excess property losses.

This means that the property losses of the company who has generated the loss
must relieve the loss against its own total income before surrendering it to a group
member.

Thus, the loss making company’s total income should be NIL before it surrenders its
property loss.

2. Excess qualifying charitable donations.

This means that the qualifying donations of the company who has generated the
loss must relieve the loss against its own total income before surrendering it to a
group member.

Thus, the loss making company’s total income should be NIL before it surrenders its
qualifying donation.

3. Trading losses.

This means that the trading losses of the company who generated them do NOT
need to relieve the loss against its total income or previous year’s income before
surrendering it to a group member.

Thus, the loss making company’s total income does not need to be NIL before it
surrenders its trading loss.

This trading loss cannot be carried back against group members income, it can only
be relieved in the corresponding period or carried forward to future periods. The
surrendering company can choose the amount to surrender as group relief.

A company may only surrender carried forward trade losses if the loss cannot be
used against its own profits.

The claimant company must calculate the profits available for offset via group relief
by first deducting all possible single company loss reliefs even if they do not intend
to claim them.

4. Non-trading loan relationship deficits.

A deficit arises on non-trade relationships when the non-trade interest expense is


greater than the non-trade interest income. All or some of this deficit can be
surrendered to group companies in the current year of carried forward to future
years.

It does not have to be set against the surrendering companies total


income first.

481
Conditions for loss relief

The loss relieved must be the lower off:

1. The loss of the surrendering company for the exact same period against which it is
being surrendered.

2. The profit of the claimant company for the exact same period against which it is
being claimed.

These periods are called “co-terminus periods”.

You will be able to understand this better with an illustration.

Illustration:

• Ilea Ltd. made a loss for the year ending 31/03/23 of (£180,000).

• William Ltd. joined the group on 01/01/2023 and made a profit of £100,000 for the
period ending 31/03/2023.

• Jane Ltd. had been a part of the group for many years and made a profit of £55,000
for the year ending 30/06/2023.

• How much loss relief can be obtained?

Solution:

Loss relieved against William Ltd.

• Only 3 months are co-terminus since William Ltd. joined the group
(01/01/23-31/03/23)

Therefore the lower of:

William: 3/12 * £100,000 = £25,000

Ilea: 3/12 * £180,000 = £45,000

• £25,000 loss can be relieved against William Ltd. profits.

Loss relieved against Jane Ltd.

• Only 9 months are co-terminus as both companies have a different year end
(01/07/22-31/03/23)

Therefore the lower of:

Jane: 9/12 * 55,000 = £41,250

Ilea: 9/12 * £180,000 = £135,000

£41,250 loss can be relieved against Jane Ltd. profits.


482
Loss memo:

22/23 trading loss  (£180,000)

Relief against William Ltd.  £25,000

Relief against Jane Ltd. £41,250

Loss to be carried forward against Ilea/group company future trading


£113,750
profits 

Illustration:

A Ltd. and B Ltd. are part of a 75% loss group. They both have 31/03 year endings.

• A Ltd. makes a trading loss of (£190,000). B. Ltd makes a profit of £180,000.

• How much of A Ltd.’s loss can be relieved?

Solution:

The lower of £190,000 and £180,000, therefore only £180,000 loss can be relieved and the
remaining loss will be carried forward against A Ltd.’s future trading profits or used for
future group relief.

483
Carried forward group relief

Carried forward group relief from 1 April 2017

A company which has a post 1 April 2017 loss carried forward may transfer all or part of
that loss to a member of the 75% group.

Losses that can be surrendered:

• Carried forward trade losses;

• Carried forward property losses;

• Carried forward non-trading loan relationship deficits;

• Carried forward management expenses;

Unlike current period group relief, the surrendering company can only surrender a
carried forward loss if it cannot use it itself.

When calculating available taxable profits against which to use the carried forward
loss relief, the claimant company must deduct its own losses first.

Note: if a company joins the group and already has carried forward losses, it cannot
surrender these losses to other group companies.

Illustration

Apple Plc has one 75% subsidiary, Banana Ltd. Their results for the year ended 31
March 2023 are as follows:

Apple Plc
Banana Ltd

£ £
Trading profit 70,000 40,000
Trading loss carried (5,000) (1,10,000)
forward from 31 March
2022
Non-trade loan 10,000 10,000
relationship income
Chargeable gain 12,000 6,000

Calculate the maximum carry forward group relief that Apple Plc can claim from
Banana Ltd.

Solution

Banana Ltd can only surrender the amount of carried forward loss that it cannot use
itself, even if it would not choose to use the loss itself:

484
Carried forward trade loss £110,000

Trading profit £(40,000)

NTLR income £(10,000)

Chargeable gain £(6,000)

Loss available for carry forward group relief £54,000

Apple Plc can only claim a loss against profits after deducting it’s own losses first:

Trading income £70,000

NTLR income £10,000

Chargeable gain £12,000

Carried forward trade loss £(5,000)

Available taxable total profits £87,000

Maximum carry forward group relief that Apple Plc can claim from Banana Ltd is
therefore £54,000 (the lower of £54,000 and £87,000)

Note: you will not be tested on group relief involving carried forward losses made
prior to 1 April 2017.

485
75% gains group

What is a 75% chargeable gains group?

This is a group in which members can:

1. Transfer assets at no gain or no loss.

The asset will be transferred between group members at its indexed cost (cost +
indexation until date of transfer).

This is similar to husband/wife or civil partner transfers for individuals.

2. Obtain group rollover relief.

Therefore one member of a group can sell a qualifying asset, and if another member
purchases a qualifying asset within the time limit, the chargeable gain on the first
asset can be rolled over against the purchase of the second asset of the other group
member.

(Rollover relief conditions must still be satisfied).

3. Chargeable gains or capital losses can be given to group members freely, to reduce
their taxable total profits as necessary.

An asset does not have to be physically moved and sold by another group member
for a chargeable gain or capital loss to arise on them, the gain or loss can simply be
transferred.

How does a company obtain membership into a 75% gains group?

• The parent company must hold a direct or indirect effective interest of more than
50% in each subsidiary.

• Subsidiary companies must own a direct interest of 75% of sub-subsidiary


companies.

Illustration:

A Ltd. owns 90% of B Ltd.

B Ltd. owns 75% of C. Ltd.

Which companies are members of this 75% gains group?


486
Solution:

All 3 companies are members.

This is because A Ltd. owns a direct interest of 90% in B Ltd. and an indirect interest of
67.5% (90% * 75%) in C Ltd.

Therefore, the parent effective interest is satisfied and the sub-subsidiary condition is
satisfied.

Illustration:

A Ltd. owns 100% of B Ltd.

B Ltd. owns 75% of C. Ltd.

C. Ltd. owns 75% of D Ltd.

Which companies are members of this 75% gains group?

Solution:

All 4 companies are members.

• This is because A Ltd. owns a direct interest of 100% in B Ltd., an indirect interest
of 75% (100% * 75%) in C Ltd, and an indirect interest of 56.25%(100% * 75% *
75%) in D Ltd.

• Additionally, B Ltd. owns 75% in the sub-subsidiary C Ltd.

• Finally, C Ltd. owns 75% in the sub-subsidiary D Ltd.

• Therefore, the parent company condition is satisfied and the sub-subsidiary


condition is satisfied.

Illustration:

Zooby Ltd. and Scrappy Ltd. are members of a 75% group. Zooby Ltd. sold a qualifying
asset for £500,000 and this resulted in a capital gain of £100,000.

• Scrappy Ltd. spent £650,000 on a qualifying asset 6 months after the sale of
Zoooby Ltd.’s asset.

What is the base cost of Scrappy Ltd.’s asset?

Solution:

As group rollover relief is available due to both assets being qualifying and purchased within
the necessary time limit, the base cost of Zooby Ltd.’s asset will be:

487
Purchase cost £650,000

Gain rolled over (£100,000)

Base cost        £550,000

No chargeable gain will result for Zooby Ltd. at present

Illustration:

If two companies are members of a capital gains group and one company transfers an
asset to another.

Will this asset be transferred at its original cost or indexed cost?

Solution:

Indexed cost

488
Syllabus A4b. The scope of CT

Syllabus: A4bi) Identify and calculate corporation tax for companies with investment business.

Companies with investment businesses

CT Implications

Companies with investment businesses

If a company makes investments and holds those investments, they are allowed to deduct
certain management expenses in relation to those investments. 

The company can also carry on a trade, however, to deduct these additional management
expenses, the company must also make and hold investments.

What are the management expenses?

These are deducted from total profits

1 Director's fees and commissions

2 Salaries of management

3 Audit fees

4 Office rent and rates

5 Bank interest

6 Capital allowances for the investment

Management expenses in excess of total profits

If management expenses are in excess of total profits, then the reliefs available are:

1 Carry forward relief against the next year's total profits

2 75% group relief


489
Illustration

C Ltd. has the following results for the year ended 31/03/2023:

Rental income £70,000

Interest receivable £20,000

Chargeable gains £3,000

Management expenses

Related to property £35,000

Other £60,000

Capital allowances

Related to property £2,300

Other £1,600

Interest payable £2,000

Director remuneration £3,000

What amount of management expenses will be carried forward?

Solution

Property income £70,000

Less:

Management expenses (£35,000)

Capital allowances (£2,300)

£32,700

Interest income

Interest receivable £20,000

Less interest payable (£2,000)

£18,000

Chargeable gains £3,000

£53,700

Less management expenses:

Other management expenses(£60,000)



Other capital allowances (£1,600)

Directors remuneration (£3,000)

Taxable total profits £Nil

Management expenses c/f £10,900


490
Syllabus: A4bii)
Close companies: Apply the definition of a close company to given situations
Conclude on the tax implications of a company being a close company or a close investment
holding company

Close companies

Close companies are

If a person runs their business as a company then they are a shareholder and employee of
the company. 

The company is a separate legal entity and has the legal rights to own assets.

If the company is UK resident or resident in the European Economic Area (EEA) and is
controlled by five or fewer shareholders and their associates or is controlled by any number
of directors and their associates, then the company is known as a close company. 

The shareholders or director-shareholders are also known as “participators”.

Special rules apply to these companies to prevent participators taking undue advantage of
corporation tax legislation by virtue of their positions of influence over the company’s
affairs.

Benefits provided to a shareholder who is not an employee of a close company

Shareholder tax implications

The shareholder is treated as receiving a dividend from the company which is subject to
income tax if the deemed dividend value exceeds the dividend nil rate band of £2,000, only
the excess value is subject to income tax at 8.75%, 33.75% or 39.35%.

The value of the deemed dividend is equivalent to the benefit which would have been
assessable if that person had been an employee of the company.

Close company tax implications

The company is treated as paying a dividend to the shareholder.

The company cannot reduce its trading profits by the expenses incurred in
connection with the benefit.

The company does not pay class 1A national insurance on the deemed
dividend.

491
Illustration

Jake Ltd. is UK resident and has 4 shareholders.

The company wants to give one of its shareholders a laptop computer.

The shareholder, John, is not a director or employee of the company. 

The company is considering 2 options to give the computer to John:

Option 1 buy the computer for £1,800 and give it to John

Option 2 give a computer that the company has already used and buy a replacement one
for the company for £1,800. The used computer has a market value of £150, and has a
balance of £Nil on the main pool. 

What should the company do?

Solution

Option 1 after tax cost

Payment (£1,800)

After tax cost £1,800


Option 2 after tax cost



Tax payment from balancing charge on main pool (£150*19%) = (£28.5)

Tax saving from AIA 100% on new purchase £1,800*19% = £342

Payment of new computer (£1,800)

After tax cost £1,487


Therefore, it is beneficial for the company to use Option 2.

Loans provided to a shareholder of a close company

Shareholder tax implications

A Loan benefit is assessable if the shareholder is also an employee of the close company. 

The shareholder who is not an employee is treated as receiving a dividend equivalent to the
loan benefit which would have been assessable.

When the loan is written off by the company then the shareholder/employee is treated as
receiving a distribution/ dividend equivalent to the loan written off.

492
Close company tax implications

The close company must pay a penalty to HMRC of 33.75% x loan provided to a
shareholder within 9 months and one day from end of the CAP.

The company can reclaim the penalty when the loan is repaid or when the
company writes off the loan.

By concession the penalty can be avoided in the following circumstances:

1) The shareholder repays the loan within the 9 month payment period.

2)  The shareholder meets the following three conditions:

- owns ≤ 5% of the shares 

- Employee of company 

- Loan ≤ £15,000

Illustration

Jake Ltd. is a UK resident company.

Jake owns 100% of the share capital in Jake Ltd. and is a director of the company. 

Jake Ltd. is giving Jake a £14,000 interest free loan. 

What will the tax implications be?

Solution

Jake is a shareholder and an employee of the company, therefore this will be taxed as the
beneficial loan benefit on Jake.


Beneficial loan benefit



£14,000*2% = £280 


The company will have to pay Class 1 A NIC on the benefit as well as a penalty. 

Class 1 A NIC £280*15.05% = £42


Penalty

33.75%*£14,000 = £4,725 This penalty is payable on the usual corporation tax payment
date

eg 9 months and 1 day after the CAP end and will be refunded to the company when the
loan is either repaid by the shareholder or written off. If the loan is written off then the
amount of the loan is treated as a distribution to the shareholder and the shareholder will be
taxed on it as if it were a dividend.


493
Syllabus: A4biii)
Identify and evaluate the significance of accounting periods on administration or winding up

Accounting periods on winding up

Winding up of a company

Liquidation

If a company decided to wind up, this does not mean that the company has gone bankrupt,
the company just wants to cease trading. 

There must be a separate corporation tax computation from the date of commencement of
winding up until the winding up has finished.

The date of commencement of winding up is the day that a liquidator is appointed to carry
out the liquidation.

On this date, one CAP will end and the CAP of winding up will begin, and continue until the
winding up has finished. 

It is very important to differentiate the penultimate and final CAPs.

Illustration

Jake Ltd. normally prepares accounts to 30/06. 

It commenced winding up and appointed a liquidator on 01/01/23.

Winding up was completed on 31/03/23.

What will the accounting periods be?

Solution

The penultimate CAP will be from 01/07/2022-31/12/22.

The final CAP will be from 01/01/23-31/03/23.


494
Syllabus: A4biv)
Conclude on the tax treatment of returns to shareholders after winding up has commenced

Tax treatment of returns on winding up

Dividends or Capital Disposal?

Tax treatment of returns on winding up

On the liquidation of a company, the liquidator will be appointed and will distribute cash or
other assets to the shareholders once the company’s creditors have been paid.

Then the shares in the company will be cancelled.

1 Shareholders are treated as receiving dividend income when the distribution


is made prior to winding up commencing. 

Tax from 0% to 39.35%

2 Shareholders are treated as disposing of their shares with proceeds equal to


the amount received on liquidation if the distribution is made after winding
up commences (during the period of liquidation).

Tax at 10%-20%.

495
Illustration

John Ltd. owns 70% of Jake Ltd.

Mr J owns the remaining 30% of Jake Ltd.

He has been the managing director of Jake Ltd. since 2010 and is an additional rate tax
payer.

Jake Ltd. commenced winding up and appointed a liquidator on 01/01/23, winding up of


the company will be completed on 31/03/2023.

Should Jake Ltd. distribute profits to John Ltd. and Mr J on 31/12/22 or 31/03/23?

Solution

For John Ltd. it will not make a difference because dividends are exempt from corporation
tax.


For Mr J, the distribution should be made on 31/03/2023 because it will be treated as a
capital disposal for him and because he owns more than 5% of the shares and is an
employee of the company, this disposal will qualify for E.R./Business Asset Disposal relief
at 10%. 


If the distribution is made before the winding up has commenced, it will be treated as
though a dividend has been given to Mr J and he will be taxed at 39.35% as he is already
an additional rate tax payer. 

496
Syllabus: A4bv)
Advise on the tax implications of a purchase by a company of its own shares

Purchase by a company of its own shares

Repurchase of shares by a company

Purchase by a company of its own shares

The Companies Act gives an unquoted company and companies quoted on the AIM the
right to purchase their own shares.

In some circumstances the shareholder is treated as making a capital gains tax disposal
rather than receiving a distribution attracting an income tax liability.

Conditions which must be met in order to be treated as a CGT disposal rather than as a
distribution are:

1 The company must be an unquoted trading company (companies quoted on


the AIM are treated as unquoted).

2 The shareholder must be resident in the UK.

3 The shares must normally have been owned by the shareholder for at least
five years.

4 The shareholder must either dispose of his entire interest or reduce their
interest in the share capital to 75% or less of the amount held before the
repurchase of shares, and

5 The shareholder must not immediately after the purchase be connected with
the company (must not control more than 30% of the issued share capital or
voting rights in the company.

497
If conditions are not met

If any one of the above provisions does not apply, then a payment for the purchase by a
company of its own shares will be treated in the normal way as a distribution.

The amount of its distribution is the excess of the payment over the amount originally
subscribed for the shares.

Note: Any legal costs and other expenditure incurred by the company in purchasing its own
shares will not be allowable against the company’s profits.

Illustration

Gary owns 10,000 shares in A Ltd.

A Ltd. has 4 equal shareholders. 

Gary will sell either 2,700 shares or 3,200 shares back to A Ltd. 

Why will the capital treatment apply if he sells 3,200 shares but not apply if he sells 2,700
shares?

Solution

It will apply if he sells 3,200 shares because only a sale of this amount will reduce his
holding to less than 75% of the original holding.

• Selling 2,700 shares



Original holding 10,000 shares/40,000 shares = 25%

Holding after sale 7,300 shares/37,300 shares = 19.5%

Reducing holding to less than 75% of original holding = 75%*25% = 18.75% 

Therefore, selling 2,700 shares has not reduced his holding to less than 75%
of the original holding and the deemed dividend treatment will apply in this
situation.

• Selling 3,200 shares



Original holding 10,000 shares/40,000 shares = 25%

Holding after sale 6,800 shares/36,800 shares = 18.5%

Reducing holding to less than 75% of original holding = 75%*25% = 18.75% 

Therefore, selling 3,200 shares has reduced his holding to less than 75% of
the original holding and the capital treatment will apply in this situation.


498
Syllabus A4c. Taxable total profits
Syllabus: A4ci) Identify qualifying research and development expenditure, both capital and
revenue, and determine the reliefs available by reference to the size of the individual company/
group

Research and development expenditure

Research and development expenditure

In order to encourage more spending on research and development expenditure

additional tax reliefs are given for qualifying research and development expenditure incurred
by companies. 

There are separate schemes for Small&Medium companies and Large companies.

• The exam question will tell you what size the company is.

Qualifying allowable revenue research expenditure includes:

• Materials

• Staff costs (salary + Class 1 secondary national insurance)

• Computer software

Scheme for SME:

1 If the company spends money on qualifying R&D, they get enhanced relief
which means that they can deduct an additional 130% of qualifying
expenditure for tax purposes.

2 If the deduction creates a trading loss, it may claim a cash repayment from
HMRC this is called a R&D tax credit which is 14.5% of the surrendered loss.

3 If this treatment is taken, then the surrendered loss cannot be carried


forward for future relief.

499
Illustration

K plc. Is a profitable manufacturing company. 

It is a small enterprise for the purposes of R&D. 

The company has recently decided to investigate the market for a new type of equipment
and has spent the following amounts in the year ended 31/12/2022:

Market research £8,000

Staff directly involved in researching the project £20,000

Administrative support for the R&D department £5,000

Heat and light in the R&D department £9,000

New software £4,000

An agency for temporary R&D staff £10,000

What tax relief is available in respect of this expenditure?

Solution

Allowable expenses that qualify for the enhanced tax relief:

Staff £20,000

Heat and light £9,000

Software £4,000

Agency staff £10,000

Total £43,000

100%+130% = 230% * £43,000 = £98,900 enhanced relief

Note:

Already charged £43,000

Additional to be charged (£98,900 - £43,000) = £55,900

Scheme for large companies:

Large companies cannot claim an additional deduction, instead they increase their taxable
income by 13% of the qualifying expenditure and then claim a 13% tax credit against their
C.T. liability. This is called ‘above the line’.

500
Calculation of the 13% tax credit:

13% of the qualifying R&D expenditure is:

• Included in the taxable income in TTP and taxed at 19% and

• then the whole 13% is deducted from the C.T. liability

For example, if the qualifying R&D expenditure is £110,000 then:

Increase TTP by £110,000 * 13% and then tax at 19% = £2,717 and,

 Then decrease C.T. liability by (13%*£110,000) £14,300 

this will have a net saving of £11,583.

Operation of the scheme

• In general the 13% tax credit is not received by the company but is
deducted from their corporation tax liability for the period concerned.

• If the amount of the 13% tax credit exceeds the company’s tax liability for
the period, the excess is paid directly to the company by HMRC.  

The repayment is restricted to the company’s PAYE and NIC liability in
respect of wages bills included in qualifying R&D expenditure.

• The amount of the 13% tax credit must be added to the company’s taxable
total profits and included as part of their income.

Illustration

C plc. Is a large company for the purposes of R&D expenditure. 

In the year ended 31 March 2023 they have spent £60,000 on qualifying R&D expenditure.

• What tax relief is available?

Solution

Increase in C.T. Liability:

13% * £60, 000 * 19% = (£1,482)

Decrease in C.T. Liability:

13% * £60, 000 = £7,800

Net saving: £6,318


501
Syllabus: A4ciii)
Determine the tax treatment of non trading deficits on loan relationships

Non trading deficit

Non trading loans

If interest payable on a non trading loan is more than the interest receivable on non trading
loans, then a non trading deficit will be created.

Any amount of the deficit can be used

• against total profits of the current chargeable accounting period

• against interest income of the previous 12 months

• against future total profits

• for group relief

Illustration

Cobble Ltd. has interest income of £10,000 and interest payable of £20,000 in the year
ended 31 December 2022, they also have trading profits of £3,000 and gift aid donations of
£1,500 in the year ended 31 December 2022 and the year ended 31 December 2023. 

How will this non trading deficit get tax relief?

502
Solution

• Non trading deficit



Interest receivable £10,000

Interest payable (£20,000)

Non trading deficit (£10,000)

• Year ended 31/12/2022



Trading profits £3,000

Non trading deficit (£1,500)

Qualifying charitable donations (1,500)


Taxable trading profits Nil


Gift aid donation wasted for the year ended 31/12/2022. 


Notice that the company can choose the amount to use in the current year and in this case
restricts the loss relief to £1,500 in order to save the gift aid deduction.

• Year ended 31/12/2023



Trading profits £3,000 

Less:

Non trading deficit (£1,500)

Trading profits £1,500

Less:

Gift aid donation £1,500)

Taxable trading profits Nil 


Again, only £1,500 was used to preserve the deduction of the gift aid donation.


503
Syllabus: A4eii)
Recognise the alternative tax treatments of intangible assets and conclude on the best
treatment for a given company and
Advise on the tax consequences of a transfer of intangible assets

Intangible fixed assets

What is an intangible fixed asset?

An intangible fixed asset are assets that cannot be touched, for example goodwill, patents,
copyrights and intellectual property. 

Companies may purchase these IFAs and incur further expenditure on them. 

As these are capital assets, they can be transferred within a 75% gains group at no gain/no
loss.

They get tax relief on these purchases by deducting amortisation of the assets from trading
profit, as amortisation is an allowable expense when calculating tax adjusted trading profit.

Alternative tax treatment of IFAs

As intangible fixed assets normally have very long lives, they are amortised over very long
periods. In this case, companies may elect for the IFA to be subject to a fixed rate
allowance of 4% per annum straight line. 

The election must be made within two years from the end of the accounting period in which
the IFA is acquired. 

On disposal of an intangible fixed asset, a company will calculate a profit of loss on the
sale. 

This will be the sale proceeds less the net book value (purchase price – amortisation until
date). A profit will increase taxable total profits and a loss will decrease taxable total profits.

Illustration

Bobble  Ltd  acquires  goodwill  during  its  nine  month  accounting  period  to 31
December 2020 for a cost of £100,000 and does not propose to amortise it, it wants to use
the alternative tax treatment.  The company has trading profits of £250,000 in this period.
Bobble Ltd. wants to sell the goodwill for £95,000 on 1 January 2023

504
• What is the allowable deduction for tax purposes for the cost of the goodwill
in the nine months to 31 December 2020?

• What will the tax adjusted trading profits be after this deduction?

• When must Bobble Ltd. make this election?

• What profit/loss will arise on the sale of the goodwill?

Solution

• Amortisation using the alternative treatment



01 April 2020 – 31 December 2020

9/12 * £100,000 * 4% = £3,000

• Tax adjusted trading profit for 9 months ended 31 December 2020



Trading profit £250,000

Less:

Amortisation (£3,000)

Tax adjusted trading profit £247,000

• The election must be made within 2 years from the end of the accounting
period in which the goodwill was acquired. Therefore, it must be made by 31
December 2022.

• Disposal of the goodwill


Total amortisation until date:



9 months to 31/12/20 £3,000

12 months to 31/12/21 (£100,000 *4%) £4,000

12 months to 31/12/22 (£100,000 *4%) £4,000

Total £11,000


Net book value of goodwill:



Cost £100,000

Amortisation until date (£11,000)

Net book value £89,000


Profit on disposal:

S.P. £95,000

NBV (£89,000)

Profit £6,000


This £6,000 will be added to the taxable total profits and increase the C.T. Payable.


505
Syllabus: A4cv)
Advise on the impact of the transfer pricing and thin capitalisation rules on companies

Transfer Pricing

HMRC wants to ensure that companies cannot reduce the total UK corporation tax by
substituting a transfer price which is below an arm’s length price for transactions between
companies where one company controls the other or both are controlled by the same
person.

The transfer pricing legislation covers not only sales but also lettings/hiring of property and
covers loan interest.

Where transfer pricing policies are under review the basic aim is to produce an arms length
price, i.e. the price which might have been expected if the parties had been independent
persons dealing with each other in a normal commercial manner unaffected by any special
relationship between them.

The OECD model will direct that the UK taxable total profits are adjusted to reflect the arms
length market value rather than the transfer price if using the transfer price results in an
overall reduction in the UK tax liability.

UK companies must apply the transfer pricing legislation in respect of transactions between
a resident and a non-resident company. 

It must also apply if both companies involved are UK resident.

There are, however, exemptions from the transfer pricing rules.

The main exemption to the transfer pricing rules applies if the advantaged company is small
or medium.

In the exam question, it will state whether the company is small or medium - if this
company is benefitting from the transfer pricing arrangement, then the prices do not need
to be adjusted to reflect market value.

Illustration

A Ltd. (large company) sells 5,000 units to B. Ltd. at £1.50. 

The market value of each unit is £3.50 

What effect will the transfer pricing legislation have on this transaction?


506
Solution

The transfer pricing legislation applies.

A Ltd. must increase its taxable total profits by £10,000 (£2*5,000)


507
Syllabus: A4cvi)
Advise on the restriction on the use of losses on a change in ownership of a company

Restriction on the use of losses

Anti-avoidance legislation

Purchasing loss making companies

The overall objective for companies forming a group is to minimise the tax liability of the
group as a whole.

Therefore an attractive features of company to be acquired is if the company being


purchased has trading losses brought forward.

There is anti-avoidance legislation that exists to prevent companies from trading in loss-
making companies.

It states that a company with a trading loss brought forward will be denied the right to
carry that loss forward in the following circumstances:

1 If there is a change in ownership of the company and



There is a major change in the nature of conduct of the company's trade
within any period of five years beginning no later than the change in
ownership and no earlier than three years before the change in ownership.

This rule applies to changes on or after 1 April 2017. Changes before 1 April
2017 are not examinable.

2 If there is a change in ownership of the company and



There is a revival of the trade after the scale of activities had become small
or negligible.

508
Illustration

Greg Ltd. purchased all of the share capital of Bob Ltd. on 01/04/2021.

On 01/04/2021 Bob Ltd. had trading losses to carry forward of (£170,000).

In the two years to 31/03/2023, Bob Ltd. made trading profits of £150,000 leaving (£20,000)
to be carried forward.

On 01/04/2023 Bob Ltd. changed the entire nature of it's trade from selling low cost bread
to selling premium bread and expensive cakes. 

Will the (£20,000) be allowed to be carried forward further?

Solution

Bob Ltd. has had a change of ownership and a change of nature of it's trade within a 5 year
period of the change of ownership, therefore the (£20,000) will not be allowed to be carried
forward.


509
Syllabus A4d. The corporation tax liability
Syllabus: A4di/ii) Assess the impact of the OECD model double tax treaty on corporation tax
and
Evaluate the meaning and implications of a permanent establishment

Permanent establishment

What is a permanent establishment?

The  rules  that  apply  to  the  taxing  of  overseas  income earned by UK Resident
companies  are  set  out  in  the Organisation for Economic Co-operation and Development
Model (OECD). 

Trading overseas:

The normal provision in tax treaties is that an overseas country will usually tax income
arising in its country from the commercial operation of a UK resident company if:

1) A trade is carried on within its boundaries

2) The profits are derived from a permanent establishment set up for that purpose

Permanent establishment

The term “permanent establishment” within an overseas country includes a place of


management, a branch, an office, a factory, a workshop or any mine. 

A UK resident company that possesses a permanent establishment trading within an


overseas country will normally be charged to tax on its overseas profits arising by UK
HMRC and the overseas authority under their own tax code.


510
Syllabus: A4diii)
Identify and advise on the tax implications of controlled foreign companies

Controlled foreign companies

A company which is resident in the UK

who wishes to set up an overseas company will be attracted to overseas countries which
have low rates of tax; these countries are called tax havens, because the company’s profits
will be taxed at a lower rate.

• If a UK company has an overseas subsidiary the normal treatment is that the


UK company will not pay UK corporation tax on the overseas dividend
remitted to the UK.

• This basically means that a UK resident company will set up a subsidiary in a


tax haven and have the subsidiary’s profits charged tax at a low rate, and then remit
dividends to the parent company in the UK, which are tax free.

However

if the overseas company qualifies as a controlled foreign company, then special rules will
apply to the taxing of the overseas company’s Taxable total profits.

Controlled foreign company definition

The controlled foreign company rules apply to owners of non-UK resident companies where
UK profits have been artificially diverted out of the UK corporation tax net, as explained
above.

A company is a CFC if it satisfies all of the following conditions:

Condition 1

 – A foreign resident company controlled from the UK

Condition 2

 – It is a foreign company resident overseas (ie resident outside of the UK)

A company is controlled by persons resident in the UK if:

511
1. A UK person or persons controls the company (>50%)

2. It is at least 40% held by a UK resident and at least 40% but no more than 55% by a
non-UK resident (the term ‘person/persons’ includes companies)

CFC Charge

If it is established that a foreign company is a CFC it may be necessary for any UK resident
company who owns at least 25% of the shares in the foreign company to pay a CFC charge
(additional corporation tax) to HMRC.

• This charge will be in respect of the chargeable profits of the foreign


company (chargeable profits are defined as income profits but not
chargeable gains, calculated using UK tax rules which have been artificially
diverted out of the UK corporation tax net).

Calculation of the CFC Charge:

[% x Chargeable profits of the CFC x C.T. Rate] – Foreign tax suffered on the chargeable
profits

Illustration:

A Ltd., a UK company with taxable profits of £20,000 for the year to 31 March 2023, holds
90% of an overseas subsidiary resident in Nemo Land. 

The subsidiary falls within the definition of  CFC. 

The overseas subsidiary has £600,000 of profits for the period. 75% of which are caught by
the CFC legislation and stand to be charged. 

The tax payable in Nemo Land is 5%

How much C.T. will A Ltd. have to pay?

Solution

UK C.T. Computation:

Taxable total profits £20,000

C.T. 19% = £3,800

Extra tax on CFC Profits:

(75%*£600,000*90%) * 19% = £76,950

Less:

DTR (5%*(75%*£600,000*90%) = (£20,250)

Total UK C.T. Payable £56,700

512
When can the CFC Charge be avoided?

The CFC charge can be avoided by a UK resident company if:

1. the foreign company did not have any chargeable profits (income profits
but not chargeable gains, calculated using UK tax rules which have been
artificially diverted out of the UK corporation tax net), or

2. the foreign company satisfies one of the exemptions listed below.

Exemptions

Avoiding the CFC charge:

Low profits exemption



 – This exemption applies if the foreign company’s profits do not exceed £500,000 and its
non-trading income does not exceed £50,000. 

For example, if the foreign company profits are £499,000 and it only has other income of
£45,000 – then the CFC Charge will not apply.

Low profit margin exemption

-    This exemption applies if the foreign company’s accounting profits are less than 10% of
its allowable expenditure. 


- For example, if the foreign company’s allowable expenditure is £100,000 and the
accounting profits are £9,000, then the CFC Charge will not apply.

Excluded territory exemption 



- This exemption applies if the foreign company is resident in a country which is specifically
listed as an excluded territory.

Tax rate is sufficiently high exemption 



- This exemption applies if the foreign company pays corporation tax overseas at a rate
which is at least 75% of the amount of tax that would have been paid if the company had
been UK resident. 


For example, if the foreign company pays tax at 14.25% or more, the CFC Charge will not
apply.

Exempt period exemption 



- There is an initial 12 month exemption from the CFC rules. This exemption will initially
apply but will not apply in the future. 


513
For example, the first 12 months of the foreign company coming under control of a UK
resident will be exempt from a CFC Charge.

Note

Make sure that you spot this in the exam, if there is a UK resident company with a
subsidiary overseas in a country with a low tax rate, it’s likely that it will be a CFC.

Illustration

K Ltd., a UK resident company, has two wholly owned subsidiaries.

1 B Inc. resident in Fishy Land where the C.T. Rate is 7%. This is an
investment company and has taxable profits of £30,000 per annum

2 C Inc. is resident in Hogwarts Land where the C.T. Rate is 18%. It has
trading profits of £2,000,000 per annum.

On which of these companies will the CFC Charge arise?

Solution:

1 Taxable profits are below £50,000 – therefore this is exempt and there will be
no CFC Charge

2 C.T. Rate is above 75% of the U.K. C.T. rate – therefore this is exempt and
there will be no CFC Charge


514
Syllabus: A4div)
Advise on the tax position of overseas companies trading in the UK

Overseas company trading in the UK

Foreign companies trading in the UK

Tax position

A non-UK resident company can be liable to UK C.T. on trading profits if it trades within the
UK, but not for trading with the UK.

C.T. is usually charged at the UK rate of 19% unless there is a double taxation treaty
specifying a lower rate.

1 Trading within the UK means either trading through a permanent


establishment or concluding contracts in the UK.

2 Trading with the UK means activities such as exporting goods to UK


customers, storing goods in the UK for customers and advertising and
marketing activities in the UK.

Illustration

M Inc is a large company resident in India. 

It manufactures mobile phones in India and sells them through a UK based agent. 

On 01/12/22 M. Inc rented a showroom and an office in London, with 2 sales managers. 

Will M. Inc. be liable to pay UK C.T. on it's trading profits?

Solution

They will be liable to pay UK C.T. on their trading profits from 01/12/22 because this is
when they started to trade within the UK. 

Before this, they were trading with the UK.


515
Syllabus: A4dv) Calculate double taxation relief

Double taxation relief (DTR)

Under UK tax law a company that is resident in the UK must pay UK C.T. on it's worldwide
income.

In the case of income arising in another country, that income may also be taxed in the
foreign country, and will be taxed in the UK, if the company is UK resident.

The rules that apply to the taxing of overseas income are set out in the Organisation for
Economic Co-operation and Development Model (OECD).

This model states that if there is no double taxation treaty between 2 countries, then double
taxation relief is available. (There will never be a treaty in your exam, you will always have to
calculate DTR)

Therefore, in order to avoid being taxed on the same income two times, double taxation
relief (DTR) is available, usually as a tax credit against the UK C.T. liability.

1 UK tax on overseas source

2 Overseas tax suffered.

Illustration

A Ltd. is UK resident.

It has one wholly owned subsidiary B. Inc. 

B. Inc. was incorporated in Barbados and trades from a permanent establishment in


Barbados. 

B. Inc. has not made an election to exempt its profits from UK C.T.

It's trading profits for FY22 are £300,000 and the C.T. rate in Barbados is 18%.

How much D.T.R will be available?

Solution

DTR is the lower of:

1) UK tax suffered (£300,000 *19%) = £57,000

2) Foreign tax suffered (£300,000 *18%) = £54,000

Therefore, DTR will be £54,000


516
Syllabus A4e. Group Structure for C.T.
Syllabus: A4eiv/v) Understand the meaning of consortium owned company and consortium
member and
Advise on the operation of consortium relief

Consortium owned company and member

Further loss relief is available if companies are structured as a


consortium

The tax reliefs available between qualifying companies where a consortium is involved are
more limited than for a 75% loss group.

Definition of a consortium

There are several parts of the definition:

1 A consortium exists where two or more companies (UK or overseas) between


them own at least 75% of another company, and each company’s holding is
at least 5%. 

For example, A Ltd owns 60% of C Ltd and B Ltd owns 20% of C Ltd, then
these companies will qualify to be in a consortium.

2 Ownership includes ordinary shares and assets and profits as for a 75%
group.

3 The investing company is known as a consortium member. 



For example, in the above situation, A Ltd and B Ltd will be the consortium
members.

4 The target company is known as a consortium company. 



For example, in the above situation, C Ltd will be the consortium company

5 The consortium company cannot be a member of a 75% loss group.

517
Consortium relief

is similar to group relief in many ways but with one main exception that losses can only be
surrendered from a cc to cm or from a cm to cc.

• For example, A Ltd can surrender losses to C Ltd and vice versa, and B Ltd
can surrender losses to C Ltd and vice versa BUT A Ltd cannot surrender
losses to B Ltd.

Current period and carried forward losses can be surrendered under consortium relief.

If the loss is made by the consortium company, the amount surrendered must first be
reduced by any potential loss relief claims against its own profits.

How much loss can be surrendered?

Between consortium company (CC) and the UK consortium members (CM), ie not between
the members. 

The maximum amount of loss that can be surrendered:

Lower of:


• % ownership x consortium company’s (target co) Taxable total profits or


Loss

• Consortium member’s (investing co) Taxable total profits/Loss

Illustration

P Ltd. has TTP £50,000 and owns 70% of C Ltd.

Q Ltd. has TTP £15,000 and owns 30% of C Ltd.

C Ltd has a loss of (£60,000)

What is the maximum consortium relief available for P Ltd and Q Ltd?

Solution

Maximum Consortium relief available is the lower of:

TTP of members/loss of consortium company * % ownership

P Ltd:

Maximum Consortium relief available is the lower of:

518
TTP £50,000

loss of consortium company * % ownership = £60,000 * 70% = £42,000

Loss relief available: £42,000 against P Ltd. profits

Q Ltd:

Maximum Consortium relief available is the lower of:

TTP £15,000

loss of consortium company * % ownership = £60,000 * 30% = £18,000

Loss relief available: £15,000 against Q Ltd. profits

Illustration

X Ltd. has a loss of (£100,000) and owns 40% of C Ltd.

Y Ltd has TTP of £15,000 and owns 38% of C Ltd.

C Ltd has TTP of £50,000.

How can X Ltd obtain loss relief within the consortium?

Solution

X Ltd. can use part of its loss to relieve up to 40% of C Ltd. TTP = £20,000

X Ltd. cannot surrender any of its loss to Y Ltd. 


519
Syllabus: A4evi) Determine pre-entry losses and understand their tax treatment

Pre-entry losses

Pre-entry losses

A group which anticipates making disposals which will give rise to substantial capital gains
might try to shelter those gains by acquiring a “capital loss company”.

This is a company which has capital losses brought forward.

The intention of the purchase of the company would be to set these capital losses against
the group’s capital gains and so reduce the group’s overall corporation tax liability.

There are tax avoidance measures in place to prevent this.

They only allow group companies with realised pre-entry capital losses to set them
against gains arising on the following types of disposals:

1 Disposals of assets which the company owned before it joined the group. 

For example, if a company has brought forward capital losses of £20,000
and sells an asset that it owned before it joined the group realising a gain of
£30,000 – it can set off it’s £20,000 brought forward loss.

2 Disposals of assets acquired by the company from outside the group since
becoming a group member. 

For example, if a company has brought forward capital losses of £20,000
and purchases an asset from a third party, after becoming a group member –

and then sells the asset realising a gain of £30,000 – it can set of it’s £20,000

brought forward loss.

520
Syllabus: A4evii/A4eviii)
Determine the degrouping charge where a company leaves a group within six years of
receiving an asset by way of a no gain/no loss transfer and
Determine the effects of the anti-avoidance provisions, where arrangements exist for a
company to leave a group

Degrouping charge

Degrouping charge

A degrouping charge may arise where a company:

1 Leaves a capital gains group

2 Within six years of acquiring an asset via a no gain, no loss transfer

3 Still owning the asset

Calculation of the degrouping charge:

Proceeds (M.V. at the date of the intra-group transfer)

Less: Cost to the group

Less: Indexation allowance

= Degrouping charge

This is basically the chargeable gain that would have arisen at the date of the original
transfer if at that time the companies had not been members of the same 75% gains group.

What do you do with this charge?

The charge will now be added to the consideration received by the selling company in
respect of the company that has left the group. If there is a degrouping loss, this will be
deducted from the consideration.


521
Note

It should be recognised that the increase to the consideration received by the selling
company will often be irrelevant due to the availability of the substantial shareholding
exemption (SSE).

• Recap – selling shares in a trading company where there is 10% ownership


overall, results in no taxable gain

• Therefore, if the SSE is available, the whole of the chargeable gain on the
sale of the shares, including the element relating to the degrouping charge
will be exempt from tax.

Illustration

Blue Ltd. sold its wholly owned subsidiary Rainbow Ltd. on 15 April 2022. 

Blue Ltd. had purchased a building on 1 August 1997 for £180,000. 

On 1 December 2012, the building was transferred to Rainbow Ltd. for £230,000. 

It’s market value on the date of the transfer was £375,000. 

Rainbow Ltd. still owned the building on 15 April 2022. 

Both companies prepare accounts to 31 March each year. 

Indexation Factor (Aug 97 - Dec 12) = 0.564

What are the tax implications of the sale of Rainbow Ltd?

Solution

When Rainbow Ltd. leaves the group, the company still owns an asset which it had
acquired from Blue Ltd. in the years preceding Rainbow Ltd.’s leaving.

Degrouping charge:

Proceeds £375,000

Less: 

Base cost (£180,000)

I.A. (Aug 97-Dec 12) ( 0.564 * £180,000)     (£101,520)

Degrouping charge £93,480

This charge is added to the consideration received by Blue Ltd. on the sale of the shares in
Rainbow Ltd.

However, any gain is likely to be exempt under the SSE rules as Blue Ltd. has owned 10%
of the shares for 12 months out of the previous 6 years.

522
Note: where an intangible asset has been transferred between the group companies at no
gain no loss within 6 years of the transferee leaving the group, no degrouping charge will
arise on the deemed disposal if the disposal qualifies for the substantial shareholding
exemption.

Companies leaving a group

Group relief ceases to be available once arrangements are in place to sell the shares
of a company.

This will usually occur sometime before the actual legal sale of the shares.

• HMRC consider that arrangements come into existence once there is


agreement in principle between the parties that the transaction will proceed. 

This is so even though such agreement is still subject to contract and not
finally binding on either party.

• HMRC will look at correspondence and details of the negotiations to


determine the date of arrangements coming into force. 

For the exam, you will be given a date on which a company is deemed to
leave a group, and from that date, group relief will cease to be available.


523
Syllabus: A4eix)
Advise on the tax treatment of an overseas branch

Tax treatment of overseas branch

Overseas Branch

Taxation for overseas branches

The profits of the branch are subject of UK corporation tax under trading profits.

The presence of the branch does not affect the profits threshold of the UK company.

If the branch makes a loss it can be relieved using the normal loss relief.

If the branch buys plant and machinery capital allowances can be claimed.

An overseas branch is simply an extension of the UK trade, and 100% of the branch profits
are assessed to UK corporation tax double tax relief (DTR) is then given where the overseas
branch’s profits are also taxed overseas. 


As an alternative to the treatment of overseas branch profits as detailed above, a company


can elect to treat the profits of its overseas branches as exempt from UK corporation tax.


Once made, the election is irrevocable and it applies to all of the company’s overseas
branches (existing and future).


The election must be made before the beginning of an accounting period to which it is to
apply.


An election will not be beneficial if a company has a loss making overseas branch as any
trading loss made by that branch would not be relievable when calculating taxable total
profits.


Even if a branch is currently profitable, a company might choose not to make an election if
double taxation relief means that there is little or no UK corporation tax liability in respect of
the overseas income.


524
If no election is made it will also mean that should the branch become loss making in the
future, the loss will be relievable in the taxable total profits of the company.

Tax implications of election to exempt profits

Advantages


If the overseas branch makes profits, then no additional UK corporation tax is payable.

Even if the UK corporation tax rate is greater than the foreign rate

Disadvantages


- If the branch is making losses with the election in place, it is not possible to
get relief for the losses made by the overseas branch.


- The election cannot be cancelled once made.



- The election applies to all overseas branches.


525
Syllabus: A4fi)
Determine the application of the substantial shareholdings exemption

Substantial shareholding exemption

If a company disposes of the whole or part of a substantial shareholding in another


company, then provided the qualifying criteria are met, any capital gain will not be a
chargeable gain and any capital loss will not be allowable.

What is a substantial shareholding?

A substantial shareholding is one where the investing company holds:

• 10% or more of the ordinary share capital; and

• 10% of the profits available for distribution to equity holders; and

• 10% of the assets available for distribution to equity holders upon a winding
up.

• The conditions must be met for a continuous 12 month period in the six
years prior to disposal.

Illustration

Peepy Ltd has owned 100% of the shares in one trading company Kreepy Ltd since 1
January 2017.  

Peepy Ltd has been offered £600,000 for the whole of the company’s share capital in
Kreepy Ltd.  

Peepy Ltd has taxable trading profits of £400,000 each year. 

The indexed cost for the shares in Kreepy Ltd was £400,000.   

What chargeable gain will arise on the sale of these shares in Kreepy Ltd?

Solution

No chargeable gain will arise on this sale. 

This is because, Kreepy Ltd. is a trading company and Peepy Ltd. owns >10% of the
shares for >12 months in the last 6 years. Therefore, this disposal will take place at no gain,
no loss. 


526
Syllabus A5: Stamp Taxes

Syllabus A5a. The scope of stamp taxes

Syllabus A5ai) Identify the property in respect of which stamp taxes are payable.

What are stamp taxes payable on?

Properties on which stamp taxes are payable

Stamp tax is a tax which is payable by companies and individuals when they buy shares
(stamp duty) and when they buy land and buildings situated in the UK (stamp duty land
tax). 

There are three types of stamp taxes.

1 Stamp duty payable on the purchase/ transfer of shares and other


marketable securities.

2 Stamp duty land tax payable on the purchase of property and lease
premiums.

3 Stamp duty reserve tax is payable where shares are transferred without
written documents.


527
Syllabus A5b. Liabilities arising on transfers

Syllabus A5bi) Advise on the stamp taxes payable on transfers of shares and securities

Shares and securities

Stamp Duty

Shares and securities

This is payable by the purchaser on the purchase/transfer of shares and securities when
transferred by a stock transfer form. 

It is not payable on newly issued shares

1 Stamp duty is payable at 0.5% of the purchase price unless the transfer is
covered by one of the exemptions.

2 The amount payable is always rounded up to the nearest £5 and is charged


on the date of the transfer form.

Stamp Duty Reserve Tax

Where shares and securities are transferred without a written document Stamp Duty
Reserve (SDRT) applies instead. 

There is no charge if the consideration is £1,000 or less. 

This is normally:

• Charged at the rate of 0.5% of the consideration payable for the shares/
securities.

• Rounded up to the nearest pence

• Levied on the date of the agreement


528
Note

If there is no consideration in money or money’s worth then there is usually no SDRT

Securities admitted to trading on a recognised growth market (eg AIM) but not listed on
that or any other market are exempt from SDRT.

Illustration

Harry had the transactions in 2022/23:

a) purchases 5,000 shares in a quoted company for £10,000

b) received 8,000 Q Plc shares from his uncle as a gift

What stamp duty/stamp duty reserve tax is payable?

Solution

a) 0.5% * £10,000 = £50

b) Nil – there is no SD payable on the gift of shares as there is no monetary
consideration


529
Syllabus A5bii)
Advise on the stamp taxes payable on transfers of land

Land

Stamp Duty Land Tax

SDLT is payable on:

Transactions involving land, unless the transaction is specifically exempt.

1 This applies to transactions in UK property and lease premiums.

2 The rate of SDLT varies depending on the purchase price and whether the
land and buildings is residential or commercial.

3 Once the purchase price exceeds the particular threshold the whole amount
is charged at the corresponding rate.

Note - the charge to stamp duty land tax on residential property in not on the ATX - UK
syllabus.

Non-residential property rates

Up to £150,000 – 0% 

£150,001 - £250,000 - 2%

£250,001 and over - 5%

530
Illustration

Ray inherited £400,000 on the death of his aunt Daisie, and he wants to buy a factory
for his new business at a cost of £400,000.

What is the amount of SDLT payable by Ray?

Solution


Factory

£150,000 x 0% =0

£100,000 x 2% = £2,000

Total £250,000

£150,000 x 5% = £7,500

Total £400,000 - SDLT £9,500 


531
Syllabus A5c/d. Exemptions and Reliefs

Syllabus A5ci/ii
Identify transfers involving no consideration.
Advise on group transactions.

Stamp taxes - exemptions and reliefs

Stamp taxes - exemptions and reliefs

The main exemptions relate to transfers where no consideration has been given.

1 Gifts at no consideration

2 Transfers of assets between 75% group companies

3 Divorce arrangements

4 Variation of wills

5 Changes in trustees

6 Takeovers, reconstructions or amalgamations

7 On the purchase of government stocks

8 On the purchase of company loan stock

9 On the purchase of unit trusts

Transactions between companies

Sales of assets between companies in the same group are exempt from SDLT and
stamp duty if:

-  One company is the beneficial owner of at least 75% of the issued share capital in
one or more other companies.

Either of the companies can be non- UK resident i.e foreign parent or party to the
transaction. When the purchasing company leaves the group within 3 years of the
property transfer, the SDLT exemption is withdrawn and SDLT becomes payable.

532
Illustration

Greg received 100,000 of £1 ordinary shares in an unquoted company Able Ltd as a gift
from his father when they were worth £60,000.

How much stamp duty is payable?

Solution

This is a gift of unquoted shares, there is no stamp duty payable on gifts.

Illustration

White Ltd. and Black Ltd. are members of a 75% group.

Black Ltd transfers an office building to White Ltd on 1 December 2022 when the
market value is £310,000

How much stamp duty is payable?

Solution

Transfer of building between companies in the same 75% group are exempt for stamp
duty, therefore there is £Nil payable.


533
Syllabus A6: Value Added Tax

Syllabus A6a. TX - UK Recap: The VAT registration


requirements

The contents of the Paper TX - UK study guide for value added tax (VAT), under headings: 

- The VAT registration requirements

VAT Registration - Compulsory and Voluntary

When is it compulsory to register for VAT?

When your sales (excluding VAT) go over the registration limit (£85,000).

There are 2 separate tests for compulsory registration:

1. Historic Turnover

2. Future Prospects

When you satisfy both tests HMRC (HM Revenue and Customs) will use the test that gives
the earlier registration date.

Historic Turnover test

At the end of every month check to see if the last 12 month sales were over £85,000.

If so, you have 30 days to tell HMRC (30 days of the end of the month in which the limit is
exceeded)

534
You are then registered for VAT from the end of the next month (or earlier if agreed)

• So let’s say the limit was exceeded in April

• You must notify HMRC by 30th May (within 30 days of the end of the month - April)

• You will be registered for VAT from 1st June

Illustration 1:

Year ended 31st December.



Sales were £96,000 (accrued evenly).

When would we become VAT registered?

• Answer

• 96,000 / 12  = 8,000 per month

• So limit is reached 85,000 / 8,000 = 10.63 months (October)

• So tell HMRC by 30th November and will be registered for VAT from 1st December

Future Prospects test

If you think the limit (£85,000) will be reached in the next 30 days alone

• then you have 30 days to tell HMRC and

• registration starts at the beginning of the 30 days you expect to reach the limit

• For example:

On 1 July, the company signed a contract valued at £100,000 for completion during
July.

The company will register for VAT from 1 July and have to notify HMRC by 30 July.

535
Illustration 2:

Guy starts to trade and in the year ended 31st December sales are expected to be
£240,000 (accrued evenly).

When would we become VAT registered?

• Answer - using the historic test because the threshold is not £85,000 in one 30
day period alone.

• 240,000 / 12  = 20,000 per month

• 85,000 / 20,000 = 4.25 month (April)

• So limit is expected to be reached in the fourth month (April)

• So tell HMRC by 30th May and registration starts on 1st June

Note: although the historic test tells us to look back 12 months, when someone starts to
trade you look back after every month as they may need to be registered before 12 months
have gone by.

Illustration 3:

The budgeted turnover of Shobha Ltd. in the first 9 months is £810 000.

The company starts trade on the 1st of July.

When must the company register for VAT?

Solution: using the future test as the threshold is exceeded in one 30 day period alone

£810 000 / 9 = £90 000 per month

Therefore, the limit would be crossed in the first month of operation (July).

HMRC will need to be notified by 30th July (within 30 days).

Registration will be effective from 01/07 (the beginning of the 30 day period).

536
De-registration

• A trader stops being liable to VAT registration when it ceases to make taxable
supplies.

The trader must notify HMRC within 30 days and will be deregistered from the date
of cessation or from an earlier agreed date.

• A trader may also deregister for VAT when its expected taxable turnover in the next
12 months is expected to fall below £83,000.

The trader may deregister for VAT if they consider this beneficial.

Illustration:

A company has been VAT registered for many years, however it has recently faced financial
difficulties and sales for the year ended 31/12/2022 are forecast to be £60,000.

• Can the company deregister for VAT?

• When will the de-registration be applicable?

Solution:

The company can request HMRC to cancel its registration because its taxable supplies for
the next 12 months are below £83,000.

The de-registration will be effective from the date on which the request is made or from an
earlier agreed date.

537
VAT implications on selling a business (deregistering permanently)

General Rule

• When a business is sold, it will cease to be registered for VAT.

The sale of the business is assumed to be a taxable supply for VAT purposes.

Therefore, all of the assets, such as plant, equipment and trading inventory owned
by the business, will need to have output tax payable on them when the business is
sold.

An exception is made if the VAT due is less than or equal to £1,000. In this situation,
VAT will not be payable.

• Illustration:

Cow plc. was being sold in the year ended 31/03/2023.

It owned plant and equipment costing £1,200,000 (VAT inclusive) and had inventory
remaining that cost £120,000 (VAT inclusive).

All of the input VAT on the inventory had been claimed in previous VAT returns.

How much output VAT will be payable on the sale of this business assuming the
plant and inventory are sold for cost?

• Solution:

VAT payable

Plant and machinery £1,200,000 * 1/6  = £200,000

Inventory £120,000 * 1/6                       = £20,000

Total VAT payable                                = £220,000

Exception to general rule (where the sale is not treated as a taxable supply)

If the business disposes of its assets and trade as a going concern, no output VAT
will be charged as it will be outside the scope of VAT if the following conditions are
met.

538
The conditions for this treatment are:

1. The business is transferred as a going concern

2. No significant break in trading

3. The same type of trade is pursued by the transferee

4. The transferee is or will become VAT registered

Voluntary registration for VAT

Even if someone is not required to register for VAT, once they are making taxable supplies,
they are allowed to.

For example, if a company makes zero rated supplies, they are not required to register for
VAT, but they are allowed to do so.

Advantages of voluntary registration:

1. Avoids late registration penalties.

2. Can recover input tax on supplies.

3. Disguises a small company to look big. (Investors may be apprehensive to invest in


a small company).

4. If a company makes zero rated supplies and standard rated purchases, then the
company will be eligible for repayments from HMRC.

Disadvantages of voluntary registration:

1. VAT added to the selling price will make an item more expensive for a final
consumer who is not VAT registered, and therefore reduce competitive advantage of
the business.

2. If the trader wants to remain competitive and still be VAT registered, then the profits
of the trader will suffer as they will have to suffer the output VAT payments on their
own, they cannot pass them on to the final consumer.

539
Illustration:

Villa sells furniture, a taxable supply. Her taxable turnover for the previous 12 months is
£68,000 and standard rated purchases are £45,000 (vat inclusive).

Villa sells to final consumers who are not VAT registered.

• Competition is high and most traders in this field are not VAT registered, therefore,
Villa cannot increase her prices. If she does, customers will go elsewhere.

• Is it beneficial for Villa to register for VAT or not?

Solution:

Profit without registering:

Sales revenue £68,000



Purchases (£45,000)

Gross profit £23,000

• Profit after registration:

Sales revenue £68,000



Purchases (£45,000)

Gross profit £23,000

Less VAT paid (£3,833) (W1)

Net profit £19,167

• W1:

VAT payable: 20/120 * £68,000 = £11,333



VAT receivable: 20/120 * £45,000 = (£7,500)

Net VAT payable         = £3,833

540
Recovery of pre-registration input VAT

Recovery of input VAT prior to registration

Input VAT incurred prior to VAT registration can be recovered on goods and services
purchased in certain circumstances. These include:

Purchased
Goods Services
item

Cannot be acquired more than 4 years Cannot be supplied more than 6


Time limit?
prior to registration. months prior to registration

Must be acquired for business Must be acquired for business


Purpose?
purposes purposes

The goods purchased that pre- Services are consumed


registration input VAT will be claimed immediately as they are
Still in hand?
on must still be in inventory prior to provided. Therefore, this is not
registration. applicable. 

541
How do we claim the pre-registration input VAT?

We treat these goods/services as being purchased on the first day of VAT registration,
therefore we will claim the input VAT when we file our first VAT return.

Illustration:

Sunny Ltd. registered for VAT on 31/01/2023.

He has to file his VAT returns quarterly, and his first return will be filed on 01/04/2023.

Sunil Ltd. purchased inventory for the business on 31/03/2022.

This inventory is still in stock. It’s VAT inclusive price is £120.

How will the input VAT for this purchase be treated?

Solution:

Sunil Ltd. will claim the £20 (input vat paid) on his first VAT return in 01/04/2023.

This is because these goods qualify to have pre-registration VAT claimed on them.

They are used for business purposes, still in stock and purchased within 4 years of VAT
registration.

542
Conditions for companies to be treated as VAT group

VAT registration requirements

Conditions:

1. 2 or more companies must be associated with each other.

That is one company must own 51% or more of the share capital in another
company, or 2 companies must be under common control.

2. All companies must be UK resident or trading from a permanent establishment in


the UK.

Consequences of Group Registration:

1. The VAT Group is treated for VAT purposes as a single company registered for VAT
on its own.

2. There will be 1 VAT registration number for the whole group.

3. One VAT return will need to be filed on behalf of the whole group.

4. The group must have a representative who fills in the VAT return.

This member will have to gather all of the output and input VAT of the individual
members and fill it in on one return.

This representative is also responsible for paying VAT on behalf of the group.

543
Advantages of a VAT Group:

1. There is no VAT on intra-group supplies

2. Only one return must be filed, therefore administration costs will be saved.

Disadvantages of a VAT Group:

1. All members remain jointly and severally liable

2. A single return may cause administration difficulties in collecting and collating the
information

3. There are special VAT schemes for businesses such as the cash, annual and flat rate
schemes.

To enter into these schemes, a business must have a turnover under a certain limit.

Since a VAT group is treated as a single company, the whole VAT group’s turnover will be
considered in comparison to the limit, when one company in the group is trying to enter into
the scheme.

Thus, it is unlikely that a company in VAT group will be eligible to enter into such schemes,
whereas if they were not in the VAT group, they would be more likely to qualify.

For example, there is a VAT group with 4 companies (A Ltd., B Ltd., C Ltd., and D Ltd.).

The annual turnover of the entire group is £5,400,000, and each individual company’s
annual turnover is £1,350,000.

B. Ltd. wants to enter into the cash accounting scheme, the company’s individual turnover
is within the limit of £1,350,000, however it cannot enter the scheme because the VAT
group’s annual turnover of £5,400,000 will be considered instead of individual company
turnover and B Ltd. will not qualify.

Illustration

Jay owns shares in a number of companies set out below

60% in A Ltd., and 80% in B Ltd.

B Ltd owns 80% in C Ltd and 51% in G Inc. (An overseas company)

Which of the above companies can be in a VAT group?


544
Solution:

A Ltd., B. Ltd., and C. Ltd can be in a VAT Group.

This is because Jay effectively owns more that 50% of the shares in each of them and they
are trading from a permanent establishment in the UK.

However, G Inc. is not trading from a permanent establishment in the UK and can therefore
not be included in the group.

545
Syllabus A6a. TX - UK Recap: Computation of VAT
liabilities

The contents of the Paper TX - UK study guide for value added tax (VAT), under headings: 
- The computation of VAT liabilities

Calculate the amount of VAT payable/recoverable

VAT payable/recoverable

VAT on standard rated supplies is 20%

Therefore, if an item is standard rated and VAT inclusive, then its total amount will be 100%
+ 20% = 120%.

• To find the VAT element alone:

VAT inclusive price * 20/120 = VAT element

Or

VAT inclusive price * 1/6 = VAT element

• VAT paid on standard rated purchases is called “input VAT” and can be claimed from
the government.

VAT charged on standard rated sales is called “output VAT” and must be paid to the
government.

The net of these 2 amounts will actually be payable/receivable from the government.

Illustration:

Mr. Mohan is self employed and has made standard rated sales of £120 (VAT inclusive) in
February 2023 and has standard rated purchases of £60 (Vat inclusive) from a VAT
registered supplier.

• What is Mr. Mohan’s VAT payable/recoverable?

Solution:

Output VAT: 20/120 * £120 = £20



Input VAT: 20/120 * 60 = (£10)

Net output VAT payable = £10


546
Understand how VAT is accounted for and administered

VAT return accounting

Quarterly accounting for VAT and electronic filing

1. On registration, the trader must charge VAT on all taxable supplies (output VAT).

2. The trader can also reclaim VAT on all taxable supplies purchased (input VAT).

3. At the end of a 3-month period, the trader accounts to HMRC for all the output tax
less the input tax on their VAT return.

4. VAT is accounted for quarterly

5. VAT registered businesses must file their returns and make payments online.

6. The deadline for submitting the VAT return and making payments electronically is 1
month and 7 days after the period has ended.

Therefore, for the period ending 31/03/2023, the return with payment can be
submitted electronically on 07/05/2023.

Illustration 1

Cow Ltd's sales (standard rated) for the first 3 months were:

January 2023 - £30,000



February 2023 - £30,000

March 2023 - £40,000

How, and when, will Cow Ltd have to submit its quarterly VAT return and pay any related
VAT liability?

Solution

Cow Ltd will have to file its VAT returns online and pay the VAT which is due electronically.

The deadline for filling the VAT return and paying any VAT is one months and seven days
after the end of each quarter.

So, in our case, Cow Ltd will pay (30,000 + 30,000 + 40,000) x 20% = £20,000 VAT on 7
May 2023 for the quarter ended 31 March 2023.


547
Illustration 2

For the quarter ended 30/06/2022, Pooja Ltd. had output vat of £10,000 and input VAT of
£7,000.

When will the company be required to file and pay the VAT liability and how much is it?

Solution:

Output VAT £10,000



Input VAT (£7,000)

Net VAT payable £3,000

• Pooja Ltd. must file the return and make the payment online on 07/08/2022.

Other points

1. Because VAT is a self-assessed tax, HMRC make control visits to VAT registered
traders.

The purpose of a control visit is to provide an opportunity for HMRC to check the
accuracy of VAT returns.

2. A business may choose to submit monthly returns but would only do so if it received
regular VAT repayments.

This would arise where the business had standard rated purchases and expenses
but made zero rated sales and hence always had more input tax than output tax and
would claim a repayment.

3. If a trader’s VAT liability exceeds £2,000,000 over a 12-month period, they must
make monthly payments on account of the VAT liability.

548
Recognise the tax point when goods or services are
supplied

What is the tax point?

The tax point is the date used to identify the VAT period which should be used to include
the output or input VAT.

Basic tax point

This is:

1. The date the goods are delivered

or

2. The date the services are performed

Actual tax point:

The actual tax point is used more frequently than the basic tax point.

The actual tax point is the earlier of:

1. The date the cash is received or paid before the goods

and

2. The basic tax point

The basic tax point date is replaced by the invoice date if an invoice is issued within 14
days of the basic tax point.

549
Here is a simple way to work your tax point out:

1. Step 1:

The basic tax point is the date that the goods are delivered or services are
performed.

Use the basic tax point if the answers to the below two steps are NO.

2. Step 2:

Is cash paid or received before the basic tax point date? (Actual tax point)

Yes – Use this date.



No – Go to step 3

3. Step 3:

Is an invoice issued within 14 days after the goods are delivered or services are
performed (basic tax point date)? (Actual tax point)

Yes – Use this date.



No – Use the basic tax point date from step 1.

You will find this easier to understand with an illustration!

Illustration:

Joe is a sole trader in business selling furniture (a standard rated supply). His year end is 31
December.

He sells furniture to Ikea on 31/05/2022 for £50,000 (Vat inclusive).

31/05/2022 Deposit of £2,000 received



30/09/2022 Goods delivered

12/10/2022 Invoice issued for goods

02/01/2023 Balance of £48,000 paid for goods.

Joe files his VAT returns quarterly.

On which VAT return will this output VAT be included?

• Here is how the transaction went:

Solution:

1. Step 1:

The basic tax point is 30/09/2022. Let us move on to the other 2 steps.

2. Step 2:

Is cash paid or received before the basic tax point date? (Actual tax point)

Yes – £2,000 deposit but the remaining has not been paid.

Therefore, for this £2,000 sale the output VAT related to it of:

£2,000 * 1/6 = £333 must be included in the VAT return filed on 30/06/2022.

550
Now, for the remaining £48,000 – we must go through the steps again.

1. Step 1:

The basic tax point is 30/09/2022. Let us move on to the other 2 steps.

2. Step 2:

Is cash paid or received before the basic tax point date? (Actual tax point)

No – Go to step 3

3. Step 3:

Is an invoice issued within 14 days after the goods are delivered or services are
performed (basic tax point date)? (Actual tax point)

Yes – Invoice was issued on 12/10/2022 and the goods were delivered on 30/09/2022.

Therefore, for this £48,000 sale the output VAT related to it of:

£48,000 * 1/6 = £8,000 must be included in the VAT return filed on 31/12/2022.

551
Information that must be given on a VAT invoice

What should a VAT invoice contain?

A VAT registered trader making a supply to another taxable person must issue a VAT invoice
within 30 days of the relevant tax point.

A VAT invoice must contain certain information including:

• VAT registration number

• The tax point

• The rate of VAT for each supply

• The VAT exclusive amount for each supply

• The total VAT exclusive amount

• The amount of VAT payable

• The invoice date and invoice number

• The type of supply

• The quantity and description of the goods supplied

• The company’s name and address

• The name and address of the customer

If a sales invoice is meant to be valid for VAT purposes i.e. a separate VAT invoice does not
need to be issued, then all of the above needs to be included in the sales invoice.

Otherwise, a separate VAT invoice will need to be issued.

A VAT invoice must be issued within 30 days of making a taxable supply.

VAT records (including VAT invoices) must normally be retained for six years. 


552
Simplified VAT invoice

A less detailed VAT invoice may be issued by a taxable person where the invoice is for a
total including VAT of up to £250.

Such an invoice must show:

1. The supplier’s name, address and registration number

2. The date of the supply

3. A description of the goods or services supplied

4. The rate of VAT chargeable

5. The total amount chargeable including VAT

Zero-rated and exempt supplies must not be included in less detailed invoices.

553
Principles regarding the valuation of supplies

Value of supply

The value of a supply is the VAT-exclusive price on which VAT is charged.

With a standard rate of 20%:

• Value + VAT = Selling price 



£100 + £20.00 = £120.00

Discounts offered

VAT is chargeable on the actual amount received where a discount is offered for prompt
payment.

1. If the discount is not taken the VAT is charged on the full sale price

2. if the discount is taken, then the VAT is based on the discounted price.

However, it is not as straightforward as that because we often don’t know when a customer
will pay.

Therefore, the supplier can charge the full amount of the VAT and then issue a credit note
for the discount if it is taken or the supplier can issue an invoice stating the terms of the
discount and that the customer can only reclaim the VAT on the amount actually paid.

554
Illustration:

Tony is a sole trader and makes standard rated sales.

He offers a discount of 5% to customers who pay within 14 days.

He makes a sale of £100 (VAT exclusive).

1) What is the output VAT charged if the customer pays within 14 days?

2) What is the output VAT charged if the customer doesn't pay within 14 days?

1) Solution ( if the customer pays within 14 days):

Sale £100
Discount (5%)  (£5)
Net Sale £95
Output VAT charged (20% * £95) = £19
Selling price  (95 + 19) = £114

2) Solution ( if the customer doesn't pay within 14 days):

Sale £100

Discount (0%)  (£0)

Net Sale £100

Output VAT charged (20% * £100) = £20

Selling price  (100 + 20) = £120

555
Zero rated and exempt supplies

Types of supplies

Standard rated supplies

A standard rated supply is taxable at 20%.

If a trader is VAT registered and makes standard rated purchases, they can reclaim input
VAT at 20%, and they must pay output VAT of 20% on their standard rated sales.

For example, an accountancy firm sold their services for £240 and made purchases of
stationery of £12. 

Both of these are standard rated items and VAT inclusive.

Therefore, the net VAT payable will be:

Output VAT:  £240 * 1/6 = £40



Input VAT: £12 * 1/6 = (£2)

VAT payable = £38

Examples:

• Stationery

• Furniture

• Computers

• Cars

• Petrol and diesel

• Accountancy fees

• Legal fees

• Advertising costs

• Confectionery

• Vans and lorries

• Sale of freehold commercial buildings within 3 years from completion

• Re-painting office premises

• Extensions to business premises


556
Zero rated supplies

A zero rated supply is taxable at 0%.

If a trader is VAT registered and makes zero rated purchases, no input VAT can be claimed,
and if they make zero rated sales, no output VAT is payable.

For example, a VAT registered trader sells baby clothes for £240.

This item is zero rated, and therefore no output VAT will be payable on the sale.

On the other hand, if a trader makes zero rated supplies and has standard rated purchases,
the trader will qualify for VAT repayments.

For example,  a VAT registered trader sells baby clothes for £240. He pays for his
advertising expenses, which cost him £120.

The trader makes zero rated supplies but standard rated purchases, and can therefore
claim the input VAT paid on his advertising expense of £20. (£120 * 1/6)

1. Basic food (not pet food or luxury items like alcohol and confectionary)

2. Sewerage services and water

3. New construction work or the sale of buildings by builders where the building is
going to be used for residential/charitable purposes

4. Drugs and medicines

5. Export of goods outside the UK.

6. Transport (but pleasure transport and transport in vehicles sitting less than 12
people is standard rated)

7. Clothing and footwear of children

8. Residential and charitable buildings

557
Exempt supplies

An exempt supply is not chargeable to VAT.

A person making exempt supplies cannot recover VAT on inputs, this is because someone
making solely exempt supplies will not have any taxable turnover and  cannot become VAT
registered.

1. Land

2. Insurance

3. Postal services

4. Financial services for example, bank charges or credit card services

5. Education

6. Health services

7. Burial and cremation services

8. Subscriptions to professional bodies

9. Sale of freehold commercial buildings owned for more than or equal to 3 years

558
Circumstances in which input VAT is non-deductible

Non-recoverable input VAT

Input VAT cannot be recovered in the following circumstances:

1. Input VAT cannot be recovered in respect of business entertainment of UK


customers

2. Input VAT cannot be recovered in relation to any items that are privately used by an
owner of a business.

For example, if an owner purchases stationery for private use, input VAT cannot be
recovered on this purchase.

3. Input VAT cannot be recovered on motor cars (unless they are used 100% for
business purposes).

This applies to both employees and owners of businesses.

A car must be used 100% for business purposes in order for input VAT to be
recovered on its purchase.

For example, an employer purchases a car for his employee.

The employee uses this car for both business and private purposes.

Input VAT cannot be recovered on the purchase price of this car.

Input VAT can be recovered in the following circumstances:

1. Input VAT can be recovered where fuel is used for private mileage (either by a sole
trader or an employee), but output VAT must be accounted for.

Output VAT is calculated according to a scale charge based on the car’s CO2
emissions.

The scale charge is VAT inclusive and will be provided to you in the exam.

2. Input VAT can fully be recovered in respect of repairs to a motor car, provided that
there is some business use.

For example, an employer purchases a car for his employee.

The employee uses this car for both business and private purposes.

Input VAT can be recovered on the repairs incurred in respect of this car.

3. Input VAT on business entertainment is recoverable if it relates to the cost of


entertaining overseas customers!

559
Illustration:

In the quarter to 31 March 2023, Shiva claimed all of the input VAT on his fuel cost (20%),
which he uses for private purposes.

The fuel cost was £1,200 (VAT inclusive).

The relevant scale charge for the car is £458.

How much VAT will be paid/reclaimed for the quarter ended 31 March 2023?

Solution

Input VAT claimed:



£1,200 * 1/6 = £200

Output VAT charged:



£458 * 1/6 = £76

Net VAT reclaimed:



£124

Illustration:

Lina Ltd. is registered for VAT. All of the sales are standard rated and all figures are inclusive
of VAT.

The following information relates to the company’s VAT return for the quarter ended
31/03/2023:

• Standard rated sales of £60,000.

• Standard rated purchases and expenses of £30,000.

• On 01/01/2023 Lina Ltd. purchased a motor car for an employee costing £12,000
VAT inclusive.

The car is used for both business and private purposes.

• Lina Ltd. paid for the petrol and repairs of the car.

The relevant quarterly scale charge is £310 for the quarter to 31/03/2023.

The scale charge is based on the C02 emissions of the car.

• The petrol and running cost of the car was £2,000 VAT inclusive.

• Calculate the VAT payable for the quarter ended 31/03/2023.

560
Solution:

• Input VAT that can be claimed in the quarter to 31/03/2023:

Standard rated exp. and purchases: 1/6 * £30,000 = £5,000



Purchase of motor car:                                                       Nil

Petrol and running cost paid for: 1/6 * £2,000 =           £333

Total input VAT that can be claimed:                          £5,333

• Output VAT payable in the quarter to 31/03/2023:

Sales:  1/6 * £60, 000:                                                    £10,000



Scale charge for petrol of car:  1/6 * £310 =                       £52

Total output VAT Payable:                                            £10,052

• Net output VAT Payable:                                                 £4,719

561
Relief available for impairment losses on trade debts

Recoverability of VAT on impairment losses (bad debts)

Normally, output VAT is accounted for when an invoice is issued.

If the sale becomes an impairment loss, the seller has paid VAT to HMRC and has not been
able to recover this from the customer.

It is possible for the supplier to reclaim this VAT on the impairment loss from HMRC
provided the following conditions are met:

1. The loss has been written off in the accounting records (the income statement)

2. 6 months has passed since the debt has been due.

If these conditions are met, then the seller can include the amount of VAT as input VAT on
the next VAT return filed and therefore get relief for it.

Illustration:

Sahil Ltd. made a sale on 31/08/2022 for £120 (VAT inclusive).



 

The buyer was given a 30-day credit period.

On 31/03/23, the debt was not paid and written off in Sahil Ltd.’s accounting records.

• Will the output VAT paid be refunded?

Solution:

• Sale £100

Output VAT £20

Selling price £120

• This £20 output was paid and included in the VAT returned filed on 30/09/2022 (VAT
returns are filed quarterly).

• The £120 was due on 30/09/2022, however it has not been paid by 31/03/23.

Therefore 6 months have passed since the debt was due to be paid and it has been
written off in Sahil Ltd.’s accounting records.

• Therefore, Sahil Ltd. will get relief for this output VAT of £20 by including it in the VAT
return filed on 31/03/23 as input VAT.

562
Default surcharge and penalty for incorrect VAT return

The default surcharge

If a taxable person:

- submits a VAT return late, or 

- submits a return on time but makes late payment of the VAT due then:

• HMRC will issue a surcharge liability notice which will specify the surcharge period
(normally 12 months).

• If within these 12 months, you don't make another default:

You will NOT pay any surcharge.

• If within these 12 months, you make another default:

1) You will have to pay a surcharge which is calculated as a % of the tax paid late and

2) The surcharge notice period will be extended for another 12 months.

Default involving late payment of VAT in the Surcharge as a % of the VAT


surcharge period  outstanding at due date

1st default 2%

2nd default 5%

3rd default 10%

4th default 15%

For the first and second default, if the surcharge due is less than £400, then nothing is
payable.

For the third and fourth default, there is a minimum of £30 payable, even if the surcharge
amount is below £30 for the quarter.


563
Illustration:

Tommy has submitted his VAT returns as follows:

Quarter ended Vat paid (£) Date submitted

31/03/22  1,200 15/05/2022

30/06/22 1,000 07/08/2022

30/09/22 3,100   05/12/2022

31/12/22 1,300 02/03/2023

VAT returns late?

What are the consequences of Tommy submitting his VAT returns late?

Solution:

• 31/03/2022

Submitted late.

Surcharge period notice issued of 12 months, ending on 31/03/23.

No default surcharge levied on initial late submission.

• 30/06/2022

Submitted and paid on time.

Still within the surcharge period, ending on 31/03/2023.

• 30/09/2022

Submitted late.

This is the first default within the surcharge period, therefore a default surcharge of
£3,100 * 2% = £62 is levied.

This is not payable by Tommy because it is less than £400.

However, the surcharge notice period will now extend to 30/09/2023.

• 31/12/2022

Submitted late.

The surcharge of £1,300 * 5% = £65 is again less than £400, therefore this will not
be payable by Tommy.

However, the surcharge notice period will now extend to 31/12/2023.

564
Errors on a VAT return

If a VAT return is submitted incorrectly, the following penalties and surcharges will apply.

Disclosed by the taxpayer

Errors disclosed by a taxpayer are either defined as small or large.

If an error occurs, then default interest and a standard penalty may be payable.

These depend on whether an error is defined as small or large.

Default interest is interest based on the delayed payment of the VAT liability.

A standard penalty may be payable depending upon the reason for the late submission.

The difference between a small and a large error are:

A small error is defined by being less than the de-minimus limit.

The de-minimus limit is the greater of:

£10,000 and

1% of turnover (subject to an upper limit of £50,000)

Consequences Small Error Large Error


Default interest
No Yes
payable?
How to disclose? On the next VAT return Separately disclose to HMRC
May be payable depending in May be payable depending on
Standard penalty?
reason of error reason of error

565
Illustration:

Bebe Ltd. has made an error relating to understated output VAT of £7,000 for the quarter to
31/12/2022.

The company’s turnover for the quarter is £200,000.

How should this error be disclosed to HMRC?

• Solution:

De-minimus limit:

The greater of:



1) £10,000

2) 1% * £200,000 = £2,000

The error is small because it is less than the de-minimus limit.

Therefore, it can be disclosed on the next VAT return.

No interest will be payable and depending on the reason for the understatement, a
standard penalty will be decided.

Disclosed by HMRC

- Default interest due regardless of size of error.



- Standard penalty.

• Standard penalty is based on the reason of inaccuracy

Genuine mistake – no penalty

Careless mistake – 30% of VAT due

Deliberate mistake – 70% of VAT due

Concealment – 100% of VAT due

566
Treatment of imports to the UK, exports from the UK

Exporting outside of the UK

• Goods/services are treated as zero rated.

• No output VAT will be added to them.

Illustration

A UK VAT registered trader supplies computers costing £10,000 (VAT exclusive) to a


company outside the UK.

The supply will be treated as zero rated and therefore no output VAT will be charged.

Whether the company outside of the UK is VAT registered or not does not matter, the
supply will be treated as though it is zero rated.

567
Coming into the UK:

• VAT must be accounted for on acquisition.

• The VAT charge is declared on the return as output VAT but can be reclaimed as
input VAT on the same VAT return.

The UK trader accounts for output VAT as the goods would be standard rated if
supplied in the UK.

The UK trader can claim back input VAT of the same amount on the same return as
the goods are used by a trader that only makes taxable supplied.

The net effect on VAT payable is therefore nil.

• The entries contra each other, therefore there is no actual VAT cost.

• The only time that there is a VAT cost is if a business makes exempt supplies, since
an exempt business cannot reclaim any input VAT.

Illustration

A UK company purchased computers costing £10,000 (VAT exclusive) from a company in


the U.S.A.

The VAT of £2,000 will be accounted for but not paid.

On receipt, the UK company will account for the £2,000 on its VAT return and can claim the
£2,000 input VAT on the same VAT return.

568
Syllabus A6a. TX - UK Recap: The effect of special
schemes

The contents of the Paper TX - UK study guide for value added tax (VAT), under headings: 

- The effect of special schemes

Operation of and advantages of VAT special schemes

Special schemes

These schemes are available to small businesses to reduce the work and amount of VAT
payable.

There are 3 schemes:

1. Cash accounting scheme

2. Annual accounting scheme

3. Flat rate scheme

569
1) Cash accounting scheme

Operation:

1. The tax point is the date on which the output tax is received and the input tax is
paid.

2. For sales, it is the date that cash is received from customers and for purchases, it is
the date that cash is paid to suppliers.

Conditions:

1. Annual taxable turnover must not exceed £1,350,000.

2. VAT returns must be kept up to date.

3. A business must leave the scheme if annual taxable supplies exceeds £1,600,000.

Advantages:

1. A business will not pay output tax until received from customers. This is a cash flow
advantage for the business.

2. The scheme provides automatic bad debt relief as output VAT will not have been
paid on a sale until the cash is received from the customer.

Illustration

Shivani has an annual turnover of £1,200,000.

All sales are standard rated and are made on credit for 60 days.

All purchases are standard rated are made on credit for 30 days.

If Shivani opts into the cash accounting scheme, when will she need to account for VAT for
her sales and purchases?

• Solution

Shivani will need to account for her standard rated sales 60 days after the sale is
made, as this is when the cash is received.

This will ensure that Shivani does not pay any output VAT for bad debts.

She will need to account for her standard rated purchases 30 days after the
purchase is made, as this is when the cash is paid.


570
2) Annual accounting scheme

Operation:

1. One VAT return is prepared each year.

2. The VAT return is due 2 months after the annual accounting VAT period, along with
the balancing payment of VAT.

3. 9 payments of VAT are made from months 4-12 during the annual accounting
period. These are (10% * VAT paid last period).

4. The final payment (2 months after the annual accounting period) is calculated as
follows:

 

(VAT payable for the year – 9 payments made during months 4-12 during the period)
= balancing payment.

Conditions:

1. Annual taxable turnover must not exceed £1,350,000.

2. VAT returns must be kept up to date.

3. A business must leave the scheme if annual taxable supplies exceeds £1,600,000.

Advantages:

1. Administration costs are saved, only one VAT return is prepared per year.

2. Regular monthly payments help the cash flow of the business, small regular
payments are made as opposed to less frequent large outflows.

3. It simplifies accounting for VAT.

Illustration:

Terry for the year ended 31/12/2021 paid VAT of £10,000.

• She was eligible to enter the annual accounting scheme and for the year ended
31/12/2022 she had VAT payable of £12,000.

• What were her payments during the year ended 31/12/2022?



When did she make these payments?

What was her balancing payment?

When did she make this payment?

571
Solution:

Payments on account:

• (10% * last year’s VAT paid)



= (10% * £10,000) = £1,000 each month

• Payments on account were made from months 4-12 during the year ended
31/12/2022. Therefore, they were made from April to December. This total 9
payments on account.

• Balancing payment:

• (VAT payable for the year – payments on account)



= £12,000 – (£1,000 * 9) 

= £3,000

• This payment is made 2 months after the year has ended. Therefore, it is made on
28/02/2023.

3) Flat rate scheme

Operation:

1. VAT payable is computed by using a flat rate %. The flat rate % differs from industry
to industry (you will be told the % in the exam)

2. This % is multiplied by the sales revenue.

3. The sales revenue used includes VAT, exempt supplies and zero rated supplies.

Note: from 6/4/17 there is a standard % of 16.5% for limited cost traders. This means that,
regardless of their industry, if they are deemed to be limited cost traders, then they must
use the rate of 16.5% (you will be told if this rate applies in the exam).

Conditions:

1. Annual taxable sales must not exceed £150,000.

2. A business must leave the scheme once turnover exceeds £230,000.

Advantages:

1. Simplicity of scheme

2. Reduces administration costs

3. Less detailed records of input and output VAT are needed.

572
Illustration:

Addi Ltd. has annual sales of £84,000.

These are standard rated and inclusive of VAT.

The company also has standard rated expenses of £4,800 (VAT inclusive).

The flat rate is 16.5%.

• Is it beneficial for the company to use the flat rate scheme or account for VAT
normally?

Solution:

Flat rate scheme:

• 16.5% * £84,000 = £13,860 payable

• Normal accounting:

• Output VAT: 20/120 * £84,000 = £14,000



Input VAT: 20/120 * £4,800 = (£800)

Net VAT payable = £13,200

• It is not beneficial for Addi Ltd. to opt into the flat rate scheme.

573
Syllabus A6ai)
Advise on the VAT implications of the supply of land and buildings in the UK

Land and Buildings

Buildings may be zero or standard rated

• Residential and charitable buildings are zero rated.

• Commercial buildings owned for less than 3 years are standard rated.

This means that on sale proceeds and rental income, output VAT will be charged at
20%. 

Input VAT on expenses can be reclaimed.

• Commercial buildings owned for more than 3 years are exempt but have an
option to tax.

This means that they are normally out of the scope of VAT, therefore no output VAT is
charged and no input VAT is reclaimed.

However, if the option to tax is used, then on sale proceeds and rental income, output
VAT will be charged at 20%, and input VAT on expenses can be reclaimed.

Purchasers may choose to opt to tax so that they can recover the input VAT paid on the
purchase of a property that has an opt to tax already on it. By doing this though, they
are promising to charge output VAT on all transactions relating to that property including
any future sale.

574
Illustration

A commercial building owned for 4 years will be sold for £1,000,000 (excl.)

Expenses relating to the sale amount to £100,000 (excl.) and its purchase cost is
£500,000 (excl.)

Should the opt to tax election be made?

Solution

Normal treatment

Sale proceeds £1,000,000



Less expenses (£100,000)

Less cost (£500,000)

Profit £400,000

Special treatment/opt to tax

Output VAT (20%*£1,000,000) = £200,000



Less Input VAT (20%*£100,000) (£20,000)

Less Input VAT (20%*£500,000) (£100,000)

Net VAT payable £80,000

Therefore, the election should not be made as it will result in a VAT payment.


575
Syllabus A6aii) Advise on the VAT implications of partial exemption

VAT implications of partial exemption

Partially exempt business

are businesses that sell taxable and exempt supplies and may be registered for VAT
compulsorily (if taxable turnover exceeds £85,000) or voluntarily.

Input tax is generally not recoverable on exempt supplies, however, if the input tax
relating to exempt supplies is less than the de-minimus limits then it becomes
recoverable.

In order to see if this is the case, there are two simplified tests to work through. If you pass test one
then you do not need to work through test two. If you fail both tests then you will need to use a
formula to calculate how much input tax is recoverable.

Partially exempt businesses

The simplified tests

Test 1:

Is total input tax less than £625 per month? and

Is exempt turnover less than 50% of total turnover?

If the answer to this test is YES then 100% of input tax can be recovered.

Test 2:

Is total input tax less input tax directly attributable to taxable supplies less than £625
per month? And

Is exempt turnover less than 50% of total turnover?

If the answer to this test is YES then 100% of input tax can be recovered.

If neither of these tests are passed then you can apply a formula to non-attributable
input tax to discover how much, if any, is recoverable.

VAT registered traders can reclaim input tax that relates to its taxable supplies
proportion, but not input VAT that relates to it’s exempt supplies, however the issue
arises when there is non-attributable input VAT, this is the input VAT that relates to
overheads. 

576
The input VAT that can be reclaimed for non-attributable input VAT must be in
proportion to the taxable supplies proportion.

Calculation of the proportion that can be reclaimed for non attributable


input VAT:

Non attributable input VAT x Taxable Turnover /Total Turnover 

For example, if a business has £70,000 taxable supplies and £30,000 exempt supplies
for the year, and it has non attributable input VAT of £9,000 – then it can reclaim
70%*£9,000 = £6,300.

If the remaining amount of £2,700 (£9,000 - £6,300) is less than £625 per month and
less than 50% of the input VAT relates to exempt supplies then this amount is also
recoverable.

£2,700/12 = £225 per month. Therefore this is recoverable too.

When calculating the amount of recoverable input tax for a quarter, the trader can now
choose to use the percentage for the previous year rather than the partial exemption
percentage for the current particular quarter.

The trader must use the same method for the whole year.

For example, if the taxable supplies percentage for 21/22 was 60%, the trader can use
this percentage to calculate the input VAT recoverable for 22/23 and then make a final
adjustment in the final quarter of 22/23 with the current year percentage.

The method used will not make any difference to the total amount of VAT recovered as
the annual adjustment will ensure that the final amount recovered is in accordance with
the figures for the whole year.

The business can now choose to enter the annual adjustment on the return for the final
period of the year rather than the first period following the end of the year.

As seen earlier, if the turnover attributable to exempt supplies is small then by


concession it can all be reclaimed.

Small is:

577
• The irrecoverable input tax must be no more than £625 per month and

• The irrecoverable input VAT must be less than 50% of the total input tax

Illustration

Jake Ltd. has the following proportion of taxable supplies:exempt supplies.

Taxable supplies 86%

Exempt supplies 14%

Input VAT

Wholly attributed to taxable sales £7,000

Wholly attributed to exempt sales £1,000

Unattributed Input VAT £3,000

Can the Input VAT attributed to exempt sales and the unattributed Input VAT be
recovered?

Solution

Yes it can if it is deemed to be small

Total Input VAT £11,000

Input VAT wholly attributed to exempt sales £1,000

Unattributed Input VAT for exempt sales (14%*£3,000) £420

Total £1,420

This will be deemed to be small as it is less than £625/month and it is less than 50% of
total Input VAT.

578
Syllabus A6aiii)
Advise on the application of the capital goods scheme

Capital goods scheme

This is a scheme that applies to partially exempt businesses (businesses with taxable
and exempt turnover) that spend large amounts of money on land and buildings or
computers and computer equipment.

Where the scheme applies, the first deduction of input tax is made in the normal way for
a partially exempt business (% of taxable supplies * input vat)  and then the input VAT
that was deducted initially is reviewed over a set adjustment period. 

The review is based on the proportion of taxable to exempt turnover over a set number
of years. 

Therefore, this scheme is to stop businesses manipulating their proportion of taxable


and exempt supplies in the period of purchase in order to reclaim more input VAT. 

For example, if a business purchased a piece of land that had input VAT of £100,000
and in the year of purchase they had 80% taxable supplies and 20% exempt supplies,
then they can recover 80% * £100,000 = £80,000 input VAT. 

However, in the following years if their taxable supplies were 50% and exempt supplies
were 50% - then it looks like they have recovered an additional amount of input VAT by
increasing their taxable supplies in the year of purchase.

Therefore, in the above example:

£100,000/10 years * (50%-80%) = £3,000 will need to be repaid each year for 10 years

579
Adjustments are made over the next 10/5 years if the proportion of exempt supplies
changes.

The annual adjustment calculation is:


Total input VAT/10 or 5 years x (% now - % in the year of acquisition)

Illustration 1

A partially exempt company purchases a computer for £100,000 plus VAT on 1 January
2021. 

In the first year to 31 March 2021, the company has 60% taxable supplies which fall to
50% in the second year and increase to 80% in the third, which will then remain the
same for future years.

The company sold the computer for £6,000 plus VAT on 10 August 2023.

Required:

Calculate the amount of input VAT recoverable in the year of purchase and the annual
adjustment required under the capital goods scheme for years 2 and 3.

Calculate the adjustment needed as a result of the sale of the computer on 10 August
2023.

Solution

• The input tax recoverable in the year ended 31 March 2021: 



Cost of computer 100,000 

Input tax = 20,000 

Recoverable input tax (20,000 × 60%) = 12,000

• Annual adjustment y/e 31 March 2022: 20,000/5 × (50 – 60) 



Pay VAT to HMRC 400

• Annual adjustment year ended 31 March 2023: 



20,000/5 × (80 – 60) 

Reclaim VAT from HMRC 800

• Annual adjustment year ended 31 March 2024 



Year of sale 

20,000/5 × (80 – 60) 

Reclaim VAT from HMRC 800

• On sale assume 100% taxable for remaining years: 

20,000/5 × (100 – 60) × 1 year = 1,600

Limited to a maximum of 1,200 

VAT charged on sale (6,000 × 20%) =1,200


580
Transfer of a going concern

No VAT is charged where:

A business is transferred /sold as a going concern

Provided the transferee business is already VAT‐registered or will become registered


immediately after the transfer and so they will be held responsible to charge output VAT
on the subsequent sale of goods

There should be no significant gap in the start of other business

Nature of the trade would not change

Illustration

Martin is registered for VAT but intends to cease trading on 31 March 2023. 

On the cessation of trading Martin can sell his entire business as a going concern to a
single purchaser.

Will output VAT be charged on the sale?

Solution

This is a sale of business as a going concern, therefore output VAT will not be charged if
the following conditions are met:

A business is transferred /sold as a going concern

Provided the transferee business is already VAT‐registered or will become registered


immediately after the transfer and so they will be held responsible to charge output VAT
on the subsequent sale of goods

There should be no significant gap in the start of other business

Nature of the trade would not change


581
Syllabus A6b. TX - UK Recap: The overall function and
purpose of taxation.

The contents of the TX - UK study guide for the UK tax system and its administration under
headings:

- The overall function and purpose of taxation in a modern economy

The Economic Purpose

Tax can be used by governments to affect:

1. Inflation

Higher tax levels should decrease inflation

2. Employment

Higher tax levels should increase employment (if spent by governments in the right
way!)

Tax can also be used by governments to affect businesses and


individuals:

• It encourages saving by offering ISAs (individual savings accounts) and tax relief for
contributions to pensions

It encourages charitable donations by offering Gift Aid tax relief

It encourages new businesses by offering investment tax relief

• It discourages motoring (causing air pollution) by fuel duties

It discourages drinking alcohol/smoking by imposing high duties etc

582
Social Justice Purpose

This can be done through Progressive taxation:

This simply means the higher earners pay a higher % of their income as taxes - thus
redistributing the wealth in society from the rich to the poor

So, income tax is an example of a progressive tax

The opposite of this is Regressive taxation where higher earners pay a lower % of their
earnings as tax

Ad Valorem Principle

Here everyone pays the same % of tax regardless of income

- e.g., VAT on a computer is the same regardless of your income

The final form of tax is Proportional Taxation

This is where the tax levels (% of earnings) remain the same regardless of income levels

583
Direct Taxes - tax paid directly to HMRC

DIRECT taxes are paid DIRECTLY to HMRC

Direct Revenue Tax

These are based on income / profits

Basically the more income or profit, the more tax you pay..

1. Example 1: Income Tax paid on different types of income

2. Example 2: Corporation Tax paid on company profits

3. Example 3: National Insurance Contributions paid on employment income / trading


income of the self employed

Direct Capital Tax

These are based on assets sold or gifted

Basically the more the asset is sold for (or the higher the gift), the more tax you pay..

1. Example 1: Capital Gains Tax

2. Example 2: Inheritance Tax

Indirect Taxes - taxes paid to HMRC indirectly through an intermediary

Basically think of this as a shop for example - you buy an item with tax on it, but you pay
the shop. The shop then pays this tax to the HMRC.

Example: VAT


584
Syllabus A6b. TX - UK Recap: Principal sources of revenue
law and practice

The contents of the TX - UK study guide for the UK tax system and its administration under
headings:

- Principal sources of revenue law and practice

HMRC

Her Majesty's Revenue and Customs

This controls all aspects of tax law in the UK

Purpose of HMRC

1. Make sure there is money available to fund public spending

2. Help families and individuals who need financial support

HMRC Main Duties:

1. Implement the tax laws

2. Oversee the tax admin

Miscellaneous HMRC stuff

Staff are known as "Officers of Revenue and Customs"

Branches are all over UK

Most taxpayers never deal with HMRC direct - instead they file their tax returns online and
pay electronically (compulsory for companies)

The responsibility for assessing how much tax is payable is down to the taxpayer under a
system called "self - assessment"

Individuals can still ask HMRC to calculate the tax though for them - companies can't


585
Structure of the UK tax system

HM Revenue and Customs (HMRC)

The treasury formally imposes and collects taxation.

The management of the treasury is the responsibility of the Chancellor of the Exchequer.

The administration function for the collection of tax is undertaken by HMRC

Commissioners

At the head of HMRC are the commissioners whose duties are:

1. To implement statue law

2. Oversee the process of UK tax administration

The main body of HMRC is divided into District offices and Accounting and payment offices

District Offices

The Commissioner appoints Officers of HMRC to implement the day to day work of HMRC

Accounts and payment offices

These concentrate on the collection and payment of tax.

586
Principal sources
Tax Legislation (Statute law)

Law so you HAVE to follow them

1. Updated annually by the Finance Act (made by the chancellor of the exchequer) -
referred to as "The Budget"

2. Statutory instruments - these give detailed guidance where necessary on points of law

Case Law

These are decisions made previously by judges in court about taxation matters

1. The "case" will help decisions be made in similar circumstances

2. The "case" rulings are binding

HMRC Guidance

These explain the laws and help interpret the law

The main types are:

1. Statements of Practice Provide clarification of how rules should be applied

2. Extra statutory concessions

These allow laws to be relaxed where their implementation would cause undue hardship

3. Internal HMRC Manuals Give guidance for their staff but are also available to the public

4. Briefs

These just provide details of a specific tax issue that's arisen

5. HMRC website, leaflets etc

Use non-technical language aimed at the general public


587
Tax EVASION is ILLEGAL

The main forms of tax evasion are:

Not giving all information

Giving false information

Tax AVOIDANCE is LEGAL

It's arranging your income to minimise tax - although HMRC are introducing anti-avoidance
legislation to lower the advantages to the taxpayer

Tax avoidance features..

No misleading information given

Can use loopholes in the tax system

The General Anti-Abuse Rule (GAAR)

This fights artificial and abusive schemes (unreasonable courses of action) which are used
to avoid paying tax

588
This is where income could get taxed under 2 different
systems

Double Taxation Agreements

These are agreements between countries over how certain items are taxed - and take
precedence over UK law

They either:

Exempt some overseas income from UK tax or

Provide tax relief if it has been taxed in 2 countries

EU Influences

The EU would like to remove differences between countries policies as these can cause
distortions and be barriers to trade for some countries

EU countries do NOT have to align their tax policies - but can jointly enact laws called
Directives

One example is the VAT directive

However the rates of VAT are not aligned...

589
Advising on tax? You have duties to the HMRC too!

Information given to HMRC must be complete and accurate

Examples:

Not declaring taxable income

Claiming un-entitled reliefs

Not notifying HMRC of their mistakes re under payment of tax etc

If the client commits an offence - you need to decide if it was an error or fraud

You then explain to the client that they should disclose the error to the HMRC

If the client won't disclose still - then you must stop representing the client and disclose
matters to HMRC if it's in the public interest or you think there might be money laundering

Professional and ethical guidance

Accountants often act for taxpayers in dealings with HMRC.

Their duties and responsibilities should be towards both clients and HMRC

The accountant must uphold standards of the ACCA that is

1. To adopt an ethical approach to work, employers and clients

2. Acknowledge the professional duty to society as a whole

3. Maintain an objective outlook

4. Provide professional high standards of service, conduct and performance at all times.

The ACCA “Code of Ethics and Conduct”

The ACCA “Code of Ethics and Conduct” sets out five fundamental principles which
members should adhere to meet these expectations, namely:

1. Integrity

2. Objectivity

3. Professional competence and due care

4. Confidentiality

5. Professional behaviour

590
Syllabus A6b. TX - UK Recap: The systems for self-
assessment and the making of returns

The contents of the TX - UK study guide for the UK tax system and its administration under
headings:

- The systems for self assessment and the making of returns

Self assessment is for income not collected at source*

*An example of tax collected at source is employment income (PAYE)

How Self-Assessment works..

Remember it is the responsibility of the taxpayer to calculate their own tax liability

1. Taxpayer is sent a notice to complete the SA (online or on paper)

2. There are deadlines for filing the SA (31/10 for paper and 31/1 online) or 3 months after
being given notice if its later

3. It covers income tax, class 2 NIC, class 4 NIC and CGT

4. Payment is due 31st January the following year (interim payments on account may also
be required)

5. 31st January the following year is known as the filing date (for both paper and online) -
different to the "actual file date"

6. Online filing - Tax returns submitted electronically automatically calculate the tax due

7. Paper filing - HMRC will (optionally) calculate it if filed by 31st October. If a taxpayer
submits a paper return on time, they can ask HRMC to calculate the tax due.

8. When the HRMC calculate the tax they make no judgement on the accuracy of the
information given to them

9. HMRC then deliver a statement of account to the taxpayer as a reminder of amounts


due

591
Companies must submit a corporation tax return within
12m of their period end

They must pay the tax within 9m + 1day of their chargeable period end (CAP) but they do
not have to file their return until 12 months after the CAP end.

Therefore many submit their corporation tax return before the 9m + 1day payment deadline
too

Features of self assessment for companies

Must be done online

Financial statements are submitted along with it - using iXBRL

Inline eXtensible Business Reporting Language (iXBRL)

iXBRL allows for the exchanging of business information electronically and tags the
accounts so they can be read by a computer

HMRC then uses online software to read the iXBRL info

Other Options

1. Integrated Software Applications

These automatically insert the iXBRL tags and produces accounts/computations in the
iXBRL format

2. Managed Tagging Services

Company outsources this tagging process

3. Conversion Software

Applies tags and so converts the info into iXBRL


592
Syllabus A6b. TX - UK Recap: The Time Limits

The contents of the TX - UK study guide for the UK tax system and its administration under
headings:

- The time limits for the submission of information, claims and payment of tax, including
payments on account

Either HMRC or Taxpayer may amend the return

HMRC

May amend any obvious errors (e.g. adding up errors) within 9 months of the date of filing

Taxpayer

May amend within 12m of the January filing date

Notification of chargeability

The onus is on the taxpayer to inform HMRC that they are liable to tax - must be done
within 6 months of the first tax year - unless there's no actual tax liability

If no notification is made then penalties will occur

593
Generally the next 31st January

This is the date for income tax, NIC class 2 & 4 and CGT

Payments on Account (POA's)

These are made if less than 80% of last years tax liability was deducted at source (unless
less than £1,000)

Therefore employees don't need to make POAs as more than 80% of their tax liability is
deducted at source

They only apply to income tax and class 4 NIC - NOT FOR CGT

DATES for POAs

1. 1st = 31st January

2. 2nd = 31st July

3. Balance on 31st January next year (normal due date)

CALCULATION of POAs

Based on last years "relevant amount" (half payable on each POA)

Relevant amount = tax due - amount deducted at source

Example

Jack’s tax bill was £10,200 - of which 2,500 was collected using PAYE

What is the amount of the POA’s?

Answer

POA 1 £3,850 (10,200 - 2,500 = 7,700/2)

POA 2 £3,850


594
Claims to reduce POAs - anytime before next 31
January

The taxpayer would do this if he expects the taxable income to be lower this year than last
(as last years was used to calculate the POA)

The grounds for the claim must be made

If the actual taxable income ends up being higher then interest will be charged on the
underpaid POA tax, and a penalty if the reduction in POA was fraudulent (and not an
innocent error)

Payment of Tax

1. Companies pay tax by self assessment

2. Estimated tax is payable 9 months and one day after the end of each accounting period
(due date), with provisions for quarterly instalment payments for ‘large’ companies.
Payment must be made electronically

3. Interest due to the HMRC on tax paid late will run from the due date to the date of
payment at a rate of 2.6% per annum.

4. Interest on overpayments of tax will run from the later of the due date or the date tax
was actually paid at a rate of 0.5% per annum.

Under self assessment interest on tax paid late will be deductible against interest
receivable.

Interest received on overpaid corporation tax will be taxable as Interest receivable

Quarterly Instalments

1. Quarterly instalments apply to large companies.

2. A large company is one with a profit exceeding £1.5M based on a single company with
no related 51% group companies and which prepares accounts for a 12 month period.

The profit limit must be divided by the number of 51% related group companies and
time apportioned for a chargeable accounting period of less than 12 months.

3. The instalments are based on the estimated current year’s liability.

4. The four quarterly instalments will be made in months 7, 10, 13 and 16 following the
start of the accounting period.

The instalments are due on the 14th of the month.


595
Thus for the accounting year ended 31 March 2023 the first quarterly instalment
payment would be due October 14 2022 followed by further payments due January 14
2023, April 14 2023 and July 14, 2023

5. Quarterly payments are not required if the company was not large in previous CAP.

596
How long should business records be kept?

Business records and personal records need to be retained for a number of years after the
tax returns have been submitted for that year.

The time for which these records need to be retained are:

Retention of records

Company’s records 6 years from the end of the accounting period

Self employed -
business and non 5 years from 31January following end of the tax year
business records

Employed 12 months from 31 January following end of the tax year

A failure to retain records could result in a penalty of up to £3,000 per accounting period.

However the maximum penalty will only be charged in serious cases. 


597
Syllabus A6b. TX - UK Recap: Compliance checks, appeals
and disputes

The contents of the TX - UK study guide for the UK tax system and its administration under
headings:

- The procedures relating to compliance checks, appeals and disputes

Appeals

Tax appeals are heard by the Tax Tribunal which is made of:

• First Tier Tribunal



Deals with most cases.

• Upper Tribunal

Deals with complex cases.

What is a tax assessment?

Opening up a tax assessment means that either the taxpayer or HMRC thinks the taxpayer
has paid too much or too little tax and wants this to be corrected.

The following table shows when a taxpayer or HMRC can open up a tax assessment.

HMRC can open a normal enquiry for a tax return, 12 months from the date of submission.

However, if HMRC suspects something more serious, they can raise a discovery
assessment.

The time limit for raising this discovery assessment depends on the reason of suspicion of
HMRC (mentioned below).

598
After HMRC raises a discovery assessment, taxpayers can raise an appeal within 30 days.

Taxpayers can make an amendment to his


12months of the January filing date
tax return

Taxpayers claim for overpayment relief 4 years from the end of the tax year

12 months from submission of the return - if


return was filed on or before the due date.

If the return was filed after the due date, the


enquiry can be opened on the anniversary of
HMRC can open an enquiry
the quarter date following the actual
submission of the return.

The quarter dates are 31 January, 30 April, 31


July and 31 October.

HMRC can raise a discovery



assessment

— No careless or deliberate behaviour 4 years from the end of the tax year

— Tax lost due to careless behaviour 6 years from the end of the tax year

— Tax lost due to deliberate behaviour 20 years from the end of the tax year

Taxpayers right of appeal 
 30 days from the assessment — appeal in


against an assessment writing

599
Syllabus A6b. TX - UK Recap: Penalties for non-
compliance

The contents of the TX - UK study guide for the UK tax system and its administration under
headings:

- Penalties for non- compliance

You have to pay an INTEREST and a PENALTY

• Late payment interest



If a tax liability is paid late, then late payment interest will be charged.

• Late payment Penalty for balancing payments



If a balancing payment is paid late, then a penalty will be charged.

• Penalties for incorrect returns



If a tax liability is understated, then a penalty might be charged.

1) Late payment interest

The rate of interest is 2.6%.

If the liability is outstanding for less than a full year, then this will be apportioned
accordingly. 

For example 

A corporation tax liability of £300,000 was outstanding for 4 months.

How much late payment interest will be payable assuming a rate of 2.6%?

4/12 * 2.6% * £300,000 = £2,600

600
2) Late payment Penalty for balancing payments

5% for tax unpaid 30 days after payment due date,

further 5% if still unpaid after 6 months and

a further 5% if still unpaid after 12 months.

3) Penalties for incorrect returns

The amount of penalty is based on the amount of tax understated.

But the actual penalty payable is linked to the taxpayer’s behaviour:

• NO PENALTY where a taxpayer simply makes a genuine mistake.

• UP TO 30% of the understated tax where a tax payer fails to take reasonable care.

• UP TO 70% of the understated tax if error is deliberate.

• UP TO  100% of the understated tax where the error is deliberate and concealed.

A penalty will be substantially reduced where the taxpayer makes disclosure, especially
unprompted disclosure to HMRC.

601
Syllabus A6bi. Offshore Matters

Syllabus A6bi)
Advise on the increased penalties which apply in relation to offshore matters.

Tax avoidance and Tax evasion

Abusive tax arrangements

A penalty has been introduced to the general anti-abuse rule (GAAR). 

It applies where the GAAR has been used to counteract tax advantages arising from
abusive tax arrangements entered into by the taxpayer. 

The penalty is 60% of the amount of the tax advantage counteracted by the GAAR.

Examples of abusive arrangements are ones that result in less income, profits or gains,
greater deductions or losses, or a claim for a repayment of tax that is unlikely to be
paid.

Examples of a tax advantage are: relief or increased relief from tax, repayment or
increased repayment of tax, avoidance of possible assessment to tax, avoidance or
reduction of charge to tax, deferral of a payment or advancement of a repayment and
avoidance of an obligation to deduct or account for tax.

HMRC may counteract these tax advantages by increasing the amount of tax due from
the taxpayer but these adjustments must be made on a ‘just and reasonable’ basis.

Offshore Matters

Finance Act 2015 increased the penalties for failure to notify chargeability to tax, late
filing and errors in respect of offshore matters. 

The level of the penalty depends on the categorisation of the overseas country
concerned (as determined by the Treasury) and the behaviour involved.

Finance Act 2016 has increased the minimum penalty for these offences, and
introduced further penalties for both the taxpayer and for those who have enabled the
offence to be carried out.

You are expected to know that these regimes exist but do not need to know the precise
amounts of the penalties that may be charged or the categorisation of particular
countries.


602
Syllabus B: Financial Decisions made by a
business

Syllabus B2. Alternative ways of achieving outcomes

a) Identify and understand that the alternative ways of achieving personal or business
outcomes may lead to different tax consequences
b) Calculate the receipts from a transaction, net of tax and compare the results of alternative
scenarios and advise on the most tax efficient course of action.

In the exam

• You will be asked to compare alternative ways of achieving personal or business


outcome and

• advise on the most tax efficient decision


The best advice here is to go and practise some past paper questions so that you are
familiar with the exam style

603
Syllabus B3. Different types of finance and investment

Advise how taxation can affect the financial decisions made by businesses (corporate and
unincorporated) and by individuals

Debt or equity?

Ways of financing a business

Raising Capital

If a company needs to raise capital to buy plant and machinery, increase working capital
or buy an investment property it has two choices: it can either finance using debt or
equity.

The examiner is going to use international accounting standard terminology, which


means that previous terminology would refer to debt as debentures or corporate bonds
but under the international accounting standard terminology the examiner will refer to
debt as loan notes.

Debt

This is the company issues loan notes.

The company must pay interest on the loan notes.

604
Tax implications

1. The cost of issuing the loan notes such as legal costs and advertising costs are an
allowable expense.

2. The interest payable on the loan notes is also an allowable expense.

3. These costs will reduce taxable trading profits if the loan is for trading purposes.

4. These costs will reduce interest receivable if the loan is for non- trading.


Equity

This means that the company is issuing shares. 

The company will make a distribution (pays a dividend to the shareholders).

Tax implications

The cost of issuing the shares is a disallowable expense.


The dividend paid to shareholders is also a disallowable expense (no effect on the tax
paid by the company).


Other ways of raising finance

Lease

If an asset is leased, then a premium will be paid and rentals each year will be paid.

The income element of the lease/no. of years of the lease, and the rentals are allowable
expenses.


Hire purchase

If an asset is hire purchased, the amount paid is an allowable expense, except if a car
that has emissions of >=50g is hire purchased, then 15% will be disallowable.


Purchase outright

If an asset is purchased outright, then it will receive capital allowances.


605
Syllabus C. Ethics

Syllabus C5/6. Ethics

Be aware of the ethical and professional issues arising from the giving of tax planning advice
and
Be aware of and give advice on current issues in taxation.

Ethical behaviour

Professional code of ethics

As a professional tax adviser or accountant it is absolutely essential that he conducts


his affairs at all times following the professional code of ethics.

Professional code of ethics

1. Objectivity – Avoid conflicts of interest; do not act for two companies one buying
shares and the other selling shares if objectivity will be compromised.


2. Professional – Professional behaviour at all times, comply with relevant laws and
avoid any action that discredits the profession.


3. Technical competence – Keep up to date with new tax rules and legislation.


4. Integrity – Must be honest and should not assist clients in committing an offence.


5. Confidentiality – Client information should not be disclosed to other parties without


the client’s permission including HMRC. The exception to this rule applies if an
accountant has knowledge or suspicion that a person has committed a money
laundering offence.

606
New Clients

Before taking on new clients – a member of the ACCA must assess:

- Risk to the integrity of the practice on accepting the work.

- Whether the practice has adequate skills and competence to carry out the work.

- On accepting new clients – a member of the ACCA must ask permission from the
client to contact the old advisers to request information.

- If the client refuses then the ACCA member should consider not acting for them.

Once the new appointment has been agreed the tax adviser should issue a letter of
engagement setting out terms and conditions. 

As a professional tax adviser or accountant it is absolutely essential that a qualified


accountant conducts their affairs at all time following the professional code of ethics.

Procedure to follow before agreeing to become tax advisers to a company or


group of companies

Information needed:

• Proof of incorporation and primary business address and registered office.

• The group structure, directors and shareholders of the company.

• The identities of those persons instructing the firm on behalf of the company and
those persons that are authorised to do so.

Actions to take:

• Consider whether becoming tax advisers would create any threats to compliance with
the fundamental principles of the professional code of ethics, for example integrity
and professional competence. 

• Where such threats exist, then the appointment should not be accepted unless the
threats can be reduced to an acceptable level via implementation of safeguards.

• Contact the existing tax advisers in order to ensure that there has been no action by
the company that would, on ethical grounds, preclude us from accepting the
appointment.


607
Dealing with HMRC

Information provided to HMRC must be accurate and complete.

An ACCA member must not assist a client to plan or commit an offence. 

If the ACCA member becomes aware of a tax irregularity:

-  They must discuss it with the client.

-  Ensure proper disclosure to HMRC.

If the client refuses to follow the advice given then the professional adviser should stop
acting for the client and must inform the client and HMRC in writing.

608
Syllabus D. Communication in an appropriate
manner

Syllabus D. Communication

Communicate with clients, hm revenue and customs and other professionals in an appropriate
manner

Communication formats

Professional Marks

Between four and six professional marks are included in each of the Professional level
papers. 

These allow students to demonstrate – and examiners to assess – particular skills and
capabilities which employers expect ACCA members to possess on qualification.

Professional marks are awarded for the overall quality of answers, and for effective
professional communication skills.

1. Writing a letter


A letter should start with the sender’s address details (put the company name and
follow with ‘Address line 1, Address line 2’, etc below it) and the date. 


The letter should commence with ‘Dear XXXXX’ as specified in the requirements.

Throughout the text of the letter, write in the first person, using phrases such as ‘I’, ‘we’,
‘your board’, and ‘our company’ to personalise.


Keep the audience and their interests in mind throughout, and refer to them on
occasions to make it clear how your answer is still clearly focused.

609

Conclude with a suitable sentence, thanking the reader for their interest or time. Use
‘Yours faithfully’ where the addressee is anonymous, such as ‘shareholders’. 

Use ‘Yours sincerely’ if the addressee is named.


2. Writing a memo


A memo is generally written to a particular person or persons (such as a board
committee or the CEO). 

A memo format will need to be headed ‘Memorandum’ and have the To/From/Date/
Subject information at the top.


The style of writing will be precise and factual, leaving little room for interpretation. 

It will be written in the first person, directly addressing the person at whom it is aimed
(‘you’) and identifying the author as ‘I’.


3. Writing a report


The purpose of a report is usually to inform, occasionally providing recommendations or
suggestions for future action. 


It will have a wider circulation than a memo and is hence more formal in style and
wording. 


Like a memo, it will be written in the first person.

The initial structure of a report is similar to that of a memo, although the heading would
be ‘Report’. 


It is good practice to provide an introduction or ‘Terms of Reference’ section at the
start, stating exactly what the report aims to cover, and to finish with a summary or
conclusion. 


Use of headings and sub-headings will not only improve presentation but also add to
the ease of reading by the audience of the report (and the marker of your exam).


610
4. Writing briefing notes


These will be required to provide information to brief a board or committee, or external


parties such as institutional shareholders, on a particular event or decision. 


They will need to be well structured, focusing on the key points at the start, with
background information provided later. 


Briefing notes, along with management reporting narratives and press statements,
should be written in the third person, referring to ‘the company’, ‘XYZ Ltd’ and ‘the
board’, rather than ‘I’ or ‘we’.


611

You might also like